Vous êtes sur la page 1sur 284

https://t.

me/MedicalBooksStore
Best of Five MCQs for the
Geriatric Medicine SCE
Best of Five MCQs for the
Geriatric Medicine SCE

Dr Duncan R Forsyth MA FRCP


Consultant Geriatrician
Department of Medicine for the Elderly,
Addenbrooke’s Hospital
Cambridge University Hospitals NHS
Foundation Trust, Cambridge, UK

Dr Stephen J Wallis MA MRCP


Consultant Geriatrician
Department of Medicine for the Elderly,
Addenbrooke’s Hospital
Cambridge University Hospitals NHS
Foundation Trust, Cambridge, UK

1
3
Great Clarendon Street, Oxford, OX2 6DP,
United Kingdom
Oxford University Press is a department of the University of Oxford.
It furthers the University’s objective of excellence in research, scholarship,
and education by publishing worldwide. Oxford is a registered trade mark of
Oxford University Press in the UK and in certain other countries.
© Oxford University Press 204
The moral rights of the authors have been asserted
First Edition published in 204
Impression: 
All rights reserved. No part of this publication may be reproduced, stored in
a retrieval system, or transmitted, in any form or by any means, without the
prior permission in writing of Oxford University Press, or as expressly permitted
by law, by licence or under terms agreed with the appropriate reprographics
rights organization. Enquiries concerning reproduction outside the scope of the
above should be sent to the Rights Department, Oxford University Press, at the
address above
You must not circulate this work in any other form
and you must impose this same condition on any acquirer
Published in the United States of America by Oxford University Press
198 Madison Avenue, New York, NY 10016, United States of America
British Library Cataloguing in Publication Data
Data available
Library of Congress Control Number: 2013956949
ISBN 978–0–9–965160–3
Printed in Great Britain by
Clays Ltd, St Ives Plc
Oxford University Press makes no representation, express or implied, that
the drug dosages in this book are correct. Readers must therefore always check
the product information and clinical procedures with the most up-to-date
published product information and data sheets provided by the manufacturers
and the most recent codes of conduct and safety regulations. The authors and
the publishers do not accept responsibility or legal liability for any errors in the
text or for the misuse or misapplication of material in this work. Except where
otherwise stated, drug dosages and recommendations are for the non-pregnant
adult who is not breast-feeding.
PREFACE

Passing the Specialty Certificate Examination (SCE) in Geriatric Medicine is compulsory for all
trainees prior to obtaining their Certificate of Completion of Training (CCT) in the specialty.
The best of five multiple choice questions format requires a sound knowledge of the principles
and practice of caring for frail older people and an ability to interpret information to solve clinical
problems. Exam technique is also important and is best achieved through practice questions.
This book seeks to fill the void in suitable material to enable specialty trainees to test their knowl-
edge and practice their exam technique before presenting themselves for the SCE. The questions
have been written in three mock exam formats, each of 100 questions, with correct answers,
explanation, and further reading suggestions provided at the end of each mock paper. Appropriate
national guidance is referenced in the explanation and further reading. The distribution of questions
closely follows the actual format of the SCE with questions designed to test all aspects of the 2010
Geriatric Curriculum (updated 2012).
Although not specifically designed for candidates preparing for the Diploma in Geriatric Medicine
(DGM), these MCQs should also prove useful preparation for the DGM.
The authors have drawn upon their extensive knowledge and experience in clinical Geriatric
Medicine, as well as their involvement in developing national guidance. We hope that this book will
prove to be an invaluable resource to all trainees in the specialty and to others who wish to test
their knowledge of Geriatric Medicine (Paediatricians and Obstetricians may not find this book
helpful!).
We would like to record our grateful thanks to both our families for their support during the writ-
ing and editing of this book. We look forward to seeing more of them now!

Duncan R Forsyth and


Stephen J Wallis
CONTENTS

Abbreviations ix

Exam 1
Questions 1
Answers 53

Exam 2
Questions 87
Answers 135

Exam 3
Questions 173
Answers 219

Index 257
ABBREVIATIONS

A&E Accident and Emergency


ABPI ankle brachial pressure index
ACE Addenbrooke’s Cognitive Examination
ACE-I angiotensin-converting enzyme inhibitor
AED anti-epileptic drugs
AF atrial fibrillation
AFP alpha foetoprotein
ALP alkaline phosphatase
AMT Abbreviated Mental Test
ANP atrial-natriuretic polypeptide
APTT activated partial thromboplastin time
ARB angiotensin receptor blocker
AREDS Age-Related Eye Disease Study
ARMD age-related macular degeneration
BAPEN British Association for Parental and Enteral and Nutrition
BCC basal cell carcinoma
BD twice daily
BDA British Dietitian Association
BE base excess
BMI body mass index
BNP brain natriuretic polypeptide
BP blood pressure
BPSD Behavioural and Psychological Symptoms of Dementia
Ca calcium
CABG coronary artery bypass surgery
CCK cholecystokinin
CGA Comprehensive Geriatric Assessment
CHSD Centre for Health Service Development
CK creatinine kinase
x Abbreviations

CKD chronic kidney disease


CLO Campylobacter-like organism
CNS central nervous system
COPD chronic obstructive pulmonary disease
Cor. Ca corrected calcium
CPN community psychiatric nurse
Cr creatinine
CRP C-reactive protein
CSF cerebrospinal fluid
CT computed tomography
CTPA CT pulmonary angiography
CVP central venous pressure
CXR chest X-ray
DC direct current
DEXA dual energy X-ray absorptiometry
DLB dementia with Lewy bodies
DNACPR Do Not Attempt CardioPulmonary Resuscitation
DoLS Deprivation of Liberty Safeguard
DRE digital rectal examination
dRVVT Dilute Russell’s Viper Venom Time
DVLA Driver and Vehicle Licensing Authority
DVT deep vein thrombosis
ECG electrocardiogram
ED Emergency department
eGFR estimated glomerular filtration rate
ENT ear, nose, and throat
EPUAP European Pressure Ulcer Advisory Panel
ERCP endoscopic retrograde cholangiopancreatography
ESC European Society of Cardiology
ESR erythrocyte sedimentation rate
ETT exercise tolerance test
FBC full blood count
GCA giant cell arteritis
GCS Glasgow Coma Score
GDS Geriatric Depression Scale
GH growth hormone
GI gastrointestinal
Abbreviations xi

GORD gastro-oesophageal reflux disease


GTN glyceryl trinitrate
Hb haemoglobulin
HRT hormone replacement therapy
IC Intermediate Care
ICA internal carotid artery
IGF insulin-like growth factor
IM intramuscular
IMCA Independent Mental Capacity Advocate
INR International Normalized Ratio
IV intravenous
IVIg IV immunoglobulin
JVP jugular venous pressure
LFT liver function test
LMWH low molecular weight heparin
LP lumbar puncture
LRTI lower respiratory tract infection
LTOT long-term oxygen therapy
LVEF left ventricular ejection fraction
M/R modified release
MCV mean cell volume
MMSE Mini-Mental State Examination
MNA The Mini Nutritional Assessment
MRI magnetic resonance imaging
MST morphine sulphate tablets
MSU mid-stream urine
MUST Malnutrition Universal Screening Tool
n-AC n-acetylcysteine
NGT nasogastric tube
NIHSS National Institutes of Health Stroke Scale
NKS The National Key Scheme
NSAID non-steroidal anti-inflammatory drug
NSTEMI non-ST elevation myocardial infarction
OD once a day
ON omni nocte (at night)
PACS partial anterior circulation stroke
PCR polymerase chain reaction
xii Abbreviations

PE pulmonary embolism
PEG percutaneous endoscopic gastrostomy
PFE pelvic floor exercises
PO per os (by mouth)
pO2 partial pressure of arterial oxygen
POCS posterior circulation stroke
PPY pancreatic polypeptide
PRN pro re nata (as required)
PRV polycythaemia rubra vera
PSA prostatic-specific antigen
PT prothrombin time
PTH parathyroid hormone
PTT partial thromboplastin time
QDS four times daily
rTPA recombinant tissue plasminogen activator 
SC subcutaneous
SIADH syndrome of inappropriate antidiuretic hormone
SJS Stevens–Johnson syndrome
SOVA safeguarding of a vulnerable adult
SPICT Supportive and Palliative Care Indicators Tool
SQiD single question in delirium
SSRI selective serotonin reuptake inhibitors
TACS total anterior circulation stroke
TDS three times daily
TGA transient global amnesia
TIA transient ischaemic attack
TSH thyroid-stimulating hormone
TUG timed get up and go test
TVT transvaginal tape
U&E urea and electrolytes
UTI urinary tract infection
VAC vacuum-assisted closure
VQ ventilation-perfusion
VTE venous thromboembolism
WCC white cell count
exam 

1 QUESTIONS

1. An 82-year-old woman with a 3-year history of a vascular dementia


was admitted with a 2-week history of worsening confusion.
Over this 2-week period she had been wandering the streets at
night looking for the children she believed lived in her house.
She had also telephoned the police several times reporting the
children to be lost and had punched her son when confronted
about her changed behaviour. She was being treated with
bendroflumethiazide 2.5mg daily for hypertension, aspirin 75mg
daily, and rivastigmine 1.5mg BD. She lived alone in a first floor flat
in a sheltered housing complex and had carers to help her wash and
dress twice daily.
No abnormalities were detected on physical examination. Laboratory
investigations, including electrocardiogram and urine dipstick, were
normal, apart from serum potassium levels of 2.9mmol/L (3.5–4.9).
Bendroflumethiazide was therefore stopped.
Following review by the consultant in old age psychiatry, she was started
on regular haloperidol 2.5mg bd. Her restlessness and wandering
worsened to such an extent that she could not sit or lie still, even for few
minutes. Haloperidol was stopped, but her condition did not change.
What is the most likely cause of her increased motor restlessness?
A. Acute psychosis
B. Dementia progression
C. Drug-induced akathisia
D. Hyperactive delirium
E. Restless leg syndrome
2 EXAM 1 | QUESTIONS

2. A 72-year-old woman saw her general practitioner with a 2-week


history of general malaise. She mentioned that over the last
3 years she occasionally leaked urine when coughing, laughing, or
lifting things, but had never suffered from urgency of micturition,
dysuria, or haematuria. Examination of her abdominal,
cardiac, and respiratory systems were normal. There was no
lymphadenopathy. She was on no medication. Dipstick testing
of her urine was positive for nitrites only. Three days later the
following normal test results were seen: full blood count (FBC),
urea and electrolytes (U&E), serum calcium, liver and thyroid
function. Urine culture showed >100 000 colony-forming units/mL
of Escherichia coli.
What is the next most appropriate step in her management?
A. Advise on pelvic floor exercises
B. External vaginal examination
C. Nothing more needs to be done
D. Refer to the continence adviser
E. Treat with antibiotics

3. A 76-year-old woman with mild Parkinson’s disease was admitted


from home to a community rehabilitation facility because of
reduced mobility secondary to pain over her left hip, 2 days after
being knocked over by a boisterous dog. The day after admission
she developed a delirium thought to be due to a urinary tract
infection (UTI). One week after admission she had developed
a grade 2 sacral pressure sore. Her daughter considered the
hospital to have been negligent in their assessments of her
mother’s risk of developing pressure sores, and requested an
independent assessment of the hospital policies and procedures.
According to NICE guidelines when should a pressure sore risk
assessment have been instigated?
A. Every day during the admission
B. If the clinical situation changes
C. Within 6hr of admission
D. Within 8hr of admission
E. Within 24hr of admission
EXAM 1 | QUESTIONS 3

4. A 69-year-old woman was seen in outpatients. She gave a story of


increasing breathlessness on exertion over the past 4 weeks. She
used to be able to walk a mile without difficulty. However, now
she was becoming breathless after a hundred metres or so. This
was associated with some central chest heaviness. The symptoms
eased within a few minutes of stopping. She had no significant
past medical history, took no medication, and was a lifelong
non-smoker.
Cardiovascular, respiratory, and abdominal examinations were
unremarkable. Her BP was 138/74mmHg lying down and 140/78
standing. A resting electrocardiogram (ECG) confirmed sinus rhythm of
76 beats/min with leftward axis deviation. There were no ST segment
changes.
With regards to current guidelines, which of the following is the
most appropriate next step in her management?
A. Coronary angiography
B. CT angiogram
C. Exercise tolerance test (ETT)
D. Medical treatment with a long-acting nitrate
E. Trial of inhaled beta-agonist

5. An 86-year-old woman presented to the emergency department


with increasing confusion. She had a past medical history of breast
cancer, osteoporosis, and hypertension. Current medications
consisted of calcium and vitamin D supplements, alendronate
70mg weekly, and lisinopril 2.5mg OD.
Cardiovascular, respiratory, and abdominal examination were
unremarkable. She scored 22/30 on a mini-mental state examination
(MMSE). When asked, she stated that she did not have any pain.
FBC and U&E were normal. Corrected serum calcium was 3.1mmol/L
(2.2–2.6). She was started on IV fluids.
Which of the following is the next most appropriate step in her
management?
A. Arrange a bone scintigram
B. Check her vitamin D level
C. Give 90mg pamidronate IV
D. Request a computed tomography (CT) of the chest/abdomen/pelvis
E. Send a paired serum calcium and parathyroid hormone level
4 EXAM 1 | QUESTIONS

6. A 70-year-old man presented with a fall in the garden after the


sudden onset of headache, vomiting, and blurred vision. He had
been fasting for religious reasons. Paracetamol and codeine did
not help the headache, which he rated as 8/10.
He was being monitored yearly in the haematology clinic because of
polycythaemia rubra vera, for which he had undergone and finished
chemotherapy 5 years earlier. He also had hypertension. His current
medication was aspirin 75mg daily and amlodipine 5mg daily.
On examination his pulse was 70beats/min, blood pressure (BP)
138/90mmHg and heart sounds were normal. Both pupils reacted
sluggishly to light. There was bilateral papilloedema, and proximal and
distal weakness in his right leg (4/5). There was no neck rigidity. There
were no other neurological abnormalities.
ECG showed sinus rhythm, with slight left ventricular hypertrophy, but
no ischaemic changes.
Blood results
Hb 20.6g/dL (13–18)
WCC 10.2 × 109/L (4–11)
Platelets 450 × 109/L (150–400)
Urea 11.4mmol/L (2.5–7.0)
Creatinine 136µmol/L (60–110)
CRP 7mg/L (<10)
Sodium 149mmol/L (137–144)
Potassium 5.6mmol/L (3.5–4.9)
Liver function, bone profile, and thyroid function were normal.
What is the most likely diagnosis?
A. Viral meningitis
B. Para-sulcal meningioma
C. Sub-arachnoid haemorrhage
D. Sub-dural haemorrhage
E. Venous sinus thrombosis

7. Which of the following statements is false?


A. By 2034, 5% of the UK population is expected to be over the age of 85 years
B. In 2012 in the UK, life expectancy at birth is less than for those aged 65 years
C. Life expectancy for a woman aged 65 years in 2007 was 12 more years
D. The female:male ratio amongst over 65s is falling and expected to continue to do so
E. The median age of the UK population is increasing
EXAM 1 | QUESTIONS 5

8. A 71-year-old woman was referred to the rapid access stroke


clinic. Her husband reported that 2 days earlier they had walked
into the house for lunch after gardening and, although physically
functioning normally, she repeatedly asked him where they were,
what was happening, and why he was in her house. By the evening
she was back to normal with no recollection for this event. She
never lost consciousness. There was no history of any previous
similar events. She was on no medication.
What is the most likely diagnosis?
A. Dementia with Lewy bodies
B. Fugue state
C. Temporal lobe epilepsy
D. Transient global amnesia
E. Transient ischaemic attack

9. A 72-year-old woman complained of a gradual deterioration


in breathing, which was starting to limit her ability to do her
shopping and walk her dog. She smoked 15 cigarettes per day
from age 15 to 66. She produced a small amount of white phlegm
each morning. Chest X-ray (CXR) was normal. Her FEV1 was 60%
of expected, improving to 70% with nebulized salbutamol.
Which of the following would be the next most appropriate
treatment for her?
A. A course of oral prednisolone
B. Pulmonary rehabilitation
C. Regular inhaled muscarinic antagonist
D. Regular inhaled corticosteroid
E. Regular oral slow release theophylline
6 EXAM 1 | QUESTIONS

10. An 84-year-old man with severe Alzheimer’s dementia was being


treated in hospital for a UTI. His inflammatory markers improved
and blood tests normalized. He became increasingly agitated
and restless. He had been shouting out and disturbing the other
patients. He had not been violent, and was not putting himself or
others at risk.
Prior to this illness he lived in a nursing home where he was dependent
for all activities of daily living. He was unable to communicate and did
not recognize close family.
Examination demonstrated a new grade 1–2 sacral pressure sore, but
was otherwise unremarkable. He was not constipated and a bladder
scan excluded recurrent urinary retention.
What is the most appropriate pharmacological intervention?
A. Haloperidol 500micrograms as required per os (PO) or intramuscular (IM)
B. Lorazepam 500micrograms as required PO or IM
C. Paracetamol 1g QDS PO
D. Quetiapine 25mg BD regularly
E. Risperidone 500micrograms regularly BD PO

11. A 79-year-old man with type 2 diabetes had been started on


ramipril 2.5mg for hypertension 1 month earlier. In addition, he
took metformin 500mg BD, gliclazide 40mg BD, aspirin 75mg OD
and simvastatin 40mg ON.
His repeat bloods showed that his creatinine had increased as
shown below:

Initial appointment This appointment


Sodium 138mmol/L (137–144) 139mmol/L (137–144)
Potassium 4.2mmol/L (3.5–4.9) 4.4mmol/L (3.5–4.9)
Urea 6.3mmol/L (2.5–7.0) 6.4mmol/L (2.5–7.0)
Creatinine 100µmol/L (60–110) 128µmol/L (60–110)

His BP remained slightly elevated at 145/84mmHg.


Which of the following is the most appropriate course of action?
A. Add amlodipine 5mg OD and review renal tests in 1 week
B. Add amlodipine 5mg OD and review in 3 months
C. Add spironolactone 25mg OD
D. Change ramipril to losartan 50mg OD
E. Stop ramipril and initiate amlodipine 5mg OD
EXAM 1 | QUESTIONS 7

12. A 70-year-old man recovering from a cerebral infarction had


painful spasticity of the stroke affected limbs, which was impairing
his quality of life. He was unable to tolerate oral medication for
his spasticity.
What would be the most appropriate drug to try by intrathecal
infusion?
A. Baclofen
B. Botulinum toxin
C. Dantrolene
D. Diazepam
E. Tizanidine

13. Four weeks after suffering a thrombotic partial anterior


circulation stroke, a 79-year-old woman was anxious to discuss
possible ways to reduce her risk of future stroke. She was
independently mobile with a zimmer frame and able to climb
stairs, but was fearful she might fall. She had atrial fibrillation
and had been on amiodarone for several years with good rate
control. She had stable CKD-4 with a creatinine clearance of
28mL/min. She drank 2 glasses of wine each day, did not smoke,
and weighed 52kg. She was normotensive and not diabetic.
What is the most appropriate advice to give?
A. Amiodarone should be changed to dronedarone
B. Aspirin has a lower bleeding risk than warfarin and should be recommended
C. Rhythm control by catheter ablation should be considered
D. She should take dabigatran, rather than warfarin due to its lower bleeding risk
E. She should take warfarin and remain on amiodarone
8 EXAM 1 | QUESTIONS

14. An 84-year-old man had angina attacks on a near daily basis


when climbing stairs. The episodes settled rapidly with rest and
occasional use of his glyceryl trinitrate (GTN) spray. He was
taking aspirin 75mg OD, simvastatin 40mg OD and lisinopril
2.5mg OD for secondary prevention of ischaemic heart disease.
He had no significant past medical history and no known drug
allergies.
On examination, he was clinically in sinus rhythm at 70 beats/min with
a BP of 130/82mmHg. His chest was clear and abdominal examination
unremarkable.
Which of the following would be the most appropriate long-acting
anti-anginal to introduce?
A. Atenolol 100mg OD
B. Bisoprolol 2.5mg OD
C. Isosorbide mononitrate modified release (M/R) 60mg OD
D. Ivabradine 2.5mg BD
E. Nicorandil 10mg BD

15. A 78-year-old man with moderate Alzheimer’s was consistently


disruptive at meal times in the residential home where he had
recently come to live. The staff of the residential home requested
that this resident was moved to a specialist dementia unit in
another home. At all other times of the day he exhibited no
behavioural disturbance and slept well at night. He was on no
medication. Physical examination was entirely normal.
What would be the most appropriate next step in his
management?
A. Admit to psychiatric unit for further assessment
B. Arrange transfer to a specialist dementia care home
C. Discuss his behaviour with his family
D. Start a cholinesterase inhibitor
E. Start low-dose antipsychotic

16. An 82-year-old woman complained for several months of


headaches, fatigue, anorexia, and dry skin. Physical examination
was normal apart from dry skin. Her corrected serum calcium
was 2.9mmol/L (2.2–2.6).
What is the most likely diagnosis?
A. Hypervitaminosis A
B. Hypervitaminosis B3
C. Hypervitaminosis B6
D. Hypervitaminosis C
E. Hypervitaminosis E
EXAM 1 | QUESTIONS 9

17. An 89-year-old man was referred to the clinic with problematic


incontinence. He was having accidents several times a day and
his family were struggling to cope with the washing. They had
particular difficulties at night, as his bed could require changing
several times. The patient and his daughter felt that the problem
was partly lack of warning, but mainly impaired mobility
preventing him from getting to the toilet in time. Indeed, he had
had several falls trying to do this.
He had a past medical history of mild cognitive impairment, postural
hypotension, and atrial fibrillation. Examination confirmed a mild
postural drop in BP and a smooth, mildly-enlarged prostate, but was
otherwise unremarkable. A post-void bladder scan did not show any
significant residual volume. A prostate-specific antigen test was normal.
Which of the following interventions would be most appropriate?
A. Alpha-blocker
B. Anti-cholinergic
C. Conveen®
D. Pad and pants
E. Residential care

18. During a multidisciplinary meeting of community staff, to review


the case management of frail older people living at home or in a
care home, the community matron asked about the potential use
of telecare and telehealth in helping her manage her increasing
caseload of frail vulnerable older people.
What is the most appropriate advice to give her about telecare
and telehealth?
A. They are most effective in multiple long-term conditions
B. They are not capable of monitoring bath floods
C. They are unlikely to be of value for care home residents
D. They may improve medication concordance
E. They will eliminate the need for carers
10 EXAM 1 | QUESTIONS

19. A 74-year-old man presented following a fall. He tripped on


a loose paving slab and was unable to keep his balance. He
sustained no significant injuries. He last fell 1 month previously
when he slipped on a door step. He had been well recently, but
has had some increased urinary frequency and mild dysuria.
He had a past medical history of diverticulosis and Addison’s
disease. Current medications consisted of hydrocortisone 20mg at
breakfast, 10mg at lunch, and 10mg at supper, with fludrocortisone
50micrograms OD.
On examination he was clinically in sinus rhythm at 60beats/min with
a lying BP of 120/80mmHg and a standing BP of 125/82mmHg. There
was no evidence of peripheral oedema. Respiratory and abdominal
examinations were unremarkable. A urine dip was positive for nitrites
and leucocytes. Co-amoxiclav was started for a UTI, in line with local
microbiology guidelines. An ECG confirmed normal sinus rhythm
Which of the following is the next most appropriate step in his
management?
A. Arrange a cardiac Holter monitor
B. Arrange bone density scanning
C. Increase fludrocortisone dose
D. Increase hydrocortisone dose
E. Initiate bisphosphonate therapy
EXAM 1 | QUESTIONS 11

20. A 79-year-old male resident in a specialist dementia care home had


recently become bed-ridden and more aggressive towards the staff.
For the last 2 days he had been difficult to feed and was very variable
in accepting fluids. Ten days earlier he had been admitted to the local
hospital after a fall and been found to have a periprosthetic fracture
of his right femur, which was deemed stable and not requiring surgical
intervention. He had bilateral hip replacements for previous fractures.
He had been discharged back to the care home 6 days ago on the
following medication: 5micrograms/hr buprenorphine patch (changed
weekly) and 9.5mg/24hr rivastigmine patch (changed daily).
On physical examination, his BP was 110/76 and pulse 60beats/min sinus
rhythm. Examination of the chest and abdomen were normal, but he
became aggressive when attempts were made to roll him onto his side
to undertake a digital rectal examination and so this was abandoned. He
lashed out at the nursing staff when they attempted personal care, but
would accept the food and drink they offered. FBC, U&Es, bone profile,
and liver function tests (LFTs) were all normal, as were chest and plain
abdominal imaging.
What is the next best step in his management?
A. Add oromorph as required
B. Add daily senna and docusate
C. Add daily strontium ranelate
D. Increase strength buprenorphine patch
E. Give intravenous (IV) ibandronate 3-monthly
12 EXAM 1 | QUESTIONS

21. A 77-year-old woman was admitted from social housing after


her landlady had raised concern that she had boarded up her
bedsit to prevent intruders, was exhibiting paranoid ideation
about the other tenants, was neglecting herself, and was living
in squalor. On admission, she was dehydrated, unkempt, and
malnourished. After being initially suspicious of the hospital
staff, she accepted food and hydration, but would not let them
bathe her. Although she was independent in mobility she would
open her bowels on her bed, then wrap the stool in tissue and
put it on the floor. She could not understand why she was in
hospital, but made no attempt to leave. When attempts were
made to discuss discharge plans with her, she had no clear idea
of how she would look after herself and said that she could
always find people to help her, although she could not name
anyone in particular. She was reluctant to accept help from
outside agencies, stating that they would spy on her and take
her money. She appeared to understand that she could not
go back to her bedsit as this had been declared uninhabitable
and said she could sleep on the streets instead. She refused to
enter in to any discussion about care home placement. The
multidisciplinary team had assessed her as lacking the mental
capacity to determine her own care needs and to be at risk
of self-neglect if living alone. The liaison mental health team
had diagnosed a personality disorder with schizoid trait. She
declined to take any medication as she considered drugs to be
unnatural.
What is the most appropriate next course of action?
A. Detain her under the Mental Health Act for treatment
B. Discharge her to live on the streets as she wished
C. Refer for an Independent Mental Capacity Advocate (IMCA)
D. Refer for Deprivation of Liberty Safeguard (DoLS) assessment
E. Refer to social services to arrange discharge to a care home
EXAM 1 | QUESTIONS 13

22. A 73-year-old man, previously well and independent, was referred


with a history of dizziness. He took no medications.
Three weeks previously he had tripped on a loose paving stone and
sustained a mild head injury, but did not lose consciousness. Since then,
he had noticed that whenever he turned his head, the room appeared
to spin. This could be particularly dramatic when he rolled to his left in
bed. There was no associated neck pain. He did not report any altered
sensation or limb weakness. The GP had prescribed prochlorperazine,
but this had not ameliorated his symptoms.
On examination, he was in sinus rhythm, with no evidence of
postural hypotension. Respiratory and abdominal examinations were
unremarkable. He had a mild refractory visual impairment, which was
corrected with spectacles. Neurological examination was normal.
An ECG confirmed sinus rhythm with a normal PR, QRS, and QT
interval.
What is the next most appropriate step in his management?
A. Cardiac Holter monitoring
B. Cervical spine X-ray
C. Hallpike manoeuvre
D. Neuroimaging
E. Referral to ear, nose, and throat (ENT) department for audiometry
14 EXAM 1 | QUESTIONS

23. An 81-year-old man with chronic stable angina was taking asprin
75mg OD, simvastatin 40mg OD, bisoprolol 2.5mg OD, nicorandil
10mg BD, and lisinopril 2.5mg OD. He was a non-smoker and was
not diabetic.
He had been unwell for 4 weeks with generalized lethargy, increasing
exertional shortness of breath and postural light-headedness. During
this time he had 3 days of loose dark stools and mild upper abdominal
discomfort, which had now settled. Subsequently, he reported his angina
was occurring on minimal exertion.
On examination, he was pale, with a sinus tachycardia of 105beats/
min. His BP was 130/70 sitting and 105/60 standing. Respiratory and
abdominal examinations were unremarkable, but he declined a digital
rectal examination. While in the clinic, he had an episode of suspected
angina and an ECG confirmed dynamic anterior lead t-wave inversion.
This settled with use of the GTN spray.
Which of the following is the most appropriate immediate course
of action?
A. GTN infusion if pain recurs and 12-hr troponin
B. If immediate troponin normal, discharge on increased dose of nicorandil
C. IV fluid with urgent bloods and review thereafter
D. Referral for acute angiography +/– angioplasty
E. Therapeutic clexane, clopidogrel, aspirin, and 12-hr troponin

24. A 69 year old man with multiple sclerosis (MS) was recovering
from his fourth episode of aspiration pneumonia within the last
12 weeks. For the last year he had been bed-bound at home and
required double-up four times a day care with hoist transfers.
He was doubly incontinent and had refused catheterization. He
was assessed as being unable to swallow safely and at extremely
high risk of further aspiration. He had completed an advanced
care plan, when originally diagnosed with MS, which stated that
he would not wish to be artificially fed under any circumstance.
When discussing his discharge arrangements from hospital
he stated that he would wish to be transferred to an assisted
suicide establishment in Switzerland as he considered his future
existence to be intolerable. He was deemed to be mentally
competent and fully understood the inference of his request.
What is the most appropriate next step in planning his discharge?
A. Arrange his discharge according to his wishes
B. Ask his family to arrange his discharge to Switzerland
C. Negotiate an alternative discharge plan with him
D. Refer him for more formal psychiatric assessment
E. Refer him to an Independent Mental Capacity Advocate
EXAM 1 | QUESTIONS 15

25. The prevalence of cardiovascular disease increases with age, and


disease symptoms and signs may be affected by the effects of
normal ageing.
Which of the following is a feature of normal cardiovascular
ageing?
A. Decreased peripheral vascular resistance
B. Decreased pulse-wave velocity
C. Increased cardiac output
D. Increased P–R interval
E. Increased resting heart rate

26. A 79-year-old man was seen in the transient ischaemic attack


(TIA) clinic. He had presented at the medical admissions unit
2 days previously with a 30-min history of sudden onset left
arm paraesthesia. He had no significant past medical history.
Examination in the clinic was unremarkable. His BP while lying
was 130/72mmHg. An ECG confirmed sinus rhythm. Blood tests
showed a random cholesterol of 5.5mmol/L (<5.2) and were
otherwise normal. The Accident and Emergency (A&E) doctors
had started him on aspirin 75mg OD and simvastatin 40mg OD.
A diagnosis of middle cerebral artery territory TIA was made.
CT head and carotid Doppler study
• CT: right-sided lacunar infarct.
• Dopplers: 70% stenosis of the left internal carotid artery (ICA). 10% stenosis of the
right ICA.
Which of the following is the most appropriate course of action?
A. Add clopidogrel 75mg OD
B. Ask the GP to titrate statin dose and monitor BP
C. Change to 80mg atorvastatin omni nocte
D. Refer to vascular surgeons for endarterectomy
E. Request magnetic resonance imaging (MRI) carotid angiography
16 EXAM 1 | QUESTIONS

27. An 87-year-old man was admitted with a worsening headache and


nausea. These were worse on lying down and coughing. He had no
significant past medical history and took no regular medications.
While being clerked he developed double vision, then had a tonic-clonic
seizure. An urgent MRI was undertaken, which confirmed venous sinus
thrombosis with a small secondary cerebral haemorrhage. He was
transferred to the stroke unit.
A baseline FBC and coagulation profile were normal.
Which of the following is the most appropriate intervention?
A. Anticoagulate with low-molecular weight heparin
B. Anticoagulate with warfarin
C. Aspirin 300mg OD for 2 weeks then aspirin 75mg OD monotherapy
D. Supportive treatment alone
E. Urgent thrombolysis

28. An 83-year-old male consulted his general practitioner regarding


an acutely swollen left knee. A diagnosis of mono-arthritis due to
osteoarthritis was made, which responded to a single injection
of triamcinolone and lignocaine. Over the next 4 weeks his BP
was found to be persistently raised at 164/102mmHg. He was
otherwise well and taking no medication. His body mass index
(BMI) was 22.9 (18.5–24.9), urinalysis was negative for proteinuria,
estimated glomerular filtration rate (eGFR) was 98mL/min
(>90mL/min), resting electrocardiogram was normal, U&Es were
normal. He was not on any medication.
What is the most appropriate initial treatment for his
hypertension?
A. Angiotensin-converting enzyme inhibitor
B. Angiotensin II receptor blocker
C. Beta-blocker
D. Calcium channel blocker
E. Thiazide diuretic
EXAM 1 | QUESTIONS 17

29. An 81-year-old woman was seen by her general practitioner at


her daughter’s request. She had phoned her daughter several
times in the previous 2 months saying that there were intruders
and children in the house. No other persons were found in the
house and all of the house locks were secure. She lived alone in a
bungalow and was independent in all activities of daily living. She
was on treatment for hypertension, type II diabetes mellitus, and
Parkinsonism (aspirin 75mg daily, ramipril 5mg daily, gliclazide
40mg BD, simvastatin 40 mg daily and Madopar® 62.5mg TDS).
She still enjoyed playing bridge and joining in the quiz night at the
local pub.
Her MMSE was 28/30. She was in atrial fibrillation with a resting heart
rate of 78beats/min. Resting BP was 128/76 with no postural drop. She
had bilateral mature cataracts. There was mild cogwheel rigidity in both
legs with normal tone in the arms. Power was 4+ in all limbs and reflexes
were normal in all limbs. There was no tremor and no cranial nerve
abnormality.
What is the most likely cause of her hallucinations?
A. Charles Bonnet syndrome
B. Dementia with Lewy bodies
C. Multi-system atrophy
D. Psychotic depression
E. Vascular dementia

30. Which of the following statements best reflects the likelihood of


reducing falls in a hospital setting?
A. Bed rails reduce falls and falls risk
B. Exercise reduces falls and falls risk in acute hospitals
C. Medication review alone has been shown to reduce falls
D. Multi-factorial interventions reduce falls and falls risk
E. Vitamin D supplementation reduces falls and falls risk
18 EXAM 1 | QUESTIONS

31. An 82-year-old woman was discharged from the orthogeriatric


unit with twice daily reablement social care after successfully
recovering from repair of a left fractured neck of femur. She was
re-admitted 4 days later in urinary retention. On day 21 of the
second admission she was independent in mobility and personal
care on the ward, and had managed the stairs. She was eager to
get home and wanted to have twice daily reablement care again.
She felt slightly anxious having been re-admitted so soon after her
first discharge. Her medication was now calcium and vitamin D,
weekly alendronic acid, regular sodium docusate. On the planned
day of discharge her daughter cancelled the care package, as she
was going on holiday, stating that her mother needed a bigger
care package and should not be discharged whilst the daughter
was away. Attempts by the care manager and ward team to
contact the daughter to discuss her concerns and actions were
unsuccessful. The patient’s MMSE was 29/30.
What is the most appropriate course of action?
A. Arrange an interim care bed in a residential home while waiting for a larger care package
B. Delay the discharge until the daughter is contactable
C. Instigate Safeguarding of a Vulnerable Adult (SOVA) proceedings
D. Restart the planned care and discharge the patient
E. Transfer to a rehabilitation unit for further therapy input

32. A 78-year-old woman presented following a fall. She had no


significant past medical history and took no regular medication.
She was fully independent and drove a car.
She fell when her foot caught on a small step at the entrance to her
house. She heard a loud crack and was unable to get up owing to severe
pain in her left hip. A neighbour called an ambulance.
An X-ray confirmed a subtrochanteric fractured neck of femur. She was
reviewed by the orthopaedic team.
Which of the following would be the most appropriate
intervention?
A. Conservative management
B. Dynamic hip screw
C. Hemiarthroplasty
D. Intramedullary nailing
E. Total hip replacement
EXAM 1 | QUESTIONS 19

33. An 83-year-old man was becoming increasingly frail and losing


physical function.
In which hierarchial order is he most likely to lose physical
function?
A. Bathing, dressing, toileting, transferring, feeding
B. Bathing, dressing, transferring, toileting, feeding
C. Dressing, bathing, toileting, transferring, feeding
D. Toileting, bathing, dressing, transferring, feeding
E. Transferring, toileting, dressing, bathing, feeding

34. A 68-year-old woman awoke one morning with a left-sided facial


weakness. She first noticed this when her orange juice dribbled
outside the corner of her mouth and when she then looked in the
mirror. She had a past history of hypertension, diabetes mellitus,
and had some cosmetic botox injections 3 weeks previously.
On examination she had a left-sided facial weakness with involvement
of frontalis on that side. Power was preserved in her upper and lower
limbs. Co-ordination was intact. There was no obvious speech deficit or
visual field loss, although she did have difficulty closing her left eye.
Which of the following is the most likely diagnosis?
A. Bell’s palsy
B. Iatrogenic paralysis
C. Lateral medullary syndrome
D. Right middle cerebral territory infarct
E. Venous sinus thrombosis

35. A 93-year-old man presented to the Emergency Department with a


‘general decline’. He had become withdrawn and lethargic. His appetite
had diminished and he had noticed some bleeding from his gums.
His daughter was concerned that he had not been managing well at
home since the death of his wife the previous year. His house was very
unkempt, he struggled with personal care, and ate a very limited diet of
sandwiches and canned ham. He frequently missed meals.
On examination, a rash was noticed on his lower limbs. This seemed to
be based around hair follicles and the hairs themselves were twisted.
Splinter haemorrhages were noted on his hands. A quiet pan-systolic
murmur is noted.
Which of the following is the most appropriate treatment?
A. Folic acid 5mg OD
B. Intravenous benzyl penicillin and gentamicin
C. Platelet transfusion
D. Thiamine 100mg BD
E. Vitamin C 250mg QDS
20 EXAM 1 | QUESTIONS

36. A 76-year-old woman was referred to the falls assessment clinic


after having six falls in as many weeks. Her timed get up and go
test score was 25sec.
Which of the following statements best describes the timed get
up and go test?
A. It cannot be completed by people with cognitive impairment
B. It is an unhelpful test in people with painful arthritis
C. It is only useful in assessing outcome of falls interventions
D. It may be helpful in diagnosing the cause of falls
E. The patient should be tested without using any walking aid

37. A 78-year-old woman attended the emergency department


for the ninth time in 6 months. She was covered in bruises of
different ages, and had fractured several ribs and both clavicles.
She was reluctant to be admitted to hospital and could not
explain her injuries. The Emergency department (ED) registrar
suspected elder abuse.
In elder abuse, which of the following is most likely?
A. The abuser has a mental health problem
B. The abuser is dependent upon the victim
C. The abuser is from outside of the family
D. The victim is financially dependent on the abuser
E. The victim has provoked the abuser
EXAM 1 | QUESTIONS 21

38. A 73-year-old man admitted 5 days previously with an


exacerbation of chronic obstructive pulmonary disease (COPD)
was scheduled for discharge the next day. His last exacerbation
was over a year ago. He had only required two courses of steroids
in his life. His usual COPD treatment consisted of a long-acting
muscarinic antagonist and a combination long-acting β-agonist/
steroid inhaler.
His other past medical history included ischaemic heart disease and
hypertension. His non-COPD medications were aspirin 75mg OD,
bisoprolol 5mg OD and simvastatin 40mg ON.
Pre-discharge arterial blood gas
pO2 8.5 (11.3–12.6kPa)
pCO2 4.8 (4.7–6.0kPa)
pH 7.4 (7.36–7.44)
BE 0.1 (–2 to 2mmol/L)

Which one of the following would be an appropriate intervention


on discharge?
A. Long-term oxygen therapy (LTOT)
B. Provide ‘stand-by’ steroids and antibiotics
C. Refer for a dual energy X-ray absorptiometry (DEXA) scan
D. Start an oral mucolytic
E. Stop β-blocker therapy

39. A 76-year-old resident of a care home, with metastatic breast


cancer, developed offensive diarrhoea after a recent bout
of constipation. She was being treated with paracetamol, a
non-steroidal anti-inflammatory drug (NSAID) and a transdermal
opiate for bone pain and intermittent pamidronate infusions for
hypercalcaemia.
What is the most likely cause of her diarrhoea?
A. Clostridium difficile
B. Diverticulitis
C. Drug-induced colitis
D. Laxative abuse
E. Overflow diarrhoea
22 EXAM 1 | QUESTIONS

40. An 89-year-old female with type 2 diabetes was managed with


metformin 500mg TDS and insulin glargine 15U nocte. Her
control had become increasingly erratic over the last 2 months,
although her HbA1c remains stable at 48mmol/mmol (target <
58mmol/mmol). Her blood glucose monitoring diary is shown
below. ‘After’ refers to 2hr post-prandial. She reports that her
diet and activity levels have not changed.

Before After Before After Before After Before


breakfast breakfast lunch lunch dinner dinner bed
Day 1 4.2 9.2 6 11 4.2 7 5.1
Day 2 17 8 4.2 8 5.7 5.5 5.2
Day 3 4.1 7.2 4.2 6.3 4.5 8 8
Day 4 21 6.2 3.9 5.2 4.1 5 5

Although not exhaustive, which of the following is the most


appropriate single intervention?
A. Add gliclazide 40mg BD
B. Increase glargine dose to 18U
C. Record nocturnal blood glucose
D. Reduce glargine dose to 11U
E. Start sitagliptin 100mg OD

41. An 84-year-old female had been found to have a fungating breast


tumour. She was adamant that she did not want any treatment
or investigation, and was deemed to have the capacity to make
these decisions. However, she found the odour both distressing
and embarrassing, which was preventing her from socializing.
Relatives feared that she may have become depressed.
Which of the following is the most appropriate intervention?
A. Activated charcoal dressing
B. Hydrocolloid dressing
C. Mirtazapine 15mg ON
D. Referral for breast biopsy
E. Tranexamic acid soaked dressing
EXAM 1 | QUESTIONS 23

42. A 90-year-old woman was admitted to hospital with a fractured


pubic ramus. On admission, she was thought to have a UTI
and was given a 5-day course of co-amoxiclav. One week after
admission she developed diarrhoea.
What is the most important next step in her management?
A. Arrange colonoscopy
B. Digital rectal examination
C. Prescribe loperamide
D. Request stool culture
E. Transfer to the isolation ward

43. A 69-year-old man was brought to A&E by his daughter. She was
concerned that he had become increasingly confused. He was in
good health until 5 days earlier, when he developed diarrhoea
and vomiting. This had now been settled for 48hr. His daughter
suspected he had caught the ‘winter vomiting bug’ from his
grandson.
On examination, he had dry mucous membranes. His pulse rate was
90beats/min and regular, and his BP was 124/82. His jugular venous
pressure (JVP) was not visible and there was no peripheral oedema. His
chest was clear, and his abdomen soft and non-tender. There was no
focal neurological deficit.
Blood test
Na 122mmol/L (137–144)
K 3.9mmol/L (3.5–4.9)
Urea 9.1mmol/L (2.5–7.0)
Cr 130µmol/L (60–110)
Urine osmolality 700mosmol/kg (350–1000)
Random cortisol (at 09:00 hours) 600nmol/L (200–700)

Which of the following is the most appropriate course of action?


A. Arrange a CT head scan
B. Fluid restrict to 1L/day
C. Infuse hypertonic 1.8% saline
D. Normal dietary intake and oral rehydration salts
E. Undertake a short Synacthen test
24 EXAM 1 | QUESTIONS

44. A 77-year-old Sri Lankan woman was visiting her family in the UK.
She complained of generalized pain and weakness, with increasing
difficulty getting in and out bed and from chairs. Her family were
worried that she might have cancer and wanted a diagnosis before
she returned home. Physical examination was normal apart from
her having bowed legs.
Investigations
Haemoglobin 10.8g/dL (11.5–16.6)
Mean cell volume 70fl (80–96)
Erythrocyte sedimentation rate 50mm/1st hour (<30)
Serum corrected calcium 2.12mmol/L (2.2–2.6)
Serum alkaline phosphatase 156U/l (45–105)
Serum albumin 35g/L (37–49)
Serum ferritin 90μg/l (15–300)
Thyroid-stimulating hormone 5.0mU/L (0.4–5.0)
Free T4 10pmol/L (10–22)

What is the most likely cause of her symptoms?


A. Cushing’s syndrome
B. Gastric adenocarcinoma
C. Hypothyroidism
D. Osteomalacia
E. Polymyalgia rheumatic

45. An 82-year-old woman was seen in the falls and bone health clinic.
She sustained a fractured left neck of femur as a result of a slip on
uneven ground. Her past medical history included chronic renal
impairment (CKD-4), type 2 diabetes mellitus, and hypertension.
She suffered from infrequent indigestion.
Which of the following would be the most appropriate course of
action to minimize her risk of future fracture?
A. Arrange a bone density (DEXA) scan
B. Commence alendronic acid 70mg weekly
C. Commence strontium ranelate 2g ON
D. Start her on 1.2g calcium + 800U colecalciferol daily
E. Start twice yearly subcutaneous (SC) denosumab injections
EXAM 1 | QUESTIONS 25

46. A 92-year-old man with a severe pneumonia had not been


responding to treatment. The multidisciplinary team agreed that
the emphasis of his management should be palliative. His family
were in agreement with this.
25hr later the patient had become increasingly agitated and was
vomiting. His family did not think he was in pain.
Which of the following is the most appropriate course of action?
A. IV fluids and nasogastric tube (NGT) insertion
B. Stat dose of subcutaneous levomepromazine
C. Prescribe diamorphine via syringe driver
D. SC diamorphine and cyclizine infusion
E. SC hyoscine hydrobromide infusion

47. Which of the following is not a function of walking equipment?


A. To enable weight redistribution to the upper limbs
B. To encourage upright body posture
C. To increase confidence and reduce fear of falls
D. To provide a wider support base and stability
E. To reduce the risk of injury when a fall occurs

48. A 76-year-old man was admitted following an episode of delirium


provoked by a lower UTI. He lived with his wife in a house that
they owned, supported with care, three times a day. He was not
known to have any pre-existing cognitive impairment.
One week later his inflammatory markers and urinary symptoms had
resolved. However, he remained confused with fluctuating attention
levels and the nurses were concerned because he was wandering on
the ward at night. He was eager to go home, but was unable to follow
discussions regarding his current care needs. His MMSE score was 16/30.
Examination was unremarkable and he was not constipated.
What is the most appropriate way to proceed?
A. Arrange a best interests meeting
B. Ask social services to arrange residential placement
C. Discharge him home as soon as care can be restarted
D. Re-discuss at next discharge planning meeting
E. Refer him to the independent mental capacity advocate service
26 EXAM 1 | QUESTIONS

49. Which of the following rehabilitation settings is most likely to be


associated with reduced length of stay and costs when compared to
post-acute care in general hospitals?
A. Day hospital rehabilitation
B. Care home-based intermediate care
C. Inpatient community hospital rehabilitation
D. Nurse-led intermediate care
E. Therapist-led intermediate care

50. A 75-year-old woman with learning difficulties was admitted from


a residential home with a 2-week history of tiredness, weakness,
and decreased mobility. She had a previous history of asthma
(1 year), hypertension (4 years), duodenal ulcer (20 years ago),
tuberculosis (60 years ago). She complained of tingling in her
fingers and toes. She had recently been treated with lamisil for a
fungal nail infection in her hands. Current medication was aspirin
75mg daily and bendroflumethiazide 2.5mg daily.
Examination
BP was 140/72 and heart rate 80beats/min, sinus rhythm. There was
a pan-systolic murmur at the left sternal edge. There were bi-basal
crackles when auscultating her chest. Neurological examination
revealed decreased power 2/5 dorsi-flexion, inversion, and eversion
R foot, decreased ankle and knee jerks on the right, equivocal right
plantar response with downgoing left plantar, reduced pinprick sensation
distally in the right limbs in a glove and stocking distribution. She walked
dragging both legs—this was more pronounced on the right.
Investigations
ESR 86mm/1st hour (<30)
Hb 9.8g/dL (11.5–16.5)
WCC 15.4 × 109/l (4–11)
Eosinophils 10.5 × 109/L (0.04–0.40)
Na 127mmol/L (137–144)
Serum osmolality 280mosmol/kg (278–305)
Urine osmolality 535mosmol/kg (350–1000)
24-hr urinary protein 0.1g (<0.2)
CXR Patchy alveolar shadowing bilaterally

What is the most likely diagnosis?


A. Allergic drug reaction
B. Churg–Strauss syndrome
C. Hyper-eosinophilic syndrome
D. Parasitic infestation
E. Occult malignancy
EXAM 1 | QUESTIONS 27

51. A 72-year-old man had been admitted with an upper


gastrointestinal bleed. He had been taking aspirin 75mg OD
for secondary prevention of ischaemic heart disease, as well as
simvastatin 40mg OD and amlodipine 5mg OD.
Upper gastrointestinal endoscopy confirmed a healing duodenal ulcer
with no stigmata of ongoing bleeding. A Campylobacter-like organism
(CLO) test was negative. The endoscopist advised starting omeprazole
40mg OD.
Which of the following statements regarding his other
medications is most appropriate?
A. Aspirin should be stopped
B. Clopidogrel should be substituted in place of aspirin
C. H. pylori eradication therapy should be started
D. No other changes are required
E. Ranitidine 150mg BD should be started

52. A 94-year-old man was admitted following a witnessed fall. He was


seen to trip on a loose rug. He sustained no injuries. His only past
medical history was of hypertension. He was taking aspirin 75mg
OD and bendroflumethiazide 2.5mg OD. He denied alcohol intake
and lived on his own without assistance. He prepared all his own
food and walked with a stick.
On examination, he appeared unkempt, but no focal abnormality
was found. The paramedics were concerned that he was struggling to
maintain a safe environment at home, and that his house was cluttered
and unclean. The fridge was empty.
Blood tests demonstrate:
• Normal renal function and LFTs.
• Normal FBC.
• Prothrombin time (PT) 20sec (11.5–15.5).
• Activated partial thromboplastin time (APTT) 35sec (30–40).
The abnormal clotting corrected with vitamin K. Which of the
following is the most appropriate course of action at this point?
A. Alcohol detoxification regime
B. Clotting factor assay
C. Dilute Russell’s Viper Venom Time testing
D. Referral to dietician and social services
E. Stop aspirin
28 EXAM 1 | QUESTIONS

53. A 73-year-old woman complained of loss of taste and that her


hair was coming out in clumps when she brushed it. Because of
concern over ‘mad-cow’ disease she had become a vegan 10 years
earlier. Her skin was noted to be dry and scaly with several poorly
healing wounds where she had been scratched by her cats. Aged
62 years she had been diagnosed with osteoporosis and treated
for 10 years with hormone replacement therapy (HRT).
A deficiency of which micronutrient is most likely to explain the
physical signs?
A. Iron
B. Thiamine
C. Vitamin A
D. Vitamin C
E. Zinc

54. A 67-year-old man with severe bilateral Parkinson’s suffered


regular, but predictable ‘off’ periods. He was taking Madopar®
125 every 3hr, amantadine 100mg BD, tolcapone 200mg TDS,
and selegiline 5mg daily. He had declined treatment with
apomorphine due to a needle phobia. He lived in a remote farm
house and his wife did not drive. His daughter and son thought
he was no longer safe to drive and would not let him drive his
grandchildren anywhere. He thought that he was still safe to
drive and was not willing to stop, as this would leave both him and
his wife isolated. His MMSE, when ‘on’ was 26/30. He was on a
3-yearly review of fitness to drive by the DVLA.
What is the most appropriate way to manage this situation?
A. Advise him to pull off the road when he is switching ‘off ’
B. Advise him to take a driving test at a DVLA driving centre
C. Notify the DVLA that he is unsafe to drive
D. Tell him that he must stop driving
E. Wait until the DVLA next seek a medical report
EXAM 1 | QUESTIONS 29

55. The family of an 84-year-old man with Parkinson’s disease were


concerned that he was becoming more confused and that he
was becoming increasingly more agitated about a man that
kept visiting his wife and having sex with her whilst she watched
television. He was also frightened that a giant snake lived in their
bathroom, so he had stopped going upstairs and was urinating
in the potted plants in the living room. He lived in a bungalow
with his wife and had carers three times a day. He attended a day
centre two mornings a week. His wife was finding it increasingly
stressful living with him.
His medication regime consisted of co-careldopa 125mg QDS,
procyclidine 25mg TDS and selegiline 5mg OD. He had no known drug
allergies.
Cardiovascular, respiratory, and abdominal examination were
unremarkable. Neurological examination showed no significant increase
in tone. There was mild dyskinesia of all limbs. He scored 14/30 on an
MMSE, but this was difficult to complete due to his fluctuating level of
consciousness.
FBC, U&E, and C-reactive protein (CRP) were all normal. A urine dip
was positive for leucocytes and protein, but negative for nitrites, blood,
and ketones.
What is the most appropriate next course of action?
A. Prescribe a course of trimethoprim
B. Prescribe risperidone
C. Prescribe rivastigmine
D. Stop the selegiline
E. Withdraw the procyclidine

56. An 89-year-old female care home resident developed pneumonia


and was treated with antibiotics in the care home, as her
preferred place of care. During this period of ill health, she
became bed-bound, had urinary incontinence, and developed a
grade 2 sacral pressure sore.
What is the most appropriate management of her incontinence?
A. Fluid restriction
B. Pads and pants
C. Regular laxatives
D. Ring pessary
E. Urinary catheter
30 EXAM 1 | QUESTIONS

57. A 72-year-old patient was discharged from hospital after a


prolonged period of rehabilitation following a left middle cerebral
artery territory ischaemic stroke. She had made a good motor
recovery, but had been left with marked dyslexia, such that she
could no longer read the paper. This was an activity she previously
enjoyed.
Which of the following best describes this change in function?
A. Disability
B. Dysfunction
C. Impairment
D. Impediment
E. Handicap

58. A 91-year-old man with mild dementia (MMSE 20/30) was ready
for discharge from hospital after recovering from a fall. The
therapy assessment recommended he had a twice daily care
package, and he said he was willing to accept this if his daughters
could not help him. His two elderly daughters stated that he
would say anything to get out of hospital and that, in the past, he
had always refused care, and expected them to provide whatever
help he demanded from them, whenever he demanded it. The
daughters said he had always been a difficult man who expected
his family to care for him, but that they no longer felt able to
provide care for him. They wanted him to be discharged to a
residential home.
What is the next most appropriate step?
A. Arrange his discharge to a residential care home
B. Arrange his discharge home with suitable care package
C. Referral to liaison psychiatry for a mental capacity assessment
D. Transfer to intermediate care for further discharge planning
E. The daughters should tell him they can no longer help with his care
EXAM 1 | QUESTIONS 31

59. A 75-year-old man was brought to the ED after being found


trying to get in to the village post office at 04.00 hours. He was
disorientated to time and place, easily distracted whilst being
examined, and smelt of urine. He was in sinus rhythm at 80beats/
min with a BP of 140/76 both lying and standing. Neurological
examination was normal. MMSE was 20/30 and he was unable
to draw a clock. He looked emaciated and had nicotine staining
of his fingers. He was not clubbed. Chest examination found
scattered rhonchi throughout both lung fields. There was no
lymphadenopathy. Abdominal and rectal examination were
normal.
The following information was obtained when his daughter arrived in
the ED. He had been losing weight for 2 months and becoming more
confused over the last 10 days. He smoked 30 cigarettes per day and
drank one large whisky each evening. He was on no medication.
Initial blood results
LFTs, blood sugar, and FBC were normal

Sodium 121mmol/L (136–146mmol/L)


Potassium 4.6mmol/L (3.5–5.3mmol/L)
Urea 2.3mmol/L (2.1–7.6mmol/L)
Creatinine 56mmol/L (51–107mmol/L)

What is the most likely diagnosis?


A. Adrenocortical insufficiency
B. Carcinoma lung
C. Congestive cardiac failure
D. Hypothyroidism
E. Urinary tract infection
32 EXAM 1 | QUESTIONS

60. A 72-year-old woman was seen in outpatients with painful


feet. She described the pain as sharp and stinging—like ‘pins
and needles’. It had been deteriorating steadily over the past
12 months. She had a past medical history of hypertension,
osteoarthritis, diet-controlled diabetes mellitus, and stroke.
On examination, she was mildly hypertensive at 150/80, with normal
heart rate and sounds. Her respiratory and abdominal examination
were unremarkable. She had attenuated ankle reflexes and a ‘stocking’
distribution decrease in light touch, pin prick, and temperature
sensation. Dorsalis pedis pulses were present bilaterally.
Which of the following medications is the most appropriate
to start?
A. Amitryptilline 10mg omni nocte
B. Duloxetine 60mg OD
C. Gabapentin 300mg OD
D. Oramorph 5mg QDS
E. Pregabalin 75mg BD

61. A 76-year-old man was troubled with nocturnal urinary frequency.


He had been seeing the continence nurse who had referred
him for further assessment. He woke up to five times a night to
urinate, passing reasonable volumes (estimated at 100–200mL)
each time. He had no significant day time symptoms.
Physical, including prostate, examination was unremarkable. There
were no abnormalities with his renal profile, FBC or prostatic-specific
antigen (PSA).
Which of the following is the least appropriate initial course of
action?
A. Advise abstention from caffeinated beverages in the afternoon/evening
B. Desmopressin therapy
C. Long-term catheterization
D. Trial of anticholinergic therapy
E. Use of a sheath-based continence system (e.g. Conveen®)
EXAM 1 | QUESTIONS 33

62. A 73-year-old man attended for a review of his diabetes. He had


been managed by diet alone and was only taking ramipril 1.25mg
OD. His latest HbA1c result was 75mmol/mol (target 48–58mmol/
mol). His BP was 142/86mmHg and a random cholesterol
5.9mmol/L.
Which of the following is the least appropriate next step in his
management?
A. Calculate 10-year cardiovascular risk
B. Initiate treatment with metformin
C. Initiate treatment with simvastatin
D. Start low dose aspirin
E. Titrate up his ramipril

63. A 75-year-old woman presented following a fall. She was initially


slightly vague regarding the mechanism, but reported that
she was moving across the living room, when her legs gave
way. She did not think she had lost consciousness. She had
not experienced preceding light-headedness, chest pain, or
palpitations.
Past medical history included atrial fibrillation (AF), diabetes mellitus,
and hypertension. Medications consisted of bisoprolol 5mg OD, aspirin
75mg OD, metformin 500mg OD.
On examination, bilateral periorbital bruising was evident. The patient
had an irregular heart rhythm at 60beats/min. Heart sounds were
normal. BP was 140/80 with no drop on standing. Respiratory and
abdominal examination were unremarkable.
An ECG confirmed AF at a rate of 48–65beats/min with a right bundle
branch block morphology. Bloods demonstrated normal electrolytes,
inflammatory markers, and troponin.
Which of the following is the next most appropriate course of
action?
A. Admit the patient for cardiac monitoring
B. Discharge and refer to community falls clinic
C. Initiate calcium and vitamin D supplementation
D. Physiotherapy review
E. Refer for tilt table testing
34 EXAM 1 | QUESTIONS

64. The son of an 82-year-old woman was concerned that his sister
has been pressuring his mother to sign cheques for her.
What is the next most appropriate course of action for you to take?
A. Ask the son to relay his concerns to social services
B. Contact the trust adult safeguarding lead
C. Discuss with the patient and assess their capacity
D. Refer the case on to the senior nursing team
E. Refer the patient to the ward social worker

65. An 86-year-old woman presented after a fall. She had not fallen
previously and recalled a rug slipping under her foot. There were no
objects for her to steady herself on and she fell to the ground. She
sustained bruising to her arm, but no fracture.
She had a past medical history of glaucoma, hypertension, and
osteoarthritis. She was taking bendroflumethiazide 2.5mg OD and
timolol 0.5% eye drops, 2 drops BD to both eyes.
Cardiovascular, respiratory, abdominal, and neurological examination
in A&E were unremarkable. An ECG showed type 1 second degree
heartblock (Wenckebach). Electrolytes and inflammatory markers were
normal.
Which of the following is the most appropriate course of action?
A. Admit to a monitored bed
B. Arrange an outpatient 24hr tape (Holter monitor)
C. Arrange for district nurses to review at home
D. Refer to cardiologists for permanent pacemaker
E. Refer to ophthalmology for review
EXAM 1 | QUESTIONS 35

66. An 84-year-old man presented with sudden onset nausea and


impaired co-ordination in his left upper and lower limb. He also
reported that his right arm and leg did not ‘feel right’.
On examination, he had a partial ptosis on the left, with a meiotic
pupil on the same side. He has impaired left-sided co-ordination in his
arm and leg. There is reduced pain and temperature sensation in his
right-sided limbs, but left side of his face.
Which of the following is the most likely working diagnosis?
A. Functional disorder
B. Left-sided laterally medullary syndrome
C. Left-sided Weber syndrome
D. Right-sided lateral medullary syndrome
E. Right-sided Weber syndrome

67. During a strategic review of rehabilitation services for older people,


a new intermediate care service had been developed in a residential
care home. The local Clinical Commissioning Group asked the provider
service to ensure there was a robust mechanism for evaluating the
effectiveness of the service.
What would be the most appropriate criteria for referral to
admission time for intermediate care?
A. As soon as a bed is available
B. Between 48 and 72hr
C. Between 72 and 96hr
D. Less than 48hr.
E. Within 7 working days

68. Frailty is recognized as a biological marker of poor health


outcomes in older people.
Which characteristic of frailty best predicts poor outcome?
A. Balance
B. Cognition
C. Gait speed
D. Social support
E. Weight loss
36 EXAM 1 | QUESTIONS

69. An 83-year-old Italian woman, who spoke little English, was brought
to hospital by her daughters who said that she was confused, could
not mobilize or transfer from bed to chair, was slurring her speech
more than usual, and was unable to swallow. They said this change had
occurred overnight. She had suffered a right hemiparesis 18 months
earlier. She was usually able to transfer and mobilize unaided, and had
some dysarthria, but her speech was normally intelligible. Since her
stroke she had occasional coughing after eating, but never choked on
food and had a tendency to dribble saliva.
On examination she was drowsy, but rousable; her speech was
unintelligible and she was drooling from the right side of her mouth.
There was a generalized mild weakness of all limbs with hyporeflexia
and a right extensor plantar. Her BP and ECG were normal. A head CT
scan showed an old left middle cerebral artery territory infarct. FBC,
electrolytes, and renal function were all normal.
Her medication was aspirin 75mg daily, simvastatin 40mg daily, and
carbamazepine 100mg BD.
Over the next 3 days she made a recovery back to her previous level
of function, but on the planned day of discharge her speech was
unintelligible, she was unable to swallow, and had to be helped by two
nurses to get out of bed.
What is the most likely diagnosis?
A. Aspiration pneumonia
B. Cardiac arrhythmia
C. Post-stroke seizures
D. Recurrent pulmonary emboli
E. Transient ischaemic attacks

70. An 84-year-old woman was admitted with a lower respiratory tract


infection (LRTI). Old crush fractures of T10 and T11 were visible on
her chest radiograph. She recalled an episode a year previously when
she had terrible back pain for a few weeks. She did not seek medical
attention as she believed it was due to ‘old age’.
She had no other relevant past medical history and took no regular
medications. Examination confirmed a mild kyphoscoliosis. She had no
abnormal neurological findings.
What is the most appropriate intervention?
A. Arrange dual energy X-ray absorptiometry (DEXA) study
B. Arrange magnetic resonance imaging of the spine
C. Arrange whole body bone scintography
D. Initiate calcium + vitamin D supplementation
E. Initiate calcium + vitamin D and a bisphosphonate
EXAM 1 | QUESTIONS 37

71. A woman with a history of a stroke and recurrent falls asked for
advice regarding reducing her risk of falling.
Which of the following would be the least appropriate to
recommend?
A. A set of perching stools for use in the kitchen and bathroom
B. A walking stick sized by measuring from the wrist to the ground
C. Four wheel rollator frame with handgrips at wrist height when elbows bent 45º
D. Installation of grab rails in her kitchen and bathroom
E. Two wheel rollator frame with handgrips at wrist height when elbows bent 15º

72. A usually fit and well 79-year-old man was seen in a geriatric
pre-assessment clinic for consideration of an elective total hip
arthroplasty. His only past medical history was of osteoarthritis
and he took no prescribed medications.
The pre-assessment was unremarkable and ‘routine’ blood tests were
sent. Concern is raised by his coagulation results.
• Normal PT.
• Raised APTT.
• Mixing studies: fails to correct.
• Dilute Russell Viper Venom Test: Negative.
Which of the following is the most likely cause of these results?
A. Christmas disease
B. Haemophillia A
C. Lupus anticoagulant
D. Over-the counter aspirin
E. Phlebotomy error

73. A 94-year-old man had been treated for a community acquired


LRTI with IV antibiotics. He had spent the first 4 days following
admission in bed and had been responding well to the antibiotics.
However, the nursing staff noted that he had developed a sacral
pressure area. The affected area was 3 × 3cm and involved
necrosis through the epidermis and down to, but not including,
muscle.
Which of the following is not an appropriate part of the
management?
A. Completion of local incident form
B. Completion of wound care plan
C. Encourage to sit out on pressure cushion
D. Referral to dietician for review
E. Use of high-specification foam mattress
38 EXAM 1 | QUESTIONS

74. Which of the following cardiovascular changes is not associated


with ageing?
A. Decreased maximal heart rate attained during exercise
B. Decreased responsiveness to β-adrenergic receptor stimulation
C. Diastolic dysfunction
D. Increased postural heart rate variability
E. Impaired nitric oxide-mediated blood vessel relaxation

75. A 78-year-old man with Paget’s disease affecting the proximal


femur had ongoing pain on weight bearing. He was taking
paracetamol 1g QDS, codeine phosphate 60mg QDS, and
ibuprofen 400mg TDS.
Bone markers
• Alkaline phosphatase 210U/L (45–105)
• Urinary hydroxyproline is significantly raised.
Which of the following is the most appropriate course of action?
A. Caudal epidural
B. Conversion to strong opioid
C. IV bisphosphonate
D. Prostate specific antigen testing
E. Radiotherapy

76. Which of the following does not contribute to the relative increase
in circulating parathyroid hormone (PTH) levels seen in elderly
populations when compared with a young adult population?
A. Reduced 1-alpha hydroxylation of vitamin D in the kidney
B. Reduced dietary calcium and vitamin D absorption
C. Reduced renal 1-alpha-hydroxylase responsiveness to PTH
D. Reduced vitamin D synthesis in the skin
E. Relative reduction in serum magnesium levels
EXAM 1 | QUESTIONS 39

77. A 95-year-old woman was brought to hospital after falling at home. Her
73-year-old son, who was her main carer, called an ambulance as he was
unable to lift her. The paramedics thought she was too unsteady on her
feet to be left at home. No injury was sustained, but she appeared very
anxious when mobilizing with the physiotherapist in ED. She was very
deaf and, despite wearing a hearing aid, frequently misunderstood what
was being said to her.
This was one of many falls related to difficulties manoeuvring around
furniture. She was admitted to hospital and put on the waiting list for an
intermediate care bed. Three days later her son-in-law, a doctor living
200 miles away, visited and told the staff that he watched his mother-in-
law get up from her chair unaided and walk a distance equivalent to
what she needed to do at home. He also said she had told him that she
found the staff’s concern that she might fall undermined her confidence
to walk with them. She normally has twice daily carers.
What is the next most appropriate step in planning her discharge?
A. Check with her son whether he needs more help
B. Continue to wait for an intermediate care bed
C. Discharge her home the next day with her usual care
D. Discharge her home the next day with increased care
E. Review her mobility with the son-in-law present
40 EXAM 1 | QUESTIONS

78. A 78-year-old man was admitted with a 10-day history of decreasing


mobility, such that he was now bed-bound. During this period he had
become increasingly agitated with paranoid ideation and had been
hallucinating. Two weeks prior to admission he had developed new onset
urinary urge incontinence, for which he was started on tolterodine.
Two days prior to admission, when he became bed-bound, a urinary
catheter was inserted by the district nurse and he was commenced on
norfloxacin orally.
His wife reported that he was normally independent in activities of daily
living and denied that he had any cognitive impairment. There was no
history of falling or drug/alcohol abuse. He had been diagnosed as having
Parkinson’s disease 5 years earlier in another part of the country. His
wife stated that he had been taking Sinemet Plus® BD since diagnosis
with no obvious side-effects. There was no previous history of psychiatric
illness. Three weeks ago they had moved into the area to be nearer
their daughter and were still in the process of unpacking. Two days after
they moved in to the area, their daughter had gone abroad for 6 weeks.
Examination of the cardiac and respiratory systems was entirely normal.
Abdominal and rectal examination did not reveal any signs of constipation
or other abnormalities. There were no focal neurological abnormalities
nor were there any signs of an extra-pyramidal disorder. He was unable to
hold a sensible conversation, was deeply suspicious of all members of the
clinical team, and claimed that his wife was trying to poison him.
Investigations
Urea 9.0mmol/L (2.5–7.0)
Cr 110µmol/L (60–110)
Na 135mmol/L (137–144)
K 4.2mmol/L (3.5–4.9)
CRP 28mg/L (<10)
Hb 12.6g/dL (13–18)
White cell count (WCC) 8.0 × 10 9/L (4–11)
Neut 6.0 × 109/L (1.5–7.0)
Urine dip
Pus cells +++
Blood ++
Ketones +
Nitrites –
What is the most appropriate first step in his management?
A. Change the antibiotic
B. Increase the dose of Sinemet
C. Sedate him with IM haloperidol
D. Stop the tolterodine
E. Start thromboprophylaxis with enoxaparin
EXAM 1 | QUESTIONS 41

79. A 92-year-old was recovering in hospital from a LRTI after falling


at home. Prior to admission, she lived alone in a small cottage
with no central heating and an outside toilet. She had been
admitted three times previously that year. Her daughter, who had
been providing a large amount of informal care, stated she felt
her mother would be safer in a residential home
The patient acknowledged that she was at risk of further falls and that
not everyone would wish to live without heating. However, she stated
that she would rather go back home, despite these difficulties and risks.
Her MMSE was 20/30.
The social worker visited the patient in the presence of her daughter.
The daughter was angry with the patient’s decision, and stated that
she would no longer provide any support for her mother. The patient
became tearful and told the social worker that, although it wasn’t what
she would wish for, she had better move to a residential home.
Which of the following would be the most appropriate course to
follow?
A. Arrange a respite placement in a residential home
B. Involve an IMCA
C. Plan for discharge home with social services care
D. Plan for discharge to a residential home
E. Re-discuss with the patient without the daughter

80. An 86-year-old woman, who lived alone, suffered a long lie


after falling at home. She took no regular medication and was
registered blind due to age-related macular degeneration. She
was admitted to hospital for rehydration and rehabilitation.
The admitting doctor expressed some concern regarding her
cognition and intended to undertake formal cognitive screening.
What would be the most appropriate assessment tool to use in
this case?
A. Addenbrooke’s Cognitive Examination
B. Category Fluency Test
C. Clock Drawing Test
D. Mini Mental State Examination
E. Montreal Cognitive Assessment
42 EXAM 1 | QUESTIONS

81. An 86-year-old man noticed some left femur pain. An X-ray


showed a sclerotic patch in the proximal femur with coarse
trabeculae. The GP felt this represented Paget’s disease and
referred the patient for advice on his management. Blood tests,
including renal profile, LFTs and FBC were normal.
Which of the following is the most appropriate next step?
A. Arrange a CT scan of the femur
B. Arrange a femoral bone scintogram
C. Check a serum prostate specific antigen level
D. Check bone specific alkaline phosphatase levels
E. Check urinary hydroxyproline levels

82. An 82-year-old man with a history of ischaemic stroke, heart


failure, and diabetes presented with sigmoid volvulus. He had
had six similar admissions over the previous 3 months. One of
these settled spontaneously, the others required endoscopic
decompression. Routine blood tests (including electrolytes) were
normal.
Which of the following would be the most appropriate
ongoing management of his condition after initial endoscopic
decompression?
A. Dietary modification
B. Loperamide 1mg TDS
C. Percutaneous endoscopic colostomy
D. Sigmoid colectomy
E. Sigmoidopexy
EXAM 1 | QUESTIONS 43

83. A 76-year-old woman presented with a deteriorating leg ulcer. Her leg
was injured 2 months previously when it had caught on some furniture.
The district nurses had been dressing the leg regularly, but the wound
continued to deteriorate.
The patient had a past history of hypertension, type 2 diabetes, ulcerative
colitis, leg claudication, and asthma. She took bendroflumethiazide 2.5mg
OD, gliclazide 40mg OD and salbutamol PRN. She was an ex-smoker. Her
last HbA1c was at the upper end of the normal range.
On examination, a well demarcated ulcer was evident on the lateral
aspect of her lower leg. Pedal pulses were difficult to feel and the
foot appeared slightly dusky. An ankle brachial pressure index (ABPI)
measurement was normal at 1.0. The wound was not malodourous and
did not seem to be infected.
Which of the following is the next most appropriate step in the
management?
A. Activated charcoal dressings
B. Formal arterial ultrasound study
C. Four layer compression bandaging
D. Topical antibiotics
E. Vacuum dressing

84. A 76-year-old woman was referred to the clinic with a leg ulcer.
She had a past medical history of diabetes and was managed by
diet alone. She had noticed some brown skin discolouration over
the lower part of both legs over the last few years. More recently,
the anterolateral part of her left leg had started oozing and the
skin had now broken down.
On examination a 5 × 5cm venous ulcer was evident. There was no
erythema nor discharge and the edges were granulating. Pedal pulses
were palpable bilaterally, the capillary refill time was normal and the
ABPI was 0.8. There was no evidence of a peripheral neuropathy.
Which of the following is the most appropriate management
advice at this stage?
A. Compression bandaging
B. Silver dressing
C. Simple non-adherent dressing
D. Swab bacteriology
E. Vascular referral
44 EXAM 1 | QUESTIONS

85. A 72-year-old man with hypertension controlled with 2.5mg


bendroflumethiazide had a 6-month history of dyspnoea on
walking his dog and when gardening. His echocardiogram showed
mild concentric hypertrophy of the left ventricle and a left
ventricular ejection fraction (LVEF) of 45%.
Which of the following drugs would be most likely to improve his
life expectancy?
A. Aldosterone
B. Amiodarone
C. Digoxin
D. Furosemide
E. Ramipril
EXAM 1 | QUESTIONS 45

86. A 69-year-old woman complained of fatigue and poor appetite.


She was uncertain as to how long these symptoms had been
present. She had been widowed 5 months having spent the
preceding year looking after her husband with lung cancer. She
could not be sure if she had lost any weight and gave no other
symptoms. She was on no medication.
On examination she was clinically anaemic with pale mucous
membranes and complexion. The liver was enlarged and irregular.
Breast examination was normal. There was no lymphadenopathy.
Digital rectal examination was normal. As there was no history of
post-menopausal bleeding, a vaginal examination was not done.
Blood tests performed
Haemoglobin 5.9g/dL (11.5–16.5)
White cell count 6.0 × 109/L (4–11)
Neutrophil count 3.9 × 109/L (1.5–7.0)
Lymphocyte count 1.9 × 109/L (1.5–4.0)
Platelet count 389 × 109/L (150–400)
Mean cell haemoglobin concentration 26g/dL (32–35)
Mean cell volume 71fL (80–96)
Mean cell haemoglobin 24pg (28–32)
Serum sodium 139mmol/L (137–144)
Serum potassium 4.6mmol/L (3.5–4.9)
Serum urea 3.2mmol/L (2.5–7.0)
Serum creatinine 90µmol/L (60–110)
Serum corrected calcium 2.4mmol/L (2.2–2.6)
Serum total bilirubin 10µmol/L (1–22)
Serum aspartate aminotransferase 99U/L (45–105)
Serum alkaline phosphatase 19U/L (1–31)
Serum c-reactive protein 3mg/L (<10)

What is the next most appropriate step in her management?


A. Blood transfusion
B. CT scan abdomen and pelvis
C. Oesophagogastroduodenoscopy
D. Start iron replacement
E. Ultrasound-guided liver biopsy
46 EXAM 1 | QUESTIONS

87. An 86-year-old man presented with painless jaundice. He


had been diagnosed with cholangiocarcinoma and had
undergone palliative biliary stenting at endoscopic retrograde
cholangiopancreatography (ERCP). He had been started on
oramorph 5mg QDS owing to pain following the procedure. 3 days
post-stent insertion, his LFTs had dramatically improved and his
pain had settled. Onward plans were being considered to increase
his support in the community.
You were asked to review him as he is being troubled by increasingly
itchy skin.
Which of the following is the most appropriate course of action at
this stage?
A. Prescribe aqueous cream with 1% menthol PRN
B. Prescribe cetirizine 10mg OD
C. Prescribe chlorpheniramine 4mg TDS
D. Prescribe ursodeoxycholic acid 600mg TDS
E. Stop the regular oramorph

88. A 90-year-old female nursing home resident was referred to


hospital because of reduced food and fluid intake immediately
after having a coryzal illness. She had a severe vascular dementia,
but seemed to recognize her family and would eat finger
food. Her son explained that if his mother had simply become
dehydrated after a viral upper respiratory tract infection and
would benefit from rehydration he would wish this, but would
not wish her to be artificially fed. He wished her to return to the
nursing home as soon as possible and, if it appeared she was going
to die, this was his preferred place of care for her.
She was unrousable, apyrexial, and hypotensive (100/68), with no
abnormalities on further physical examination. Her FBC was normal,
serum sodium 184mmol/L (137–144), urea 15mmol/L (2.5–7) and
creatinine 250µmol/L (60–110).
What is the most appropriate management plan?
A. Artificial hydration and nutrition via NGT
B. IV rehydration
C. Place her on a palliative care pathway
D. Return immediately to the nursing home
E. IV antibiotics and rehydration
EXAM 1 | QUESTIONS 47

89. An 82-year-old woman was discharged on the 8th post-operative


day from the ortho-geriatric unit, with twice daily reablement
social care, after successfully recovering from repair of a left
fractured neck of femur. She was re-admitted 4 days later in
urinary retention.
What is the most likely cause of her urinary retention?
A. Cauda equina syndrome
B. Constipation
C. Resolving delirium
D. Urinary tract infection
E. Vaginal prolapse

90. A 75-year-old male was referred to the geriatric team for


consideration of transfer to the local intermediate care service
after having bowel resection and colostomy for Duke’s B colonic
carcinoma.
What patient characteristic is most likely to identify their inability
to benefit from rehabilitation?
A. A trial of rehabilitation
B. Depressive illness
C. Hypoactive delirium
D. Moderate dementia
E. Unwillingness to participate

91. A 78-year-old man with Parkinson’s disease of 6 years


duration was seen 2 days post-colon resection for Duke’s B
adenocarcinoma bowel. He was vomiting continuously and
complaining of abdominal discomfort. Abdominal CT showed
a massively dilated and fluid-filled stomach. He was put on to
nasogastric suction and IV fluids. His usual Parkinson’s medication
was Sinemet Plus® ×1 at 07.00, 11.00, 16.00, and 21.00 hours.
What is the most appropriate advice to give the surgical team
regarding his Parkinson’s medication?
A. Change Sinemet® to equivalent dosing of dispersible Madopar®
B. Crush Sinemet® and give via the NGT
C. Omit his Sinemet® until he can take oral medication again
D. Substitute a rotigotine patch for his Sinemet®
E. Substitute melt selegiline for his Sinemet®
48 EXAM 1 | QUESTIONS

92. The wife of an 82-year-old male with Lewy body dementia


enquired as to whether she could claim for attendance
allowance.
What is the most appropriate advice to give her regarding
attendance allowance?
A. Her application must be verified by a doctor
B. It does not cover caring for someone with dementia
C. It is tax-free and not means tested
D. It must be used to pay for additional care
E. The amount is reduced by any personal savings

93. A 70-year-old male with moderate dementia (MMSE 16/30) was


disruptive in the care home where he lived.
What is the most appropriate management strategy to try to
reduce his disruptive behaviour?
A. Ignore the behaviour
B. Prescribe an antipsychotic
C. Provide diversion therapy
D. Reward good behaviour
E. Reminiscence therapy

94. An 80-year-old Caucasian man saw his general practitioner


because he was worried about a 5 × 7mm brown and black lesion
on his forehead. Over the last few months it had started to
get darker in colour and to itch. The border was regular but it
was slightly raised. There was no lymphadenopathy and he was
otherwise well.
What would be the most appropriate action for his GP?
A. Biopsy the lesion
B. Reassure him and plan to review in 1 month
C. Reassure and advise to come back if worried
D. Refer for a routine dermatology opinion
E. Refer for urgent dermatology opinion
EXAM 1 | QUESTIONS 49

95. A 67-year-old man had severe deforming psoriatic arthropathy


affecting both hands. He used adaptive cutlery and elbow crutches
to walk. He also had osteoarthritis of both hips, which made rising
from a chair difficult.
Which of the following would be of greatest help to him for
toileting?
A. Bedside commode
B. Conveen®
C. Grab rails by the toilet
D. Raised toilet seat
E. Pads and pants

96. A 78-year-old bachelor wished to return to his home after


recovering from a right middle cerebral artery ischaemic stroke.
He was independent in transfers and walked with a stick. He
had residual mild (grade 4) weakness of his left limbs. He had a
tendency to overbalance when standing without his stick. He was
able to perform a wet shave independently, and could also get in
and out of the bath, independently, using a bath rail.
What aid is most likely to benefit him when dressing?
A. Dressing sleeve
B. Dressing stick
C. Leg lifter
D. Trouser clip
E. Zip pull
50 EXAM 1 | QUESTIONS

97. A previously independent 87-year-old man was brought to the


A&E department. He was unable to provide a history himself. His
daughter stated that his speech had become muddled during the
course of the day. He started becoming drowsy a few hours ago
and was noted to be feverish.
On examination he was pyrexial at 38.5˚C. His Glasgow Coma Score
(GCS) was 11 (E2, V4, M5). There was evidence of meningism with
neck stiffness and photophobia. He was dysphasic. Cardiovascular,
respiratory, and abdominal examinations were unremarkable. There
was no abnormal peripheral neurology.
He was started on ceftriaxone and benzylpenicillin for a presumptive
meningitis
His CT head scan is shown in Fig. 1.1.

Figure 1.1╇ Patient’s CT head scan.

Which of the following is the next most appropriate step in his


management?
A. Echocardiography
B. Level 2 care
C. Lumbar puncture
D. Meningococcal polymerase chain reaction (PCR) on blood sample
E. Neurosurgical review
EXAM 1 | QUESTIONS 51

98. Which of the following changes in respiratory function best explains


the decline in the partial pressure of arterial oxygen (arterial pO2) with
ageing?
A. Absence of change in total lung capacity
B. Decreased elastic recoil of the lungs
C. Decreased closing volume of the airways
D. Increased diffusion capacity of carbon monoxide
E. Reduced maximum oxygen consumption

99. A 72-year-old female with idiopathic Parkinson’s disease had


been treated with ropinirole 3mg three times a day over the last
4 years and now had no tremor and minimal extrapyramidal
rigidity in her left limbs. She was completely independent in all
domestic and daily activities of living.
Her husband reported that she had become more forgetful in the last
year. He also described her recent (4 months) pre-occupation with
cleaning and re-arranging the kitchen cabinet on a daily basis from
breakfast to late at night, she became very anxious if she was prevented
from cleaning. She no longer bothered going out shopping or for Sunday
lunch with friends—activities that she previously enjoyed. She stated
that she only cleaned as was necessary.
After a non-ST elevation myocardial infarction (NSTEMI) 8 months
earlier she was taking aspirin 75mg daily, clopidogrel 75mg daily,
ramipril 5mg daily, and lansoprazole 15 mg daily.
She scored 26/30 on MMSE and 4/15 on the Geriatric Depression Scale.
There was no evidence of postural hypotension.
What is the cause of her current interest in the kitchen cabinet?
A. Acute psychosis (hypomania)
B. Depression
C. Dementia
D. Dopamine dysregulation syndrome
E. Impulse control disorder
52 EXAM 1 | QUESTIONS

100. ╇A 69-year-old woman complained that her cheeks and all the
fingers of both hands became white when she was emotionally
stressed. This would last a few minutes or several hours, and
ended with the skin becoming flushed and tingly. She was being
treated with regular gold injections and a NSAID for deforming
rheumatoid arthritis. Examination revealed pitting scars at the
tips of several fingers.
What is the most appropriate treatment to recommend?
A. A beta-blocker
B. A calcium channel blocker
C. A prostaglandin
D. Biofeedback relaxation
E. Topical nitrates
exam

1 ANSWERS

1. C.╇ Acute psychosis should have responded, to some extent, to haloperidol. Akathisia or motor
restlessness can occur as a side effect of treatment with neuroleptics. It is an uncontrollable urge
to continuously move the legs, with a tendency to persist even after the offending drug is stopped.
Anxiety and agitation secondary to delirium usually respond to haloperidol. Dementia progression
is slow unless exacerbated by a vascular event, which was not apparent in this case. Symptoms of
restless leg syndrome usually occur at night or when the person is resting, causing them to get up
and walk around. New development of such symptoms should prompt a search for iron deficiency.
Akathisia is a frequent and common adverse effect of treatment with antipsychotic (neuroleptic)
drugs. This syndrome consists of subjective (feeling of inner restlessness and the urge to move),
as well as objective components (rocking while standing or sitting, lifting feet as if marching on
the spot, and crossing and uncrossing the legs while sitting). Antipsychotic-induced akathisia
can be classified according to time of onset in the course of antipsychotic treatment (acute,
tardive, withdrawal, and chronic akathisia). Reported prevalence rates vary widely between
5 and 36.8%. Numerous risk factors for acute akathisia have been described and the exact
pathophysiology of akathisia is still unknown. Since akathisia is a drug-induced adverse effect,
optimal management involves its prevention, rather than treatment. Standardized titration and the
use of novel antipsychotics are successful measures of prevention. Propranolol or other lipophilic
beta-blockers seem to be the most consistently effective treatment for acute akathisia. There is
nothing in the literature to guide a clinician when treatment with beta-blockers fails. Addition of
benzodiazepines would appear to be a sensible next choice, especially if subjective distress persists.
If all of these drugs are unsuccessful, amantadine or clonidine can be tried. Other agents that
have been investigated include ritanserin, piracetam, valproic acid (sodium valproate), and tricyclic
antidepressants. Evidence on the treatment of tardive akathisia is unsatisfactory.

Further reading
A.D.A.M. Medical Encyclopedia. Restless legs syndrome.
Available at: www.ncbi.nlm.nih.gov/pubmedhealth/PMH0001810/
Kane JM, Fleischhaker WW, Hansen L, et al. Akathisia: an updated review focusing on
second-generation antipsychotics, J Clin Psychiat 2009; 70(5): 627–43. Available at: http://www.
psychiatrist.com/abstracts/abstracts.asp?abstract=200905/050901.htm#

2. B.╇ She has asymptomatic bacteriuria (clinically significant bacteriuria without symptoms related
to the urinary tract), which does not require antibiotic treatment. This occurs in approximately
20% of females at this age. Antibiotic treatment risks: bacterial resistance, candidiasis, and
Clostridium difficile. Asymptomatic bacteriuria may be associated with urge incontinence, but not
stress incontinence (which she has had for several years anyway). Before referral to the continence
adviser, who might well advise pelvic floor exercises, an external pelvic floor examination needs to
be performed to exclude a cystocele or vaginal prolapse.
54 EXAM 1 | ANSWERS

Further reading
Cormican M, Murphy AW, Vellinga A. Interpreting asymptomatic bacteriuria. British Medical Journal
2011; 343: 364–7. Available at:  http://www.bmj.com/content/343/bmj.d4780?sso=
Dubeau CE, Kuchel GA, Johnson T, et al. Incontinence in the frail elderly.
Available at:  http://www.icsoffice.org/Publications/ICI_4/files-book/comite-11.pdf
European Association of Urology. EAU Guidelines. Available at:  www.uroweb.org/guidelines
European Association of Urology. Guidelines on urinary incontinence.
Available at:  http://www.uroweb.org/fileadmin/user_upload/Guidelines/16%20Urinary%20Incontinence.pdf
Grabe M, Bjerklund-Johansen TE, Botto H, et al. Guideline on urological infection. European
Association of Urology 2010.
NICE. The management of urinary incontinence in women, Clinical Guideline 40.
Available at:  http://www.nice.org.uk/nicemedia/live/10996/30281/30281.pdf
Scottish Intercollegiate Guidelines Network. Management of urinary incontinence in primary care.
Available at:  http://www.sign.ac.uk/pdf/sign79.pdf
Vasavada SP. Urinary incontinence.
Available at:  http://emedicine.medscape.com/article/452289-overview#showall

3. C.╇ Pressure ulcers generate considerable costs to the NHS, and a potentially unnecessary
burden of ill health and poor quality of life to the individual. Most, if not all pressure sores are
preventable. All patients should have an assessment within 6hr of admission to hospital and then
daily, if at risk, or, if not at risk, when their clinical situation changes. This woman was at risk of
developing pressure sores because of her reduced mobility and should have had a risk assessment
as per NICE guidelines within 6hr of her admission and again when her clinical condition changed.

Further reading
NICE. Pressure ulcer prevention: pressure ulcer risk assessment and prevention including the use
of pressure-relieving device (beds, mattresses and overlays) for the prevention of pressure ulcers in
primary and secondary care, Clinical Guideline 7, 2003. Available at: http://www.nice.org.uk/nicemedia/
live/10928/29180/29180.pdf
Royal College of Nursing. Pressure ulcer risk assessment and prevention: recommendations 2001.
Available at:  http://www.rcn.org.uk/__data/assets/pdf_file/0003/78501/001252.pdf

4. B.╇ The clinical features are convincing for angina, however, she has a lack of risk factors other
than age. This leaves her with a moderate (about 60–70%) chance of having significant coronary
artery disease. NICE guidelines (2010) suggest that she should be offered non-invasive functional
imaging. Exercise testing is no longer recommended to diagnose or exclude stable angina in people
without known coronary artery disease. CT angiogram would, therefore, be the next most
appropriate step here. Alternatives would include stress-echo or myocardial perfusion scans.
Medical treatment without further testing may be appropriate if the patient is too frail for non-
invasive tests, or too frail to proceed to angiography if the results were positive.

Further reading
NICE. Chest pain of recent onset, Guidance CG95. Available at:  http://www.nice.org.uk/CG95

5. E.╇ The hypercalcaemia could be due to many causes. Checking a parathyroid hormone (PTH)
level will help differentiate between excessive calcium (be it due to dietary excess, a destructive
bone lesion or excessive vitamin D) and inappropriate resorption from bone. Physiologically, the
EXAM 1 | ANSWERS 55

PTH should be suppressed in the presence of hypercalcaemia. If it is raised or, indeed, normal, this
is inappropriate and would confirm a diagnosis of primary hyperparathyroidism.
The initial management should be to rehydrate the patient and stop exogenous calcium
supplements. Pamidronate should be reserved for when such first line treatment is not effective in
correcting the serum calcium.
If the PTH level is suppressed, and cessation of the exogenous calcium does not remedy the
problem, then other causes of hypercalcaemia should be considered. These include lytic bone
lesions, either associated with her previous breast cancer, or as yet unidentified malignancy. Bone
scintography and/or further CT imaging may prove useful, and choice of investigation would
depend further on clinical history and examination.

Further reading
Green TE. Hypercalcaemia in emergency medicine. Medscape.
Available at:  http://emedicine.medscape.com/article/766373-overview

6. E.╇ This man developed venous sinus thrombosis as a result of hyper-viscosity due to
reactivation of polycythaemia rubra vera (PRV) and dehydration secondary to fasting. Intracranial
tumours, especially a bleed into a para-sulcal meningioma, can present in a similar manner—the clue
to the diagnosis lies in the history especially development of symptoms in relation to dehydration.
Meningitis is excluded by normal inflammatory markers and no signs of meningeal irritation, the
latter also helps exclude sub-arachnoid haemorrhage. A sub-dural haematoma large enough to
cause bilateral papilloedema is likely to have more pronounced neurological deficit than slight leg
weakness. His leg weakness was a result of focal haemorrhage due to raised venous pressure.

Further reading
Family Practice Notebook. Polycythaemia rubra vera.
Available at:  htttp://www.fpnotebook.com/hemeonc/Marrow/PlycythmRbrVr.htm
Medscape. Venous sinus thrombosis.
Available at:  http://emedicine.medscape.com/article/1162804-overview#

7. C.╇ The 2007–9 data predict a life expectancy of a further 20.2 years for women and 17.6 years
for men aged 65 (i.e. 85.2 years for women and 82.6 years for men).
The life expectancy for children born in 2007–9 is 77.9 for men and 82 years for women. For those
who are 65 it is 82.8 and 85.4 years. The cohort of 65-year-olds have survived childhood and
‘middle-age’ illness, which impact on overall life expectancy.
The female:male ratio is projected to fall from 156 women/100 men in 1984 to 118 women/100
men in 2034.
The median age in the UK was 35 in 1984, 39 in 2009, and is projected to be 42 in 2034.

Further reading
Office for National Statistics. Available at:  http://www.ons.gov.uk/ons/rel/mortality-ageing/focus-on-
older-people/older-people-s-day-2010/focus-on-older-people.pdf

8. D.╇ By definition, dementias are slowly progressive and do not suddenly occur. So whilst people
with dementia with Lewy bodies may manifest marked fluctuations in both physical and cognitive
function this is not a dementia. The implication is that her husband has been with her all along and
so would have seen seizure activity. This is a classical story for transient global amnesia. There is no
suggestion of any transient neurological deficit so this is not a TIA.
56 EXAM 1 | ANSWERS

Further reading
Mayo Clinic. Transient global amnesia. Available at:  http://www.mayoclinic.com/health/
transient-global-amnesia/DS01022
Medscape. Transient global amnesia. Available at:  http://emedicine.medscape.com/
article/1160964-overview#

9. C.╇ She has partially reversible (stage 2 moderate) COPD. NICE guidelines recommend a regular
inhaled beta-agonist or muscarinic antagonist as first line treatment. She has already stopped
smoking. Initial management would be to recommend as-required short-acting beta-agonist inhaler,
but you are not given this option. Option E would be added if the patient remained symptomatic
despite regular inhaled bronchodilator. Option A would be for acute exacerbations or, rarely,
for severe COPD. Theophylline should only be used after a trial of short-acting bronchodilators
and long-acting bronchodilators, or in patients who are unable to use inhaled therapy. Pulmonary
rehabilitation is a multidisciplinary approach to managing COPD aimed at improving concordance
and should be offered to all those who consider themselves functionally impaired by their COPD.
Pulmonary rehabilitation alone would be inappropriate without some bronchodilator therapy;
hence, C is the correct answer.

Further reading
NICE. Chronic obstructive pulmonary disease. Clinical Guideline CG101 (updated 2010). Available at: 
http://publications.nice.org.uk/chronic-obstructive-pulmonary-disease-cg101

10. C.╇ The first line approach should be to reverse any factors that may be driving the acute
deterioration and delirium. This would necessitate a full examination and review of the patient,
rather than sedation as a first line intervention. In this instance, it is likely that the pressure sore is
painful, but that the patient is unable to communicate this verbally. A non-verbal pain assessment
tool (e.g. Doloplus-2) may be useful.
Analgesia may be sufficient to control his symptoms and thus terminate the acute delirium.
Hypnotics and antipsychotics should be reserved for when other conservative measures fail to help
and/or the agitation becomes such that it puts the patient or others at risk.

Further reading
Lefebvre-Chapiro S: The DOLOPLUS 2 scale—evaluating pain in the elderly. European
Journal of Palliative Care 2001; 8: 191–4. Available at:  http://www.haywardpublishing.
co.uk/journal_search_results.aspx?JournalID=4&sw=0&yrFrom=0&yrTo=0&sa=Lefeb
vre-Chapiro&ef=False&notw=0&alw=True
NICE. Delirium, Clinical guideline CG103. Available at: http://publications.nice.org.uk/delirium-cg103
Woods B, Moniz-Cook E. Dementia: pain relief—a first line response to agitation in dementia? Nature
Reviews Neurology 2012; 8: 7–8. Available at: http://www.nature.com/nrneurol/journal/v8/n1/full/
nrneurol.2011.192.html

11. A.╇ It is common for there to be a slight increase in creatinine following initiation of an
angiotensin-converting enzyme inhibitor (ACEI). NICE recommend that if there is a fall in eGFR (of
25% or less) or a rise in creatinine (of 30% or less) that the renal profile be reviewed in a week. If
the change is greater, or if hyperkalaemia develops, the ACE inhibitor should be withdrawn.
Losartan would have a comparable effect to ramipril on the renal profile.
EXAM 1 | ANSWERS 57

Further reading
NICE. Chronic kidney disease, Clinical guideline CG73. Available at: http://publications.nice.org.uk/
chronic-kidney-disease-cg73

12. A.╇ A multidisciplinary approach is important to managing spasticity. Physiotherapy, the use
of casts and splints, surgical release of contractures, and medication may all play a part. Baclofen
exerts its effects pre- and post-synaptically by inhibiting spinal reflexes. Intrathecal baclofen has been
shown to improve walking speed, functional mobility ratings, and spasticity; and may be beneficial in
individuals who are unable to tolerate oral anti-spasticity drugs.
Dantrolene acts on peripheral muscles by inhibiting release of calcium from the sarcoplasmic
reticulum during muscle contraction. Diazepam has muscle relaxant, anxiolytic, hypnotic,
and anticonvulsant actions. It acts by enhancing GABAA mediated inhibition in the central
nervous system. Tizanidine acts centrally pre- and post-synaptically as an agonist of the alpha-2
adrenergic receptor. Botulinum toxin works by binding to presynaptic acetylcholine vesicles
at the neuromuscular junction of skeletal muscle, where it prevents release of the excitatory
neurotransmitter acetylcholine.

Further reading
Remy-Neris O, Triffreau V, Bouilland S, Bussel B. Intrathecal baclofen in subjects with spastic
hemiplegia: assessment of the antispastic effect during gait. Archives of Physical Medicine and
Rehabilitation 2003; 84: 643–50. Available at: http://www.ncbi.nlm.nih.gov/pubmed/12736875

13. E.╇ Her CHA2DS2-VASc score is 4 and so formal anticoagulation is recommended not aspirin.
Aspirin is only recommended if the CHA2DS2-VASc score is 0–1. Dabigatran is recommended as
an alternative to warfarin in high risk patients or patients who cannot tolerate warfarin. 150mg BD
has a superior stroke prevention risk to warfarin, but an increased gastrointestinal (GI) bleeding
risk. 110mg BD has not been proven to be worse than warfarin at reducing stroke risk and has a
reduced GI bleeding risk. Dabigatran is contraindicated in severe renal impairment with creatinine
clearance <30mL/min and is, therefore, not suitable in this case.
As this patient has good rate control and is asymptomatic there is no reason to change to dronedarone.
She has had atrial fibrillation for many years and so is unlikely to attain rhythm control by drugs or
cardioversion; however, catheter ablation is not appropriate unless she remains symptomatic.

Further reading
Camm AJ, Kirchof P, Lip GY, et al. Guidelines for the management of atrial fibrillation: the task force
for the management of atrial fibrillation of the European Society of Cardiology (ESC). European Heart
Journal 2010; 31: 2369–429. Available at: http://www.escardio.org/guidelines-surveys/esc-guidelines/
GuidelinesDocuments/guidelines-afib-FT.pdf
NICE. Atrial fibrillation—dabigatran etexilate: guidance, Guideline TA249.
Available at: http://guidance.nice.org.uk/TA249/Guidance/pdf/English
NICE. Atrial fibrillation (stroke prevention)—rivaroxaban: guidance, Guideline TA256.
Available at: http://guidance.nice.org.uk/TA256/Guidance/pdf/English

14. B.╇ NICE advocate beta-blockers or calcium channel blockers as first line long-acting anti-
anginals. Isosorbide mononitrate, nicorandil and ivabradine are considered second line. Bisoprolol
2.5mg OD is a more appropriate starting dose than atenolol 100mg OD

Further reading
NICE. Stable angina, Clinical guideline CG126. Available at: http://guidance.nice.org.uk/CG126
58 EXAM 1 | ANSWERS

15. C.╇ If his behaviour can be understood it may be modifiable or accommodated. It transpired
he was a regular after dinner speaker and by allowing him to stand and say something at meal times
his behaviour modified. Prescribing drugs for dementia sufferers without just cause is bad practice.
Many people with dementia are prescribed antipsychotics, although they are not always necessary
and often prescribed without review. This results in approximately four people suffering adverse
drug events per year and five people dying.

Further reading
All Party Parliamentary Group on Dementia. Always a last resort: inquiry into the prescription of
antipsychotic drugs to people with dementia living in care homes. London: HM Government, 2008.
Available at: http://alzheimers.org.uk/site/scripts/download.php?fileID=322
Dementia action alliance. Antipsychotics. Available at: http://www.dementiaaction.org.uk/news/
article/6/the_right_prescription_a_call_to_action_on_the_use_of_antipsychotic_drugs_for_people_
with_dementia
Lawler B. Behaviour and psychological symptoms of dementia. British Journal of Psychiatry 2002; 181:
463–5. Available at: http://bjp.rcpsych.org/content/181/6/463.full
Warner J. Behavioural and psychological symptoms of dementia (BPSD).
Available at: http://www.ucl.ac.uk/dementia/edtraining/bpsd.ppt#289,1

16. A.╇ Many older people take vitamin supplements in the belief that these are good for health,
even when they eat a healthy diet. It is always important to ask about over the counter medication
and specifically to enquire about vitamin and food supplements, which patients may not otherwise
mention, as they are not considered to be medications.
Hypervitaminosis A would explain her symptoms and is also associated with hepatic dysfunction
and leukopenia. Hypervitaminosis D is associated with hypercalcaemia, polyuria and metastatic
calcification. Hypervitaminosis B3 (niacin) is associated with flushing, burning sensation of the skin,
nausea and vomiting, hepatotoxicity, and dizziness. Hypervitaminosis C may be associated with
diarrhoea and an increase risk of developing oxalate renal stones. Hypervitaminosis B6 may cause a
sensory neuropathy. There are no known toxic effects of vitamin E, although there is a theoretical
increased risk of haemorrhage.

Further reading
Linus Pauling Institute, Micronutrient Information Centre (LPIMIC). Vitamin A. Available at: http://lpi.
oregonstate.edu/infocenter/vitamins/vitaminA/
LPIMIC. Vitamin B3. Available at: http://lpi.oregonstate.edu/infocenter/vitamins/niacin/
LPIMIC. Vitamin B6. Available at: http://lpi.oregonstate.edu/infocenter/vitamins/vitaminB6/
LPIMIC. Vitamin C. Available at: http://lpi.oregonstate.edu/infocenter/vitamins/vitaminC/
LPIMIC. Vitamin E. Available at: http://lpi.oregonstate.edu/infocenter/vitamins/vitaminE/

17. C.╇ The most likely cause of incontinence here is difficulty getting to the toilet in time. The
information provided does not support bladder outflow obstruction or urge incontinence,
but further history is required to be certain. In any event, alpha-blockade would likely have a
deleterious effect on the postural hypotension. Anticholinergics would likely exacerbate the mild
cognitive impairment and the evidence for stress incontinence would need to be stronger before
trialling them.
Containment methods, although less preferable to treating the cause, would reduce the need to
mobilize to the bathroom, thereby reducing both episodes of incontinence and his risk of falling.
EXAM 1 | ANSWERS 59

The simplest intervention would be to provide bedside bottles for him to use. A conveen, if he is
able to tolerate it, would provide better containment than pad and pants, and is less likely to result
in skin irritation.
It should be possible to support this patient at home with the previously mentioned interventions.

Further reading
NICE. Urinary incontinence, Clinical guideline CG40.
Available at: http://www.nice.org.uk/guidance/CG40
NICE. The management of lower urinary tract symptoms in men, Clinical guideline CG97.
Available at: http://guidance.nice.org.uk/CG97

18. D.╇ There are a variety of technologies that can help with medication concordance, e.g. pill
dispensers with alarm functions, SMS message reminders. The limited evidence base for the
effectiveness of telehealth and telemedicine comes from the management of specific conditions
(e.g. diabetes and heart failure), rather than individuals with multiple comorbidities. Water sensors
can be used to transmit information to central hubs to warn of bathroom floods, which may be
useful, for example, in maintaining independent living for people with dementia. These technologies
may improve efficiency within care homes and can help reduce fall risks. Most care is hands on and
so, whilst telehealth and telecare may reduce the routine input from carers, technology will not fully
replace the carers!

Further reading
Kerr B, Hurst K, Clark S. Telehealthcare and mental health: using telehealthcare effectively in the
support of people living with mental disorder. London: Kings Fund, 2011. Available at: http://kingsfund.
koha-ptfs.eu/cgi-bin/koha/opac-detail.pl?biblionumber=103952
Kings Fund. The impact of telehealth: a review of the evidence. London: Kings Fund. Available
at: http://www.kingsfund.org.uk/topics/technology_and_telecare/telehealth_evidence.html
Martin S, Kelly G, Kernohan WG, McCreight B, Nugent C. Smart home technologies for health and
social care support. Cochrane Database of Systematic Reviews. 2005; Issue 4: CD006412.
Available at: http://www.ncbi.nlm.nih.gov/pubmed/18843715
Poole T. Telecare in older people. London: Kings Fund, 2006.
Available at: http://kingsfund.koha-ptfs.eu/cgi-bin/koha/opac-detail.pl?biblionumber=37212

19. B.╇ There are no clinical features to suggest he is under-replaced with either mineral or
corticosteroids. Similarly there are no features to suggest an Addisonian crisis or severe sepsis.
Increasing the steroid doses is therefore not warranted. Indeed, it may be the case that the
hydrocortisone dosing is too high.
There are no features to suggest syncope and examination and ECG are unremarkable. Cardiac
Holter monitoring is not warranted.
Addison’s disease is associated with osteoporosis and given the multiple falls, a bone density scan is
warranted and consideration made to anti-resorptive therapy depending on the result.

Further reading
Griffing GT. Addison disease. Medscape 2012.
Available at: http://emedicine.medscape.com/article/116467-overview
NICE. Osteoporosis fragility fracture risk, Clinical guideline CG146.
Available at: http://www.nice.org.uk/nicemedia/live/13857/60400/60400.pdf
60 EXAM 1 | ANSWERS

20. D.╇ His aggression seems to relate to movement and would indicate that his pain control is
inadequate. He is not demonstrating obvious symptoms of opioid toxicity: nausea and vomiting,
dizziness, drowsiness, sweating, or hallucinations; although he may be constipated and he is more
disturbed in behaviour than usual. As his only medication is given by transdermal patch and he is
resident in a specialist dementia care home, these are clues that he is unlikely to be able to request
PRN medication and cannot be relied upon to take oral medication. Bone prophylaxis with a
bisphosphonate should be considered and as he is unlikely to be compliant with oral treatment,
strontium would not be suitable either. Intravenous ibandronate might be considered for longer-
term bone protection.

Further reading
AGS Panel on Persistent Pain in Older Persons. The management of persistent pain in older persons.
Journal of the American Geriatric Society 2002; 50: S205–24. Available at: http://www.geriatricpain.org/
Content/Resources/CPGuidelines/Documents/AGS%20Panel%20on%20Persistent%20Pain%20in%20
Older%20Persons.pdf
American Geriatrics Society Panel on the pharmacological management of persistent pain in older
persons. Pharmacological management of persistent pain in older persons. Journal of the American
Geriatric Society 2009; 57: 1331–46. Available at: http://www.geriatricpain.org/Content/Resources/
CPGuidelines/Documents/AGS%20Pharmacological%20Management%20of%20Persistent%20
Pain%20in%20Olders%20Persons%20(2009)%20(2).pdf

21. D.╇ She has been neglecting herself to the point of dehydration and malnutrition, has rendered
her bedsit unfit for human habitation and is deemed to lack the mental capacity to care for herself
or determine her care needs. She is not a danger to others and her primary psychiatric diagnosis
of personality disorder is untreatable. She needs to be in a place of safety and given that she has
not tried to leave hospital is likely to settle wherever she is placed. However, as she will not discuss
care home placement she would be deprived of her liberty and a DoLS procedure would be most
appropriate as this would include appointing an IMCA.

Further reading
Deprivation of Liberty Safeguards: Available at: http://www.dh.gov.uk/prod_consum_dh/groups/dh_
digitalassets/@dh/@en/documents/digitalasset/dh_091867.pdf; http://www.dh.gov.uk/prod_consum_dh/
groups/dh_digitalassets/@dh/@en/documents/digitalasset/dh_087309.pdf
Mental Capacity Act 2005 (England). Available at: http://webarchive.nationalarchives.gov.uk/+/http://
www.dca.gov.uk/legal-policy/mental-capacity/mca-cp.pdf

22. C.╇ The patient gives a clear history of vertigo precipitated by head movements. This would
be consistent with a diagnosis of benign paroxysmal positional vertigo. It is likely that the mild head
trauma damaged his utricle and dislodged some otoliths into his semicircular canals. A Hallpike
manoeuvre is simple to undertake in clinic and may prove diagnostic. An Epley manoeuvre may then
prove curative.
The lack of syncope, normal cardiac examination and unremarkable ECG make an arrhythmia
unlikely. Also, the patient is describing vertigo, rather than pre-syncope.
Head injury may also result in cervical spine injury, and while this could impair the proprioceptive
feedback from the cervical spine and cause ‘dizziness’ it would be atypical in the absence of pain.
There is no evidence of focal neurological deficit and no loss of consciousness, making the diagnosis
of subdural haemorrhage unlikely despite the head trauma. It would be preferable to pursue the
simpler bedside Hallpike test first.
EXAM 1 | ANSWERS 61

Menière’s disease involves the triad of vertigo, hearing impairment and tinnitus. There is no
suggestion of a hearing impairment or tinnitus in the history and the acute onset in this case is out
of keeping with the more gradually progressive cause in Menières.

Further reading
Fife TD. Benign paroxysmal positional vertigo. Seminars in Neurology 2009; 29(5): 500–8.
Available at: https://www.thieme-connect.de/DOI/DOI?10.1055/s-0029-1241041

23. C.╇ The abdominal discomfort, diarrhoea, and symptoms consistent with anaemia are
concerning for a sub-acute upper GI bleed. The modest postural drop in BP and tachycardia imply a
degree of hypovolaemia. It is likely that relative hypoperfusion of his coronary arteries and anaemia
are behind the exacerbation of his angina. Treatment should be to restore his circulating volume
and correct the anaemia. Fluid resuscitation and confirmation of his blood count are therefore the
most appropriate next steps in his management.
GTN infusion, antiplatelets, and heparin could exacerbate any bleeding or hypotension, and worsen
coronary artery oxygen supply.

24. C.╇ The Coroners and Justice Act, 2009 states that it is an offence to assist or encourage the
suicide or attempted suicide of another in individual; the act covers England, Wales and Northern
Ireland. Therefore both options A and B are potentially illegal acts and place the discharging team at
risk of prosecution, whether or not the individual goes ahead with assisted suicide.
From the information given he would seem to be mentally competent and neither requires referral
for more formal psychiatric assessment nor the appointment of an Independent Mental Capacity
Advocate.

Further reading
Coroners and Justice Act 2009. Available at: http://www.legislation.gov.uk/ukpga/2009/25/contents

25. D.╇ The fundamental age-related change in arterial function is impairment of distensibility
and, thus, of the cushioning function of the aorta and its major branches, associated with an
enhancement in pulse wave velocity. In the resting ageing heart, there are largely no alterations of
systolic function, with preserved ejection fraction and stroke volume; because resting heart rate is
unchanged or only minimally reduced with ageing, cardiac output is also preserved.

Further reading
Craft N, Schwartz JB. Effects of age on intrinsic heart rate, heart rate variability, and AV conduction in
healthy humans. American Journal of Physiology—Heart 1995; 268(4): H1441–52.
Available at: http://ajpheart.physiology.org/content/268/4/H1441.short
Ferrari AU, Radaelli A, Centola M. Invited review: aging and the cardiovascular system. Journal of Applied
Physioliology 2003; 95(6): 2591–7. Available at: http://www.jappl.org/content/95/6/2591.full

26. B.╇ Although there is a potentially significant left internal carotid territory lesion, the left arm
and leg symptoms would correlate with a right cerebral (hence right internal carotid territory)
lesion. Endarterectomy is only indicated in cases of symptomatic disease and would not be
indicated here. In cases of borderline significant stenosis, which might be considered for surgery,
imaging in two domains may be useful, but variations in modality would not be large enough to alter
the indications in this case.
62 EXAM 1 | ANSWERS

Clopidogrel currently does not have a licence for TIA. It is recommended by NICE as the first
line agent for ischaemic stroke, but they advise aspirin + dipyridamole M/R for patients who have
had a TIA.
The patient has only just started on simvastatin, which is recommended as first-line by NICE. If he
fails to reach target (<4mmol/L) then an increase in dose and/or alternative agent(s) should be
considered. They should therefore be referred back to their GP for ongoing cardiovascular risk
factor optimization and review.

Further reading
NICE. Clopidogrel and M/R dipyridamole for the prevention of occlusive vascular events, Guideline
TA210. Available at: http://guidance.nice.org.uk/TA210
NICE. Lipid modification, Clinical guideline CG67. Available at: http://guidance.nice.org.uk/CG67
NICE. Stroke, Clinical guideline CG68. Available at: http://www.nice.org.uk/CG68

27. A.╇ Current NICE guidelines suggest that patients diagnosed with cerebral venous sinus
thrombosis (including those with secondary cerebral haemorrhage) should be given full dose
anticoagulation treatment (initially with heparin and then warfarin) unless precluded by other
co-morbidities.

Further reading
NICE. Stroke, Clinical guideline CG68. Available at: http://www.nice.org.uk/CG68
McElveen WA. Cerebral venous thrombosis.
Available at: http://emedicine.medscape.com/article/1162804-overview

28. D.╇ Updated NICE guidance recommends offering drug treatment to people of any age with
stage 2 hypertension (average home ambulatory BP >150/95 or clinic BP >160/100) irrespective
of the presence of target organ damage, cardiovascular disease, renal disease, or 10-year risk of
cardiovascular disease.

Further reading
British National Formulary. Available at: http://bnf.org/bnf/bnf/61/204016.htm
NICE. CG127: Hypertension clinical management of primary hypertension in adults, Clinical guideline
CG127 (updated). Available at: http://guidance.nice.org.uk/CG127

29. A.╇ She had visual impairment secondary to bilateral cataracts and apart from visual
hallucinations she did not have significant cognitive impairment affecting her daily activities.
Dementia with Lewy bodies is characterized by early impairments in attention, executive and
visuospatial function and hallucinations with memory affected later in the course of the disease.
The patient did not show any signs of paranoia and continued to participate in group activities.
Although she had signs of vascular Parkinsonism characterized by ‘lower body Parkinsonism’ she did
not have dementia. Apart from the Parkinsonism there were no other features (autonomic, urinary,
cerebellar or corticospinal dysfunction) to support a diagnosis of multi-system atrophy.

Further reading
Jacob A, Prasad S, Boggild M, Chandratre S. Charles Bonnet syndrome—elderly people and visual
hallucinations. British Medical Journal 2004; 328: 1552.
Available at: http://www.bmj.com/content/328/7455/1552.full
EXAM 1 | ANSWERS 63

Litvan I, Bhatia KP, Burn DJ, et al. SIC Task Force appraisal of clinical diagnostic criteria for Parkinsonian
disorders, Movement Disorders Society Scientific Issues Committee report. Movement Disorders 2003;
18(5): 467–86. Available at: http://www.movementdisorders.org/UserFiles/SICreport.pdf
Manford M, Andermann F. Complex visual hallucinations: clinical and neurobiological insights. Brain 1998;
121: 1819–40. Available at: http://brain.oxfordjournals.org/content/121/10/1819.full.pdf
McKeith I. Consensus guidelines for the clinical and pathologic diagnosis of dementia with Lewy bodies
(DLB): report of the Consortium on DLB International Workshop. Journal of Alzheimer’s Disease 2006;
9(Suppl. 3). Available at: http://www.ncbi.nlm.nih.gov/pubmed/16914880
RNIB. Charles Bonnet Syndrome. Available at: http://www.rnib.org.uk/eyehealth/eyeconditions/
conditionsac/Pages/charles_bonnet.aspx

30. D.╇ There is evidence multifactorial interventions reduce falls and risk of falling in hospitals and
may do so in nursing care facilities, but no recommendations can be made regarding any particular
component of these programmes. Vitamin D supplementation was effective in reducing the rate of
falls in nursing care facilities, as was a review of medication by a pharmacist. There was no evidence
other interventions targeting single risk factors reduced falls and this included exercise interventions.
However, exercise in the subacute hospital setting appeared effective.
However, the evidence base is limited due to the small number of hospital studies, and the difficulty
isolating effects of individual components of treatments that involved multiple components, and the
variability of interventions. Bed rails have been shown to increase falls risk in confused older people.

Further reading
Cameron ID, Murray GR, Gillespie LD, et al. Cochrane summary: interventions for preventing falls in older
people in nursing care facilities and hospitals. 2013. Available at: http://summaries.cochrane.org/CD005465/
interventions-for-preventing-falls-in-older-people-in-nursing-care-facilities-and-hospitals

31. D.╇ The patient has expressed her wishes regarding care at discharge, which seems appropriate
to her needs and concerns regarding the early readmission. With a Mini Mental State Examination
(MMSE) of 29/30 it is likely that she is competent to make these decisions and so her discharge
should be re-instigated. There is nothing in this case to suggest that the daughter is going to be
involved in her mother’s care and so her presence is not necessary for a safe discharge. As a
competent adult the patient is the person with whom discharge planning should be arranged. Whilst
the daughter may have similar anxieties to her mother, regarding the early readmission, she has no
legal right to interfere with her mother’s care and so, whilst her concerns need to be acknowledged
they need not influence the care planning with her mother. The patient is independent on the ward
and so does not need transfer to a rehabilitation unit. The daughter should not have cancelled
her mother’s care, but a SOVA is not required as this would be more appropriate if the mother
required safe-guarding under the Mental Capacity Act 2005.

Further reading
Department of Health. Ready to go, Department of Health guidance on discharge planning from
hospital and IC. Available at: http://www.dh.gov.uk/prod_consum_dh/groups/dh_digitalassets/@dh/@
en/@ps/documents/digitalasset/dh_116675.pdf
‘Ready to go’ – Department of Health guidance on discharge planning from hospital and intermediate
care. Available at: http://www.dh.gov.uk/prod_consum_dh/groups/dh_digitalassets/@dh/@en/@ps/
documents/digitalasset/dh_116675.pdf
HM Government. Mental Capacity Act 2005 (England). Available at: http://webarchive.nationalarchives.
gov.uk/+/http://www.dca.gov.uk/legal-policy/mental-capacity/mca-cp.pdf
64 EXAM 1 | ANSWERS

32. D.╇ NICE advocate replacement arthroplasty in patients with a displaced intracapsular fracture
owing to the risk of avascular necrosis. If an individual is medically fit, previously independent and
not cognitively impaired, then a total hip replacement is preferred. Otherwise a hemiarthroplasty is
indicated.
Extramedullary implants, such as a dynamic hip screw should be used in preference to an
intramedullary nail in patients with trochanteric fractures. Intramedullary nailing is more appropriate
for subtronchanteric fracture and would be preferable in this case.
Conservative management is only rarely indicated as even in the presence of severe co-morbidities,
the prognosis is usually worse with prolonged bed rest and the surgical intervention itself provides
pain relief and can be considered, in part, a palliative intervention.

Further reading
NICE. CG124. Hip fracture, Clinical guideline CG124.
Available at: http://www.nice.org.uk/guidance/CG124

33. A.╇ There is a hierarchial relationship in the decline and recovery of physical function in older
people. The ability to bathe, dress, toilet, transfer and feed oneself are lost sequentially, regardless
of the cause; and are regained in reverse order.

Further reading
Spector WD, Katz S, Murphy JB, et al. The hierarchical relationship between activities of daily living and
instrumental activities of daily living. Journal of Chronic Disease 1987; 40: 481–9.
Available at: http://www.ncbi.nlm.nih.gov/pubmed/3597653

34. A.╇ Unilateral facial weakness without upper sparing is most likely due to a Bell’s palsy. If the
botulinum toxin injections were responsible, onset would have been sooner.
Central causes, such as lateral medullary syndrome, stroke, or venous sinus thrombosis would all
result in frontal sparing.

Further reading
NICE. Stroke, Clinical guideline CG68. Available at: http://guidance.nice.org.uk/CG68
Taylor DC, Keegan BM. Bell Palsy. Medscape 2012.
Available at: http://emedicine.medscape.com/article/1146903-overview

35. E.╇ The perifollicular haemorrhage with corkscrew hairs is consistent with scurvy. This is also
responsible for the haemorrhage in his gums and likely to be contributing to the non-specific
symptoms.
Vitamin C supplementation should be commenced and other nutritional deficiencies looked for and
corrected. Intervention should occur to ensure a ‘balanced diet’ in the future.

Further reading
Goebel L, et al. Scurvy. Medscape Aug 2011.
Available at: http://emedicine.medscape.com/article/125350-overview

36. D╇ The timed up and go test (TUG) is a measurement of mobility. The original purpose of
the TUG was to test basic mobility skills of frail elderly patients. The test has been used in other
populations, including people with arthritis, stroke, and vertigo. It includes a number of tasks
such as standing from a seating position, walking, turning, stopping, and sitting down, which are
all important tasks needed for a person to be independently mobile. It provides an opportunity
EXAM 1 | ANSWERS 65

to assess falls risk and may also be useful in monitoring response to interventions. Cognitively
impaired individuals are more likely to be unable to follow the instructions, but do not presume that
everyone with cognitive impairment will be unable to complete this test.
For the test, the person is asked to stand up from a standard chair and walk a distance of
approximately 10 feet (3m), turn around and walk back to the chair, and sit down again. The
individual uses his/her usual footwear and can use any assistive walking device they normally use.
The person is seated with his/her back to the chair, their arms resting on the arm rests, and any
walking aid they may use should be in hand. Timing, using either a wristwatch with a second hand or
a stop watch, begins when the individual starts to rise from the chair and ends when he/she is once
again seated in the chair. The normal time required to finish the test is between 7–10sec. Individuals
who cannot complete the task in that time, probably have some mobility problems, especially if
they take more than 20sec.

Further reading
American College of Rheumatology. Timed up and go (TUG). Available at: http://www.rheumatology.
org/practice/clinical/clinicianresearchers/outcomes-instrumentatation/TUG.asp

37. A.╇ Prevalence estimates of elder abuse are difficult to get as much is either not reported
or not recognized. Abusers often have mental health problems, are within the family and in a
caring role; they are more likely to be financially dependent upon the victim and the provocation
is stress and frustration within their caring role. Abuse may be financial, physical, psychological,
verbal, sexual, or neglect. Consider elder abuse when there are unexplained or hard to explain
injuries; when the older person seems fearful of their carer; when a supposedly ‘failing’ individual
unexpectedly ‘flourishes’ in hospital; where there is significant carer strain; when the carer is
known to have mental health problems; when there are repeated unexpected hospital admissions
suggesting poor adherence to medication, which should be being supervised.

Further reading
Action on Elder Abuse. What is elder abuse. Available at: http://www.elderabuse.org.uk/Mainpages/
Abuse/abuse.html
House of Commons Health Committee, Elder Abuse, Second Report of Session 2003–04. Available
at: http://www.publications.parliament.uk/pa/cm200304/cmselect/cmhealth/111/111.pdf

38. B.╇ NICE guidelines encourage involvement of patients in ‘self-management’ and advocate a
stand-by home course of antibiotics and corticosteroids. NICE guidelines currently only advocate
osteoporosis treatment for individuals receiving frequent courses of oral corticosteroids.
This patient does not meet criteria for LTOT therapy, which requires a pO2 of < 7.3 kPa in a period
of clinical stability. NICE only advocate mucolytics in patients with chronic cough productive of
sputum who improve with their use, and not use routinely to prevent exacerbations. The benefits
of beta-blockers in ischaemic heart disease are well recognized. A systematic Cochrane review
failed to demonstrate any adverse effect of cardio-selective beta-blockers on lung function or
respiratory symptoms compared to placebo and more recent studies (Short et al., 2011) have
suggested a reduction in mortality with chronic beta-blocker use in COPD. COPD patients should
therefore not have them withheld.

Further reading
BNF (Oxygen Section). Available at: http://www.bnf.org/bnf/index.htm
NICE. Chronic obstructive pulmonary disease, Clinical guideline 101.
Available at: http://guidance.nice.org.uk/CG101
66 EXAM 1 | ANSWERS

Salpeter SR, et al. Cardioselective beta-blockers for chronic obstructive pulmonary disease.
Cochrane Database of Systematic Reviews 2008. Available at: http://onlinelibrary.wiley.com/
doi/10.1002/14651858.CD002992/abstract
Short, et al. Effect of β blockers in treatment of chronic obstructive pulmonary disease: a
retrospective cohort study. British Medical Journal 2011; 342: d2549.
Available at: http://www.bmj.com/content/342/bmj.d2549.full

39. E.╇ Constipation with spurious diarrhoea (overflow) is most likely in someone who has recently
been constipated (and probably still is); who is on an opiate and requires regular treatment for
hypercalcaemia.

Further reading
Basson MD. Constipation Available at: http://emedicine.medscape.com/article/184704-overview
Guandalini, S. Diarrhea Available at: http://emedicine.medscape.com/article/928598-overview

40. D.╇ The HbA1c is satisfactory, which would suggest further intensification of therapy is not
indicated. In addition, the majority of blood glucose values are < 6mmol, which would suggest over-
treatment in the context of recurrent hypoglycaemia. The occasional high fasting BMs may suggest
that there is significant hypoglycaemia overnight, with rebound hyperglycaemia. While it would
be useful to suggest recording nocturnal blood glucose occasional to confirm this, there is ample
evidence to suggest an immediate reduction in basal glargine levels.

Further reading
NICE. CG66: ‘Type 2 Diabetes’. Clinical guideline 66. Available at: http://guidance.nice.org.uk/CG66

41. A.╇ Activated charcoal dressings are effective at reducing malodour. Consideration should also
be given as to whether the wound may be infected, e.g. with pseudomonas or anaerobes. These
may also result in malodour and treatment may help.
Tranexamic acid soaked dressings may be useful for persistently bleeding ulcers.

Further reading
McManus J. Principles of skin and wound care: the palliative approach. End of Life Care 2007;
1(1): 8–19. Available at: http://endoflifecare.co.uk/journal/0101_palliative.pdf

42. B.╇ Whilst anyone developing diarrhoea in hospital after receiving antibiotics must be screened
for Clostridium difficile the most important first step is digital rectal examination to exclude
constipation with overflow diarrhoea. Following a fractured pelvis she will almost certainly have been
immobile and treated with opioid analgesia. Appropriate examination and correct diagnosis could
prevent unnecessary bed/ward moves for this patient and lower her risk of developing delirium.

Further reading
NICE. Faecal incontinence: the management of faecal incontinence in adults. Clinical guideline 49.
Available at: http://www.nice.org.uk/nicemedia/pdf/CG49NICEGuidance.pdf

43. D.╇ The most likely cause for the confusion is hyponatraemia resulting from salt and fluid losses
secondary to the recent GI illness. The associated hypovolaemia is driving vasopressin secretion and
a high urine osmolality.
Now that the patient is able to eat and drink normally, the situation should start to correct.
Oral rehydration salts will help this. If the patient were unable to manage sufficient oral intake,
EXAM 1 | ANSWERS 67

rehydration with isotonic saline would be appropriate. The target rate of correction should not
exceed 0.5–1mEq/L/hr.
The clinical picture can be explained by the hyponatraemia. There is no focal neurology. A CT head
scan is not currently indicated, but may need consideration if there is no improvement with first line
treatment.
Fluid restriction would further dehydrate the patient, and stimulate increased vasopressin levels.
Hypertonic saline runs the risk of correcting the hyponatraemia too rapidly, which may precipitate
central pontine myelinolysis
Addison’s disease may result in hypotension and hyponatraemia. The association with the GI illness
and reasonable random cortisol make this less likely.

Further reading
Simon EE, Batuman V. Hyponatremia.
Available at: http://emedicine.medscape.com/article/242166-overview#a0199

44. D.╇ The story is fairly typical of someone with osteomalacia and the bowed legs suggests she
has had rickets as a child, although could be a distractor and she actually had Paget’s of the legs (you
are not told if this was anterior bowing). The TSH and free T4 may be consistent with borderline
hypothyroidism, but this would not explain her symptoms. An erythrocyte sedimentation rate (ESR)
of 50 may not be abnormal in this age group and is certainly not high enough to make polymyalgia
rheumatica a likely cause of her symptoms. The low calcium, raised alkaline phosphatase, low
albumin and iron deficiency anaemia (the ferritin is low normal, but with a raised ESR the ferritin
is likely to be even lower) suggest a dietary cause for her osteomalacia. Osteoporosis may be
associated with Cushing’s syndrome, but you are given no features to make you consider Cushing’s
syndrome to be a possibility. Gastric adenocarcinoma might explain her anaemia and non-metastatic
manifestations of malignancy might explain her fatigue, but neither would explain her pain and other
biochemical abnormalities.

Further reading
Family Practice Notebook. Bone disorders.
Available at: http://www.fpnotebook.com/rheum/Bone/Ostmlc.htm

45. E.╇ NICE report that ‘Denosumab is recommended as a treatment option for the secondary
prevention of osteoporotic fragility fractures only in postmenopausal women at increased risk of
fractures who are unable to comply with the special instructions for administering alendronate
and either risedronate or etidronate, or have an intolerance of, or a contraindication to, those
treatments.’ The low eGFR in this patient would constitute such a contraindication. Denosumab
is not contra-indicated in renal failure, although care must be taken to monitor for hypocalcaemia.
Alfacalcidol should therefore also be given.
NICE guidelines suggest treatment without DEXA scans in females aged over 75yrs who sustain
fragility fractures rendering this unnecessary. While considered ‘first line’ by NICE for secondary
prevention of osteoporotic fractures in women over the age of 75 who have sustained a fragility
fracture, aldendronate is contra-indicated at an eGFR of less than 35ml/min/1.73m2. With CKD-4,
the patient would have an eGFR of 15–29ml/min/1.73m2. The dyspepsia may also raise concerns
regarding GI tolerability. Similarly, the summary of product characteristics for strontium ranelate
says to avoid if eGFR less than 30mL/min/1.73m2.
68 EXAM 1 | ANSWERS

Given her degree of renal impairment, she is likely to need activated 1-alpha hydroxycholecalciferol
(alfacalcidol) instead of colecalciferol as the ability of the kidneys to hydroxylate the colecalciferol
will likely be impaired. Paired PTH and calcium levels should also be assayed.

Further reading
British National Formulary. Available at: http://www.bnf.org.uk
NICE. Secondary Prevention Osteoporosis. Guideline TA161.
Available at: http://www.nice.org.uk/TA161
NICE. Denosumab. Guideline TA204. Available at: http://guidance.nice.org.uk/TA204

46. B.╇ Levomepromazine is a phenothiazine, which has anti-emetic and sedative effects, and
would, therefore, target the present symptoms. There is no obvious evidence of pain, so strong
opioid analgesia may be unnecessary and may only serve to worsen nausea. Regular re-assessment
for pain and consideration of diamorphine should be undertaken if there is no response to other
treatments though.
The patient has not required previous subcutaneous boluses, and an infusion may therefore not
be necessary but should be considered if more than two stat doses of medication are required
over 24hr.
Hyoscine hydrobromide is an anti-muscarinic drug, which may be effective in drying-up secretions.
It crosses the blood brain barrier and may worsen confusion. There is no indication here.
Midazolam would be first-line choice, but will not ameliorate the nausea.
There is no evidence that intravenous hydration enhances palliative care. Good mouth-care should
continue to be offered.

Further reading
British National Formulary. Available at: http://www.bnf.org.uk
The National Council for Palliative Care. Available at: http://www.ncpc.org.uk/

47. E.╇ Appropriately-sized walking aids are beneficial both during rehabilitation and as longer-
term strategy to help those with permanent difficulty in walking. An appropriately selected aid may
reduce the risk of falling, but paradoxically may increase the risk of injury if a fall does occur, as the
faller may injure themselves on the aid and/or it may prevent them breaking the fall.

Further reading
Disability Living Foundation. Choosing walking equipment.
Available at: http://www.dlf.org.uk/factsheets/Choosing_Walking_Equipment.pdf
Elmamoun M, Mulley G. Walking sticks and frames for patients with neurological disorders. Practical
Neurology 2007; 7: 24–31. Available at: http://pn.bmj.com/content/7/1/24.extract

48. D.╇ The lack of pre-existing cognitive difficulty and the acute insult in the form of a urinary tract
infection suggest that the confusion is driven by a delirium. These may outlast the trigger event.
The patient currently lacks capacity to decide on discharge planning. Owing to the confusion
and wandering, he could not be safely managed at home. In cases where a patient lacks capacity,
clinicians should arrange a best interests meeting to decide the safest and least restrictive course
of action. If a patient has no representative, then a review by an independent mental capacity
advocate should be sought.
EXAM 1 | ANSWERS 69

However, any ‘best interests’ decision should only be taken if it is deemed unlikely that a patient will
regain capacity, or if it needs to be taken before they do so. In this case, it is likely that the delirium
will resolve now the trigger has settled. This may be expedited by good nutrition, hydration and
avoiding constipation or unnecessary medications.
His situation should therefore be reviewed on a daily basis and discharge planned when it is deemed
safe to do so.

Further reading
NICE. Delirium: Diagnosis, prevention, and management. Clinical guideline 103.
Available at: http://publications.nice.org.uk/delirium-cg103
Mental Capacity Act 2005. Available at: http://www.legislation.gov.uk/ukpga/2005/9/contents

49. C.╇ A Cochrane systematic review of the randomized controlled trial evidence suggests that
day hospital care is at least equivalent in effectiveness to comprehensive geriatric assessment in
traditional settings, but may be more expensive. Early discharge to a community hospital providing
multi-disciplinary care is associated with increased independence and similar cost-effectiveness
compared to post-acute care in general hospitals. Nurse-led units are associated with longer lengths
of stay and so are unlikely to be cost-effective. A single trial of rehabilitation in a care home setting
found considerable increase in length of stay without additional health gain or reduction in need for
long-term institutional care.
There are no trial data on therapist led intermediate care (IC) services, but one would anticipate
similar outcomes to nurse-led units.

Further reading
British Geriatric Society. Intermediate Care. Guidance to Commissioners and Providers of Health
and Social Care. Available at: http://www.bgs.org.uk/index.php/topresources/publicationfind/
goodpractice/363-intermediatecare
Forster A, Young J, Lambley R, Langhorne P. Cochrane systematic review: medical
day hospital care for the elderly vs. alternative forms of care. Cochrane Summaries,
2009; January 21. Available at: http://summaries.cochrane.org/CD001730/
medical-day-hospital-care-for-the-elderly-versus-alternative-forms-of-care
Griffiths PD, Edwards ME, Forbes A, et al. Cochrane systematic review: effectiveness of IC in nursing-
led in-patient units. Cochrane Summaries, 2009; January 21. Available at: http://summaries.cochrane.org/
CD002214/effectiveness-of-intermediate-care-in-nursing-led-in-patient-units

50. B.╇ The main clue in this case is the recent history of asthma, presence of eosinophilia, and
sensorimotor neuropathy (all of which suggests an autoimmune phenomenon with possible
vasculitis). Hyper-eosinophilic syndrome causes eosinophilic infiltration of the lungs, but neuropathy
is not seen. Parasitic infection can give rise to asthma and eosinophilia, but not neuropathy. Occult
malignancy is not associated with asthmatic symptoms, although neuropathy can be a main feature.
The underlying cause of Churg–Strauss syndrome is not known. Whilst there are isolated case
reports of an association with mesalazine, propylthiouracil, methimazole, and freebase cocaine,
there is no known linkage with aspirin, bendroflumethiazide, or lamisil.

Further reading
Farid-Moayer M. Churg–Strauss syndrome.
Available at: http://emedicine.medscape.com/article/333492-overview
70 EXAM 1 | ANSWERS

Johns Hopkins Vasculitis Center. Churg–Strauss syndrome (CSS). Available at: http://www.
hopkinsvasculitis.org/types-vasculitis/churgstrauss-syndrome-css/

51. D.╇ Current NICE guidelines advocate continuing aspirin providing there are no stigmata of
recent bleeding, but suggest the use of a proton pump inhibitor. NICE advises that in patients
taking clopidogrel, the decision to continue or stop it should be made with the involvement of
an appropriate specialist. There is no evidence to suggest that clopdiogrel is safer than aspirin at
reducing future bleeding risk. Neither is there evidence to suggest that H2 receptor antagonists in
conjunction with a proton pump inhibitor confer more benefit. The negative CLO test excludes
H. pylori infection and the need for eradication therapy.
Given the above, no further changes are required.

Further reading
NICE. Acute upper gastrointestinal bleeding: management. Clinical Guideline 141. Available at: http://
publications.nice.org.uk/acute-upper-gastrointestinal-bleeding-management-cg141
NICE. Managing dyspepsia in adults in primary care, Clinical guideline 17.
Available at: http://www.nice.org.uk/cg17
NICE. MI secondary prevention, Clinical guideline 48.
Available at: http://www.nice.org.uk/guidance/CG48

52. D.╇ Correction of the clotting abnormality with vitamin K implies either warfarin usage or
nutritional deficiency. The latter is most likely owing to the problems identified at home.

53. E.╇ Symptoms of zinc deficiency include eczema on the face and hands, hair loss, mental apathy,
defects in the reproductive organs, delayed sexual maturation, menstrual irregularities, decreased
growth rate, and impaired mental development. Deficiency may also lead to postnatal depression,
loss of the senses of taste and smell, anaemia, poor appetite, impaired conduction and nerve
damage, white spots on the nails, mental disorders, susceptibility to infections, delayed wound-
healing, and impotence in men.
Common features of zinc deficiency include dry scaly skin, poor wound healing, easily
pluckable hair, hypoguesia, and night blindness. Vitamin C deficiency is associated with follicular
hyperkeratosis, petechiae, bleeding gums, and poor wound healing. Iron deficiency is associated
with, angular stomatitis, glossitis, and spooning of the nails. Vitamin A deficiency is associated
with: follicular hyperkeratosis, night blindness, keratomalacia, and hypoguesia. Thiamine deficiency is
associated with: muscle tenderness, paraesthesia, and hyporeflexia.
Meat and fish provide most of our dietary zinc, as the amino acids bind zinc and make it more
soluble. Zinc from fruit, vegetables, and cereals are less readily available for absorption due to
binding by phytates and oxalates. Zinc absorption also decreases with age and in women on HRT.

Further reading
Ahmed T, Haboubi N. Assessment and management of nutrition in older people and its importance
to health. Clinical Interventions in Aging 2010; 5: 207–16.
Available at: http://www.ncbi.nlm.nih.gov/pmc/articles/PMC2920201/
Principles and Practice of Geriatric Medicine. Pathy MSJ. 2006.
Available at: http://lib.myilibrary.com/Open.aspx?id=33981&src=2

54. B.╇ As his ‘off ’ periods are predictable he may be safe to drive, although his children’s concerns
suggest otherwise. He is not cognitively impaired when ‘on’ and so should be considered capacious,
EXAM 1 | ANSWERS 71

although his judgement may be affected by the fear of being isolated if he loses his car. There is
not enough reason to break confidentiality and approach the DVLA directly. In this circumstance, it
seems too long to wait 3 years before his next medical fitness to drive assessment is due; anyway,
this is the maximum period of license given by the DVLA and does not preclude earlier assessment
if considered appropriate. The best solution is to advise him to go to an approved driving test
centre, but warn him that if he fails this test he will not be allowed to drive home!

Further reading
DVLA. At a glance medical fitness to drive.
Available at: www.dft.gov.uk/dvla/medical/~/media/pdf/medical/at_a_glance.ashx

55. E.╇ The impairment in MMSE, acute onset, and fluctuating conscious level are consistent with
an acute delirium. It is possible that there is a degree of underlying Parkinson’s associated dementia
too; further collateral history and/or use of assessment tools such as the IQCODE, could help
explore this further.
Procyclidine is an anti-muscarinic, which may help to reduce drug-induced extrapyramidal
symptoms. The anti-cholinergic effects can, however, have a detrimental effect on cognitive
function. Weaning and stopping the procyclidine may help terminate the delirium and improve
baseline cognitive function.
In the absence of new focal neurology, head injury, or suspicion of central nervous system (CNS)
infection, neuroimaging is not a first-line investigation.
The negative nitrites, normal CRP, and lack of symptoms (other than confusion) make urosepsis
less likely than other causes, which should be pursued first. Sending a specimen for microbiology
may be appropriate meanwhile.
The atypical antipsychotic risperidone is licensed for use in acute delirium in the UK although
extrapyramidal side-effects are lower with atypical than typical antipsychotics one should use them
with caution in Parkinson’s patients. We are not told how long this man has had an extra-pyramidal
disorder, if it is less than one year then there is a possibility that this episode reflects emergent
dementia with Lewy bodies and so antipsychotics should be avoided until this can be excluded.
If there is no or limited improvement upon withdrawing the procyclidine then the next step would
be to consider withdrawing the selegiline.
Rivastigmine is potentially beneficial in Parkinson’s disease with dementia, but this diagnosis has
not been established and so whilst delirium is being excluded/treated it would be inappropriate to
initiate rivastigmine.

Further reading
British National Formulary. Available at: http://www.bnf.org/bnf/index.htm
Inouye S, van Dyck C, Alessi C, et al. Clarifying confusion: the confusion assessment method. Annals of
Internal Medicine 1990; 113: 941–8. Available at: http://www.ncbi.nlm.nih.gov/pubmed/2240918
Jorm AF, Korten AE. Assessment of cognitive decline in the elderly by informant interview. British
Journal of Psychiatry 1988; 152: 209–13. Available at: http://www.ncbi.nlm.nih.gov/pubmed/3167337
NICE. Delirium: diagnosis, prevention, and management, Clinical guideline 103.
Available at: http://guidance.nice.org.uk/CG103
Seppi K, Weintraub D, Coelho M, et al. The Movement Disorder Society evidence-based medicine
review update: treatments for the non-motor symptoms of Parkinson’s disease. Movement Disorder
2011; 26(Suppl 3): S42–80. Available at: http://www.ncbi.nlm.nih.gov/pubmed/22021174
72 EXAM 1 | ANSWERS

56. E.╇ Assuming she will recover from her acute illness, she should recover mobility and no longer
be bed-bound; this is likely to require input from the community physiotherapist or IC team. Thus,
we are looking for a temporary solution to her incontinence, but one that will protect the already
broken skin, to promote healing of her sacral pressure sore. Temporary catheterization is the most
appropriate option, provided she is agreeable.
Insensible fluid losses are likely to have increased during this acute illness and fluid intake to
have decreased. Further fluid restriction will not improve the incontinence and may cause other
problems due to dehydration. Pads and pants will not protect the already damaged skin, no matter
how absorbent the pads are. Although she may well be constipated with this acute illness, laxatives
would only be necessary if she is uncomfortable and, as she is bed-bound, these may cause further
problems of faecal soiling, impairing wound healing. There is no suggestion in the history that she
has a prolapse.

Further reading
Dubeau CE, Kuchel GA, Johnson T, et al. Incontinence in the frail elderly.
Available at: http://www.icsoffice.org/Publications/ICI_4/files-book/comite-11.pdf
European Association of Urology. Guidelines on urinary incontinence. Available at: http://www.
uroweb.org/fileadmin/user_upload/Guidelines/16%20Urinary%20Incontinence.pdf
NICE. The management of urinary incontinence in women, Clinical guideline 40.
Available at: http://www.nice.org.uk/nicemedia/live/10996/30281/30281.pdf
Scottish Intercollegiate Guidelines Network. Management of urinary incontinence in primary care.
Available at: http://www.sign.ac.uk/pdf/sign79.pdf

57. E.╇ An accepted classification that is used as a framework to assess patients for rehabilitation
consists of four levels:
• Pathology: the disease process itself.
• Impairment: the impact of the disease process on function.
• Disability: the restriction on ability to perform a task owing to the impairment.
• Handicap: the negative impact of a disability on an individual preventing them performing a
task they would otherwise have been able to and wish to do.

Further reading
Young J. Caring for older people British Medical Journal 1996; 313: 677.
Available at:  http://www.bmj.com/content/313/7058/677.full

58. B.╇ Whilst arrangements are being made for his discharge home with an appropriate care
package his daughters should explain to him that they no longer feel able to participate in his care
and that they will not respond to his demands for them to do so. This may be hard for them to
do, but they can then be supported by the multidisciplinary team to reinforce the need for the
care package. The geriatric team should have the expertise to assess his mental capacity and
should not need to refer to liaison psychiatry just for this. He has expressed a wish to go home
and agreed to have the care package recommended, he has also stated that this would be if his
daughters could not help any longer—this is all consistent with him being capacious. The care plan
has been identified and agreed, so transfer to intermediate care for further discharge planning is
inappropriate and not a function of intermediate care anyway. Discharge to residential care is not in
his best interests, but simply serving the wishes of his daughters.
EXAM 1 | ANSWERS 73

Further reading
Mental Capacity Act 2005 (England). Available at:  http://webarchive.nationalarchives.gov.uk/+/http://
www.dca.gov.uk/legal-policy/mental-capacity/mca-cp.pdf
Department of Health (England). Intermediate Care—Halfway Home. Updated Guidance for the
NHS and Local Authorities. London: DoH, 2009. Available at: http://www.dh.gov.uk/prod_consum_dh/
groups/dh_digitalassets/@dh/@en/@pg/documents/digitalasset/dh_103154.pdf

59. B.╇ He is hyponatraemic, euvolaemic (no postural hypotension) with otherwise normal
electrolytes and renal function making adrenocortical insufficiency unlikely. There is no information
to suggest fluid overload and so congestive cardiac failure is unlikely. Hypothyroidism can cause
hyponatraemia and cognitive impairment, but the cognitive impairment here is acute, rather than
chronic and so is likely to be delirium secondary to his hyponatraemia; the fact that he is a heavy
smoker and losing weight makes it most likely that he has syndrome of inappropriate antidiuretic
hormone secretion secondary to lung cancer. The fact that he smells of urine is a red herring and
does not in itself imply urine infection.

Further reading
Clayton JA, Le Jeune IR, Hall IP. Severe hyponatraemia in medical in-patients: aetiology, assessment and
outcome. Quarterly Journal of Medicine 2006; 99: 505–11.
Available at: http://qjmed.oxfordjournals.org/content/99/8/505.full.pdf+html
Schrier RW, Bansal S. Diagnosis and management of hyponatremia in acute illness. Current Opinion in
Critical Care 2009; 4(6): 627–34. Available at: http://www.ncbi.nlm.nih.gov/pmc/articles/PMC2716705/
Wakil A, Ng JM, Atkin SL. Investigating hyponatraemia. British Medical Journal 2011; 342: d1118.
Available at: http://www.bmj.com/content/342/bmj.d1118.full (requires BMA or Athens account).

60. B.╇ The history and clinical findings are consistent with a peripheral neuropathy. Her diabetes is
the likely cause.
Duloxetine is recommended as a first line neuropathic analgesic for diabetic patients and should be
tried first line. If this fails, other neuropathic agents may be considered. Neuropathic pain is often
not opiate sensitive.

Further reading
British National Formulary. Available at: http://bnf.org/bnf/index.htm
NICE. Neuropathic pain—pharmacological management, Clinical guideline 96.
Available at: http://www.nice.org.uk/CG96

61. C.╇ This history is consistent with urge incontinence. Initial conservative treatment should
involve a review of fluid intake and appropriate advice. Anticholinergics may facilitate bladder
relaxation and ease symptoms. Although unlicensed, desmoperssin can improve water retention by
the kidneys and when given at night reduce nocturia. Careful electrolyte monitoring is required.
If conservative and medical measures fail, then a containment system may become appropriate. The
risk of infections with an indwelling catheter makes this the least preferable solution.

Further reading
NICE. The management of lower urinary tract symptoms in men, Clinical guideline 97. Available at:
http://guidance.nice.org.uk/CG97
NICE. Urinary incontinence, Clinical guideline 40. Available at: http://www.nice.org.uk/guidance/CG40
74 EXAM 1 | ANSWERS

62. A.╇ NICE recommend aggressive treatment of cardiovascular risk factors in patients with
diabetes who should be assumed to be ‘high risk’. There is consequently no benefit in calculating
10-year risk and use of the Framingham risk equation in such situations is a NICE ‘Do Not’
recommendation.

Further reading
NICE. Type 2 Diabetes, Clinical guideline 66. Available at: http://guidance.nice.org.uk/CG66
NICE. Lipid modification, Clinical guideline 67. Available at: http://guidance.nice.org.uk/CG67

63. A.╇ The vague recollection and significant facial injury are highly suggested of a transient loss
of consciousness. While such reflex mechanisms may be slowed in older age, conscious individuals
who are falling would normally break the fall with their arms, or in the event they are unable to
do that, turn their head to one-side. Failure of all these protective mechanisms may imply loss of
consciousness. In the absence of a significant postural drop in BP, arrhythmia must be considered,
and significant pauses and/or bradycardia excluded. A reduction in the beta-blocker dose is likely
indicated.

Further reading
NICE. Transient loss of consciousness in adults, Clinical guideline 109.
Available at: http://guidance.nice.org.uk/CG109

64. C.╇ Everyone is responsible for ensuring the safeguarding of individuals in their care. It is not
appropriate to merely re-direct the son to an alternative agency. If validated, it is possible that there
could be a safeguarding issue here with potential financial abuse.
The first step should be to assess the capacity of the patient concerned and whether they consent
to further involvement of the safeguarding team. If they give consent or if they lack capacity and a
‘best interests’ decision is being taken they should be referred according to local pathways.
It may be appropriate for more junior members of the healthcare team to escalate to more senior
members in the first instance, but the possibility of abuse should not be ignored.

Further reading
Department of Health. Safeguarding Adults: the role of health services. Available at: http://www.
dh.gov.uk/en/Publicationsandstatistics/Publications/PublicationsPolicyAndGuidance/DH_124882

65. E.╇ There are no features of syncope in the history and environmental hazards are likely to be
the main factor contributing to the fall. The ECG findings are incidental and there are no associated
adverse features. It is possible that the second degree heart block is being exacerbated by systemic
absorption of her beta blocker eye drops. If these can be changed, the arrhythmia may settle.
Further rhythm analysis and/or cardiology involvement should be considered after ophthalmology
review.
There is no indication for admission given the lack of other adverse features. A pacemaker
would not be indicated if the block settled after drug review. There is no merit in undertaking a
further 24-hr tape unless a change to treatment is implemented. This would be appropriate after
withdrawal of her beta blocker eye drop.
Ideally, the patient would be assessed in parallel in their home environment by a falls prevention
team, but there is no particular indication for the district nurses to review.
EXAM 1 | ANSWERS 75

Further reading
Sovari A, et al. Second-degree atrioventricular block.
Available at: http://emedicine.medscape.com/article/161919-overview
Stewart WC, Castelli WP. Systemic side effects of topical beta-adrenergic blockers. Clinical Cardiology
1996; 19: 691–7. Available at: http://onlinelibrary.wiley.com/doi/10.1002/clc.4960190904/pdf

66. B.╇ Lateral medullary (also known as Wallenberg) Syndrome is a posterior circulation
stroke syndrome consisting of ipsilateral ataxia, ipsilateral Horner’s syndrome and contralateral
spinothalamic tract disruption. It usually results from pathology in the distal vertebral or posterior
inferior cerebellar arteries.
Weber syndrome consists of contralateral upper and lower limb weakness (corticospinal tract) and
ipsilateral gaze weakness. This results from disease in the territory of the posterior cerebral artery
affecting the midbrain.

Further reading
The Internet Stroke Centre. Lateral Medullary Syndrome (Wallenberg Syndrome).
Available at: http://www.strokecenter.org/professionals/stroke-diagnosis/stroke-syndromes/
lateral-medullary-syndrome-wallenberg-syndrome/
Shane Tubbs R. Circle of Willis Anatomy.
Available at: http://emedicine.medscape.com/article/1877617-overview

67. D.╇ Transfer to intermediate care services (in the study quoted this was to community hospitals,
i.e. bed-based IC) is associated with improved independence at 6 months for older people needing
post-acute rehabilitation. However, transfer later than 48hr after referral was associated with worse
outcomes at 6 months, as measured by the Nottingham extended activities of daily living.

Further reading
Green J, Young J, Forster A, Bogle S. Post-acute transfer of older people to IC services: the sooner the
better. Age and Ageing 2007; 36: 589–92.
Available at: http://ageing.oxfordjournals.org/content/36/5/589.long

See also
Green J, Young J, Forster A. A randomized controlled trial evaluation of locality-based community
hospital care for older people. British Medical Journal 2005; 331: 317–22. Available at: www.bmj.com/
content/331/7512/317
Young J, Green J, Forster A, et al. Postacute care for older people in community hospitals: a
multicenter randomized, controlled trial. Journal of the American Geriatric Society 2007; 55(12): 1995–
2002. Epub 2007 Nov 2. Available at: http://www.ncbi.nlm.nih.gov/pubmed/17979957

68. C.╇ Of the frailty phenotype, only gait speed has been shown to be associated with survival.

Further reading
Studenski S, Perera S, Patel K, et al. Gait speed and survival in older adults. Journal of the American
Medical Association 2011; 305(1): 50–8.
Available at: http://jama.jamanetwork.com/article.aspx?articleid=644554
Fried LP, Tangen CM, Walston J, et al. Frailty in older adults: evidence for a phenotype. Journals of
Gerontology. Series A: Biological Sciences and Medical Sciences 2001; 56(3): M146–57.
Available at: http://biomedgerontology.oxfordjournals.org/content/56/3/M146.short
76 EXAM 1 | ANSWERS

69. C.╇ The daughters describe an event that has occurred overnight and exaggerated their
mother’s residual post-stroke deficit, as well as generating a generalized weakness, which
gradually resolved, i.e. Todd’s paresis. As this is something that has affected all of her brain, a
TIA can be ruled out. Although she has residual neurological deficit from her stroke she has
been mobile and so pulmonary emboli seems unlikely. The normal blood tests make aspiration
pneumonia unlikely, anyway she would only have aspirated saliva overnight unless something else
happened. A cardiovascular event is a possibility with transient drop in cardiac output causing
global neurological deficit. A normal ECG does not exclude cardiac arrhythmia, but the recurrent
nocturnal nature of events makes post-stroke seizure more likely.

Further reading
Ibrahim AlEissa E. First adult seizure.
Available at: http://emedicine.medscape.com/article/1186214-overview
Myint PK, Staufenberg EFA, Sabanathan K. Post-stroke seizure and post-stroke epilepsy. Postgraduate
Medical Journal 2006; 82: 568–72.
Available at: http://www.ncbi.nlm.nih.gov/pmc/articles/PMC2585721/pdf/568.pdf

70. E.╇ The X-ray findings are consistent with an incidental osteoporotic crush fracture. Current
NICE guidelines would support treatment with a bisphosphonate without the need for further
imaging.

Further reading
NICE. Osteoporosis secondary prevention, Guideline TA161.
Available at: http://guidance.nice.org.uk/TA161

71. C.╇ Correct sizing of walking aids is important to enable them to be used safely. Walking sticks
should be measured from the wrist height to the ground and frames should be sized so that the
grips are at wrist height when the elbows are bent at 15º. A quad stick may be preferable to a frame
in patients with weakness following a stroke as they do not rely on the weak side.
Appropriate home modifications such as perching stools, shower stools, and grip rails can further
reduce risk of falls.

Further reading
Disability Living Foundation. Choosing walking equipment.
Available at: http://www.dlf.org.uk/factsheets/Choosing_Walking_Equipment.pdf
Elmamoun M, Mulley G. Walking sticks and frames for patients with neurological disorders. Practical
Neurology 2007; 7: 24–31. Available at: http://pn.bmj.com/content/7/1/24.extract

72. E.╇ Aspirin usage would not affect standard PT and APTT studies.
Inherited coagulation defects such as haemophilia A and B would normally present in childhood.
Acquired haemophilia can occur when autoantibodies against clotting factors are produced. This is
however, rare and would be associated with excessive bleeding. In this case, a factor deficiency is
excluded by the failure to correct with mixing studies. These involve mixing the blood sample with
pooled plasma. Replacing missing factors in this should correct any factor deficiency and ‘normalize’
the APTT.
A non-specific coagulation inhibitor, such as a lupus anticoagulant, would also inhibit clotting, even
after addition of pooled plasma. The Dilute Russell’s Viper Venom Time (dRVVT) test involves
adding a venom-derived pro-coagulant to the sample. The anticoagulant continues to inhibit
EXAM 1 | ANSWERS 77

coagulation and this is prolonged. A confirmatory test can be undertaken with the addition of an
excess of phospholipid, which should correct the result. A negative dRVVT test excludes a lupus
anticoagulant.
The most likely explanation is of a pre-analytical error. This could be EDTA contamination of the
citrated sample, or under-filling of the citrated sample. In the absence of other clinical features to
suggest a coagulopathy, a repeat should be undertaken in the first instance.

Further reading
Gretheln et al. Acquired Hemophilia. Medscape.
Available at: http://emedicine.medscape.com/article/211186-overview
Lab Tests Online UK. aPTT.
Available at: http://labtestsonline.org.uk/understanding/analytes/aptt/tab/test

73. E.╇ NICE recommend that individuals with a grade 3–4 pressure ulcer should, as a minimum
provision, be placed on alternating pressure mattresses or a sophisticated continuous low pressure
system. A high-specification foam mattress may be appropriate for a grade 1–2 ulcer as a minimum
in conjunction with very close observation, but should be upgraded if there is any concern.
Frequent repositioning, sitting out with appropriate pressure relief and optimal nutrition should all
be pursued. NICE advocate that all grade 2 and above pressure sores be treated as a local clinical
incident.

Further reading
NICE. Pressure ulcers: the management of pressure ulcers in primary and secondary care, Clinical
guideline 29. Available at: http://www.nice.org.uk/guidance/index.jsp?action=byID&o=10972

74. D.╇ The ageing heart demonstrates impaired relaxation and associated diastolic dysfunction. It
is also clear that endothelial dysfunction increases with age, and this is thought to occur as a result
of impaired nitric oxide mediated relaxation, although the precise mechanism is unclear. Impaired
endothelial relaxation is associated with cardiovascular disease.
The responsiveness to sympathetic, parasympathetic, and circulating catecholamine stimulation
decreases, with reduced autonomic mediated changes in heart rate. The response to exercise and
changes in posture are therefore attenuated.

Further reading
Fillit HM, Rockwood K, Woodhouse K (eds). Effects of ageing on the cardiovascular system.
Brocklehurst’s Textbook of Geriatric Medicine and Gerontology (pp. 91–6), 7th edn. Maryland
Heights: Saunders 2010.

75. C.╇ Bisphosphonates are well recognized to help reduce pain associated with Paget’s disease.
The raised alkaline phosphatase (ALP) and urinary hydroxyproline support this diagnosis in
conjunction with the X-ray. If urinary bone markers were normal, it would be prudent to check
prostate specific antigen levels to exclude prostate cancer.
There is no indication for radiotherapy. While a strong opioid may be needed, it is worth exploring
whether bisphosphonates can manage the pain in the first instance in order to avoid opioid side
effects. Similarly, an epidural would usually be reserved until all other options have been explored.

Further reading
Alikhan MM. Paget’s disease. Medscape.
Available at: http://emedicine.medscape.com/article/334607-overview
78 EXAM 1 | ANSWERS

76. E.╇ With increasing age average partial thromboplastin time (PTT) levels may be up to 35%
higher than in a younger adult population. This is thought to result from lower circulating activated
vitamin D levels. Calcium homeostasis is maintained, in spite of the low activated vitamin D levels,
by an increase in PTH—a mild secondary hyperparathyroidism.
Chronic hypomagnesaemia, which may be seen in nutritional deficiencies in the elderly, impairs PTH
secretion.

Further reading
Fulop T. Hypomagnesemia. Medscape.
Available at: http://emedicine.medscape.com/article/2038394-overview
Fillit HM, Rockwood K, Woodhouse K (eds). Disorders of the parathyroid glands. Brocklehurst’s
Textbook of Geriatric Medicine and Gerontology (Chapter 91, pp. 755–759), 7th edn. Maryland
Heights: Saunders, 2010.

77. E.╇ The son-in-law may have no reason to lie, but as the staff are concerned that more
rehabilitation may be needed they should confirm his assertion that his mother-in-law’s mobility is at
her usual level, and that she is simply unnerved by the unfamiliar staff and their apparent anxieties.
This all sounds very plausible, especially if she has trouble hearing what the staff are saying. She
may have more faith in walking with her son-in-law than with her son, as she knows her son cannot
pick her up if she falls. If the son-in-law is correct, then the IC bed is not required, but it would be
prudent to check with the patient and her main carer, her son, whether they feel any extra care is
needed before her discharge, rather than assuming the care package is adequate or increasing her
care with no thought as to whether this is appropriate to her and her son’s needs.

Further reading
Department of Health. Ready to go. Planning the discharge and the transfer of patients from hospital
and intermediate care. Available at: http://www.dh.gov.uk/prod_consum_dh/groups/dh_digitalassets/@
dh/@en/@ps/documents/digitalasset/dh_116675.pdf
Hinkle JL, et al. Examining assessment tools for discharge planning. Nursing Times 2008; 43:
32–5. Available at: http://www.nursingtimes.net/examining-assessment-tools-for-discharge-
planning/1910945.article
Shepperd S, McClaran J, Phillips CO. Discharge planning from hospital to home. The Cochrane
Library. Available at: http://onlinelibrary.wiley.com/doi/10.1002/14651858.CD000313.pub3/abstract;jse
ssionid=C0E46967BE215837EBBBE9C974A9491E.d01t01

78. D.╇ He is most likely to be suffering from acute delirium due to the anti-muscarinic side-effects
of tolterodine.
The recent onset of incontinence may have been due to a urinary tract infection, but there is
no evidence of one now. Therefore changing antibiotic is inappropriate. There is no evidence of
Parkinsonian symptoms at this time and so no need to adjust the dose of levodopa, indeed the
absence of extrapyramidal signs brings in to question the diagnosis or he is very well treated!
Furthermore, increasing the dose of Sinemet may aggravate his psychotic symptoms. His agitation
and paranoia should be managed without resort to further pharmacological intervention if at all
possible; haloperidol is contra-indicated in Parkinson’s disease and so if sedation were required
lorazepam might be a more preferred choice. Thromboprophylaxis with a low molecular weight
heparin is important as this man is at high risk of venous thromboembolic disease, but is not the
most important first step in the management of his delirium.
EXAM 1 | ANSWERS 79

Further reading
NICE CG103. Delirium: diagnosis, prevention, and management, Clinical guideline 103. Available at:
http://guidance.nice.org.uk/CG103
Woodford HJ, George J. Cognitive assessment in the elderly: a review of clinical methods. Quarterly
Journal of Medicine 2007; 100: 469–84.
Available at: http://qjmed.oxfordjournals.org/content/100/8/469.full

79. E.╇ The patient does have an impaired MMSE, but on the basis of the evidence presented
appears to understand, weigh up, and retain the information pertinent to her initial decision. She
therefore appears to have capacity to make discharge planning decisions. Capacity necessitates that
a decision be made free from duress. It is unclear whether the news the daughter will not support
the patient is exerting an unreasonable emotional pressure on the patient, or merely altering
the information she needs to weigh to reach a decision. Further discussion without the daughter
present would enable this to be ascertained and for alternatives, such as social services provided
care, to be explored.
Temporary respite placement may be necessary if decisions cannot be reached, but ideally a secure
discharge plan would be made from the ward. It is inappropriate to finalize these plans without
further discussion, as there remains doubt about the patient’s true wishes and capacity.
An IMCA would only be necessary if the patient lacked capacity and had no other advocate to act
in their best interests.

Further reading
Mental Capacity Act 2005. Available at: http://www.legislation.gov.uk/ukpga/2005/9/contents

80. B.╇ Dementia and visual impairment are amongst the most common medical conditions in
older age and may frequently co-exist. There is a possible association between the degree of visual
impairment in age-related macular degeneration and cognitive impairment.
Cognitive testing in visually-impaired older people is under-researched and many tests rely upon
vision. Of the tests shown only the category fluency test is vision independent. This entails naming
as many different animals as possible within one minute and can also be extended to assess verbal
fluency by asking the individual to name as many words beginning with the same letter of the alphabet
in one minute. This test is incorporated in the Addenbrooke’s Cognitive Examination (ACE).
An alternative strategy might be to use one of the other tests and discount the questions requiring
vision, but in the case of the clock drawing test this nullifies any testing as it is all vision dependent.
A version of the MMSE for the blind (MMblind), in which all items requiring image processing are
omitted, has been validated. The proportion of vision dependent questions is: 3/30 in the MMSE;
33/100 in the ACE; and 8/30 in the Montreal Cognitive Assessment.

Further reading
Reischies FM, Geeiselmann B. Age-related cognitive decline and vision impairment affecting the
detection of dementia syndrome in old age. British Journal of Psychiatry 1997; 171: 449–51.
Available at: http://www.ncbi.nlm.nih.gov/pubmed/9463604
Clemons TE, Rankin MW, McBee WL. Cognitive impairment in age-related eye disease study: AREDS
report no. 16. Archives of Opthalmology 2006; 124(4): 537–43.
Available at: http://www.ncbi.nlm.nih.gov/pubmed/16606880
80 EXAM 1 | ANSWERS

81. C.╇ Metastatic prostate disease is a common differential for a sclerotic bone lesion and is
important to rule out at this stage. Bone scintography will also be needed, but a total body scan
should be conducted to look for remote lesions.
Serum alkaline phosphatase levels will usually be raised in active Paget’s disease, but need not
always be so. Hydroxyproline levels are usually raised in Paget’s and this can help with the diagnosis.
It may be appropriate to pursue these after the bone scan and PSA.

Further reading
Alikhan MM. Paget’s disease. Medscape.
Available at: http://emedicine.medscape.com/article/334607-overview

82. C.╇ Percutaneous endoscopic colostomy is a minimally invasive procedure that anchors the
sigmoid to the abdominal wall, preventing volvulus and enabling decompression.
Loperamide would contribute to constipation, which can predispose to volvulus.
The risks of anaesthesia and surgery are significant and render sigmoid colectomy unfavourable.
There is also a high risk of anastamotic breakdown if a primary repair is carried out.
A defunctioning stoma may be an option.
Sigmoidopexy involves surgically fixing the sigmoid to the abdominal wall. It requires either
a laparoscopy or laparotomy. Major abdominal surgery and the anaesthetic risk would carry
significant mortality risk in this patient.

Further reading
NICE. Percutaneous endoscopic colostomy—guidance, Guidance IPG161.
Available at: http://guidance.nice.org.uk/IPG161/Guidance/pdf/English
Thornton SC. Sigmoid and cecal volvulus. Medscape.
Available at: http://emedicine.medscape.com/article/197322-overview

83. B.╇ Infection, malignancy, and vascular insufficiency should all be considered in non-healing
ulcers. The location and well-demarcated lesion in this case is suggestive of an arterial component.
This is supported by the appearance of the foot. Although an ABPI of 1.0 may be considered
normal, vessel calcification (such as is common in diabetes or with increasing age) may give a false
negative result.
In the absence of overt infection, swabbing the wound would be more appropriate than empirical
antibiotics.

Further reading
Scottish Intercollegiate Guidelines Network (SIGN). Management of chronic venous leg ulcers.
Available at: http://www.sign.ac.uk/pdf/sign120.pdf

84. C.╇ Current evidence suggests that compression bandaging is the preferred intervention
for improving healing rates in venous ulceration. Caution should be used if there is evidence of
impaired arterial supply (ABPI < 0.8) or in patients with diabetes who may have heavily calcified
vessels and spuriously normal ABPIs. Compression would therefore not be advocated in this case.
Failure to respond to more simple measures may warrant formal vascular review and a closely
supervised trial of compression. Similarly there is no evidence of significant vascular disease
warranting referral at this stage.
Swabs should only be taken in cases of evident infection, as cultures of skin flora in clinically non-
infected wounds may confound management.
EXAM 1 | ANSWERS 81

Simple non-adherent dressings are recommended as first line treatment. More expensive
hydrocolloid, silver or honey dressings should be held in reserve for non-responsive cases and/or
after expert review.

Further reading
Scottish Intercollegiate Guidelines Network (SIGN). Management of chronic venous leg ulcers.
Available at: http://www.sign.ac.uk/pdf/sign120.pdf

85. E.╇ ACE inhibitors are proven to improve life expectancy in all grades of heart failure.
Aldosterone has been shown to decrease mortality in patients with NYHA grade 3–4 heart failure
and would be further down the prescribing pathway than an ACE inhibitor.

Further reading
NICE. Chronic heart failure. National guideline for diagnosis and management in primary and
secondary care. Clinical guideline 108.
Available at: http://www.nice.org.uk/nicemedia/live/13099/50514/50514.pdf

86. A.╇ Colonic malignancy would best explain this presentation and so the investigation of choice
would be CT scan of the abdomen and pelvis possibly followed by colonoscopy for tissue diagnosis
if a lesion is identified. If the CT does not identify any tumour and confirms liver metastases then
an alternative approach for a tissue diagnosis would be to perform a liver biopsy. Upper GI tumour
is a possibility and so oesophago-gastroduodenoscopy may be required if CT is unhelpful. Whilst
she may well need to have iron replacement therapy, the priority must be to relieve symptoms
and also ensure she has a ‘reserve’ lest there is any substantial blood loss before a diagnosis can be
established. So, the priority in her management is the blood transfusion.

Further reading
British Columbia Guidelines and Protocols. Iron deficiency—investigation and management.
Available at: http://www.bcguidelines.ca/pdf/iron_deficiency.pdf
Goddard AF, James MW, McIntyre AS, Scott BB. British Society of Gastroenterology. Guidelines for the
management of iron deficiency anaemia. Available at: http://www.bsg.org.uk/pdf_word_docs/iron_def.pdf

87. E.╇ The normalization of his LFTs suggests that the itch is not related to biliary obstruction.
Oramorph is well recognized to cause itching and given that pain is controlled, this should be
discontinued before considering other possible treatments, such as antihistamines or topical agents.

Further reading
NHS Lothian ‘Palliative Care Guidelines—Itch’.
Available at: http://www.palliativecareguidelines.scot.nhs.uk/documents/itch.pdf

88. B.╇ This may be a reversible situation and she should be given the benefit of rehydration with
an intention to return to the nursing home once she has been rehydrated. This is not inconsistent
with the principles of palliative care; there is no immediate suggestion that she is dying (although
she well might). As her son is against artificial nutrition for his mother, it is reasonable (as a best
interests decision) to attempt rehydration alone and see if she starts eating again once rehydrated.
There is no obvious reason to commence antibiotics. There is no suggestion that she is distressed
or in pain, so diamorphine is not indicated.
82 EXAM 1 | ANSWERS

Further reading
NHS Lothian. Supportive and Palliative Care Indicators Tool (SPICT ™). Available at: http://www.
palliativecareguidelines.scot.nhs.uk/documents/SPICT_June2012.pdf
Mental Capacity Act 2005 (England). Available at: http://webarchive.nationalarchives.gov.uk/+/http://
www.dca.gov.uk/legal-policy/mental-capacity/mca-cp.pdf

89. B.╇ Eight days after repair of a fractured neck of femur she is likely to still be taking opioid
analgesia and so constipation is the most likely cause of her urinary retention. As she is only
receiving twice daily social care at discharge, the implication is that she is mobile, Nonetheless, she
will be less mobile than usual and so at risk of constipation.
Cauda equina syndrome may cause bowel and bladder disturbance, but is unlikely as you are not
given any of the suggestive features: low back pain; unilateral or bilateral leg pain radiating from the
buttocks down the back of the thighs; saddle paraesthesia; lower extremity weakness and sensory
disturbance; reduced or absent lower limb reflexes.
There is no mention of symptoms to suggest vaginal prolapse; dependent upon the severity of the
prolapse this is more likely to be associated with stress incontinence.
Delirium is associated with cholinergic deficit (hence the propensity for anticholinergic drugs
to cause confusion) and so one could hypothesize that urinary retention might be more likely
in a delirious patient, but it is more likely that urinary retention in a delirious patient would be
secondary to constipation associated with the cause of delirium or the immobility, dehydration,
and poor nutritional intake associated with being delirious. Urinary retention may lead to a urinary
infection due to stagnation of urine, rather than the other way round.

Further reading
Cauda equina and conus medullaris syndromes. Available at: http://emedicine.medscape.com/
article/1148690-overview#aw2aab6b2b4
Hshieh TT, Fong TG, Marcantonio ER, Inouye SK. Cholinergic deficiency hypothesis in delirium: a
synthesis of current evidence. Journal of Gerontology. Series A: Biological Sciences and Medical Sciences
2008; 63(7): 764–72. Available at: http://biomedgerontology.oxfordjournals.org/content/63/7/764.
short (accessible via Athens log in).

90. E.╇ Before making a judgement on whether a patient may or may not have ‘rehabilitation
potential’ it is important to undertake a complete review of their current and pre-morbid
status and determine rehabilitation goals with the individual, assuming that they are competent
to make such decisions. Any patient willing to participate in rehabilitation should be given a
trial of rehabilitation without any assumptions being made about their ability or inability to
benefit. Depression may hinder rehabilitation goal attainment and should be treated. Dementia
should not be used as a barrier to rehabilitation, but should be considered when designing a
rehabilitation programme for the individual; those with behavioural disturbance and/or severe
dementia, with a proven inability to follow instructions (verbal, written, or signed) will only be
proven to have an inability to benefit from rehabilitation when the staff attempt rehabilitation
with them. Hypoactive delirium must be allowed to resolve before making a judgement
regarding rehabilitation potential, which again will be through a subsequent trial of rehabilitation.
Only a failure to establish rehabilitation goals (lack of willingness to participate not due to
cognitive impairment or depression) with the individual will predict an inability to benefit from
rehabilitation.
EXAM 1 | ANSWERS 83

91. D.╇ Parkinson’s patients must get their medication on time when in hospital and may suffer
pain, worsening of their Parkinson’s symptoms, delirium and be more prone to complications of
immobility and dysphagia as well as increased length of stay due to being under-medicated. He
has an ileus and so cannot have oral medication as absorption cannot be predicted. Melt selegiline
is a possibility, but as this is a weak drug it may not provide adequate symptom control and if he
continues to vomit despite the nasogastric suction absorption cannot be guaranteed. The rotigotine
patch is the best option although opinion differs as to how quickly this might start to achieve
adequate plasma concentrations.

Further reading
Brennan KA, Genever RW. Managing Parkinson’s disease during surgery. British Medical Journal 2010;
341 c5718. Available at: http://dx.doi.org/10.1136/bmj.c5718 (Published 1 November 2010).
Parkinson’s Disease Society. To stop their condition getting out of control, people with Parkinson’s
need their medication on time – every time Get it on time. Available at: http://www.parkinsons.org.
uk/sites/default/files/publications/download/english/b104_getitontimebooklet_0.pdf; also available at:
http://www.parkinsons.org.uk/sites/default/files/publications/download/english/b124_ideasforaction.pdf

92. C.╇ Attendance allowance is a tax free benefit available to anyone aged over 65 years, who
needs help with personal care, irrespective of their underlying diagnosis. It is not means tested and
does not require a medical report. It is meant to assist in the cost of care, but does not have to be
used to pay for care.

Further reading
HM government. Attendance allowance.
Available at: http://www.direct.gov.uk/en/disabledpeople/financialsupport/dg_10012438
Patient.co.uk. Attendance allowance.
Available at: http://www.patient.co.uk/health/Attendance-Allowance.htm

93. C.╇ It is likely that he is bored and so diversion therapy, engaging him in some stimulating activity
that he is interested in is an appropriate strategy. Simply ignoring the behaviour will not make it go away.
Antipsychotic medications are over prescribed in care home residents and should only be prescribed
for behavioural and psychological symptoms of dementia if the person is a danger to themselves or
others. Rewarding good behaviour is unlikely to work and is potentially punitive and denies his human
rights. Reminiscence therapy may be a suitable distracter, but is a second best option.

Further reading
All Party Parliamentary Group on Dementia. Always a last resort: Inquiry into the prescription of
antipsychotic drugs to people with dementia living in care homes. 2008 Available at: http://alzheimers.
org.uk/site/scripts/download.php?fileID=322
NHS Choices. Antipsychotic use in dementia. Available at: http://www.nhs.uk/news/2009/10October/
Pages/Antipsychotic-use-in-dementia.aspx
Alzheimer’s Society. Antipsychotic drugs. Available at: http://alzheimers.org.uk/site/scripts/documents_
info.php?documentID=548

94. E.╇ There are some worrying features that this might be a melanoma. The ABCDE warning
signs are:
• Asymmetry.
• Border irregularity.
84 EXAM 1 | ANSWERS

• Colour variability.
• Diameter more than 6mm or increasing in size rapidly.
• Extra features: bleeding, crusting, itching, elevation, or palpable nodule.
A fast track dermatology cancer referral is appropriate. The only treatment shown to work is wide
excision. Prognosis is determined by Breslow thickness (95–100% 5-year survival if <1mm; 50%
5-year survival if >4mm).

Further reading
Royal College of Physicians (London). The prevention, diagnosis, referral, and management of
melanoma of the skin. Concise Guidelines Number 7, 2007. Available at: http://www.rcplondon.ac.uk/
sites/default/files/concise-melanoma-2007.pdf

95. D.╇ A bedside commode offers nothing over using the toilet and will be as hard to get up from.
He would be unable to put a conveen on and anyway this is inappropriate as continence is not an
issue. Grab rails would help him to be able to grip and pull up, which he is unlikely to be able to do
easily. A raised toilet seat would be the best option. Pads and pants would be unnecessary do not
help with toileting and risk encouraging incontinence.

96. D.╇ He has limited functional limb disability, but will need to do most of his dressing in a seated
position to minimize falls risk. Thus, the aid that is most likely to help him is the trouser clip. Trouser
clip is designed to reduce the need to bend when pulling on trousers; it clips to the trousers and can
then attached to the upper clothing while seated, as the user stands the trousers are held in place.
A dressing sleeve might be useful to someone with more disability, who had difficulty dressing and
need something to facilitate clothes sliding on more easily.
A dressing stick is a wooden stick with a rubber tip at one end and a double wire hook at the other,
designed to pull on or push off garments that cannot be reached easily, e.g. socks or jacket sleeves.
A leg lifter is more often required after hip surgery although it can be useful in other conditions.
However, given that he has minimal residual weakness this is unlikely to be of use to him.
A zip pull will help when manual dexterity or strength cause difficulties in pulling up zips.

Further reading
The following website provides a useful insight in to the various domestic appliances that might benefit
people with all degrees of disability. Available at: www.essentialaids.com

97. E.╇ The CT head shows a left maxillary/frontal sinusitis with frontal lobe involvement. The free
gas and parafalcine enhancement are suggestive of an early abscess. This needs urgent neurosurgical
review and consideration for drainage.
In the absence of a likely source (sinusitis here), echocardiography should be considered to
investigate the possibility of infective endocarditis.
Level 2/3 care is likely to be needed, but again should be pursued after a definitive management
plan is agreed with the neurosurgeons.
A lumbar puncture should be deferred until after discussion with the neurosurgeons, as the abscess
may predispose to raised intracranial pressure and brainstem coning.
Meningococcal PCR on an EDTA blood sample can be diagnostic and should be considered in all
cases of meningitis where a LP is difficult/not possible.
EXAM 1 | ANSWERS 85

Further reading
Hasbun R. Meningitis. Medscape (updated 7 July 2011).
Available at: http://emedicine.medscape.com/article/232915-overview

98. B.╇ The decline in arterial pO2 with ageing is largely due to ventilation-perfusion mismatch due
to decreased elastic recoil of the lungs and resultant increased closing volume of the airways. The
absence of change in total lung capacity plays no role. The diffusion capacity of carbon monoxide
decreases with ageing, but this is of uncertain significance in respect of the fall in arterial pO2.
Maximum oxygen consumption is a measure of overall cardiopulmonary function.

Further reading
Sharma G and Goodwin J. Effect of aging on respiratory system physiology and immunology. Journal of
Clinical Interventions in Aging 2006; 1(3): 253–60.
Available at: http://www.ncbi.nlm.nih.gov/pmc/articles/PMC2695176/

99. E.╇ Impulse control disorders are defined as the failure to resist an impulse, drive, or temptation
to perform an act. Whilst these may often be harmful to the individual or others they may be
quite benign as in this case. In Parkinson’s disease such behaviours include: compulsive gambling,
hypersexuality, walkabout, punding, and the dopamine dysregulation syndrome. The most effective
treatment is readjustment of the dopaminergic medication with a careful balance between motor
and behavioural symptoms. Such behaviours are not linked to either depression or dementia in
Parkinson’s disease. Dopamine dysregulation syndrome is a specific impulse control disorder with
a need for increasing doses of dopamine replacement therapy, most typically levodopa. The MMSE
of 26/30 is above the usual cut-off of 23 for diagnosing a dementia. The history does not support a
diagnosis of hypomania or depression

Further reading
Gallagher S. Treating Parkinson’s disease: dopamine dysregulation syndrome and impulse control. Available
at: https://www.national.slam.nhs.uk/wp-content/uploads/2010/09/NP-Steve-Gallagher-Research.pdf
SIGN Diagnosis and pharmacological management of Parkinson’s disease, Guideline 113.
Available at: http://www.sign.ac.uk/pdf/sign113.pdf

100. B.╇ This woman has secondary Raynaud’s phenomenon, which occurs in 10% of people with
rheumatoid arthritis. Secondary Raynaud’s is often more severe than primary Raynaud’s, occurs
at an older age (but can occur at any age), is more often associated with a raised ESR and positive
anti-neutrophil antibodies or anti-extractable nuclear antibodies.
The severity of her disease, with scarring already occurring means she is likely to develop ulceration
and possibly gangrene. Whilst non-drug treatments should form the foundation of her management
pharmacological treatment will be needed. Non-drug treatment includes: avoiding extremes of
temperature, keeping extremities warm, reducing stress, rotating the arms like windmills during an attack
to promote vasodilation or placing the hands under the armpits, and avoid carrying bags by the handles.
Relaxation biofeedback to reduce stress levels has not been shown to be effective in a Cochrane review.
Beta-blockade will worsen symptoms and should be avoided. Vasodilation with a calcium channel
blockade is the treatment of choice. Prostaglandins would be the next line treatment if calcium
channel blockers cannot be tolerated or are ineffective; most studies report on the use of intravenous
iloprost. Topical nitrates may reduce the severity, but not the duration or frequency of attacks.
86 EXAM 1 | ANSWERS

Further reading
Goundry B, Bell L, Langtree M, et al. Diagnosis and management of Raynaud’s phenomenon. British
Medical Journal 2012; 344: e289. Available at: www.bmj.com/content/344/bmj.e289
Raynaud's & Scleroderma Association. Raynaud's & Scleroderma: A Guide for Health Professionals.
Available at: www.raynauds.org.uk/images/stories/PDF/hpbooklet2011.pdf
exam

2 QUESTIONS

1. An 86-year-old woman was fitted with bilateral behind the ear


hearing aids. When she was reviewed in the audiology clinic her
daughters complained that she still could not hear them when
they visit her at home and her hearing aids were being worn. The
technician examined the devices and felt that they were working
normally. Otoscopy was unremarkable.
What is the most likely explanation?
A. Acoustic feedback
B. Battery failure
C. Impacted ear wax
D. Misadjusted volume
E. Not wearing the aids

2. A 78-year-old man with Parkinson’s disease of 6 years duration


was admitted for elective resection of a Duke’s B adenocarcinoma
bowel. His usual Parkinson’s medication was Sinemet Plus® ×1
at 07.00, 11.00, 16.00, and 21.00 hours. Two years earlier he had
been unable to tolerate adjunctive medication with ropinirole
due to psychotic side-effects. His MMSE 1 month pre-operatively
was 21/30.
Which of the following interventions is most likely to reduce the
post-operative risk of delirium?
A. Ensuring he gets his Sinemet at standard ward drug rounds
B. Maintaining his usual dosing schedule of Sinemet
C. Prescribing rivastigmine peri-operatively
D. Prescribing a small dose of lorazepam each evening peri-operatively
E. Reducing his levodopa dosage peri-operatively
88 EXAM 2 | QUESTIONS

3. Parkinson’s patients often have difficulty turning over in bed


at night.
Which of the following is most likely to make turning in bed
easier?
A. Bed rail
B. Hospital bed
C. Monkey pole
D. Rope ladder
E. Silk sheets

4. An 83-year-old woman presented to A&E with central chest pain.


She had no significant past medical history, was a non-smoker,
and took no regular medication. She usually had a good exercise
tolerance and regularly walked several miles. She had no previous
or subsequent exertional symptoms.
The examination, ECG and 12-hr troponin undertaken in the ED
were normal and she was referred for an exercise tolerance test. She
managed to comply with this and achieved Stage 4 of the Bruce protocol
and 95% of her target heart rate. She stopped due to fatigue.
The exercise test technician reported that they observed a maximum
30/15mmHg rise in BP from her baseline of 120/70.
Which of the following is the next most appropriate step in her
management?
A. Arrange CT of the coronary arteries
B. Arrange a myocardial perfusion scan (e.g. MIBI)
C. Arrange percutaneous coronary angiography
D. Reassure and discharge with advice for if symptoms recur
E. Start an oral long acting nitrate
EXAM 2 | QUESTIONS 89

5. An 84-year-old woman was brought to the accident and


emergency department. Her family were concerned that she had
been deteriorating for several months. She had gained weight,
continually complained of feeling cold, and they felt her thought
processes had slowed. They wondered if she was developing
dementia. More recently, she had become dizzy on standing up,
and had nearly lost consciousness while doing so earlier that day.
On examination, her BMI was 32. Her BP was 120/60 with a heart
rate of 60beats/min regular. Respiratory, abdominal, and neurological
examinations were unremarkable.
Initial investigation
Na 133mmol/L (137–144)
K 5.0mmol/L (3.5–4.9)
Urea 7.0mmol/L (2.5–7.0)
Cr 105µmol/L (60–110)
Thyroid-stimulating hormone (TSH) >100mU/L (0.4–5.0)
Full blood count: C-reactive peptide and corrected calcium were normal.
ECG: sinus rhythm 55beats/min with 1st degree heart block.
Urine Dipstix: positive for nitrites and leucocytes.
Lying and standing BP: showed a drop of 30/10mmHg.
Which of the following is the next most appropriate course of action?
A. Arrange an urgent CT head scan
B. Commence trimethoprim for a likely UTI
C. Start IV replacement with liothyronine sodium
D. Start 25micrograms oral thyroxine daily
E. Undertake a synacthen test

6. A 78-year-old man presented with a 4-month history of increasing


shortness of breath. His exercise tolerance had reduced from 800m
to 10m and he was no longer able to get upstairs to bed. He had slept
in a chair for the last 1 month. He was a non-smoker and retired tax
inspector. Six months earlier he had a left total knee replacement from
which he had made an uneventful recovery. He was on no medication.
His brain natriuretic polypeptide (BNP) level was 50pg/mL (<100pg/mL)
and D-dimer 680ng/mL (<500ng/mL). CXR was normal as was resting
ECG. Oxygen saturation was 87% (>95%) on air.
What is the most likely cause of his shortness of breath?
A. Chronic heart failure
B. Chronic obstructive pulmonary disease
C. Late-onset asthma
D. Recurrent pulmonary emboli
E. Gastro-oesophageal reflux disease
90 EXAM 2 | QUESTIONS

7. Which of the following statements is not true?


A. Medical advances are the principle driver for the increase in life-expectancy at birth
B. The fastest growing segment of the UK population is the over 80s
C. The number of centenarians is predicted to rise from 5523 in 1996 to 95 000 in 2066
D. The world’s population over 65years is growing at a rate of 2.4% annually
E. UK birth rates have fallen below the natural replacement of 2.1 children per woman

8. A 74-year-old woman developed sudden painless loss of vision in


her right eye. Fundoscopic examination was compatible with a
diagnosis of central retinal vein occlusion.
Which of the following is least important in her further
management?
A. Assessing BP
B. Random blood glucose
C. Serum lipid profile
D. Treating co-existent glaucoma
E. Treatment with aspirin

9. The 82-year-old wife of an 84-year-old man with Alzheimer’s


dementia visited her general practitioner for a routine check of
her BP. During this consultation she stated that she was finding it
increasingly difficult to care for her husband, but that she would
not wish to put him in to a care home.
Which of the following characteristics is not well correlated with
low well-being in carers?
A. Behavioural disturbance in care recipient
B. Being a male care giver
C. Carer’s range of coping strategies
D. Extent of social support network
E. Strength of relationship

10. A 72-year-old woman had a 2-year history of difficulty in naming


objects, picture recognition, and comprehension. Recall and
visuospatial testing were normal. Her language had gradually
deteriorated and, although it remained fluent, it consisted
primarily of stock phrases and was reduced in significant content.
Repeated cognitive testing in other domains showed little change
and her husband reported no significant behavioural difficulties.
What is the most likely diagnosis?
A. Alzheimer’s disease
B. Functional dysphasia
C. Progressive non-fluent aphasia
D. Semantic dementia
E. Vascular dementia
EXAM 2 | QUESTIONS 91

11. A 72-year-old woman with osteoarthritis had increasing pain


in her left knee. This was bad first thing in the morning and
eased over a few minutes as she ‘got going’. She was particularly
troubled on climbing the stairs and was only doing this
when absolutely necessary. She had no other affected joints.
Examination confirmed crepitus in the left knee, but was
otherwise unremarkable. Her BMI was 30kg/m2. She was taking
no medication.
Which of the following is the least appropriate course of action at
this point?
A. Advise she lose weight
B. Recommend glucosamine supplements
C. Recommend regular paracetamol
D. Refer to a physiotherapist for review
E. Topical NSAID

12. A 69-year-old man with diet-controlled type-II diabetes


complained of recent onset uncomfortable muscle aches in his
arms. He had been hospitalized 3 months earlier with a non-ST
elevation myocardial infarction. Since his discharge he was taking
aspirin 75mg daily, simvastatin 40mg daily, bisoprolol 5mg daily
and ramipril 5mg daily. His BP was 120/72mmHg sitting and
standing.
Blood tests
Haemoglobin 14.2g/dL (13–18)
WCC 7.5 × 109/L (4–11)
CRP 10mg/L (<10)
TSH 5.0mU/L (0.4–5.0)
Creatine kinase 210U/L (24–195)

What is the most likely diagnosis?


A. Hypothyroidism
B. Polymyalgia rheumatic
C. Polymyositis
D. Simvastatin side-effect
E. Viral myalgia
92 EXAM 2 | QUESTIONS

13. A 70-year-old man presented with sudden onset headache,


vomiting, and blurred vision. He had been fasting for religious
reasons. Paracetamol and codeine did not help the headache,
which he rated as 8/10.
He was being monitored yearly in the haematology clinic because
of polycythaemia rubra vera, for which he underwent and finished
chemotherapy 5 years earlier. He also had hypertension. His current
medication was aspirin 75mg daily and amlodipine 5mg daily.
On examination his pulse was 70beats/min, BP 138/90mmHg, and heart
sounds were normal. Both pupils reacted sluggishly to light. There was
bilateral papilloedema, and proximal and distal weakness in his right
leg (4/5). There was no neck rigidity. There were no other neurological
abnormalities.
ECG showed sinus rhythm, slight left ventricular hypertrophy, but no
ischaemic changes.
Blood results
Hb 20.6g/dL (13–18)
WCC 10.2 × 109/L (4–11)
Platelets 450 × 109/L (150–400)
Urea 11.4mmol/L (2.5–7.0)
Creatinine 136µmol/L (60–110)
CRP 7mg/L (<10)
Sodium 149mmol/L (137–144)
Potassium 5.6mmol/L (3.5–4.9)
Liver function, bone profile, and thyroid function were normal.
What would be the most appropriate imaging to confirm the
diagnosis?
A. CT head scan without contrast
B. CT head scan with contrast
C. MR cerebral arteriography
D. MRI head diffusion weighted imaging
E. MR cerebral venography
EXAM 2 | QUESTIONS 93

14. A 77-year-old woman with newly-diagnosed AF had a 24-hr


tape. This had confirmed her to be in permanent AF at a rate
ranging from 55 to 75beats/min. She had a past medical history
of hypertension, diabetes mellitus, and congestive cardiac failure.
She had never had a stroke or TIA. She had had three falls in the
last year—all attributed to environmental hazards.
She was taking metformin 500mg OD, ramipril 2.5mg OD, and
furosemide 40mg OD.
Echocardiography demonstrated bi-atrial enlargement, mild left
ventricular impairment, and no significant valvular disease.
Which of the following would be the most appropriate drug to
introduce?
A. Aspirin
B. Clopidogrel
C. Diltiazem
D. Dipryidamole
E. Warfarin

15. An 84-year-old woman with dementia was to be discharged from


hospital to a dementia registered care home. Her family asked for
a needs assessment of eligibility for NHS Continuing Health Care
prior to her discharge.
Which of the following is not directly included in this assessment?
A. Cognition
B. Medication
C. Nutritional status
D. Vision
E. Wandering

16. As the secondary care adviser to a Clinical Commissioning Group


you are asked for advice in helping determine to what extent
they should invest in telehealth and telecare for older people with
long-term conditions.
Which of the following best describes the evidence base for these
interventions?
A. There is no evidence of superiority to usual care
B. There is no evidence that older people benefit
C. They are most effective in multiple long-term conditions
D. They have no place in dementia management
E. They have no place in managing single long-term conditions
94 EXAM 2 | QUESTIONS

17. A 72-year-old woman presented to the continence clinic. She


was usually fit and well, and ate a normal diet. Her only relevant
past medical history was of two vaginal deliveries at ages 35 and
38 years. No instrumentation was required and she sustained no
tears. Over the last 5 years she developed problems with faecal
incontinence. She originally noticed that she was leaking faeces
when coughing several years ago. The situation had deteriorated
such that she was now leaking most days.
Her weight has been stable for the last 5 years. She reported that the
stools were formed (Bristol Stool Chart 4) with no evidence of blood
or mucus. Abdominal examination is unremarkable. Digital rectal
examination showed an empty rectum, with reduced sphincter tone.
She had no overt cognitive problems and was independently mobile.
Baseline blood tests arranged by her GP were normal.
Which of the following is the next most appropriate step in her
investigation/management?
A. Anorectal manometry and physiology studies
B. Flexible sigmoidoscopy
C. Pudendal nerve studies
D. Referral to a physiotherapist specializing in continence
E. Trial of loperamide

18. An 87-year-old woman needed to be hospitalized because she


had suffered a stroke. She lived alone with her son, who had
Alzheimer’s dementia and was dependent upon his mother, who
was his sole carer. He could not be left alone.
What is the most appropriate action plan for her son?
A. Arrange a live-in carer as soon as possible
B. Arrange urgent home care four times a day
C. Arrange urgent respite care in a residential home
D. Admit the son to hospital with his mother
E. Manage the mother at home with care four times a day
EXAM 2 | QUESTIONS 95

19. A 76-year-old Parkinson’s patient was admitted for an elective


inguinal hernia repair. His Parkinson’s regime consisted of
co-careldopa 125mg QDS. He took no other medications and
had no known drug allergies. He had an episode of polymyalgia
rheumatica 3 years previously, but had stopped his steroids more
than 1 year previously, with no relapse in symptoms.
The surgical team were concerned that post-operative nausea may
affect absorption of his Parkinson’s medications and sought your
guidance about which anti-emetic would be the most appropriate to try.
Which of the following would is most appropriate?
A. Domperidone 10mg TDS
B. Haloperidol 3mg OD
C. Metoclopramide 10mg TDS
D. Ondansetron 4mg TDS
E. Prochlorperazine 10mg TDS

20. A 69-year-old woman, who had suffered from Benson’s syndrome


for 7 years, fell and sustained a comminuted fracture of her left
ankle. This was fixed using a periosteal plate, which subsequently
became infected. The plate was removed and the ankle fused.
The Abbey pain scale was used to assess her pain and oral opiate
adjusted according to the pain assessment. She was also treated
with IV antibiotics for radiographically proven osteomyelitis.
Three weeks later both the nursing staff and family found it
impossible to feed her. There were no new focal neurological
abnormalities and her inflammatory markers were now normal.
What is the most likely cause for the onset of her dysphagia?
A. Cerebral infarction
B. Opiate toxicity
C. Progressive dementia
D. Sub-dural haemorrhage
E. Urinary tract infection

21. Nutritional assessment is an important part of a Comprehensive


Geriatric Assessment (CGA). Care home residents are known to
be at particular risk of malnutrition.
Which of the following is most likely to be of value in assessing
nutritional status in a newly-admitted resident to a nursing home?
A. Body Mass Index
B. Malnutrition Universal Screening Tool
C. Mini-nutritional Assessment
D. Quetelet index
E. Serum albumin
96 EXAM 2 | QUESTIONS

22. A 94-year-old woman was being being treated on your ward.


She presented with confusion and a homonymous hemianopia.
CT imaging demonstrated a primary intracerebral tumour with
significant mass effect. She responded well to dexamethasone
with improvement in her neurology. She was independently
mobile on the ward without aids and the occupational therapists
raised no concerns about her ability to cope on discharge.
The cancer multidisciplinary team recommended supportive palliative
care, and estimated a prognosis of 2–3 months.
Which of the following is most appropriate with regards to her
discharge?
A. Discharge without additional support and discuss case with GP
B. Fast track referral for continuing health care
C. Referral for full continuing health care assessment
D. Referral for social services care assessment
E. Rehabilitation in a community hospital

23. A 91-year-old man was admitted with a LRTI. He was initially


treated with IV co-amoxiclav. This was converted to oral
co-amoxiclav at 48hr and he completed a 7-day course. His past
medical history included hypertension, type 2 diabetes mellitus,
and osteoarthritis. He took bendroflumethiazide 2.5mg OD,
amlodipine 5mg OD, and ibuprofen 400mg TDS PRN. In addition
to the co-amoxiclav, he was prescribed paracetamol PRN and
enoxaparin 40mg OD SC for thromboprophylaxis.
He became deconditioned as a result of his acute illness and required a
period of rehabilitation on the ward. On ‘routine’ blood testing 2 weeks
into his admission, it was noted that he had become hyperkalaemic.
Admission 2 Weeks Reference
Na 136mmol/L 142mmol/L 137–144mmol/L
K 5.1mmol/L 5.8mmol/L 3.5–4.9mmol/L
Urea 6.4mmol/L 6.6mmol/L 2.5–7.0mmol/L
Creat 99µmol/L 106µmol/L 60–110µmol/L

Which drug is most likely to be responsible for the


hyperkalaemia?
A. Amlodipine
B. Bendroflumethiazide
C. Co-amoxiclav
D. Enoxaparin
E. Ibuprofen
EXAM 2 | QUESTIONS 97

24. A 76-year-old woman had been treated for psychotic depression


for the last 40 years. She lived alone in a flat with regular
community psychiatric nurse (CPN) input and review by a
consultant psychiatrist. In the last few weeks, her behaviour had
become more erratic and she had stopped letting her regular
CPN into the flat, saying that she did not know who she was. After
several visits, the CPN managed to get the patient to agree to be
admitted to the local psychiatric unit and, given her compliance,
formal section under the Mental Health Act was not deemed
necessary. Her behaviour continued to deteriorate: she would not
get out of bed; she would not engage in conversation; she refused
to take her antidepressant and antipsychotic medication; and she
refused to eat or drink.
What is the most appropriate next course of action regarding her
feeding?
A. Arrange insertion of a feeding NGT
B. Continue to try to get her to eat and drink normally
C. Refer for a percutaneous endoscopic gastrostomy
D. The hospital should seek guidance from the courts
E. Treat her under Section 3 of the Mental Health Act

25. In an 85-year-old man with chronic cardiac insufficiency, secondary to


ischaemic heart disease, on maximum medical treatment, which of the
following is not an indicator of possible need for palliative care?
A. Increasing need for personal care
B. Moving in to a residential home
C. Onset of breathlessness at rest
D. Progressive decline in weight in the last year
E. Two hospital admissions in the last year
98 EXAM 2 | QUESTIONS

26. A 79-year-old woman attended the ED with her husband. She was
usually fit and active, but had exerted herself more than usual
on a strenuous walk that day. They had been for lunch in a local
restaurant, and an hour or so later, her husband noticed that
she was behaving unusually. She seemed disorientated, and was
unable to recall getting up and going on the walk. She did not
know where she was and would apparently forget what she was
told after just a few moments. This was causing her to become
anxious. She had no difficulty recognizing her husband or recalling
events prior to that morning.
What is the most likely diagnosis?
A. Delirium
B. Fugue state
C. Migraine
D. Transient global amnesia
E. Transient ischaemic attack

27. Which of the following statements regarding CGA is not true?


Elderly people admitted to hospital receiving a CGA are:
A. Less likely to be admitted to an institution
B. Less likely to die during the acute admission
C. Less likely to experience functional decline
D. More likely to show benefit on cognitive measures
E. No different from the control group at 12 months’ follow-up

28. A 73-year-old woman with rheumatoid arthritis suffered recurrent


UTIs. Her daughter thought this was due to her mother’s
difficulty with personal care, due to her deformed hands and
restricted shoulder movements.
Which appliance would best address this problem?
A. Femidom
B. Lubidet
C. Radar key
D. Splash sentry
E. Whiz freedom
EXAM 2 | QUESTIONS 99

29. An 83-year-old woman with osteoporosis and deforming


rheumatoid arthritis of both hands and feet was having difficulty
mobilizing following hip surgery. She lived alone in a ground floor
flat and usually had twice daily care.
Which type of walking aid is most likely to be beneficial?
A. Electric wheelchair
B. Four wheeled walking frame
C. Standard gutter frame
D. Standard tripod walking aid
E. Walking trolley

30. An 83-year-old woman had had four falls at home in the last
month. She had bilateral early cataracts and age-related
macular degeneration; her footwear was badly worn, uneven
heels of 4cm height. Her daughter stated that her mother had
become more forgetful over the last year and often forgot to
take her medication (aspirin 75mg daily, simvastatin 40mg
daily, bendroflumethiazide 2.5mg daily, lisinopril 10mg daily,
paracetamol 1g QDS).
What aspect of assessment will have the greatest impact on
onward management of her falls risk?
A. Assessment of cognition
B. Ensuring correct spectacles are worn
C. Measuring lying and standing BP
D. Medication review
E. Provision of suitable footwear
100 EXAM 2 | QUESTIONS

31. The geriatric unit multidisciplinary team was planning the


hospital discharge of a 66-year-old man with Parkinson’s dementia
complicated by frequent falls, excessive day-time somnolence,
unpredictable bouts of aggressive behaviour (day and night),
and orthostatic hypotension (70mmHg) unresponsive to a
combination of fludrocortisone, domperidone, midodrine,
and abdominal compression. At best he could manage his own
personal care, and at worst could not and would resist assistance.
Admission to hospital had occurred 10 weeks earlier, after he fell at
home, sustaining multiple rib fractures and a haemopneumothorax.
The local Parkinson’s specialist had regularly reviewed his medication
(rivastigmine, modafenil, midodrine, fludrocortisone, Sinemet®,
domperidone, mirtazapine, calcium, vitamin D, and alendronate) and
considered these to be optimized.
Prior to this hospital admission he lived alone with support from his
ex-wife and a CPN. He fell regularly and had, in recent months, started
wandering from his home and was losing weight. He had been resistant
to carers coming in to his home. His ex-wife repeatedly stated that the
only appropriate discharge plan was that he went home as he would not
wish to be in a care home with older people. He was assessed as not
having the mental capacity to determine his own care needs or where
these would be best provided. He did not recognize the dangers of
mobilizing without his walking frame, and frequently failed to use this or
threw it at the nursing staff.
What is the most appropriate discharge plan?
A. Home with a live-in carer
B. Home with assistive technology
C. Specialist dementia nursing home
D. Transfer to old age psychiatry ward
E. Transfer to a rehabilitation unit

32. An 84-year-old man presented with falls. His vitamin D level was
17nmol/L (< 25nmol/L demonstrate deficiency, 25–50nmol/L are
suboptimal).
Given that he has adequate dietary calcium intake, which of the
following is the most appropriate treatment?
A. 100 000U IM colecalciferol stat, then 800U PO colecalciferol OD
B. 100 000U IM ergocalciferol stat, then 800U PO colecalciferol OD
C. 100 000U PO colecalciferol stat, then 800U PO colecalciferol OD
D. 250nanograms 1-alfacalcidol daily
E. 800U PO colecalciferol OD without stat dose
EXAM 2 | QUESTIONS 101

33. The daughter of a 74-year-old man with mild cognitive


impairment (MMSE 21/30) was concerned that he had lost 7.5%
of his body weight over the last 6 months. He lived alone with his
daughter visiting every week to take him out for his household
shopping. He was a life-long non-smoker and retired foundry
worker. He did not understand why his daughter was concerned
as he had no symptoms.
Investigations
Haemoglobin 12.9g/dL (13–18)
WCC 8.5 × 109/L (4–11)
Sodium 141mmol/L (136–146mmol/L)
Potassium 4.6mmol/L (3.5–5.3mmol/L)
Urea 4.3mmol/L (2.1–7.6mmol/L)
Creatinine 96mmol/L (51–107mmol/L)
CRP 22mg/L (<10)
Serum corrected calcium 2.40mmol/L (2.2–2.6)
Serum alkaline phosphatase 66U/L (45–105)
Serum albumin 37g/L (37–49)
Serum lactate dehydrogenase 100U/L (10–250)
Random blood sugar 5.2mmol/L (3–6)
Plasma free T4 20pmol/L (10–22)

Urinalysis, faecal occult bloods, and chest X-ray were normal.


What is the next most appropriate step in his management?
A. CT—chest, abdomen, and pelvis
B. Measure serum tumour markers
C. Nutritional assessment by dietician
D. Three months watchful waiting
E. Upper and lower bowel endoscopy

34. A previously independent 59-year-old man presented 5hr after a


sudden onset left-sided hemiparesis. He was found to be in atrial
fibrillation. He had never previously required medical attention.
An MRI of his head was suggestive of an acute right middle cerebral
artery territory stroke with an estimated volume of 150cm3 involved.
He was drowsy with a GCS of 12. His National Institutes of Health
Stroke Scale (NIHSS) score was 15. Other systemic examination was
unremarkable.
Which of the following is the most appropriate intervention?
A. Angiography and clot retrieval
B. Cardioversion
C. Decompressive craniectomy
D. Therapeutic low molecular weight heparin
E. Thrombolysis
102 EXAM 2 | QUESTIONS

35. An 83-year-old man was admitted to a community rehabilitation


ward following a prolonged hospital stay with pneumonia. He
was immobile in bed for 2 weeks and had become deconditioned.
During this period he developed bilateral grade 2 pressure sores
on his heels. Other pressure areas were intact. He had normal
capillary refill and palpable pedal pulses. His ankle-brachial
pressure indices (ABPI) were 0.8 bilaterally.
He demonstrated good potential to rehabilitate on the ward and on
discharge to the rehabilitation facility was able to transfer independently
and mobilize a short distance with minimal assistance from one person.
He was eating at least three-quarters of all meals and his weight was
stable.
Which of the following is the most important in managing his
pressure ulcers?
A. Change to an alternating pressure mattress
B. Encourage him to sit out and practise mobilizing
C. Enrol him on a high protein diet
D. Referral for arterial duplex ultrasound studies
E. Referral to community tissue viability team

36. A 75-year-old man was found by his wife having apparently fallen
off his commode in the bedroom. When she found him he was
more confused than usual and appeared to be in severe pain.
In the ED he was found to have a fractured right clavicle, flail
right chest, right haemothorax, and fractured left humerus.
Neurological examination was consistent with Parkinson’s disease
with no other neurological deficit.
His wife was distraught that he should have sustained such severe
injuries in the short time that she had left him to answer the telephone
downstairs. Parkinson’s disease had been diagnosed 4 years earlier.
He had been changed from monotherapy with a dopamine agonist to
levodopa 8 months earlier because of developing cognitive impairment.
In the last 4 months, he had started on an acetylcholinesterase inhibitor
and the dose gradually titrated up. He was not on any other medication.
He did not normally suffer any motor fluctuations.
What is the most likely cause for his fall?
A. Convulsion
B. Elder abuse
C. Orthostatic hypotension
D. Dyskinesias
E. Transient ischaemic attack
EXAM 2 | QUESTIONS 103

37. The frailty index of accumulated deficits shows a clear


relationship between higher scores and mortality.
Which of the following is least likely to explain the differential
mortality between men and women?
A. A higher accumulation of deficits in men
B. A lower risk of mortality in women
C. Female ‘fertility-frailty pleiotropy’
D. Male ‘fitness-frailty pleiotropy’
E. The Gompertz–Makeham model for mortality risk

38. An 89-year-old man required emergency surgery for a


strangulated inguinal hernia.
Which of the following conditions, treated with warfarin, does not
require bridging treatment with therapeutic dose low molecular
weight heparin peri-operatively?
A. AF and previous stroke
B. Bi-leaflet metallic aortic valve replacement
C. Deep vein thrombosis 2 months previously
D. Metallic mitral valve replacement
E. Pulmonary embolism 1 month previously

39. The manager of a residential home requested a medical review


of a 71-year-old female resident, with vascular dementia, who was
constantly scratching her hands. There was a diffuse scaly rash
on both palms and soles, with several vesicles on her palms and
scratch marks. There was no rash elsewhere on her body. She
had lived in the residential home for 3 years and not been noted
to have a rash before. She required prompting, but no assistance
with her personal care and was independently mobile. There
were no other residents or staff with a similar rash.
What is the most likely cause of her rash?
A. Atopic eczema
B. Contact dermatitis
C. Pompholyx
D. Psoriasis
E. Scabies
104 EXAM 2 | QUESTIONS

40. A 74-year-old man presented with seizures. He had been advised


to inform the driver and vehicle licensing authority (DVLA) and to
refrain from driving for the time being.
On review in the outpatient department, it became apparent that he
had not followed this advice. He continued to drive as he had no other
way of reaching the shops. He did not think it was ‘the government’s
business’ and planned on continuing to drive.
Which of the following is the most appropriate course of action?
A. Contact the DVLA without further discussion with the patient
B. Contact the patient’s GP and ask them to explore further
C. Document your advice carefully and take no further action
D. Explain that you will need to contact the DVLA yourself and do so
E. Provide the patient with written documentation of your discussion

41. An 82-year-old woman with metastatic breast cancer had been


having increasing difficulty managing her tablets. She had been
taking long-acting morphine sulphate (MST) 45mg BD and her
symptoms had been well controlled on this regime. She had no
difficulty with liquid medications.
Which of the following would be most appropriate to replace the MST?
A. 10micrograms/hr buprenorphine patch
B. 30mg/24hr diamorphine SC
C. 50mg diclofenac TDS
D. 25micrograms/hr fentanyl patch
E. 15mg oramorph 4-hourly

42. A 73-year-old man was brought to the ED after his neighbours


had called for an ambulance as they were concerned that he
was making an unusual amount of noise in his house. He lived
alone with his dog. The ambulance crew reported that the
house was squalid, but the dog appeared well cared for. In ED,
he was unkempt, dehydrated, restless, refused to be examined,
and frequently asked to walk to the toilet. He was unsteady on
his feet, but could walk unaided and would not let anyone into
the toilet with him. His only next of kin was a niece who lived in
the same village and saw him regularly. He refused to take any
medication, but would accept drinks when offered.
What is the most appropriate next step in his management?
A. Ask him what is bothering him
B. Contact his niece for further information
C. Dipstick test his urine before starting antibiotics
D. Prescribe an antipsychotic to reduce his agitation
E. Start antibiotics for presumed UTI
EXAM 2 | QUESTIONS 105

43. A 93-year-old woman presented with three episodes of collapse,


with transient loss of consciousness. She denied any chest pain,
breathlessness, or cough. She had suffered some dysuria recently.
Her stools had not changed in colour or frequency. Her past
medical history included oesophageal adenocarcinoma. This had
initially been treated with brachytherapy, but the management
was now supportive only. Her original prognosis was 6 months.
She was dysphagic to solids, but could manage fluids and purées.
On examination, she had an elevated JVP, a heart rate of 105beats/min,
with regular normal heart sounds. She had a lying BP of 125/80mmHg,
sitting 110/70mmHg. There was mild oedema of her ankles, worse
on the left. Her abdomen was soft and non-tender. There were a few
scattered crackles in her chest.
Admission ECG and chest X-ray were unremarkable.
Initial blood tests
Na 137mmol/L (137–144) Hb 12.5g/dL (11.5–16.5) pO2 8.7kPa (11.3–12.6)
K 4.5mmol/L (3.5–4.9) WCC 12.5╇ × 109/L (4–11) pCO2 3.2kPa (4.7–6.0)
Urea 10.2mmol/L (2.5–7.0) Platelets 230╇ × 109/L (150–400) H+ 40nmol/L (35–45)
Cr 110µmol/L (60–110) CRP 47mg/L (<10) Base –1mmol/L (±2)
excess
(BE)

Initial treatment was with IV fluids and antibiotics for presumed


urosepsis.
Next day on the ward she had a further collapse during physiotherapy
and responded to a further IV fluid bolus. Her observations prior to
therapy had been normal.
Which of the following is the most appropriate next investigation?
A. 24-hr heart recording
B. Carotid sinus massage
C. CT pulmonary angiography
D. Septic screen
E. Synacthen test
106 EXAM 2 | QUESTIONS

44. An 86-year-old woman was failing to cope at home after her


93-year-old diabetic husband was admitted to hospital 3 days
previously, with an ischaemic foot. They had never been separated
in 63 years of marriage. She was neither eating nor drinking, and
was neglecting her personal hygiene. She had type 2 diabetes,
which was normally well controlled on metformin 500mg BD. Her
daughters were concerned that she had not been managing her
medication for several months. Physical examination was entirely
normal. She was independently mobile, but failed to initiate tasks
independently. FBC, U&Es, CRP, and urinalysis were all normal.
When questioned, she knew her husband was in hospital.
What is the most likely cause for this presentation?
A. Delirium
B. Dementia
C. Depression
D. Drug overdose
E. Grief reaction

45. An 84-year-old woman was admitted with a fractured neck of


femur. She underwent emergency hemiarthroplasty 72hr after
admission. Her next of kin enquired about her prognosis.
Which of the following statements is incorrect?
A. At 12 months, 60% are limited at either feeding, dressing, or toileting
B. Delirium post-operatively is not associated with increased mortality
C. One-year mortality is over 50% in those admitted from institutional care
D. Surgery within 48hr is associated with a shorter length of stay
E. The 1-year mortality risk following fracture is about 30%
EXAM 2 | QUESTIONS 107

46. An 82-year-old female presented with a fractured shaft of femur.


Her leg had been aching for several days. She had been turning to
let her husband pass her in the corridor when she heard a crack
and fell to the floor.
She had been on steroids for polymyalgia rheumatica for many years.
6 years previously she had sustained a Colles fracture, and was started
on calcium and vitamin D supplementation, and weekly alendronate
70mg. She also took omeprazole 20mg OD. She drank 30U of alcohol a
week. Her X-ray is shown in Fig. 2.1.

Fig. 2.1╇Patient’s X-ray.

Which of the following is most likely to have contributed to her


risk for this fracture?
A. Excessive alcohol consumption
B. Long-term steroid use for polymyalgia rheumatic
C. Parental history of femoral fracture
D. The use of alendronate
E. The use of omeprazole
108 EXAM 2 | QUESTIONS

47. An 84-year-old man was being treated for severe depression.


His oral intake over the past 2 weeks had been minimal. He
was deemed to have capacity and initially refused supplemental
nasogastric feeding. After electroconvulsive therapy, his mental
state improved. He had a degree of oropharyngeal dysphagia,
and the speech and language therapists agreed that it would be
preferable to meet his nutritional needs via the nasogastric route
to give his swallow time to improve. He now agreed to this.
Blood tests showed normal FBC, LFTs, sodium, potassium, and calcium.
The phosphate was 0.4mmol/L (0.8–1.4)
Which of the following would be least appropriate?
A. Correct hypophosphataemia before starting feed
B. Daily multivitamin supplementation
C. IV vitamin B prior to starting feed
D. Oral thiamine 300mg daily
E. Start feed at 50% of energy requirements and titrate

48. An 84-year-old man was being seen in the day rehabilitation


unit following a series of falls. Medical review unmasked subtly
impaired co-ordination of his left upper limb and to a lesser
extent his left leg. A clinical diagnosis of posterior circulation
stroke was made and later confirmed with neuroimaging.
Which of the following assessment tools is least appropriate to
monitor his progress?
A. Berg Balance Scale
B. NIH Stroke Scale (NIHSS) score
C. Rivermead score
D. Tinnetti score
E. Timed Up and Go Test

49. Effective team working is a crucial aspect of effective


rehabilitation services.
Which of the following service models is likely to be most
efficient?
A. Inter-disciplinary
B. Intra-professional
C. Multi-disciplinary
D. Trans-disciplinary
E. Uni-disciplinary
EXAM 2 | QUESTIONS 109

50. A 79-year-old woman with Alzheimer’s dementia had recovered


from a delirium caused by pain from osteoporotic wedge fractures
of the third and fourth lumbar vertebrae. She was extremely
variable in her mobility, needed prompting to eat and drink, was
unpredictable with her continence, and needed assistance with
personal care. She lacked the capacity to determine her own care
needs, was considered unsafe to continue living alone with care
four times a day, and was waiting for a care home placement.
On the fourth week of her hospital stay, despite receiving
thromboprophylaxis with low molecular weight heparin (LMWH),
she developed a left ileofemoral deep vein thrombosis. Systemic
examination and laboratory investigations excluded secondary
causes of osteoporosis and overt malignancy.
What is the most appropriate maintenance treatment for her
deep vein thrombosis?
A. Compression hosiery and life-long dabigatran
B. Compression hosiery and life-long warfarin
C. Compression hosiery and 6 months warfarin
D. Life-long therapeutic dose of LMWH
E. Six months therapeutic dose of LMWH

51. A 79-year-old woman with type 2 diabetes took metformin 500mg


TDS, aspirin 75mg OD, and simvastatin 40mg OD. Initially, she
had been on gliclazide monotherapy, but this was switched owing
to poor hypoglycaemia awareness. She was overweight, with a
BMI of 30kg/m2.
Her blood tests showed an HbA1c of 75mmol/mmol (target 48–58mmol/
mmol).
Which is the most appropriate alteration to her diabetes
treatment?
A. Exenatide 5micrograms SC BD
B. Gliclazide 40mg BD
C. Insulin glargine OD
D. Insulin Novomix 30 BD
E. Sitagliptin 100mg OD
110 EXAM 2 | QUESTIONS

52. An 84-year-old male patient in residential care, with vascular


dementia (MMSE 7/30), had been concerning his GP and wife. He
had become increasingly withdrawn and disengaged with eating.
He showed no interest in food and just held it in his mouth. He
did not cough or appear to aspirate when swallowing. The family
felt that he was losing significant amounts of weight.
The care home staff reported that he had become constipated and that
they had noticed some bright red blood on the paper after he had had
his bowels open.
Physical examination was unremarkable. He was observed to swallow
safely. There was a loaded rectum on digital examination, but no
obvious masses or bleeding. Blood tests that had been arranged showed
a haemoglobin level of 12g/dL (13–18) with an MCV of 82fL (80–96).
Which of the following is the most appropriate course of action?
A. Old age psychiatry referral
B. Refer for nasogastric feeding
C. Refer for percutaneous endoscopic gastrostomy
D. Speech and language therapy review
E. Suppositories and oral laxatives

53. A 72-year-old female reported that she often wet herself and
had taken to wearing pads. This had been happening for the last
5 years and had meant that she now declined social engagements.
She was more likely to leak urine if she strained. She had tried
toileting even if she felt she did not need to, but this had not
stopped her wetting herself. Her bowels were opened once each
day. She was on no medication.
Which of the following statements is most appropriate regarding
the use of pelvic floor exercises in the management of her
incontinence?
A. Pelvic floor exercises are best done in the privacy of her home
B. Pelvic floor exercises have no role to play
C. Pelvic floor strength and endurance is best measured by digital vaginal examination
D. She should be given a leaflet describing pelvic floor exercises
E. She should be invited to a pelvic floor exercise group
EXAM 2 | QUESTIONS 111

54. An 84-year-old female attending the falls assessment clinic was


found to have a resting BP of 166/104mmHg with no postural
drop. Cardiovascular examination was otherwise normal as was
her resting electrocardiogram. Her recurrent falls were attributed
to a combination of age-related macular degeneration, bilateral
early cataracts and multiple environmental hazards within the
home. Her BMI was 24.6 (18.5–24.9).
What is the most appropriate initial management of her
hypertension?
A. Advise her to lose weight
B. Arrange ambulatory BP monitoring
C. Recommend she buy a home BP monitor
D. Repeat her BP at her next clinic visit
E. Start a calcium channel blocker

55. An 82-year-old man with moderate to severe Alzheimer’s


dementia was admitted with a UTI. He was agitated, distressed,
and a risk to both himself and other patients. He required three
doses of IM 0.5mg haloperidol over 48hr due to his aggressive
behaviour.
He was taking the antibiotics for his UTI, and was eating and drinking.
He was taking his prescribed analgesia and did not appear to be in
pain. He was being nursed in a well-lit side room by the same nursing
staff who were continually trying to orientate him.
Which of the following is the most appropriate course of action?
A. Continue PRN haloperidol alone
B. Initiate 0.5mg risperidone BD
C. Initiate 10mg olanzapine OD
D. Initiate 100mg quetiapine BD
E. Switch to PRN lorazepam
112 EXAM 2 | QUESTIONS

56. A previously healthy and independent widow was admitted to


hospital with acute confusion after being treated by her GP for a
presumed UTI 5 days earlier. On admission, she was aggressive,
paranoid, and thought she was being pursued by vampires. Her
FBC, U&E, CRP, calcium, LFTs, chest X-ray, and urine cultures
were all normal. She was on no medication. A week later she was
calmer, and waiting for the man of her dreams to come and take
her away to their love-nest. At this time, her MMSE was 13/30
and her CT-head scan showed a minor degree of cortical atrophy.
When moved, she complained of pain in her right hip; X-ray
showed severe osteoarthritic changes and no fracture.
Three weeks into her admission she needed emergency abdominal
surgery for a strangulated incisional hernia. Three weeks post-surgery
she had a persistent delusion regarding her ‘lover’ and refused to get
out of bed, as this was where he wanted her to be. Her MMSE was
now 17/30.

09.00 hours plasma cortisol 700nmol/L (200–700)


TSH 0.3µIU/mL (0.4–3.0)
Free T4 20pmol/L (10–22)

What is the most likely diagnosis?


A. Cushing’s syndrome
B. Delirium
C. Dementia
D. Psychotic depression
E. Thyrotoxicosis
EXAM 2 | QUESTIONS 113

57. An 89-year-old woman with post-stroke epilepsy presented in


status and required IV lorazepam to terminate her seizures. She
had been admitted on three occasions with seizures over the past
4 months.
Her usual treatment was sodium valproate 800mg BD. This had been
titrated up over her previous admissions from a starting dose of 300mg
OD, with little apparent improvement. She had no other past medical
history and other medications consisted of aspirin 75mg OD and
simvastatin 40mg ON.
The seizures terminated and the next day the patient was alert and
felt well. A CT head scan had shown no new change, and inflammatory
markers and electrolytes were within normal limits. She remained
in hospital for physiotherapy, but became increasingly drowsy and
delirious. The nursing staff had witnessed no overt seizure activity. 1
week later she was found in her chair with a GCS of 11. There was no
focal neurology and bedside observations were stable. A repeat CT head
was unchanged.
Which of the following is the most appropriate course of action?
A. Add in levetiracetam 250mg BD
B. Increase dose of sodium valproate
C. Load with IV phenytoin
D. Reduce dose of sodium valproate
E. Supportive care only
114 EXAM 2 | QUESTIONS

58. An 82-year-old woman presented to the falls clinic having had


three falls in the past few months.
She attributed these to simple trips and overbalancing. She had no
significant past medical history. Her only medication was temazepam
10mg ON for insomnia.
On examination, she was found to be in sinus rhythm at 70beats/
min. She had a normal BP, with no orthostatic drop. Respiratory and
abdominal examinations were unremarkable. Fundoscopy was not
possible owing to bilateral cataracts. The remainder of the neurological
examination was within normal limits. A Hallpike manoeuvre was
negative.
An ECG confirmed sinus rhythm and baseline bloods showed:
• Normal renal function.
• Normal FBC.
• TSH 2.5mU/L (0.4–5.0).
• Vitamin D 80nmol/L (60–105).
Which of the following interventions is unlikely to further reduce
her falls risks?
A. Cataract surgery
B. Gradual withdrawal of benzodiazepines
C. Otago strength and balance exercises
D. Supplemental vitamin D
E. Tai Chi lessons
EXAM 2 | QUESTIONS 115

59. An 84-year-old woman was diagnosed as having acute coronary


syndrome after developing central chest pain radiating to both
arms, with associated nausea and dizziness. There were no ECG
changes and her 12-hr troponin I was 2.6µg/L (<0.1). Her other
blood tests were all normal. There was a spiculated mass in
the right mid-zone on her chest X-ray, which had been noted
6 months earlier and had been assumed to be a primary lung
cancer (she had declined further investigations).
She had a 5-year history of Parkinson’s disease and for the last
18 months this had been associated with stable non-bothersome
hallucinations. Her cognition was normal, MMSE 28/30. Her usual
medication was alendronic acid 70mg weekly, daily calcium and vitamin
D supplements, and co-beneldopa 125 TDS. She needed repeated
doses of diamorphine for her cardiac pain. She was commenced on a
beta-blocker, angiotensin-converting enzyme inhibitor (ACE-I), aspirin,
clopidogrel, and a statin.
Over the next 2 days she complained that her hallucinations had
increased in frequency and were more bothersome. Physical
examination remained unchanged.
What is the most likely explanation for the change in her
hallucinations?
A. Advancing Parkinson’s disease
B. Cerebral metastases
C. Charles Bonnet Syndrome
D. Dementia with Lewy bodies
E. Delirium due to drug toxicity

60. An 84-year-old man with benign prostatic hyperplasia was


attending the falls clinic. He presented with postural hypotension
and had been taking tamsulosin M/R 400micrograms OD. This had
been stopped.
3months later, he reported that he no longer became light-headed on
standing, but had increasing difficulty passing urine and a poor stream
quality.
Which of the following would be the most appropriate medication
to suggest?
A. Doxazosin 2mg ON
B. Finasteride 5mg OD
C. Oxybutinin M/R 5mg OD
D. Solifenacin 5mg OD
E. Terazocin 1mg ON
116 EXAM 2 | QUESTIONS

61. An 84-year-old woman had a history of incomplete bladder


emptying thought to relate to previous complications around
the time of child birth. She self-catheterized twice daily and was
able to void to some extent in between. Her residual volume in
the morning was usually about 200mL and about 400mL in the
evening. She had no other relevant past medical history and took
no medications.
She had recently started being troubled by incontinence. She described
episodes where she leaked urine on moving. She had no problems in the
morning, but the situation deteriorated during the course of the day.
She did not have difficulties overnight.
Which of the following is the most appropriate intervention?
A. Adjust timings of catheterization
B. Initiate anticholinergic
C. Provide supply of continence pads
D. Refer for suprapubic catheter
E. Treat with an alpha blocker

62. A 92-year-old woman with COPD and hypertension was managed


with salbutamol, 2 puffs PRN, aspirin 75mg OD, and amlodipine
5mg OD.
She had had a recent exacerbation, during which her symptoms were
not well controlled and she had been using her reliever (salbutamol)
inhaler in excess of five times a day. Her baseline FEV1 was 70% of
predicted.
Which of the following would be the most appropriate alteration
to her usual treatment?
A. Current therapy with standby oral corticosteroids
B. Long-acting beta agonist and corticosteroid inhaler
C. Starting a long-acting beta agonist
D. Starting an inhaled corticosteroid
E. Starting mucolytic therapy
EXAM 2 | QUESTIONS 117

63. A 97-year-old woman was admitted to hospital from a local


nursing home with a fever. She was found to have a left-sided
pneumonia. She was started on IV antibiotics.
The ward staff were concerned about the patient. They noticed multiple
bruises of various ages. They had also identified a grade 2 pressure sore
and felt that the patient was cachectic. They were worried she was not
being well looked after in the nursing home. On review of the imaging,
left-sided rib fractures were noticed. These could have contributed to
her hypostatic pneumonia.
What is the most appropriate initial course of action?
A. Contact the patient’s general practitioner
B. Contact the patient’s next of kin
C. Discuss with the patient and assess her capacity
D. Initiate a safeguarding notification with social services
E. Seek further collateral history from the care home

64. An 85-year-old man presented being drowsy with a left


hemiparesis. Neuroimaging demonstrated a large primary
glioblastoma. He had diamorphine, glycopyrrhonium, midazolam,
and cyclizine prescribed on a PRN basis.
Over the next 24hr he became increasingly agitated and distressed. He
was vomiting and complaining of a severe headache that was worse lying
down. His hemiparesis had worsened. He had settled slightly following a
small dose of diamorphine.
Which of the following is the next most appropriate add-on
treatment?
A. SC dexamethasone infusion
B. SC diamorphine infusion
C. SC haloperidol infusion
D. SC midazolam infusion
E. SC cyclizine infusion
118 EXAM 2 | QUESTIONS

65. An 89-year-old man was recovering from a stroke. He has a


persistent left hemiparesis, left-sided neglect and ‘at-risk’ swallow.
Previously, he was living with his wife in their own home. The
house was registered solely in his name. They had no other
significant assets or income. The patient and multidisciplinary
team agreed that, because of his ongoing nursing and care needs,
permanent placement in a nursing home should be pursued.
A continuing care checklist was undertaken and he had scored 1A, 2Bs,
and 8Cs.
Which of the following funding options is most likely to apply?
A. Continuing health care funded indefinitely
B. Continuing health care funded with review at 3 months
C. Fully self-funded
D. Fully social services funded
E. Self-funded with registered nursing care contribution

66. An 82-year-old woman presented with a history of self-limiting


left-sided facial droop and associated mild left upper and lower
limb weakness.
She had started feeling non-specifically unwell earlier in the day. Her
daughter then noticed that her speech was slurred and commented
on the facial asymmetry. Over the next half an hour, the patient
then noticed her left leg became numb and started dragging while
walking and her left arm felt heavy. The symptoms improved over
the subsequent half an hour. When she arrived in the ED she had a
persistent unilateral headache with associated nausea and photophobia.
On examination she was apyrexial. Kernig’s sign was negative. Power,
tone, co-ordination, sensation, and reflexes were intact throughout.
Blood tests were unremarkable.
Which of the following is the most likely diagnosis?
A. Encephalitis
B. Glioblastoma multiforme
C. Migraine with aura
D. Right middle-cerebral artery territory stroke
E. Transient ischaemic attack
EXAM 2 | QUESTIONS 119

67. An elderly couple, both with dementia, received care in their


one bedroom cottage four times a day. The family, who lived
in the same village, were concerned that this level of care was
inadequate as the couple had started to wander out of the house
at night. The couple had always expressed the wish to remain in
their own home and not move into institutional care.
What is the most appropriate next step in their management?
A. Arrange a live-in carer
B. GPS tracker phone
C. Move to a residential home
D. Property exit sensor
E. Wrist actigraph

68. An 82-year-old man with inoperable metastatic bowel cancer


was admitted to hospital with postural hypotension secondary
to adrenal insufficiency caused by adrenal metastases. He had
lost more than 15% of his body weight in the last month, despite
having a good appetite. The postural hypotension responded to
treatment with fludrocortisone, but he remained too feeble to
mobilize. The oncologist recommended symptomatic palliative
care. His community matron phoned the ward to enquire about
what the discharge plans were likely to be. She informed the
multidisciplinary team that he was the main carer for his wife
who had dementia. Their son had moved in to live with his
mother whilst his father was in hospital as she could not manage
with her usual three-times-a-day care package. The family were
considering a residential home placement for their mother. The
father had previously expressed a wish to die at home
Which of the following would be the most appropriate course of
action?
A. Arrange care home placement for him and his wife
B. Discharge home with palliative care support
C. Fast-track discharge to a nursing home
D. Transfer to the local hospice
E. Transfer to the oncology ward
120 EXAM 2 | QUESTIONS

69. A 70-year-old woman had a 3-month history of non-bloody, watery


diarrhoea. This occurred day and night, and was associated with
urgency and occasional incontinence. She also complained of
lower abdominal pain, fatigue, and mild weight loss of 3kg. She
was on no medication.
Which of the following tests is most likely to confirm a diagnosis
of microscopic colitis?
A. Colonic biopsies
B. Colonoscopy
C. Faecal elastase
D. Sigmoidoscopy
E. Stool culture

70. Which of the following best describes osteoporosis?


A. Bone density of 1.5 SDs below peak bone mass
B. Condition of inadequate bone mineralization
C. Fracture occurring as a result of low bone mineral density
D. Imbalance of bone resorption and bone formation
E. State in which osteoblastic activity exceeds osteoclastic

71. An 84-year-old woman had increasing difficulty managing at


home. She struggled to mobilize and reported that, once she
was standing, she could manage with her rollator frame, but had
increasing difficulty in rising from her chair.
Clinical examination confirmed a degree of proximal muscle weakness.
She had impaired times on a TUG test. Blood tests, including vitamin D,
TSH, and creatine kinase (CK) were unremarkable.
What is the most appropriate intervention at this stage?
A. Admit for physiotherapy
B. Arrange a home care package
C. Check chair heights and styles at home
D. Recommend an electric riser-recliner chair
E. Suggest residential home placement
EXAM 2 | QUESTIONS 121

72. An 89-year-old man was admitted with a UTI. He had been living
alone without social support. The paramedics reported that
his house was unkempt. The only food appeared to be cans of
beans. His hospital records showed that he had lost 15kg since
his last attendance 3 years previously. He now weighed 62kg. His
BMI was calculated as 21kg/m2. He was tachycardic at 110beats/
min, with a BP of 170/90. His chest was clear. There was mild
suprapubic tenderness, and a urine dip was positive for nitrites
and leucocytes. Urine cultures were sent.
Blood tests
CRP 240mg/L (<10)
Hb 9.1g/dL (13–18)
MCV 105fL (80–96)
Platelets 267 × 109/L (150–400)
WCC 18 × 109/L (4–11)
Na 137mmol/L (137–144)
K 4.4mmol/L (3.5–4.9)
Urea 6.1mmol/L (2.5–7.0)
Cr 50µmol/L (60–110)

Which of the following prescriptions is least appropriate?


A. Amlodipine 5mg OD
B. Enoxaparin 40mg OD SC
C. Gentamicin 180mg OD
D. Pabrinex: 1 pair of vials OD for 5 days
E. Paracetamol 1g QDS

73. Which of the following is not an effect of the renin-angiotensin system


following a fall in circulating volume?
A. Increased aldosterone secretion
B. Increased atrial-natriuretic peptide secretion
C. Increased systemic vascular resistance
D. Increased vasopressin release
E. Stimulation of thirst centres

74. Which of the following is not thought to be associated with an


accelerated ageing process?
A. Chronic infection
B. Free radical damage
C. Maternal calorific restriction
D. Reduced insulin-like growth factor (IGF)
E. Telomere shortening
122 EXAM 2 | QUESTIONS

75. An 84-year-old woman was confirmed to have osteoporosis


following a DEXA scan. She had a past medical history of epilepsy,
hypothyroidism, polymyalgia rheumatica, depression, and reflux
oesophagitis.
She was currently taking levothyroxine 150micrograms OD, phenytoin
300mg OD, sertraline 100mg OD, prednisolone 7mg OD, tramadol 50mg
QDS, and omeprazole 40mg OD.
Blood tests
Showed a normal renal profile and FBC.
• TSH: 0.01mU/L (0.4–5.0).
• T4: 25.2pmol/L (10–22).
Which of the following is least likely to have contributed to her
osteoporosis?
A. Levothyroxine
B. Omeprazole
C. Phenytoin
D. Prednisolone
E. Tramadol
EXAM 2 | QUESTIONS 123

76. A 96-year-old man was found by his daughter at home. She


had visited without prior arrangement. He had taken 72g of
paracetamol 4hr previously. He explained to her that he had had
enough of living and wished to die. She called an ambulance.
In the ED, he again maintained that the overdose was intentional. He
had been planning it for several months and had made appropriate
arrangements regarding his last will and testimony. He had left video
communications for friends and family. He regretted the fact that
his daughter found him, and the distress this had caused her, but was
adamant he wished to die. He appeared calm and rational.
He was reviewed by the on-call psychiatry team who felt that he was
neither depressed nor cognitively impaired. There was no evidence of
undue pressure or coercion from family. They did not feel he had an
acute mental illness.
Laboratory tests show that paracetamol levels were above the level at
which treatment is recommended. The prothrombin time was double
the normal range. The patient agreed to hospitalization, but refused
treatment with n-acetylcysteine (n-AC) and said that he did not wish to
have any active treatment. He understood this might result in his death,
but he still wished to die. A do-not-attempt-resuscitation order was
completed.
What is the most appropriate course of action at this point?
A. Admit patient for ongoing symptom control
B. Admit to high dependency area for close urine output and central venous pressure (CVP)
monitoring
C. Instigate treatment with n-AC under the Mental Capacity Act
D. Instigate treatment with oral methionine
E. Section the patient under section 5.2 of the Mental Health Act and initiate treatment
with n-AC

77. A Clinical Commissioning Group wanted to enhance its


intermediate care service by developing an admissions-avoidance
pathway from home to rehabilitation beds in a local
residential home.
What is of primary importance in designing such a service?
A. Defining the medical staffing arrangements
B. Determining appropriate therapy skill mix
C. Ensuring all patients have a clear diagnosis
D. Ensuring interdisciplinary team working
E. Ensuring regular clinical governance meetings
124 EXAM 2 | QUESTIONS

78. An 83-year-old man was admitted following a fall. He had a past


medical history of hypothyroidism, diabetes mellitus, stable angina,
and osteoarthritis. His current medications were: levothyroxine
125micrograms OD, metformin 500mg BD, aspirin 75mg OD,
simvastatin 40mg OD, and co-codamol 8/500 PRN (max. 8 tablets/day).
He described an increasing frequency of falls over the past few weeks.
They occurred at all times of day. He was rather vague about the exact
mechanism. The most recent fall happened while he was carrying an
empty cup through to the kitchen. He did not think that he was turning
at the time, and was uncertain whether he lost consciousness or not.
He had not experienced recent chest pain and had never had any
palpitations. He usually took care when moving from a lying to standing
position, and made sure to steady himself before setting off.
On examination, bruising on his forehead was evident. His chest was clear
and abdominal examination was unremarkable. His pulse was 55beats/min
regular, with a sitting BP of 125/79 and standing BP of 110/72. His heart
sounds were normal and there was no evidence of peripheral oedema.
His ECG rhythm strip is shown in Fig. 2.2.

Fig. 2.2.╇ ECG rhythm strip.


Blood tests
Na 132mmol/L (137–144)
K 5.3mmol/L (3.5–4.9)
Ur 4.5mmol/L (2.5–7.0)
Cr 65µmol/L (60–110)
WCC 7.2 × 109/L (4–11)
Hb 13g/dL (13–18)
MCV 80fL (80–96)
Platelets 165 × 109/L (150–400)
CRP 17mg/L (<10)
TSH 1.02mU/L (0.4–5.0)

Which of the following would be the most appropriate next


investigation?
A. Cardiac Holter monitoring
B. Carotid sinus massage
C. Echocardiograph
D. Synacthen test
E. Tilt table testing
EXAM 2 | QUESTIONS 125

79. A 73-year-old woman was recovering on the rehabilitation unit


following a left middle cerebral artery territory ischaemic stroke.
At presentation she had a right hemiplegia with power 3/5 in the
right upper and lower limbs. She had improved such that power
was 4+/5 in the right lower limb and 3/5 in the right upper limb.
Which of the following is the most appropriate walking aid for her
to use?
A. A four leg gutter frame
B. A four leg, no wheel (Zimmer) frame
C. A four leg, two wheels (Rollator) frame
D. A four-point (quad) stick
E. A standard walking stick

80. A 73-year-old man had been referred by his GP owing to concerns


regarding his deteriorating cognitive function. The patient had
little insight into his difficulties; however, his wife was able to
describe how he had become increasingly forgetful over the
past 9 months. She was prompted to see the GP when he got
lost walking back from the local newsagents. She was worried
about how she could continue to support him at home. His only
previous medical history was of beta-blocker-induced syncope.
Cardiovascular, respiratory, abdominal, and neurological examinations
were unremarkable. An Addenbrooke’s Cognitive Examination score
(ACE) was 65/100, with a corresponding Mini-Mental State Examination
(MMSE) score of 17/30. This was comparable with the score of 18/30
recorded by his GP in the referral letter. The patient scored 2/15 on a
Geriatric Depression Scale (GDS).
Baseline blood tests were unremarkable. An ECG confirmed sinus
rhythm with first-degree heart block and a left bundle branch block
pattern. A CT head scan showed mild atrophy with no evidence of
cerebrovascular disease.
What would be the most appropriate agent to prescribe at
this stage?
A. Donepezil 5mg OD
B. Memantine 5mg OD
C. Mirtazapine 15mg OD
D. Risperidone 500micrograms BD
E. Rivastigmine 1.5mg BD
126 EXAM 2 | QUESTIONS

81. A 69-year-old woman was seen in outpatient clinic because of


recent confusion. She and her family had first noticed symptoms
3 weeks previously, when her speech had become more hesitant
and she had difficulty finding some words. Her condition had
gradually deteriorated, being worse on some days than on others.
Over the last couple of days she had also noticed some tingling in
her right hand.
She had a past medical history of type 2 diabetes mellitus, hypertension,
and rheumatoid arthritis. She was taking aspirin 75mg OD, lisinopril
2.5mg OD, folic acid 5mg weekly, and methotrexate 5mg weekly. She
was a non-smoker.
Examination confirmed a moderate, mixed, predominantly expressive
dysphasia. Power was subtly reduced in the right upper limb. Owing to
mild cataracts, fundoscopy was not possible.
Blood tests
D-dimer Normal
FBC 10g/dL (11.5–16.5)
WCC 7.2 × 109/L (4–11)
MCV 78fL (80–96)
Platelets 352 × 109/L (150–400)

Which of the following is the most likely diagnosis?


A. Haemorrhagic stroke
B. Ischaemic stroke
C. Recurrent TIA
D. Space-occupying lesion
E. Venous sinus thrombosis

82. A 69-year-old woman presented following a syncopal fall. Initial


examination was unremarkable, but she was admitted to a
monitored bed for observation. The on-call doctor was called
to see her because the nurses noted a change in heart rhythm.
A 12-lead ECG showed polymorphic ventricular tachycardia
(torsades de pointes).
On re-assessment, she was normotensive with a GCS of 15 and denied
any chest pain.
Which of the following treatment options is most appropriate?
A. Direct current (DC) cardioversion
B. IV amiodarone
C. IV magnesium sulphate
D. IV verapamil
E. Overdrive pacing
EXAM 2 | QUESTIONS 127

83. An 84-year-old woman had a non-healing leg ulcer. She reported


that she first noticed the ulcer after banging her leg on a
table 9 months earlier and despite multiple dressing regimes from
the district nurses, it continued to deteriorate.
Her past medical history included hypertension, osteoporosis, and
diabetes mellitus (diet-controlled).
The ulcer was on the anterior aspect of the superior part of her left
shin and measured approximately 6 × 7cm. The edges were sloughy and
there was a slight malodour. Pedal pulses were easily palpable in both
feet and capillary refill was normal. A recent wound swab demonstrated
skin flora only.
Which of the following is the most appropriate next step?
A. Course of oral antibiotics
B. CT angiography of the legs
C. Multiple punch biopsies
D. Silver-based dressing regime
E. Vacuum-assisted closure (VAC) dressing

84. A 78-year-old woman with a past history of stroke was


admitted to the ward. She was dysphasic. Her family reported
that she was communicating less, was withdrawing from any
touch and had not been sleeping as well as usual. She had
been non-compliant with oral medications. She usually took
bendroflumethiazide 2.5mg OD, simvastatin 40mg ON, and
aspirin 75mg OD.
Examination demonstrated longstanding contractures in her left
lower limb, with some ulceration over the left lateral malleolus.
Cardiovascular, respiratory, and abdominal systems were unremarkable.
Her bowels were open regularly and a rectal examination showed soft
stool only.
Baseline investigations
• Bloods: normal FBC, U&E, CRP.
• CXR: unremarkable.
• Urine dip: nitrite negative, leucocyte negative, protein 1+.
Which of the following is the most appropriate course of action?
A. Prescribe a 10micrograms/hr buprenorphine patch
B. Prescribe a 12 micrograms/hr fentanyl patch
C. Prescribe norfloxacin 400mg BD for 3 days
D. Prescribe paracetamol 1g QDS
E. Prescribe risperidone 500micrograms BD orally
128 EXAM 2 | QUESTIONS

85. A 75-year-old man had a sudden onset of vertigo and nausea


6 days earlier. This had lasted several hours and he had felt
unbalanced ever since, but this was improving day by day. He
had not fallen. This episode had been preceded by a sense of ear
fullness, hearing loss, and a low-tone roaring noise in his right ear.
Neurological examination was normal, there was no ear wax, and
no postural hypotension. He recalled having had a similar, but less
severe episode 10 years earlier. He asked if he would be able to
drive a car once his balance recovered.
What is the most appropriate advice to give him?
A. He can drive once his symptoms resolve
B. He can drive 1 month after symptoms resolve
C. He should take a driving test at a DVLA driving centre
D. He should notify the DVLA of this episode
E. He will not be able to drive again

86. An 86-year-old man developed multiple large, itchy blisters on


his lower limbs over a 5-week period. Some of the blisters had
broken. He was on no medication and was otherwise well.
What is the most likely diagnosis?
A. Bullous pemphigoid
B. Dermatitis herpetiformis
C. Impetigo
D. Pemphigus foliaceous
E. Stevens–Johnson syndrome
EXAM 2 | QUESTIONS 129

87. A 94-year-old man was admitted with a left middle-cerebral


artery territory ischaemic stroke. He had made very poor
progress over the past few weeks and remained bed bound, with a
dense hemiparesis, mixed dysphasia, and ‘at-risk’ swallow. He had
had two aspiration pneumonias. The family and multidisciplinary
team agreed that he was dying and ongoing care should be
supportive.
On day 10 he had a nocturnal tonic clonic seizure, which lasted 10 min
and was terminated with 10mg rectal diazepam. A repeat CT head scan
showed the known infarct, but no new changes. He had two further
self-terminating seizure the next day, which his family found very
distressing.
On assessment he had a GCS of 6, bubbly upper airways noises, but
clear chest and seemed comfortable. IV access had not been possible.
Which of the following is the most appropriate intervention?
A. Continuous SC diamorphine infusion
B. Continuous SC midazolam infusion
C. Nasogastric levetiracetam BD
D. Rectal carbamazepine BD
E. Rectal diazepam following seizure

88. An 80-year-old woman was brought to the ED, having been found
wandering around the local shopping precinct asking people how
she could get home, but unable to tell them where she lived.
Physical examination, FBC, LFTs, corrected calcium, blood urea
and serum electrolytes, electrocardiogram, CXR, and urinalysis
were entirely normal. She was able to give the contact details for
her daughter, who was spoken to on the telephone. The daughter
stated that the family had been concerned about her mother’s
deteriorating memory for about 2 years. This was the first time,
to their knowledge, that she had wandered and got lost; normally,
she was independent in her shopping, but bought very little in
the way of nourishing food. Increasingly, the family had been
helping her to manage her finances. With her agreement, plans
were underway to sell her house and for her to move to sheltered
accommodation.
What is the next most appropriate step in managing this
situation?
A. Admission for assessment under the Mental Health Act
B. Arrange community mental health team assessment at home
C. Assess the patient’s mental capacity to determine their care needs
D. Refer to the memory clinic to confirm the diagnosis of dementia
E. Urgent medical admission to confirm the diagnosis of dementia
130 EXAM 2 | QUESTIONS

89. An 89-year-old, previously healthy man developed a right


basal pneumonia. He lived alone and was unable to maintain
adequate nutrition or hydration. He was admitted to hospital, for
treatment with IV antibiotics and fluids. His condition gradually
declined and he died 3 days later.
Which of the following best explains the reduction in acquired
immunity with ageing?
A. Decreased B-cell replenishment
B. Increased germinal centre reactivity
C. Increased nasal production of IgA
D. Increased T-cell production
E. Involution of the thymus

90. A 79-year-old woman with dementia was recovering from a


fall when, despite receiving thromboprophylaxis with LMWH,
she developed a left ileofemoral deep vein thrombosis.
The occupational therapist is considering her suitability for
intermediate care services.
Which of the following is not included in the Barthel Index?
A. Bathing
B. Cognition
C. Dressing
D. Grooming
E. Toileting

91. A 78-year-old man attending his routine Parkinson’s review was


noted to have a brown pigmented lesion on his cheek, with some
areas darker than others. The margins were irregular. The lesion
had grown in size over the last year and started to change in
colour.
What is the most likely diagnosis?
A. Lentigo maligna melanoma
B. Pigmented basal cell carcinoma
C. Pigmented naevus
D. Seborrhoeic keratosis
E. Solar lentigo
EXAM 2 | QUESTIONS 131

92. Four weeks after suffering a thrombotic partial anterior


circulation stroke, a 79-year-old woman was anxious to discuss
possible ways to reduce her risk of future stroke. She was
independently mobile with a Zimmer frame and able to climb
stairs, but was fearful she might fall. She had AF and had been on
amiodarone for several years with good rate control. She drank
2U of wine each day, did not smoke, and weighed 52kg. She was
normotensive and not diabetic.
What is the most appropriate treatment to recommend?
A. Aspirin and amiodarone
B. Cardioversion and warfarin
C. Catheter ablation and warfarin
D. Warfarin and dronedarone
E. Warfarin and amiodarone

93. A 98-year-old female resident of a nursing home developed a


sacral pressure sore. She was quadriplegic after falling down the
stairs in her own home 12 months earlier and had lived in the
care home ever since. She had a long-term in-dwelling urinary
catheter and was assisted with feeding. Her daughter was very
angry that her mother had developed a pressure sore.
What is the most appropriate advice to give her?
A. Complain to the Care Quality Commission
B. Complain to her Member of Parliament
C. Move her mother to another nursing home
D. Speak to the care home manager
E. Write to the local newspaper

94. A 79-year-old man was admitted to hospital after having lost


consciousness. His wife witnessed the event and described him as
staring into space, then falling to the ground jerking from head
to toe. He went blue and was incontinent. When he came round
he was vague and appeared confused for several minutes. Physical
examination was entirely normal.
What is the most appropriate first investigation?
A. Electrocardiogram
B. Electroencephalography
C. MRI head scan
D. Serum prolactin
E. 24-hr Holter monitoring
132 EXAM 2 | QUESTIONS

95. An 83-year-old female resident of a dementia-registered care


home regularly passed a formed motion into her pants after
eating. She was mobile in the care home and did not give any
warning that she was going to defaecate. She had been prescribed
senna 1 year earlier, when she was taking codeine for low back
pain. Her current medication was donepezil 10mg OD, aspirin
75mg OD and simvastatin 40mg nocte. Digital rectal examination
(DRE) revealed normal formed stool with slightly reduced
anal tone. Although she was unable to understand the sensory
neurological examination, the anal sphincter did contract on DRE.
What is the most likely explanation for her incontinence?
A. Intrarectal contractions with no increase in external sphincter tone
B. Laxity of the anal sphincter and levator ani due to laxative abuse
C. Loss of muscle tone in the levator ani with loss of the anorectal angle
D. Reduced anal squeeze pressure due to declining muscle strength
E. Segmental peristalsis of the colon with reduced anal sensation

96. An apparently previously healthy and independent 84-year-old


woman developed a strangulated para-umbilical hernia whilst
recovering from a UTI, complicated by mixed delirium.
Post-operatively she had a further episode of delirium
characterized by erotic delusions. Efforts to get her out of bed
and into a chair were met with fierce resistance, which was
thought to be pain-related behaviour due to severe osteoarthritis
of the right hip. Despite apparently adequate analgesia she
continued to make little progress with the physiotherapists and
was content to lie in bed. She needed supervision to feed herself
and did not spontaneously interact with the staff.
What is the most likely reason for the failure to achieve her
rehabilitation goals?
A. Alzheimer’s dementia
B. Clash of personalities
C. Endogenous depression
D. Hypoactive delirium
E. Staffing shortages
EXAM 2 | QUESTIONS 133

97. A 93-year-old woman presented to the ED. Her carers were


concerned because she was difficult to rouse. On arrival her GCS
was 12. She was pyrexial at 38.5°C. She had a sinus tachycardia,
but otherwise cardiovascular, respiratory, and abdominal
examination were unremarkable. She was mildly photophobic
and had a degree of neck rigidity. On peripheral nervous system
examination, power was reduced in her left lower limb and she
had an up-going left plantar. There was no papilloedema, and
pupils were equal and reactive. The provisional diagnosis was
bacterial meningitis.
Which of the following is true?
A. An EDTA blood sample should be sent for polymerase chain reaction testing
B. Benzylpenicillin 1.2g QDS should be started immediately, even before lumbar
puncture (LP)
C. Contact tracing will be required if S. pneumoniae meningitis is confirmed
D. LP can safely be performed in the absence of a CT head
E. She should be started on high dose steroids (e.g. dexamethasone 2g QDS)

98. The wife of a 75-year-old male called their general practitioner


because he had developed a right hemiplegia while they were
eating breakfast. Thirty minutes later, whilst she was on
the telephone his symptoms resolved completely. He was a
diet-controlled type 2 diabetic and was on no medication. He
undertook home BP monitoring and his BP always read between
130/70 and 136/74.
What is the most appropriate management?
A. Advise his wife to give 75mg aspirin and refer to local stroke service to be seen within 24hr
B. Advise his wife to give 300mg aspirin and refer to local stroke service to be seen
within 1 week
C. Advise his wife to give 300mg aspirin and refer to local stroke service to be seen
within 24hr
D. Advise his wife to give 300mg aspirin and review in the GP surgery within 24hr
E. Advise his wife to give 75mg aspirin and review in the GP surgery within 1 week
134 EXAM 2 | QUESTIONS

╇ 99.╇An 89-year-old female resident of a specialist dementia care


home was noted to have an ulcerating lesion above the lateral
malleolus of her left ankle. The lesion was largely obscured by
a large haematoma; however, the upper edge was visible and
seemed to be rolled, friable, and irregular. She had mild bilateral
ankle oedema. She had lived in the home for 4 years and had not
previously been noticed to have any leg ulcers.
What is the most likely diagnosis?
A. Marjolin’s ulcer
B. Necrobiosis lipoidica diabeticorum
C. Pyoderma gangrenosa
D. Squamous cell carcinoma
E. Venostasis ulcer

100.╇A 79-year-old man with Parkinson’s disease had recently


been started on rivastigmine 1.5mg BD as he had developed
troublesome hallucinations. In the last year, he had experienced
increasing frequency of falls, where he would fall backwards
when walking or turning. He had also found increasing difficulty
climbing and descending stairs, and had to do this on all fours.
He was able to wash and dress himself independently, albeit
slowly. He sometimes needed assistance to get in and out of bed,
and out of a chair. His only other medication was co-beneldopa
125 QDS. He attended the ED because of a bleeding toe after
having some chiropody. His wife stated that she was concerned
about the number of falls that he was having.
What is the most appropriate next step in managing his
Parkinson’s?
A. Arrange occupational therapy assessment
B. Arrange physiotherapy assessment
C. Arrange for the Parkinson’s nurse to review him at home
D. Measure lying and standing BP
E. Refer to the falls clinic for assessment
exam

2 ANSWERS

1. A.╇ The commonest reason for lack of benefit from hearing aids is not wearing them. A period
of learning is required to achieve benefit from hearing aids. If they are used intermittently (such as
when the daughters visit), the benefit is limited.
Battery failure, impacted ear wax and maladjusted volume have been excluded by the technician,
although these would all also result in impaired hearing and later symptoms such as indistinct
speech (volume too low) or a complaint that the individual was shouting or that there was whistling
(volume too high).
Acoustic feedback is the commonest reported problem by older people which then causes them
to stop wearing their hearing aids (a Danish study reported acoustic feedback occurs in 32% aged
70–79 and 40% aged >80; difficulty with volume control 20% and 29% of the two age groups
respectively; difficulty inserting aid 16% and 29%).

Further reading
Knott L. Deafness. Available at: http://www.patient.co.uk/doctor/Deafness.htm
National Academy on an Aging Society. Hearing loss: a growing problem that affects quality of life.
National Academy on an Aging Society No. 2, December 1999.
Available at: http://www.agingsociety.org/agingsociety/pdf/hearing.pdf
Ross M. Troubleshooting your hearing aid. Available at: http://www.healthyhearing.com/content/
articles/Accessories/Service/7835-Troubleshooting-your-hearing-aid

2. B.╇ Parkinson’s patients must get their medication on time when in hospital and may suffer
pain, worsening of their Parkinson’s symptoms, delirium, and be more prone to complications of
immobility and dysphagia as well as increased length of stay due to being under-medicated. It is
essential to maintain his usual dosing schedule and not move to an institutionalized regime. He may
become paranoid and suspicious at worst, just anxious at best about the un-necessary change to his
drug regime. There is poor quality evidence that rivastigmine may improve outcomes in delirium,
there is no evidence that it can lessen incidence. Lorazepam may be appropriate management if
delirium develops to reduce agitation, if he is a danger to himself or others and would be preferred
to an antipsychotic which might worsen his Parkinson’s. Ensuring he is nourished and hydrated, pain
is managed, he is kept orientated and ward moves (within and between) are avoided to reduce
delirium risk, as will the staff being aware that he is at high risk due to his age, cognitive impairment
and need for adherence to medication regime.

Further reading
Brennan KA. Genever—Managing Parkinson’s disease during surgery. British Medical Journal 2010;
341: c5718. Available at: http://www.bmj.com/content/341/bmj.c5718 (requires BMA or Athens
account).
136 EXAM 2 | ANSWERS

NICE. Delirium diagnosis, prevention and management, Clinical guideline CG103.


Available at: http://guidance.nice.org.uk/CG103
Parkinson’s UK. Get it on time campaign. Available at: http://www.parkinsons.org.uk/sites/default/files/
publications/download/english/b124_ideasforaction.pdf

3. E.╇ A bed rail can be helpful when trying to sit up, get out of bed, or turn one way in bed.
A hospital bed may be useful for nursing and positioning of immobile patients who need assistance,
but does not make it easier for the patient to turn. A monkey pole and rope ladder may be useful
to assist the patient raise themselves up in bed, but will not help with turning. Silk or satin sheets
reduce friction and make turning easier, it is wise to only recommend the lower sheet to be silk as a
silk top sheet may slide off the patient and then leave them with the difficulty of moving in bed and
leaning out to pick it up off the floor!

4. D.╇ The history given is fairly non-specific and the usual good exercise tolerance is reassuring.
NICE guidance would suggest that risk stratification with a scoring system and then non-invasive
imaging would be most appropriate to rule out coronary artery disease. However, although
no-longer part of the NICE investigation pathway, the patient has had a normal exercise test. This
coupled with the low pre-test probability based on history would suggest a more conservative
course first-line.

Further reading
NICE. Chest pain of recent onset. Clinical guidelines CG95.
Available at: http://www.nice.org.uk/CG95
Rull G. Exercise tolerance testing.
Available at: http://www.patient.co.uk/doctor/Exercise-ECG-Testing.htm

5. E.╇ The laboratory tests confirm that this patient is hypothyroid, however, it is imperative
to exclude hypoadrenalism before initiating treatment with thyroxine. Failure to do so may
provoke a hypoadrenal crisis. The postural hypotension as well as the raised potassium and low
sodium suggest this as a possibility. If treatment with thyroxine is necessary more urgently, e.g. in
myxoedemic coma, steroid cover should be given pending formal exclusion of hypoadrenalism.
In non-myxoedemic coma patients, once hypoadrenalism has been excluded, treatment should be
with oral thyroxine. IV liothyronine should only be considered in myxoedemic crises.
Although the urine analysis may be positive, the patient has no symptoms of urosepsis or other
markers of sepsis. Treatment is therefore not warranted.
The lack of focal neurology and an alternative explanation for the symptoms means that
neuroimaging should not be the first line course of action.

Further reading
Murray, et al. Deterioration of symptoms after start of thyroid hormone replacement. British Medical
Journal 323: 332. Available at: http://www.bmj.com/content/323/7308/332.1

6. D.╇ A serum BNP level less than 100pg/ml (29pmol/l) in an untreated patient makes a diagnosis
of heart failure unlikely. The absence of wheeze makes reversible airways disease unlikely. He is
a non-smoker, and has no occupational risk for chronic obstructive pulmonary disease. The chest
X-ray did not show evidence of either current pneumonia, bronchiectactic changes, or fibrosis
to suggest recurrent aspiration due to gastro-oesophageal reflux disease (GORD). He is elderly,
hypoxic, and has had recent surgery and will have been less active than usual, making recurrent
EXAM 2 | ANSWERS 137

pulmonary emboli the most likely diagnosis. The D-dimer level is unhelpful in this case and may also
be raised in older people. In patients with suspected pulmonary embolism and previous venous
thromboembolic disease, a negative D-dimer test result seems to safely rule out a recurrent
event. However, the proportion of negative results is lower in such patients, reducing the clinical
usefulness of the D-dimer test in this group.

Further reading
NICE. Management of chronic heart failure in adults in primary and secondary care. Clinical guideline
CG108. Available at: http://www.nice.org.uk/nicemedia/live/13099/50517/50517.pdf
Health Hub from Cleveland Clinic. B-type Natriuretic Peptide (BNP) blood test.
Available at: http://my.clevelandclinic.org/heart/services/tests/labtests/bnp.aspx
Taylor Thompson B, Hales C. Diagnosis of pulmonary embolism.
Available at: http://www.uptodate.com/contents/diagnosis-of-acute-pulmonary-embolism
Lab Tests Online. D-dimer.
Available at: http://labtestsonline.org/understanding/analytes/d-dimer/tab/test
British Thoracic Society guidelines for the management of suspected acute pulmonary embolism.
British Thoracic Society Standards of Care Committee Pulmonary Embolism Guideline. Thorax
2003;58:470–84. Available at: http://www.brit-thoracic.org.uk/Portals/0/Guidelines/Pulmonary%20
Embolism/Guidelines/PulmonaryEmbolismJUN03.pdf
British Thoracic Society Standards of Care Committee. D-dimer in suspected pulmonary embolism
(PE). A statement, 2006. Available at: http://www.brit-thoracic.org.uk/Portals/0/Guidelines/
Pulmonary%20Embolism/Guidelines/ddimerstatement.pdf
Le Gal G, Righini M, Roy PM, et al. Value of D-dimer testing for the exclusion of pulmonary embolism
in patients with previous venous thromboembolism. Archives of Internal Medicine 2006; 166(2):
176–80. Available at: http://archinte.ama-assn.org/cgi/reprint/166/2/176.pdf

7. A.╇ The increase in life expectancy is more likely due to improved public health and social
programmes, nutrition and access to basic health care rather than to ongoing advances in medicine.
The associated fall in infant mortality results in an increased predicted life-expectancy. Medical
advances are more likely to be responsible for the increase in remaining life expectancy for those
reaching retirement age.

Further reading
Office for National Statistics. Focus on older people. Available at: http://www.ons.gov.uk/ons/rel/
mortality-ageing/focus-on-older-people/2005-edition/focus-on-older-people.pdf

8. E.╇ The main risk factors for developing retinal vein occlusion are: hypertension,
hypercholesterolaemia (cholesterol >6.5mmol/l), diabetes, renal impairment, and glaucoma.
An association has been reported with thrombophilia (antiphospholipid antibody syndrome,
hyperhomocysteinaemia), multiple myeloma, myeloproliferative disorders, and systemic
inflammatory diseases (Behcet’s, polyarteritis nodosa, sarcoidosis, Wegener’s granulomatosis, and
Goodpasture’s syndrome).
The detection and management of underlying medical conditions is aimed at preventing recurrence
of venous occlusion, particularly in the other eye, and to reduce cardiovascular risk factors. Aspirin
is not recommended in the management of retinal vein occlusion as there is no clear evidence of
association between retinal vein occlusion and risk of cardiovascular or cerebrovascular disease nor
protection against retinal vein occlusion in the contralateral eye.
138 EXAM 2 | ANSWERS

Further reading
Kiire CA, Chong NV. Managing retinal vein occlusion. British Medical Journal 2012; 344: 37–43.
Available at: http://www.bmj.com/content/344/bmj.e499?tab=responses
The Royal College of Opthalmologists. Interim Guidelines for Management of Retinal Vein Occlusion,
2010. Available at: www.rcophth.ac.uk/core/core_picker/download.asp?id=728

9. B.╇ Behavioural disturbance in the care recipient, being a female carer, poor strength of the
relationship between carer and care recipient, weak social network and a narrow range of coping
strategies have all been shown to correlate well with low well-being in carers.

Further reading
BMA. Working with carers: guidelines for good practice, 2009.
Available at: www.bma.org.uk/healthcare_policy/community_care/Workingwithcarers.jsp
Eahar K, Owen A, Williams K, et al. Effective caring: a synthesis of the international evidence on carer
needs and interventions. Centre for Health Service Development (CHSD). Paper 27, 2007. Available
at: www.health.gov.au/internet/main/publishing.nsf/Content/035CFE9F19D3745CCA2574640021602
7/$File/CHSDEffecCaringV2Final.pdf
Department of Health. Carers. 2011. www.dh.gov.uk/en/SocialCare/Carers/index.htm
NICE-SCIE. Dementia. The NICE -SCIE guideline on supporting people with dementia and their
carers in health and social care. National clinical practice guideline 42 (updated version) 2011.
Available at: www.nice.org.uk/nicemedia/live/10998/30320/30320.pdf
Princess Royal Trust for Carers. Supporting carers: an action guide for general practitioners
and their teams. Sheffield University. Available at: http://static.carers.org/files/prtc-introductory-
section-3660.pdf

10. D.╇ The primary progressive aphasias consist of semantic dementia and progressive non-fluent
aphasia. Semantic dementia, as represented in this scenario, consists of severe and progressive
problems of naming, category fluency, and comprehension. Speech remains fluent. Diagnosis
requires a 2 year period of progressive language impairment in the relative absence of other
cognitive deficits. Progressive non-fluent aphasia presents with preserved comprehension, but
hesitant speech with marked word finding difficulties.

Further reading
Sewell M, Vigario A, Sano M. Neuropsychology in the diagnosis and treatment of dementia. In: Fillit
H, Rockwood K, Woodhouse K. (eds) Brocklehurst’s Textbook of Geriatric Medicine and Gerontology, 7th
edn, Chapter 53. Philadelphia: Saunders.

11. B.╇ NICE advocate weight loss and exercise as first line treatments. Advice on building
muscles that support the knee joint may be appropriate from a physiotherapist. Paracetamol is
recommended as a first line analgesic and may be augmented with topical NSAIDs and/or capsaicin.
If these measures don’t help, opiates or oral NSAIDs should be considered. NICE recommend that
glucosamine or chondroitin products should not be used for the treatment of osteoarthritis.

Further reading
NICE. Osteoarthritis: the care and management of osteoarthritis in adults, Clinical guideline CG59.
Available at: http://publications.nice.org.uk/osteoarthritis-cg59/guidance
EXAM 2 | ANSWERS 139

12. D.╇ 5–10% of patients given statins in clinical trials suffer from myalgia; the absolute risk in
general populations may be higher. The normal thyroid-stimulating hormone (TSH) and CRP make
(A and B) unlikely. Although the CK level is slightly raised this is more compatible with exercise than
mysositis. The absence of viral symptoms makes (E) unlikely.

Further reading
Buettner C, Davis RB, Leveille SG, et al. Prevalence of musculoskeletal pain and statin use. Journal of
General Internal Medicine 2008; 23(8): 1182–6.
Available at: http://www.ncbi.nlm.nih.gov/pmc/articles/PMC2517983/

13. E.╇ This man developed venous sinus thrombosis as a result of hyper-viscosity due to
reactivation of polycythaemia rubra vera and dehydration secondary to fasting. Intracranial
tumours, especially bleed into a para-sulcal meningioma, can present in a similar manner—the clue
to the diagnosis lies in the history especially development of symptoms in relation to dehydration.
Meningitis is excluded by normal inflammatory markers and no signs of meningeal irritation, the
latter also helps exclude sub-arachnoid haemorrhage. Sub-dural haematoma large enough to cause
bilateral papilledema is likely to have more pronounced neurological deficit than slight leg weakness.
His leg weakness was a result of focal haemorrhage due to raised venous pressure.
The best method of visualizing the dural venous sinuses and larger cerebral veins is MR venography.
CT venography is a viable alternative, but is not an option you are given. The other investigations
mentioned may, to some extent, delineate the extent of focal haemorrhage and dilatation of
venous sinuses. In cerebral venous sinus thrombosis, MRI and CT may show an infarct pattern that
does not follow the distribution of an arterial occlusion. MRI may also show absence of flow void in
the venous channels. In the absence of a haemorrhagic component, the infarct may not show for up
to 48–72hr on CT.

Further reading
McElveen WA. Cerebral venous thrombosis.
Available at: http://emedicine.medscape.com/article/1162804-overview#

14. E.╇ Her CHADS2 score is 4, giving her an 8.5% risk of stroke per year (or 9.8% if scoring with
CHA2DS2-VASc Score). It is well recognized that therapeutic anticoagulation would reduce her
risk of embolic stroke. Historically, clinicians have concerned about warfarinizing fallers, but this risk
is overstated. Out of the listed medications, warfarin would therefore be the drug of choice. Had
they been listed, a novel oral anticoagulant (noac), such as apixaban, dabigatran, or rivaroxaban
may also be appropriate. If therapeutic anticoagulation were not feasible, or deemed inappropriate,
aspirin would be the second line choice. There is no evidence to suggest superiority of clopidogrel
or dipyridamole in this situation.
The 24-hr tape shows her heart rate is stable and does not need rate controlling medication.
Indeed, diltiazem is negatively inotropic and could worsen her heart failure.

Further reading
NICE. Apixaban for preventing stroke and systemic embolism in people with nonvalvular atrial
fibrillation, Guideline TA275. Available at: http://guidance.nice.org.uk/TA275/
NICE. Atrial fibrillation, Clinical guideline CG36. Available at: http://guidance.nice.org.uk/CG36
NICE. Dabigatran etexilate for the prevention of stroke and systemic embolism in atrial fibrillation,
Guideline TA249. Available at: http://guidance.nice.org.uk/TA249/
140 EXAM 2 | ANSWERS

NICE. Rivaroxaban for the prevention of stroke and systemic embolism in people with atrial
fibrillation, Guideline TA256. Available at: http://guidance.nice.org.uk/TA256/
Pugh D, Pugh J, Mead G. Attitudes of physicians regarding anticoagulation for atrial fibrillation: a
systematic review. Age & Ageing 2011; 40: 675–83.
Available at: http://ageing.oxfordjournals.org/content/40/6/675.full.pdf
Task Force for the Management of Atrial Fibrillation of the European Society of Cardiology. Guidelines
for the management of Atrial Fibrillation. ESC guidelines. European Heart Journal 2010; 31: 2369–429.
Available at: http://www.escardio.org/guidelines-surveys/esc-guidelines/GuidelinesDocuments/
guidelines-afib-FT.pdf

15. D.╇ Any individual whose health care needs change is eligible to be assessed as to the
complexity of their health needs in order to determine whether they are eligible for continuing
health care funding. This is especially true, if they are changing residence (i.e. moving in to a care
home). This assessment takes in to account the following domains:
• Behaviour.
• Cognition.
• Communication.
• Psychological/emotional needs.
• Mobility.
• Nutrition.
• Continence.
• Skin (including wounds and ulcers).
• Breathing.
• Symptom control through drug therapies and medication (includes a need for supervision of
complex medication regimes).
• Altered states of consciousness.
• Other significant care needs.
Each item is assessed according to its nature, intensity, complexity, and unpredictability. Vision is not
directly taken in to consideration, but may influence behaviour, communication, emotional needs,
ability to maintain nutrition and continence.
Where a person’s primary need is a health need, they are eligible for NHS continuing health care.
Deciding whether this is the case involves looking at the totality of the relevant needs. Where an
individual has a primary health need and is therefore eligible for NHS continuing health care, the
NHS is responsible for providing all of that individual’s assessed needs, including accommodation, if
that is part of the overall need.
If the person does not qualify for NHS continuing health care, their local authority will be responsible
for assessing their care needs and providing services if they are eligible. However, if they don’t qualify
for NHS continuing health care, but are assessed as having health care or nursing needs, they may still
receive some care from the NHS. For someone who lives in their own home, this could be provided as
part of a joint package of care, where some services come from the NHS and some from social services.
If the person moves into a nursing home, the NHS may contribute towards their nursing care costs.

Further reading
Department of Health (England). The National Framework for NHS Continuing Healthcare and
NHS-funded Nursing Care. July 2009 (revised). Available at: http://www.dh.gov.uk/prod_consum_dh/
groups/dh_digitalassets/documents/digitalasset/dh_103161.pdf
EXAM 2 | ANSWERS 141

NHS Choices. Practical support. Available at: http://www.nhs.uk/CarersDirect/guide/practicalsupport/


Pages/NHSContinuingCare.aspx

16. A.╇ There is an inadequate evidence base to support or refute the use of telehealth or
telemedicine in either health or social care. Around one-third of studies have shown no benefit
of telehealth over usual care; very few studies have shown a negative impact or demonstrated
statistically significant benefits. Most studies have been limited to specific conditions (specifically
diabetes and heart failure). Study design, in those that have examined the use of telehealth in
people with multiple long-term conditions, often fail to separate the impact of the technology
provided from the wider model of care management, which is often introduced at the same time.

Further reading
Kerr B, Hurst K, Clark S. Telehealthcare and mental health: using telehealthcare effectively in the
support of people living with mental disorder. King’s Fund Library Database, 2011. Available at: http://
kingsfund.koha-ptfs.eu/cgi-bin/koha/opac-detail.pl?biblionumber=103952
King’s Fund. The impact of telehealth: a review of the evidence. The King’s Fund, London. Available at:
http://www.kingsfund.org.uk/topics/technology_and_telecare/telehealth_evidence.html
Martin S, Kelly G, Kernohan WG, McCreight B, Nugent C. Smart home technologies for health and
social care support. Cochrane Database of Systematic Reviews 2005; Issue 4: CD006412. Available at:
http://www.ncbi.nlm.nih.gov/pubmed/18843715
Poole T. Telecare in older people: Wanless social care review. London: King’s Fund, 2006. Available at:
http://kingsfund.koha-ptfs.eu/cgi-bin/koha/opac-detail.pl?biblionumber=37212

17. D.╇ The symptoms are likely due to pelvic floor weakness, to which she is predisposed by the
two vaginal deliveries. Physiotherapy, with or without bio-feedback, may be sufficient to improve
pelvic floor and internal sphincter strength without necessitating more invasive tests or procedures.
Pressure studies and ultrasound are invaluable in planning surgical intervention. They should be
considered if initial management methods fail, but are not appropriate first line.
While excluding lower GI malignancy needs to be considered, this patient has a good history for
stress incontinence, going back over several years with no warning features and normal blood tests.
While it is increasingly recognized that pudendal nerve damage from childbirth can contribute to
internal sphincter weakness, there is no clear consensus on treatment, although pudendal nerve
stimulation has been tried. Non-invasive strategies should be tried first.
The history and examination are suggestive of pelvic floor/sphincter incontinence. While
loperamide may provide some symptomatic relief, it may also exacerbate the situation by causing
faecal loading with overflow. It is preferable to address the root cause.

Further reading
NICE. Faecal incontinence: The management of faecal incontinence in adults, Clinical guideline CG49.
Available at: http://guidance.nice.org.uk/CG49/

18. C.╇ Ideally, maintaining the son in his own environment is the best choice and has least risk to him.
However, this can rarely be achieved in a crisis situation, especially on an open-ended basis (it is not
possible to predict how well his mother will recover or how long this will take). He cannot be left alone,
and so options B and E are unreasonable; furthermore, option E still includes his mother in his care
when we are not made aware of the level of her disability and is also denying her access to appropriate
stroke services. He does not need to be hospitalized, and would be separated from his mother
(single sex accommodation), as well as exposed to the risks of hospitalization, e.g. hospital-acquired
142 EXAM 2 | ANSWERS

infection. A move to urgent respite is necessary until such time as it is clear what level of recovery his
mother has made and what care package they might both need.

19. A.╇ Haloperidol, metoclopramide, and prochlorperazine all have anti-dopaminergic activity
and would adversely affect the symptoms of Parkinson’s, and could even trigger an episode of
neuroleptic malignant syndrome.
Although it also has anti-dopaminergic activity, domperidone does not cross the blood–brain
barrier and may, therefore, be used safely. Its mild prokinetic effects will promote resumption of
normal gut motility. Ondansetron (5-HT antagonist) and cyclizine (antihistamine) could be used
safely, but lack the prokinetic benefits and as robust an evidence base.

Further reading
British National Formulary. Available at: http://www.bnf.org

20. B.╇ Dysphagia may occur with any stroke (site or severity), but the absence of new focal
neurology makes this unlikely. Dysphagia may occur as dementia progresses, but is more likely
to present as an increasing difficulty rather than as a sudden problem. It is unlikely that 3 weeks
after the initial trauma a sub-dural haemorrhage would manifest solely with dysphagia. The normal
inflammatory markers suggest a UTI with delirium as the cause of her dysphagia is unlikely.
It is most likely that as her infection is resolving and her ankle has been fused that pain has been
lessening and the Abbey pain scale has been misinterpreting her abnormal behaviour to be due to
pain rather than being due to opiate induced delirium.

Further reading
Croisile B. Benson’s syndrome or posterior cortical atrophy.
Available at: https://www.orpha.net/data/patho/GB/uk-Benson.pdf
Zwakhalen SMG, Hamers JPH, Abu-Saad HH, Berger MPF. Pain in elderly people with severe
dementia: a systematic review of behavioural pain assessment tools. BMC Geriatrics 2006; 6:
3. Available at: http://www.ncbi.nlm.nih.gov/pmc/articles/PMC1397844/

21. B.╇ Body Mass Index (BMI) is not always easy to calculate in older people due to difficulties in
measuring height in kyphotic and/or scoliotic individuals and those with poor muscle tone (although
height can be calculated by other methods such as arm span). This is further compounded in sick
individuals, who it may not be possible to weigh or measure their height. Furthermore, BMI may
alter due to accumulation of oedema or ascites, and so not accurately reflect nutritional status.
The Malnutrition Universal Screening Tool (MUST) is recommended as a screening tool by the
National Institute of Clinical Excellence (NICE), the British Association for Parental and Enteral
and Nutrition (BAPEN) and the British Dietitian Association (BDA). It takes 3–5min to complete.
Although it includes BMI it also takes in to account history of unexplained weight loss, and acute
illness effect, to identify adults who are malnourished or at risk of malnutrition.
The Mini Nutritional Assessment (MNA) was specifically designed for older patients, and takes less
than 15 minutes to complete.
Both MUST and MNA are potentially useful in the scenario given, but as MUST is quicker, provides
alternative means of assessing BMI if height cannot be measured/is unknown, and also provides
alternative strategies for assessing risk, it would be the preferred option.
The Quetelet index enables calculation of BMI.
EXAM 2 | ANSWERS 143

Serum albumin can be affected by factors other than nutritional state, e.g. inflammation and
infection, limiting its usefulness as a marker of nutritional status in acutely unwell patients. Its long
half-life also limits its utility for looking at short-term changes in protein and energy intake.

Further reading
Ahmed T, Haboubi N. Assessment and management of nutrition in older people and its importance
to health. Clinical Interventions in Aging 2010; 5: 207–16.
Available at: http://www.ncbi.nlm.nih.gov/pmc/articles/PMC2920201/
BAPEN. Malnutrition Universal Screening Tool.
Available at: http://www.bapen.org.uk/pdfs/must/must-full.pdf
Guigoz Y. The Mini Nutritional Assessment (MNA®). Review of the literature—What does it tell us?
Journal of Nutritional Health & Aging 2006; 10(6): 466–85.
Available at: http://www.mna-elderly.com/publications/341.pdf
NICE. Nutrition support in adults: oral nutrition support, enteral tube feeding and parenteral nutrition,
Clinical guideline CG32. Available at: http://publications.nice.org.uk/nutrition-support-in-adults-cg32/
guidance#screening-for-malnutrition-and-the-risk-of-malnutrition-in-hospital-and-the-community

22. A.╇ Although the overall prognosis is poor, the physiotherapist and occupational therapy
assessments show that this patient is currently independent with her mobility and self-care. There is
therefore no indication for rehabilitation, or indeed instigation of a care package.
Her GP needs to be informed of her prognosis, so that regular review can be undertaken. It would
be worth referring to the community palliative care teams to assist with this monitoring. When her
condition deteriorates, such that she needs additional support, this should be arranged.
Given the likely rapid deterioration in her condition, it would be appropriate to initiate a fast-
track continuing care referral before she develops significant needs. This would help clarify funding
sources when care is needed. It would not, however, be appropriate to delay discharge while
arranging this. The next most appropriate step is therefore early discharge with involvement of the
GP, who can continue with this process in the community.

Further reading
Department of Health. NHS Continuing healthcare practice guidance. London: DoH. Available
at: http://www.dh.gov.uk/en/Publicationsandstatistics/Publications/PublicationsPolicyAndGuidance/
DH_115133
Department of Health. The national framework for NHS continuing healthcare.
London: DoH. Available at: http://www.dh.gov.uk/en/Publicationsandstatistics/Publications/
PublicationsPolicyAndGuidance/DH_103162

23. D.╇ Heparin can suppress adrenal secretion of aldosterone and cause hyperkalaemia. This
is more likely in patients with diabetes mellitus and pre-existing renal impairment. The risk of
hyperkalaemia increases with the duration of therapy, but is usually reversible on cessation. It is
advisable to monitor potassium levels in prolonged heparin treatment.
Ibuprofen could be nephrotoxic and induce hyperkalaemia and renal impairment. However, the
otherwise normal U&E rule this out. Amlodipine, bendroflumethiazide and co-amoxiclav are not
associated with hyperkalaemia.

Further reading
Edes TE, Sunderrajan EV. Heparin-induced hyperkalaemia. Archives of Internal Medicine 1985;
145(6): 1070–2. Available at: http://www.ncbi.nlm.nih.gov/pubmed/4004433
144 EXAM 2 | ANSWERS

EMC. Summary of Product Characteristics—‘Clexane’ Available at: http://www.medicines.org.uk/


EMC/medicine/10054/SPC/Clexane+Forte+Syringes/
British National Formulary. Available at: http://bnf.org/bnf/index.htm

24. A.╇ The situation is unlikely to improve whilst she is not taking her antidepressants and
antipsychotic medication and, indeed, is deteriorating, so simply continuing in attempts to gain her
compliance are likely to be fruitless and lead to worsening physical and mental state.
While a formal capacity assessment is required before instigating any treatment, this is not listed as
an option. Given that the patient has a known psychiatric illness and is not interacting with the staff
(a likely manifestation of her psychiatric illness), she cannot be assumed to have the mental capacity
to determine her own care and treatment needs at this stage. She entered in to hospital voluntarily
and is not making any attempt to leave and so it would be appropriate to treat her under a ‘best
interests’ decision after discussing with her next of kin without having to seek a court ruling. Even if
the patient had not presented voluntarily and had required sectioning under the Mental Health Act,
this would only allow treatment of the underlying psychiatric disorder. Additional treatments, such
as feeding, would still need to occur on a best-interests basis.
Placement of a NGT will enable her medication and nutrition to be provided. This is a safer
alternative to a percutaneous endoscopic gastrostomy (PEG), which would also take longer to
organize. If the NG were not tolerated, it may need to be considered.

Further reading
Mental Capacity Act 2005 (England). Available at: http://webarchive.nationalarchives.gov.uk/+/http://
www.dca.gov.uk/legal-policy/mental-capacity/mca-cp.pdf
Mental Health Act 1983. Available at: http://www.legislation.gov.uk/ukpga/1983/20/section/2
Mental Health Act 2007. Available at: http://www.legislation.gov.uk/ukpga/2007/12/contents

25. E.╇ The increasing need for personal care, move into any institutional care (residential or
nursing), worsening breathlessness and progressive decline in weight over the last year all reflect
declining function and suggest a potential need for palliative care intervention. Two hospital
admissions in the last year (it is not specified what the reason might be) does not trigger a possible
need for palliative care, whereas two or more in the last 6 months (if they are in anyway related to
the underlying disease process) does.

Further reading
NHS Lothian. Supportive and Palliative Care Indicators Tool (SPICT ™). Available at: http://www.
palliativecareguidelines.scot.nhs.uk/documents/SPICT_June2012.pdf
Gold Standards Framework. The GSF Prognostic Indicator Guidance. Available at: http://www.
goldstandardsframework.org.uk/cd-content/uploads/files/General%20Files/Prognostic%20
Indicator%20Guidance%20October%202011.pdf

26. D.╇ Transient global amnesia (TGA) is a well recognized phenomenon consisting of transient
disruption of memory function in the absence of other neurological signs. It often follows a precipitant
of physical exertion or emotional stress. Immediate recall and remote memory are preserved, but
there is usually retrograde amnesia going back several hours, as well as anterograde amnesia. This
often results in high anxiety in the patient who is unable to assimilate what is happening to them.
The mechanism is unclear, although a migraine variant or seizures have been conjectured. Patients
with TGA have a low incidence of stroke on follow-up, making underlying cerebrovascular disease
EXAM 2 | ANSWERS 145

a less likely cause. The overall prognosis is good. It is however, important to consider and exclude
stroke and/or seizure activity particularly in high risk individuals.
Patients with a ‘fugue state’ have amnesia for personal identity and events. The lack of a fluctuating
course, inattention and any other areas of cognitive impairment preclude a delirium as the cause.

Further reading
Caplan L, Chedru F, Lhermitte L, Mayman C. Transient global ischaemia and migraine. Neurology 1981;
31: 1167. Available at: http://www.neurology.org/cgi/content/abstract/31/9/1167
Inouye SK, van Dyck CH, Alessi CA, Balkin S, Siegal AP, Horwitz RI. Clarifying confusion: the confusion
assessment method. A new method for detection of delirium. Annals of Internal Medicine 1990;
113(12): 941–8. Available at: http://www.ncbi.nlm.nih.gov/pubmed/2240918
NICE. Delirium: diagnosis, prevention and management, Clinical guideline 103, 2010.
Available at: http://www.nice.org.uk/nicemedia/live/13060/49909/49909.pdf
Sucholeiki R. Transient global amnesia. eMedicine.
Available at: http://emedicine.medscape.com/article/1160964-overview

27. E.╇ In their meta-analysis, Ellis et al. show that patients undergoing CGA are more likely to be
alive and in their own homes at the end of scheduled follow-up compared with patients receiving
standard care. In addition, they are less likely to living in residential care and perform better on
cognitive measures.
The effects are significant out to at least one year. For the primary outcome ‘living at home’
number needed to treat was 33 at median follow-up of 12 months and 17 at a median follow-up of
6 months.

Further reading
Stuck AE, Iliffe S. Comprehensive geriatric assessment for older adults should be standard practice
according to a wealth of evidence. Editorial. British Medical Journal 2011; 343: d6799.
Available at: http://www.bmj.com/content/343/bmj.d6799.extract
Ellis G, Whitehead MA, Robinson D, O’Neill D, Langhorne P. Comprehensive geriatric assessment for
older adults admitted to hospital: meta-analysis of randomized controlled trials. British Medical Journal
2011; 343: d6553. Available at: http://www.bmj.com/content/343/bmj.d6553

28. B.╇ Femidom is a female condom.


The Lubidet is essentially a bidet that replaces the existing toilet seat and lid, and can be installed on
most toilets. The gentle warm water wash and warm air dry can be operated by hand, foot, elbow,
or shoulder operated controls.
Pre cut RADAR (Royal Association for Disability and Rehabilitation) keys are available in two head
sizes (the larger one is easier to grip), and are for use with RADAR disable toilet locks.
When passing urine from a seated position a splash sentry deflects urine down into the toilet.
It can be clipped to the toilet bowl, raised toilet seat or toilet frame. It is of more use for men
than women.
The Whiz Freedom may be of use to women who need to urinate in a standing, seated, or
lying position. It can be folded and transported in a bag or pocket and will spring back to its
original shape.

Further reading
Mobility Smart. Available at: http://www.mobilitysmart.cc/p-guard-for-gentlemen-who-sit.html
146 EXAM 2 | ANSWERS

29. C.╇ It is most likely that her care is to help with dressing and meals at either end of the day as
a result of her deforming arthritis; she may manage in the middle part of the day with drinks and
snacks provided by her morning carers. She is unlikely to be able to grip a walking frame (B and D),
and whilst a walking trolley might seem helpful for her to transfer plates and cups from the kitchen
to her dining/living area again she is unlikely to be able to grip the handle and steer it. A wheelchair
is not a walking aid!

Further reading
Disability living foundation. Choosing walking equipment.
Available at: http://www.dlf.org.uk/factsheets/Choosing_Walking_Equipment.pdf

30. A.╇ In this case all of these may be important in reducing her falls risk. However, assessment
of cognition is the most crucial component of a comprehensive falls risk assessment, as any
intervention to minimize falls risk relies upon the individual being able to understand and cooperate
with that intervention.

Further reading
American Geriatrics Society/British Geriatrics Society. Clinical practice guidelines for falls prevention
in older adults. 2010. Available at: http://www.americangeriatrics.org/health_care_professionals/
clinical_practice/clinical_guidelines_recommendations/2010/
Best Practice. Assessment of falls in the elderly.
Available at: http://bestpractice.bmj.com/best-practice/monograph/880.html

31. C.╇ Whilst his ex-wife appeared to be acting as an advocate for him, her expectations of his
care, needs, and safety seem unrealistic. Her expectations need considerate managing by the
multi-disciplinary team, with the help of the community psychiatric nurse whom she has known
for some time. It may be that she is experiencing guilt following their divorce and blames herself
for the fact that he can no longer live in his own home. The degree of variability in his behaviour
and functional abilities would require more than one live in carer, and his aggression would pose
considerable risk to the carers, especially as he has not been receptive to having other people
in his house in the past. Assistive technology could not help in this situation and transfer to the
psychiatry unit is simply failing to address the discharge needs. He has no obvious rehabilitation
potential and so transfer to a rehabilitation setting would also be inappropriate.
Ultimately the decision on most appropriate discharge placement will be determined by a
‘best interest’s decision’, preferably without having to disenfranchise the ex-wife by appointing
an IMCA.

Further reading
Department of Health. Ready to go? Planning the discharge and the transfer of patients from hospital
and intermediate care. Available at: http://www.dh.gov.uk/prod_consum_dh/groups/dh_digitalassets/@
dh/@en/@ps/documents/digitalasset/dh_116675.pdf
Mental Capacity Act 2005 (England). Available at: http://webarchive.nationalarchives.gov.uk/+/http://
www.dca.gov.uk/legal-policy/mental-capacity/mca-cp.pdf
Shepperd S, McClaran J, Phillips CO, et al. Discharge planning from hospital to home. The Cochrane
Library. Available at: http://onlinelibrary.wiley.com/doi/10.1002/14651858.CD000313.pub3/abstract;jse
ssionid=C0E46967BE215837EBBBE9C974A9491E.d01t01
EXAM 2 | ANSWERS 147

32. C.╇ The blood tests show that the patient is osteomalacic with levels of 25-OH colecalciferol
<25nmol/L. This places him at risk of osteoporotic fractures and proximal myopathy. Correction
of this may improve his long-term rehab potential and would require a stat bolus dose followed by
maintenance therapy. Maintenance therapy alone may be sufficient in individuals who are only mildly
deficient.
Given that vitamin D is highly fat-soluble, oral dosing is preferable and more rapidly hydroxylated,
and PO colecalciferol is, therefore, more appropriate for the initial ‘booster’ dose.
Activated forms of vitamin D, such as alfacalcidol, are only required when renal failure prevents
hydroxylation.

Further reading
British National Formulary. Available at: http://bnf.org
CDC. Fat soluble vitamins & micronutrients: vitamin D. 
Available at: http://www.cdc.gov/nutritionreport/99-02/pdf/nr_ch2b.pdf
Vieth R, Bischoff-Ferrari H, Boucher BJ, et al. The urgent need to recommend an intake of vitamin D
that is effective. American Journal of Clinical Nutrition 2007; 85: 649–50.
Available at: http://www.ajcn.org/content/85/3/649.full.pdf

33. C.╇ In this case major organic (especially malignant) disease is unlikely. His baseline
investigations are normal. A CRP of 22 in isolation should not be given undue weight and should
be considered normal at this age. In the absence of any clues within the history and physical
examination then published evidence suggests that further investigation is not immediately
warranted and that three months of ‘watchful waiting’ is advisable and to be preferred to
undirected and invasive testing. Serum tumour markers are not useful screening tools and should be
directed to monitoring treatment or detecting recurrence early.
Malnutrition has a high prevalence in older age and is more likely in men who live alone and those
with dementia. All older people with unintentional weight loss should have a nutritional assessment
and this should be the next step in his management before going on to ‘watchful waiting’.

Further reading
McMinn J, Steel C, Bowman A. Investigation and management of unintentional weight loss in older
adults. British Medical Journal 2011; 342: 754–9.
Available at: http://www.bmj.com/content/342/bmj.d1732.pdf%2Bhtml

34. C.╇ The delayed presentation puts the patient outside the time period in which reperfusion
therapies (thrombolysis or clot retrieval) are likely to provide any benefit. There is, however, risk
of on-going deterioration as the infarcted tissue becomes oedematous, and swells, and referral for
consideration of decompressive craniectomy should be considered.
Anticoagulation would increase the risk of haemorrhagic transformation and cardioversion is
contraindicated in stable patients with uncertain onset.

Further reading
NICE. Stroke: diagnosis and initial management of acute stroke and transient ischaemic attack (TIA),
Clinical guideline CG68. Available at: http://guidance.nice.org.uk/CG68
NIH Stroke Scale. Available at: http://www.ninds.nih.gov/doctors/NIH_Stroke_Scale.pdf
148 EXAM 2 | ANSWERS

35. B.╇ The heel ulcers probably result from his prolonged immobility during the hospital stay.
The mainstay of treatment is to relieve the pressure and improve his ability to reposition. This is
achieved through the ongoing physiotherapy and sitting out.
NICE suggest that a high-specification foam mattress may be appropriate for a grade one or two
ulcer as a minimum in conjunction with very close observation, but should be upgraded to an
alternating pressure mattress if there is any concern. On this basis, given that his mobility and ability
to reposition are improving, there is no automatic need to change his mattress. Further pressure
relief adjuncts, such as heel troughs may be of benefit overnight.
The community tissue viability team will be able to support the ongoing management, but are not a
treatment in their own right. The type of pressure sore is not consistent with a vascular pathology
and the normal examination and ABPI scores are reassuring. Vascular imaging would not be a
first line investigation. Providing he is eating well and not at nutritional risk, a modified diet is not
recommended. If a nutritional deficiency is identified, expert dietician input should be sought.

Further reading
NICE. Pressure ulcers: the management of pressure ulcers in primary and secondary care, Clinical
guideline CG29. Available at: http://www.nice.org.uk/guidance/index.jsp?action=byID&o=10972

36. A.╇ He had recently been started on an acetylcholinesterase inhibitor and the dose was
being increased. These drugs lower seizure threshold. Carer strain increases with increasing
demands placed on the carer (this equates to physical severity of Parkinson’s and degree of
cognitive impairment). It is important to consider elder abuse, as these injuries are excessive for
just having fallen of the commode. The wife has given a reasonable explanation of events and
seems appropriately concerned, making abuse less likely. Autonomic dysfunction accompanying
Parkinson’s disease might explain a fall, but would not usually result in such severe injuries if he
had simply fallen trying to stand up from the commode, in his wife’s absence. You are told he did
not have motor fluctuations and so dyskinesias are an unlikely explanation. The only neurological
abnormalities were those of the Parkinson’s, the absence of acute neurological deficit does not
imply he has had a TIA.

Further reading
British National Formulary. Available at: http://bnf.org/bnf/bnf/current/61145.htm
Drug Information Online. Rivastigmine. Available at: http://www.drugs.com/mmx/rivastigmine.html
Electronic Medicines Compendium. Available at: http://www.medicines.org.uk/emc/
House of Commons health Commission. Elder abuse. Available at: http://www.publications.parliament.
uk/pa/cm200304/cmselect/cmhealth/111/111.pdf and http://www.nlm.nih.gov/medlineplus/elderabuse.
html

37. A.╇ The underlying principle of the frailty index, or deficits accumulation model of frailty, is
that frailty is measured by the quantity of health deficits accumulated throughout life, rather than
the nature of those deficits. Despite having a higher frailty index score (more accumulated deficits),
than men of the same age, mortality rates are lower for women. Health deficits might have greater
impact on mortality in older males due to their lower physiological reserves (male ‘fitness-frailty
pleiotropy’), i.e. the price to pay for more optimal physiological functioning in youth is a lower
threshold for system failure in old age. Reduced energy and nutritional demands in younger women
might result in greater physiological reserves in old age and greater longevity of women who have
fewer children (female ‘fertility-frailty pleiotropy’).
EXAM 2 | ANSWERS 149

The Gompertz–Makeham model states that mortality is dependent upon vitality more than age, i.e.
frailty is more important than chronology.

Further reading
Golubev AG. The issue of feasibility of a general theory of aging. I. Generalized Gompertz-Makeham
Law. Advances in Gerontology 2009; 22: 60–73.
Available at: http://www.ncbi.nlm.nih.gov/pubmed/19827677
Hubbard RE, Rockwood K. Frailty in older women. Maturitas 2011; 69: 203–7.
Available at: http://www.ncbi.nlm.nih.gov/pubmed/21570783

38. B.╇ Warfarin should usually be stopped 5 days before invasive procedures and an INR checked
the day before, so that vitamin K can be given if the INR remains above 1.5.
The risks of perioperative anticoagulation are outweighed by the risk of recurrent DVT and/or
embolic stroke in high risk patients (previous stroke in AF, AF and multiple risk factors, metallic
mitral valve). The risk of embolic stroke in mechanical aortic valves is less than with mitral valves
and is outweighed by the risk of perioperative bleeding if anticoagulated.
Patients with venous thromboembolism (VTE) more than 3 months previously can be given
prophylactic low molecular weight heparin, rather than therapeutic bridging.

Further reading
ClinicalTrials.gov. PERIOP 2—A Safety and Effectiveness Study of LMWH bridging therapy versus
placebo bridging therapy for patients on long term warfarin and require temporary interruption of
their warfarin. Available at: http://www.clinicaltrials.gov/ct2/show/NCT00432796
ClinicalTrials.gov. Effectiveness of bridging anticoagulation for surgery (The BRIDGE Study).
Available at: http://www.clinicaltrials.gov/ct2/show/NCT00786474?term=bridge+warfarin&rank=2
Keeling D, Baglin T, Tait C. Guidelines on anticoagulation with warfarin, 4th edn. British Journal of
Haematology. Available at: http://www.bcshguidelines.com/documents/warfarin_4th_ed.pdf

39. C.╇ Pompholyx is an endogenous form of eczema (not caused by allergen or irritant) on
the palms or soles, which presents with itching, vesicles, and scales. Irritant and allergic contact
dermatitis present with the signs of eczema (itch, symmetrical erythema, vesicles, scaling, papules,
fissuring, and lichenification), but with a history of exposure to an irritant, e.g. solvents, detergents,
rubber gloves, cement, oil, nickel, cobalt. A history of regular hand washing or a cut-off of the
rash at the wrists (wearing of rubber gloves) will help identify the irritant. With eczema, there is
often evidence of rash in other areas, e.g. flexural sites (forearms, behind the knees, behind the
ears, eyelids) and the fingernails may be pitted. Psoriasis is not usually itchy and there are often well
defined plaques on the palms or elsewhere on the body (elbows, knees, ears, scalp, umbilicus, soles
of feet) as well as nail pitting onycholysis and subungal hyperkeratosis. Scabies is unlikely as there
is no suggestion of prolonged skin contact with her carers and no history of similar rash in other
residents or staff, although asymptomatic contacts do not exclude the diagnosis completely.

Further reading
Primary Care Dermatology Society. A–Z of clinical guidance—how to use.
Available at: www.pcds.org.uk/clinical-guidance-and-guidelines/a-z-of-diagnosis
NICE. NHS Clinical Knowledge Summaries. Clinical topics: skin and nails.
Available at: www.cks.nhs.uk/clinical_topics/by_clinical_specialty/skin_and_nail
150 EXAM 2 | ANSWERS

40. D.╇ It is the duty of the licence holder to inform the DVLA of any changes in their fitness
to drive.
If a patient lacks capacity and is not able to understand the need to inform the DVLA (e.g. because
of a dementia), current GMC guidance suggests you should inform the DVLA yourself.
If the patient has capacity, but disputes your assessment, you should support them in obtaining a
second opinion and advise them not to drive in the meantime.
If the patient continues to drive when they may not be fit you should make every reasonable effort
to persuade them to stop and inform the DVLA. If you do not manage, or if you discover they
are continuing to drive against advice, you should contact the DVLA and disclose relevant medical
information to the medical adviser. Before contacting the DVLA you should try to inform the
patient of your decision to disclose personal information and you should inform them in writing
when you have done so.

Further reading
Driver and Vehicle Licensing Agency. DVLA’s current medical guidelines for professionals—conditions
A to C. Available at: http://www.dft.gov.uk/dvla/medical/~/media/pdf/medical/at_a_glance.ashx
GMC. Confidentiality: reporting concerns about patients to the DVLA or the DVA. Available at: http://
www.gmc-uk.org/Confidentiality_reporting_concerns_DVLA_2009.pdf_27494214.pdf

41. D.╇ A 25micrograms/hr fentanyl patch would provide equivalent opioid dosing in the
most convenient route. The oramorph, although an equivalent dose, is likely to result in peaks
and troughs, and hence unreliable analgesia. A SC infusion would be unnecessarily invasive.
A 10micrograms/hr buprenorphine patch would be a significant reduction in the opiate dose.
Diclofenac tablets are generally smaller than MST, but the oral route is compromised and it
is preferable to avoid long-term NSAIDs owing to the risks of gastrointestinal and renal side
effects.
As a general rule:
• The breakthrough dose of oral morphine should be 1/6 the 24-hr oral dose.
• Subcutaneous diamorphine dose should be 1/3 the 24-hr oral morphine dose.
• A 12micrograms/hr fentanyl patch is equivalent to 45mg morphine/24hr.

Further reading
British National Formulary. Available at: http://www.bnf.org
National Council for Palliative Care. Available at: http://www.ncpc.org.uk/

42. A.╇ Further background information from his niece may indeed be helpful, but the first step
must be to try and find out from him what is agitating him. His frequent trips to the toilet may be
a clue, but a UTI is not the only possible cause of apparent frequency (as he will not let anyone in
with him you cannot be sure he is using the toilet or, if he is, that it is to pass urine) and you do not
know whether he has dysuria or not. He may be constipated, have diarrhoea or have frequency
related to prostate enlargement or even be in urinary retention.
His unkempt state, but the dog appearing well cared for may be relevant—he may be neglecting
himself whilst ensuring the dog is well fed and cared for.
He is not apparently at risk of harming himself or others so it would be inappropriate to sedate
him and this might make the situation worse by increasing his risk of falling, as well as increasing his
EXAM 2 | ANSWERS 151

risk of other complications, e.g. hypostatic pneumonia, pressure sores, venous thromboembolic
disease, further dehydration, and poor nutritional intake.

Further reading
Banerjee S. The use of antipsychotic medication for people with dementia: time for action A report
for the Minister of State for Care Services. Available at: http://www.dh.gov.uk/prod_consum_dh/
groups/dh_digitalassets/documents/digitalasset/dh_108302.pdf
British Geriatrics Society. Comprehensive assessment for the older frail patient in
hospital. Available at: http://www.bgs.org.uk/index.php/topresources/publicationfind/
goodpractice/195-gpgcgassessment
Scottish Intercollegiate Guidelines Network (SIGN). Management of patients with dementia, Guideline
86. Available at: http://www.sign.ac.uk/pdf/sign86.pdf
Sellas MI. Elder abuse. Medscape.
Available at: http://emedicine.medscape.com/article/805727-overview
Torke AM, Sachs GA. Self-neglect and resistance to intervention: ethical challenges for clinicians.
Journal of General Internal Medicine 2008; 23(11): 1926–7.
Available at: http://www.ncbi.nlm.nih.gov/pmc/articles/PMC2585692/
Ward KT, Reuben DB. Comprehensive geriatric assessment. Uptodate.
Available at: http://www.uptodate.com/contents/comprehensive-geriatric-assessment

43. C.╇ Although she denied chest pain or subjective shortness of breath, there is evidence of
raised right-sided heart pressures, sinus tachycardia, postural collapse, and hypoxia. Her active
cancer, relative immobility, and possible sepsis render her high risk for venous thromboembolism.
She should be anticoagulated and a confirmatory test undertaken.
There is no evidence of cardiac arrhythmia, making a primary cardiac cause less likely. Were
it an option, echocardiography could be helpful in looking for evidence of right heart strain.
Hypoadrenalism could be caused by adrenal metastases, but would not explain the hypoxia. This
should be considered if the CT pulmonary angiography (CTPA) is negative.

Further reading
Timmons S, Kingston M, Hussain M, et al. Pulmonary embolism: differences in presentation between
older and younger patients. Age & Ageing 2003; 32: 601–5.
Available at: http://ageing.oxfordjournals.org/content/32/6/601.full.pdf

44. B.╇ The acute nature means that delirium should be considered, but there is no suggestion
of fluctuating altered consciousness or inattention. Whilst she may well be missing her husband it
would be unusual for a grief reaction to manifest in this way, especially as she knows where he is. It
is also unlikely that she would become so depressed that she was neglecting herself, although one
should consider that she might be worried as to her husband’s well-being and prognosis. There is
no past history of psychiatric illness and so drug overdose is unlikely. It is most likely that she has
a dementia, especially as she has not been managing her medication for some months. As she is
diabetic this may well be a sub-cortical dementia due to small vessel disease, which would manifest
as failure to initiate despite being physically able to do tasks (‘can do, but does not initiate’). It is
likely that her husband was the ‘brains’ and told her what to do.

Further reading
Dementia diagnosis and assessment.
Available at: http://learning.bmj.com/learning/search-result.html?moduleId=5004452
152 EXAM 2 | ANSWERS

NICE. Delirium: diagnosis, prevention and management, Clinical guideline CG103.


Available at: http://guidance.nice.org.uk/CG103
NICE. Dementia. Available at: http://www.nice.org.uk/nicemedia/live/10998/30317/30317.pdf
NICE. Depression, Clinical guideline CG90. Available at: http://guidance.nice.org.uk/CG90
Woodford HJ, George J. Cognitive assessment in the elderly: a review of clinical methods. Quarterly
Journal of Medicine, 2007; 100: 469–84.
Available at: http://qjmed.oxfordjournals.org/content/100/8/469.full

45. B.╇ The mortality of hip fracture at one year is about 30%. Approximately 10% of people with
fractures die within the first month of the fracture. Most of the deaths are due to the associated
co-morbidities and not just the fracture. A fractured neck of femur is a marker of a frail individual.
Surgery within 48hr is associated with shorter length of stay and reduction in the time patients are
dependent or in severe pain.
Delirium, chest infection, and heart-failure peri-surgery are all associated with increased mortality
both acutely and at 1 year. Fractures in patients from institutional care are associated with higher
mortality than those from their own homes.

Further reading
National Osteoporosis Society. Osteoporosis facts and figures v1.1.
Available at: http://www.nos.org.uk/Document.Doc?id=47
NICE. Hip fracture: the management of hip fracture in adults, Clinical guideline CG124.
Available at: http://publications.nice.org.uk/hip-fracture-cg124/guidance
SIGN. Management of hip fracture in older people, Guideline 111.
Available at: http://www.sign.ac.uk/pdf/sign111.pdf
Cullen MW, Gullerud RE, Larson DR, et al. Impact of heart failure on hip fracture outcomes: a
population based study. Journal of Hospital Medicine 2011; 6(9): 507–12.
Available at: http://www.ncbi.nlm.nih.gov/pubmed/22042721

46. D.╇ The X-ray shows an atypical subtrochanteric fracture. These have been associated with
long-term bisphosphonate use and are thought to result from the inhibition of remodelling of small
cortical stress fractures.
Excessive alcohol, steroid use, proton pump inhibitors, and parental hip fractures are all associated
with increased risk of osteoporotic hip fracture, but this would usually be at the neck of femur.

Further reading
MRHA Drug Safety Update. Bisphosphonates: atpical femoral fractures.
Available at: http://www.mhra.gov.uk/Safetyinformation/DrugSafetyUpdate/CON120213
Shane E, Burr D, Ebeling P, et al. Atypical subtrochanteric and diaphyseal femoral fractures: report of a
task force of the american society for bone and mineral Research. Journal of Bone and Mineral Research
2010; 25: 2267–94. Available at: http://onlinelibrary.wiley.com/doi/10.1002/jbmr.253/full

47. A.╇ NICE now recommend starting feeding at a lower rate in patients at risk of re-feeding
syndrome with electrolyte correction in parallel. This is a shift from previous advice of correcting
electrolyte abnormalities first.
EXAM 2 | ANSWERS 153

Further reading
Mehanna HM, Moledina J, Travis J. Refeeding syndrome: what it is, and how to prevent and treat it.
British Medical Journal 2008; 336: 1495. Available at: http://www.bmj.com/content/336/7659/1495
NICE. Nutrition support in adults: oral nutrition support, enteral tube feeding and parenteral nutrition,
Clinical guideline CG32. Available at: http://publications.nice.org.uk/nutrition-support-in-adults-cg32

48. B.╇ The NIH Stroke Scale is used to gauge the severity of a stroke. It may be used to assess
recovery from stroke, but is not sensitive enough to detect progressive improvement in balance and
mobility. The TUG could be used to monitor such an improvement, however, the Rivermead Score
and Berg Balance Scale would be best suited to this.

Further reading
Hubbard R, Eeles E, Rockwood K. Impaired mobility. In: Fillit HM, Rockwood K, Woodhouse
K (eds) Brocklehurst’s Textbook of Geriatric Medicine and Gerontology, 7th edn (pp. 886–93).
Philadelphia: Saunders, 2010.

49. D.╇ Unidisciplinary and intra-professional both define a team of professionals who are all
from the same profession, such as three physical therapists collaborating on the same case.
Interdisciplinary defines a group of health care professionals from diverse fields who work in
a coordinated fashion toward a common goal for the patient. Such teams generally work in a
better coordinated way than multi-disciplinary teams, which comprise representatives of different
disciplines who coordinate the contributions of each profession, which are not considered to
overlap, in order to improve patient care. Multidisciplinary teams are therefore hindered by
professional boundaries with an absence of crossover of responsibilities, e.g. only the occupational
therapist will do a wheelchair assessment. Whilst these boundaries tend to be less obvious in
inter-disciplinary teams, trans-disciplinary teams are composed of members of a number of
different professions cooperating across disciplines to improve patient care through practice or
research. Trans-disciplinary working better defines the absence of professional boundaries and the
integration of research evidence in to practice. In such teams, it is not just the physiotherapists who
do stairs practice, and not just the speech and language therapist who assesses swallow.

50. B.╇ Graduated, knee high, elastic compression stockings (30–40mmHg) worn for 2 years after
first episode of proximal deep vein thrombosis (DVT) reduces the frequency of post-phlebitic
syndrome. They should probably also be recommended after ileo-femoral deep vein thrombosis.
Dabigatran etexilate is recommended, by NICE, as a possible treatment to reduce the risk of VTE
in adults who have surgery to replace their hip or knee joint. It is not recommended for treatment
of venous thromboembolic disease.
This patient has had an ileo-femoral DVT unrelated to a major reversible risk factor and so should
have at least 6 months treatment with warfarin and, as her risk, of further VTE, will remain high
(variable mobility and an ileo-femoral DVT occurring whilst receiving thromboprophylaxis) she
should be considered for life-long anticoagulation, with periodic reassessment of the risks and
benefits of continued anticoagulation.
Therapeutic sub-cutaneous low molecular weight heparin might be recommended in patients
where warfarin is contraindicated and in cancer patients. Provided there are no problems getting
this patient to take her medication, she will be supervised in the care home and there are no other
reasons specified to suggest that warfarin is contraindicated. Long-term treatment with heparin runs
a risk of osteopenia.
154 EXAM 2 | ANSWERS

Further reading
Goldhaber SZ, Jenkins JS, Kline JA, et al. Management of massive and submassive pulmonary embolism,
iliofemoral deep vein thrombosis, and chronic thromboembolic pulmonary hypertension: a scientific
statement from the American Heart Association. Circulation, 2011; 123: 1788–1830. Available at:
http://circ.ahajournals.org/content/early/2011/03/21/CIR.0b013e318214914f.full.pdf+html
NICE. Dabigatran etexilate for the prevention of venous thromboembolism after hip or knee
replacement surgery in adults, Guideline TA157, 2008. Available at: http://www.nice.org.uk/ta157
Scottish Intercollegiate Guidelines Network. Antithrombotic therapy, Section 2: Venous
thromboembolism. Available at: http://www.sign.ac.uk/pdf/qrg129.pdf

51. E.╇ The HbA1c of 75mmol/mmol shows unacceptable control and intensification of diabetes
treatment is needed. Insulin and gliclazide would both likely lead to weight gain, which could
further increase insulin resistance. They are also likely to exacerbate problems with hypoglycaemia.
Exenatide (a GLP-1 mimetic) is recommended by NICE for use as a third-line agent in patients with
a BMI >35kg/m2.
Sitagliptin (a DPP-4 inhibitor) is indicated for use as a second line agent where sulphonylureas are
contraindicated. They are associated with a lesser risk of hypoglycaemia.

Further reading
British National Formulary. Available at: http://bnf.org
Electronic Medicines Compendium.
Available at: http://www.medicines.org.uk/emc/medicine/19609/SPC/
NICE. Type 2 diabetes—newer agents (a partial update of CG66), Short guideline CG87.
Available at: http://www.nice.org.uk/CG87shortguideline

52. E.╇ Lack of interest in food and deterioration in oral intake is a recognized consequence of
advanced dementia. There is no convincing evidence of the benefit of tube feeding to improve
survival, aspiration rates, pressure ulcers, functional status, or patient comfort.
There is no overt sign of aspiration, so ‘routine’ speech and language therapy input is unlikely to
provide benefit.
It is possible that the change in behaviour and lack of intake could be secondary to his constipation,
so this should be addressed in the first instance. If depression is thought likely, a trial of an appetite
stimulating anti-depressant (e.g. mirtazapine) could be considered.
A dietician review and provision of supplements could also be considered.

Further reading
Finucane TE, Christmas C, Travis K. Tube feeding in patients with advanced dementia: a review of the
evidence. Journal of the American Medical Association 1999; 282(14): 1365–70.
Available at: http://jama.jamanetwork.com/article.aspx?articleid=191992
Mitchell SL, Teno JM, Kiely DM, et al. The clinical course of advanced dementia. New England Journal of
Medicine 2009; 361: 1529–38. Available at: http://www.nejm.org/doi/pdf/10.1056/NEJMoa0902234

53. E.╇ Pelvic floor exercises (PFE) are potentially useful for treating stress and mixed urinary
incontinence. PFE have not been shown to be effective in urge incontinence, but expert opinion
states that they may have a role in conjunction with bladder training. Pelvic floor strength and
endurance can adequately be assessed by either digital rectal or vaginal examination; digital rectal
examination may also be preferable to many women compared to digital vaginal examination.
EXAM 2 | ANSWERS 155

PFE regimens are as effective if delivered in groups or individually, however, around 35% of patients
do not wish to participate in group sessions. PFE programmes need to be tailored to the individual
if they are to be achievable. PFE can be done anywhere the person wishes to and most people can
manage to do them quite discretely. PFE are probably better demonstrated than described in a
leaflet, as this allows the individual to practice in the presence of the continence adviser.

Further Reading
NICE. The management of urinary incontinence in women, Clinical guideline CG40.
Available at: http://www.nice.org.uk/cg40
Scottish Intercollegiate Guidelines Network. Management of urinary incontinence in primary care, a
national guideline 79. Available at: http://www.sign.ac.uk/pdf/sign79.pdf

54. B.╇ Her initial BP reading is high, but she may be stressed by the clinic visit or have genuine
‘white coat hypertension’. Updated NICE guidance recommends ambulatory BP monitoring in this
situation to improve the accuracy of diagnosis. Such practice has been shown to be cost effective
and prevent unnecessary prescribing of antihypertensives. If she already had a home BP monitor
(one recommended by the British Hypertension Society), then it would be reasonable to get her to
measure her own BP—average of two consecutive readings, at least one minute apart, twice daily
for at least 4 days and, ideally, 7 days. She is not overweight.

Further reading
British Hypertension Society. BP Monitors.
Available at: http://www.bhsoc.org/bp-monitors/bp-monitors/
NICE. Hypertension clinical management of primary hypertension in adults (update), Clinical guideline
CG127. Available at: http://guidance.nice.org.uk/CG127

55. B.╇ Given that other reversible and non-pharmacological options have been explored, a course
of regular antipsychotic may help to moderate his hyperactive episodes and reduce the risk to
the patient and others. Only rispderidone has a license for such use in the UK. While quetiapine
and olanzapine are used in this way, and olanzapine or haloperidol is recommended by NICE, the
doses suggested here are too large to start with. Any antipsychotic prescribed should be reviewed
regularly.

Further reading
British National Formulary. Available at: http://www.bnf.org
NICE. Delirium: diagnosis, prevention and management, Clinical guideline CG103.
Available at: http://www.nice.org.uk/cg103

56. B.╇ The initial history is suggestive of a delirium post treatment for a UTI. It is not uncommon
for delirium to manifest after the precipitant has been removed (cf. jet-lag). Delirious symptoms
may persist for weeks or months after removal of the precipitating cause(s), however, persistent
delirium is more common in those with more severe illness(es); longer duration of acute illness and
more severe prior cognitive impairment. The second acute illness will have delayed resolution from
her original delirium and she may still have an unresolved precipitant—pain from her osteoarthritis.
The normal cortisol and thyroid function exclude Cushing’s and thyrotoxicosis as rarer causes
of psychotic illness. The absence of prior psychiatric illness makes a first episode of psychotic
depression less likely. The family describe an acute confusional state in someone previously
independent, so an unrecognized dementia, although possible, is unlikely to be the sole explanation,
although this may be a delirium on a background of previously unrecognized dementia.
156 EXAM 2 | ANSWERS

Further reading
Brownlie BEW, Rae AM, Walshe JWB, Wells JE. Psychoses associated with thyrotoxicosis—‘thyrotoxic
psychosis.’ A report of 18 cases, with statistical analysis of incidence. European Journal of Endocrinology
2000; 142: 438–4. Available at: http://www.eje-online.org/content/142/5/438.full.pdf
Kirk LF, Hash RB, Katner HP, Jones T. Cushing’s disease: clinical manifestations and diagnostic evaluation.
American Family Physician 2000; 62(5): 1119–27.
Available at: http://www.aafp.org/afp/2000/0901/p1119.html
NICE. Delirium: diagnosis, prevention and management, Clincial guideline CG103.
Available at: http://guidance.nice.org.uk/CG103
NICE. Depression: the treatment and management of depression in adults (update), Clinical guideline
CG90. Available at: http://guidance.nice.org.uk/CG90
NICE. Depression in adults with a chronic physical health problem: Treatment and
management, Clinical guideline CG91. Available at: http://publications.nice.org.uk/
depression-in-adults-with-a-chronic-physical-health-problem-cg91
Rodda J, Walker Z, Carter J. Depression in older adults.
Available at: http://www.bmj.com/content/343/bmj.d5219.extract

57. D.╇ It is likely that the patient has been non-compliant with therapy at home. This could account
for the recurrent seizures and lack of improvement with increased doses of sodium valproate.
Now that she has stayed in hospital she is taking the medication, effectively being started at a dose
of 800mg BD. Addressing the poor compliance and reducing in the dosing should reverse the
deterioration.

Further reading
NICE. The epilepsies: diagnosis and management, Clinical guideline CG137. Available at: http://
publications.nice.org.uk/the-epilepsies-the-diagnosis-and-management-of-the-epilepsies-in-adults-and-
children-in-primary-and-cg137

58. D.╇ Vitamin D supplementation is only thought to beneficial in reducing falls’ risk in individuals
with low levels.

Further reading
Gillespie LD, Robertson MC, Gillespie WJ, Lamb SE, Gates S, Cumming RG, Rowe BH. Interventions
for preventing falls in older people living in the community. Cochrane Database of Systematic Reviews
2009, Issue 2: CD007146. Available at: http://onlinelibrary.wiley.com/doi/10.1002/14651858.
CD007146.pub3/abstract

59. E.╇ The most likely explanation is that this is due to the diamorphine. Parkinson’s disease
symptoms change gradually and any sudden change usually has an external precipitant, i.e. is a
reaction to a ‘stressor’, be it either physical or emotional. Cerebral metastases from her presumed
primary lung cancer must be considered, but in the absence of other neurological deficit, is less
likely than drug toxicity. You are given no information about her vision and so, whilst Charles
Bonnet syndrome is defined as hallucinations due to poor visual acuity in the presence of normal
cognition, there is no reason to invoke a new diagnosis. She is said to be cognitively intact and so
cannot have a diagnosis of Dementia with Lewy bodies.
EXAM 2 | ANSWERS 157

Further reading
NICE. Delirium: Diagnosis, prevention and management. Clinical guideline CG103.
Available at: http://guidance.nice.org.uk/CG103

60. B.╇ While alpha blockade is usually first line for treatment of symptoms related to benign
prostatic hyperplasia, they can be associated with postural hypotension. This is usually worse
with terazosin and doxazosin and less pronounced with Tamsulosin M/R. Given resolution of the
orthostatic drop on stopping this treatment, this class of drug should be avoided. Finasteride (an
anti-androgen) is a reasonable alternative.
Oxybutinin and solifenacin are anticholinergics. In the context of BPH, they risk precipitating
bladder outflow obstruction.
Referral to an urologist for consideration of surgery would also be an option.

Further reading
British National Formulary. Available at: http://www.medicinescomplete.com/mc/bnf/current/
NICE. Lower urinary tract symptoms: the management of lower urinary tract symptoms in men,
Clinical guideline CG97.
Available at: http://publications.nice.org.uk/lower-urinary-tract-symptoms-cg97

61. A.╇ The most likely explanation is that she is experiencing overflow incontinence once her
bladder is ‘full’. The lack of symptoms in the morning and overnight, correspond with the periods
after draining her bladder. Re-catheterizing herself earlier in the day, or even a third time, may be
sufficient to control her symptoms. If this is not successful, a long-term catheter may be needed.
Anticholinergics may further impair the ability to void in between catheterizations. Although
an alpha blocker may reduce urethral tone, this is likely to increase leaking and not address the
underlying pathophysiology relating to her traumatic delivery. Additional containment methods such
as continence pads should be a last resort.

Further reading
NICE. Urinary incontinence: the management of urinary incontinence in women, Clinical guideline
CG40. Available at: http://www.nice.org.uk/guidance/CG40

62. C.╇ The NICE stepwise management of COPD guideline would suggest addition of a long-
acting beta agonist or long-acting muscarinic antagonist at this point. If this increase does not
adequately control symptoms, then therapy should be escalated (in a stepwise fashion) to a
combination long-acting beta agonist and inhaled steroid inhaler and subsequently triple therapy
with a long-acting beta agonist, long-acting muscarinic antagonist, and inhaled corticosteroid.
There may be some emerging evidence that long-acting muscarainic antagonists are, in fact,
preferable to long-acting beta agonists as a step-up from reliever treatment alone, as they may have
a disease modifying role. Future guidelines may place them as first line long-acting therapy.
There is no role for mucolytic therapy in patients without a chronic cough, or to prevent
exacerbations.

Further reading
NICE. Chronic obstructive pulmonary disease: Management of chronic obstructive pulmonary disease
in adults in primary and secondary care (partial update), Clinical guideline CG101.
Available at: http://guidance.nice.org.uk/CG101
158 EXAM 2 | ANSWERS

Decramer M, Celli B, Kesten S, et al. Effect of tiotropium on outcomes in patients with moderate
chronic obstructive pulmonary disease (UPLIFT): a prespecified subgroup analysis of a randomized
controlled trial. Lancet 2009; 374: 1171–8. Available at: http://www.thelancet.com/journals/lancet/
article/PIIS0140-6736(09)61298-8/abstract

63. C.╇ The first step in taking matters further is to discuss with the patient and assess her capacity.
She may have a perfectly legitimate explanation for how the injuries have been sustained (e.g.
falls). Even if there is no good explanation, the patient needs to be informed and to consent to
involvement of other agencies. Individuals with capacity have the right to make decisions about
their safety, even if others think these decision are unwise. If an individual lacks capacity, then action
should be taken in their best interests and the case referred to the local safeguarding team.
However, the patient lives in a care home. If there is concern that other residents are potentially at
risk if it transpires that the injuries are a result of poor care, there may be justification in escalating
the situation even if the patient does not explicitly consent. If this is felt to be needed, senior advice
should be sought and the patient informed of what is happening. Every effort should be made to
maintain their confidentiality if at all possible.

Further reading
Department of Health—Safeguarding adults: the role of health services. Available at: http://www.
dh.gov.uk/en/Publicationsandstatistics/Publications/PublicationsPolicyAndGuidance/DH_124882

64. A.╇ It is likely that his headache and nausea result from raised intracranial pressure.
Dexamethasone may relieve this and also provide a degree of temporary neurological improvement
by reducing any oedema. He has only required one dose of PRN diamorphine, so continuous
infusion of this is not indicated at this stage.

Further reading
British National Formulary. Available at: http://www.bnf.org.uk
National Council for Palliative Care. Available at: http://www.ncpc.org.uk/

65. D.╇ The patient has check-listed negative for continuing health care. A positive checklist consists
of one of the following: 1A*, 2As, 1A and 4Bs, or 5Bs.
The negative checklist means that his ongoing social care will be means tested. To include his house
in his assets would render his wife homeless and social services would, therefore, not count this.
However, a charge may be applicable when assets are released, e.g. if the wife chooses to sell the
house. The lack of other significant assets means he will qualify for full social services funding in the
interim.
If he were to have had sufficient assets then he would be eligible for the full cost less the nursing
contribution (£108.70/week in 2011), which would be paid for by the NHS.

Further reading
Department of Health. The national framework for NHS continuing healthcare and NHS funded
nursing care—July 2009 (revised). Available at: http://www.dh.gov.uk/prod_consum_dh/groups/
dh_digitalassets/documents/digitalasset/dh_103161.pdf
Department of Health. Social care.
Available at: http://www.dh.gov.uk/health/category/policy-areas/social-care/
Directgov. Health and social care assessments. Available at: http://www.direct.gov.uk/en/
DisabledPeople/HealthAndSupport/ArrangingHealthAndSocialCare/DG_4000436
EXAM 2 | ANSWERS 159

Department of Health. Legislation. Available at: http://www.dh.gov.uk/en/Publicationsandstatistics/


Legislation/Actsandbills/index.htm

66. C.╇ The headache, photophobia, and nausea are consistent with migraine. The neurological
deficit is likely due to a preceding aura.

Further reading
Chawla J. Migraine headache. Medscape.
Available at: http://emedicine.medscape.com/article/1142556-overview
Hand PJ, Kwan J, Lindley RI, et al. ‘Distinguishing between stroke and mimic at the bedside’ Stroke 2006;
37: 769–75. Available at: http://stroke.ahajournals.org/content/37/3/769.full

67. D.╇ It is a one-bedroom property so there is nowhere for a live-in carer to sleep; even if
there were somewhere, the carer would still need some means of knowing that the couple were
up and had left the house. GPS tracker phone or other device would help locate the couple (if
they remembered to take it with them), but would not alert to the fact that they had left the
house (wandered off ). Their expressed wish is to avoid institutional care and so this should be a
last resort. A wrist actigraph, worn like a wristwatch, can be useful to alert to someone moving
about, but would not tell you where they were and so would be more of a nuisance than benefit
in monitoring the couple’s safety. A property exit sensor could alert the family if the front door is
opened during the night, enabling someone to then go and fetch the couple back to the house.

Further reading
Department of Health. Building telecare in England, 2005. Available at: http://www.leonardo-energy.
org/sites/leonardo-energy/files/root/pdf/2005/dh_4115644.pdf
Francis J, Holmes P. Ethical issues in the use of telecare, SCIE Report 30, 2010.
Available at: http://www.scie.org.uk/publications/reports/report30.pdf
Kerr B, Hurst K, Clark S. Telehealthcare and mental health: using telehealthcare effectively in the
support of people living with mental disorder. London: King’s Fund, 2011.
Available at: http://kingsfund.koha-ptfs.eu/cgi-bin/koha/opac-detail.pl?biblionumber=103952
Martin S, Kelly G, Kernohan WG, et al. Smart home technologies for health and social care support.
Cochrane Database of Systematic Reviews 2005; Issue 4: CD006412.
Available at: http://onlinelibrary.wiley.com/doi/10.1002/14651858.CD006412.pub2/abstract
Poole T. Telecare in older people: Wanless social care review. London: King’s Fund, 2006.
Available at: http://kingsfund.koha-ptfs.eu/cgi-bin/koha/opac-detail.pl?biblionumber=37212

68. B.╇ He has advanced cancer which has already been deemed for palliative care by the oncology
service and he has no need for symptom control at this time. His wish was to die at home with his
wife present. The family have made it clear that she will need residential home placement once
their father dies; if he is placed in a nursing home she would not be able to join him unless it was a
dual registered home; similarly, he has too high care needs for residential placement. So, it would
be most appropriate to meet his preferred place of care with the family and social worker having a
residential placement ready for his wife once he dies. He does not have long to live, so this should
be a fast-track discharge with palliative care support for him and possibly a temporary increase in
care for his wife.

Further reading
Gold Standards Framework. Available at: http://www.goldstandardsframework.org.uk/
160 EXAM 2 | ANSWERS

NHS. Preferred priorities for care.


Available at: http://www.endoflifecare.nhs.uk/tools/core-tools/preferredprioritiesforcare

69. A.╇ The history will usually allow clinical differentiation from other causes of diarrhoea and
help identify potentially reversible risk factors, e.g. use of ranitidine, sertraline, aspirin. Initial
investigation with blood tests, radiological examinations, and stool microscopy and culture, are
typically normal in microscopic colitis, unless there is co-existent pathology. Therefore, lower bowel
endoscopy is necessary to make the diagnosis. The macroscopic findings of flexible sigmoidoscopy
and colonoscopy are usually unremarkable. For this reason, if microscopic colitis is suspected it is
necessary to request random colonic biopsies, to make a histological diagnosis.

Further reading
Goosenberg E. Collagenous and lymphocytic colitis. Medscape.
Available at: http://emedicine.medscape.com/article/180664-overview
Williams JJ, Beck PL, Andrews CN, et al. Microscopic colitis—a common cause of diarrhoea in older
adults. Age & Ageing 2010; 39(2): 162–8.
Available at: http://ageing.oxfordjournals.org/content/39/2/162.full

70. D.╇ Osteoporosis is a condition in which the density and quality of bone is reduced. It results
from a mis-match between peak bone density, bone resorption, and bone deposition. Osteomalacia
describes inadequate bone mineralization of an otherwise normal bone matrix.
The World Health Organization define a person as having osteoporosis when their bone mineral
density measurement (T-score) is less than 2.5SD below that of a reference population of young,
healthy adults.
A complex interplay between environmental, hormonal, and cellular pathways result in preferential
osteoclast activity (resorb bone) in excess of that of osteoblasts (lay down new bone).
Osteoporosis increases the risk of fracture, but is not defined by it. A low-trauma fracture in the
presence of osteoporosis is labelled a fragility fracture. Owing to the increasing prevalence of
osteoporosis with age, particularly in females, NICE suggests treating all women aged over 75 years
with fragility fractures as if they have osteoporosis, without the need for bone mineral density
testing.

Further reading
International Osteoporosis Foundation. What is osteoporosis.
Available at: http://www.iofbonehealth.org/what-osteoporosis-1
Jacobs-Kosmin D. Osteoporosis. Medscape.
Available at: http://emedicine.medscape.com/article/330598-overview

71. C.╇ It is important to exclude any environmental factors that may be impairing her ability to
function at home. It is more difficult to stand from a low chair and provision of a suitable chair,
with arm rests to enable upper limb strength to be recruited to assist standing may be sufficient to
enable her to continue independently. A full occupational therapy home assessment may be useful
as there may be a role for other pieces of equipment or adaptations, such as toilet frames.
Arranging care and or suggesting placement may ultimately result in further disempowering the
patient. Similarly, regular use of a riser-recliner chair can result in muscle atrophy and greater
difficult in standing from a normal chair.
Phyisotherapy would be more appropriate in the home environment once other issues have been
addressed.
EXAM 2 | ANSWERS 161

Further reading
Podsiadlo D, Richardson S. The timed Up & Go: a test of basic functional mobility for frail elderly
persons. Journal of the American Geriatrics Society 1991, 39(2):142–8.
Available at: http://ukpmc.ac.uk/abstract/MED/1991946

72. E.╇ The BNF suggests that a maximum dose of 3g paracetamol per 24hr be prescribed to
patients with hepatocellular insufficiency, chronic alcoholism, chronic malnutrition, or dehydration.
The limited dietary intake, weight loss, and raised MCV all point to chronic malnutrition in this case.
He is hypertensive, amlodipine would be an appropriate choice following the British Hypertension
Society Guideline.
40mg is the correct prophylactic dose of enoxaparin for a medical patient. He has normal renal
function and BMI.
Gentamicin may be an appropriate antibiotic for urosepsis (depending on local policy). 3mg/kg OD
is appropriately at the lower end of the dosing spectrum. Close monitoring of renal function and
hearing would be essential.
Pabrinex would be appropriate given the poor nutritional state

Further reading
British National Formulary. Available at: http://www.bnf.org
Williams B, Poulter NR, Brown MJ. British Hypertension Society Guidelines for hypertension
management 2004 (BHS-IV): summary.
Available at: http://www.bhsoc.org/pdfs/Summary%20Guidelines%202004.pdf

73. B.╇ A fall in BP results in renal artery hypoperfusion, reduced delivery of sodium to the
distal tubules and sympathetic activation. This results in increased renin secretion from the
juxtaglomerular cells. Renin acts upon angiotensinogen, cleaving it to angiotensin-1. This is in turn
converted to angiotensin-2 (A2). A2 acts (amongst other things) to:
• Constrict resistance vessels and raise systemic vascular resistance.
• Promote aldosterone secretion (hence, sodium retention in kidneys).
• Stimulate vasopressin release, increasing water reabsorption from the collecting ducts.
• Stimulate thirst centres.
• Stimulate cardiac and vascular hypertrophy.
The effect of these is to increase circulating volume, vascular resistance, and hence BP.
Atrial-natriuretic polypeptide (ANP) is not activated by the renin-angiotensin system and, in
many ways, antagonizes it. ANP is released from the atria in response to high BP and promotes
vasodilatation, inhibits renin secretion, and promotes sodium and fluid excretion.

Further reading
Klabune RE. Cardiovascular physiology concepts—renin-angiotensin-aldosterone system.
Available at: http://www.cvphysiology.com/Blood%20Pressure/BP015.htm
Klabune RE. Cardiovascular physiology concepts—atrial and brain natriuretic peptides.
Available at: http://www.cvphysiology.com/Blood%20Pressure/BP017.htm

74. D.╇ Work on C. elegans and the insulin/IGF pathway have suggested that reductions in IGF
and growth hormone (GH) levels are protective against ageing. GH receptor knockout and IGF-1
receptor heterozygote dwarf mice are long-lived. The cellular pathway for this effect is not clear,
162 EXAM 2 | ANSWERS

but is thought to be mediated through Daf-16 in C. elegans, which corresponds to FOXO in


mammals.
Chronic infection results in increased cell damage, inflammation, and free-radical production. Free
radical damage is postulated to cause cellular protein and DNA damage, disrupting cellular function
and ultimately causing cell death.
Telomere shortening is thought to limit the number of cell divisions a given cell can make and
thereby impede repair mechanisms.
Maternal caloric restriction may have an adverse effect on organogenesis and epigenetic
programming, influencing the later development of cardiovascular disease and diabetes.

Further reading
Fillit HM, Rockwood K, Woodhouse K. Brocklehurst’s Textbook of Geriatric Medicine and Gerontology, 7th
edn, 2010, Chapters 1–7. Saunders Elsevier, Philadelphia.

75. E.╇ Glucocorticoids, proton pump inhibitors, thyrotoxicosis, and certain anti-epileptic drugs
(including phenytoin, carbamazepine, and valproate) are well recognized to cause osteoporosis.
Prolonged excess thyroxine replacement, as evident in this case, can do the same. There is no
association between tramadol and osteoporosis.

Further reading
British National Formulary. Available at: http://bnf.org
Khalili H, Huang ES, Jacobson BC, et al. Use of proton pump inhibitors and risk of hip fracture in
relation to dietary and lifestyle factors: a prospective cohort study. British Medical Journal 2012; 344:
e372. Available at: http://dx.doi.org/10.1136/bmj.e372.
National Osteoporosis Foundation. Medicines that may cause bone loss.
Available at: http://www.nof.org/articles/6

76. A.╇ There is no evidence to suggest that the patient is suffering a disease of the mind or brain.
He is able to understand, retain, and weigh-up pertinent information and still expresses a wish to
decline treatment. He has capacity and his wishes should be respected.
It would be sensible at this point to obtain a second opinion from an independent senior clinician
and to encourage the patient to discuss further with friends, family, and spiritual advisers.
Although the decision to decline treatment may seem illogical to the treating team, it should be
respected. Life-prolonging treatment is therefore not appropriate and the emphasis should be on
symptom control.
Neither the Mental Capacity Act, nor the Mental Health Act provide authority for clinicians to treat
patients who lack capacity for physical conditions. The mental health act can provide authority for
forced treatment of psychiatric conditions. Treatment of physical conditions in both instances is on
a common-law basis.

Further reading
National Archives. Mental Capacity Act 2005.
Available at: http://www.legislation.gov.uk/ukpga/2005/9/contents
Department of Health. Mental Health Act 1983
Available at: http://www.dh.gov.uk/en/Publicationsandstatistics/Legislation/Actsandbills/DH_4002034
EXAM 2 | ANSWERS 163

77. C.╇ When designing any service, it is most important to determine the patient group(s) that
the service is for. In order to ensure ‘the right patient in the right place at the right time’ one must
know what is wrong with the patient to ensure the service and staff can best meet that patient’s
needs. A clinical diagnosis is therefore of paramount importance. Staffing skill mix and medical input
will then be determined by the type of patients being managed by the service; for instance, not all
step-up care services will necessarily require consultant input beyond the initial diagnostic phase, or
may only need it on request. Interdisciplinary team working may have benefits over other styles of
team working, but each team must be allowed to develop its optimum working practice. The clinical
governance structure is then important to ensure that the service is performing optimally and safely.

Further reading
Department of Health (England). Intermediate care—Halfway home. Updated guidance for the NHS
and Local Authorities. London: DoH, 2009. Available at: http://www.dh.gov.uk/prod_consum_dh/
groups/dh_digitalassets/@dh/@en/@pg/documents/digitalasset/dh_103154.pdf
National audit of intermediate care report. 2012. Available at: http://www.nhsbenchmarking.nhs.uk/
projects/partnership-projects/National-Audit-of-Intermediate-Care.php

78. D.╇ The facial injury would be consistent with a syncopal fall. There is evidence of postural
hypotension on standing from a sitting position, although formal lying and standing BP recordings
should be pursued. The electrolytes demonstrate mild hyponatraemia and hyperkalaemia. These
features could be explained by hypoadrenalism, which ought to be excluded with a synacthen test.
Cardiac examination and resting ECG are unremarkable and this points against the need for a 24-hr
tape or echocardiography. There is already evidence of postural hypotension and NICE do not
recommend tilt testing as a first line investigation.
If the synacthen test is normal, carotid sinus massage would be appropriate as a subsequent
investigation.

Further reading
NICE. Transient loss of consciousness (‘blackouts’) management in adults and young people, Clinical
guideline CG109. Available at: http://www.nice.org.uk/nicemedia/live/13111/50452/50452.pdf

79. D.╇ Gait rehabilitation is important after stroke. Use of inappropriate aids can result in
overcompensation and long-term gait abnormalities. While a stick used in the unaffected hand
has been shown to reduce sway and move the centre of body mass towards the sound limb, this
beneficial effect is more marked with a four point stick.
This patient would be unable to use the Zimmer, rollator, or gutter frame owing to the limb
weakness.

Further reading
Disability Living Foundation. Choosing walking equipment. Available at: http://www.dlf.org.uk/
factsheets/Choosing_Walking_Equipment.pdf
Elmamoun M, Mulley G. Walking sticks and frames for patients with neurological disorders. Practical
Neurology 2007; 7: 24–31 Available at: http://pn.bmj.com/content/7/1/24.extract

80. B.╇ The cognitive decline over nine months, coupled with impaired cognitive test scores is
consistent with a diagnosis of moderate severity dementia. The most likely subtype given the CT
findings is Alzheimer’s. The first line agents would usually be acetylcholinesterase inhibitors such as
donepezil or rivastigmine. However, the abnormal ECG and history of syncope would make these
less preferable.
164 EXAM 2 | ANSWERS

Memantine is a glutamate receptor anatagonist, which is licenced in the UK for use in severe
Alzheimer’s disease, or in moderate disease where acetylcholinesterase inhibitors are
contra-indicated.
It is always worth considering depression, but the lack of clinical features and reassuring score on
the geriatric depression scale count against this.
There are no features of challenging behavioural or psychological symptoms that would warrant
treatment with an anti-pschotic such as risperiodne.

Further reading
British National Formulary. Available at: http://www.bnf.org/bnf/index.htm
NICE. Dementia: supporting people with dementia and their carers in health and social care, Clinical
guideline CG42. Available at: http://www.nice.org.uk/CG42

81. D.╇ Although anaemia might be expected with the rheumatoid arthritis and methotrexate use,
the low MCV raises concerns of iron deficiency. A GI malignancy with cerebral metastases needs to
be excluded.
Strokes, like other vascular events, are sudden onset and do not fluctuate. It is possible for initial
strokes to extend, or to be exacerbated by haemorrhagic transformation, but this would not be
expected over such a timescale. Similarly the progressive, but fluctuating nature of the presentation
is not consistent with recurrent TIAs.
Although a venous sinus thrombosis can present with fluctuating symptoms, the negative d-dimer
result makes this less likely.

Further reading
NICE. Stroke: diagnosis and initial management of acute stroke and transient ischaemic attack (TIA),
Clinical guideline CG68. Available at: http://www.nice.org.uk/CG68
McElveen WA. Cerebral venous thrombosis. Medscape.
Available at: http://emedicine.medscape.com/article/1162804-overview

82. C.╇ If the patient were compromised, then DC cardioversion would be the treatment of
choice. However, given the often paroxysmal nature of Torsades, it frequently recurs. In a stable
patient, drug treatment to suppress early after depolarization will terminate the arrhythmia.
Magnesium achieves this by decreasing the influx of calcium. Amiodarone and verapamil both
prolong the QT interval and can further exacerbate the situation. Overdrive pacing may prove
successful, but non-invasive methods should be tried first.

Further reading
Dave J. Torsade de Pointes. Medscape.
Available at: http://emedicine.medscape.com/article/1950863-overview

83. C.╇ Squamous cell carcinoma should be considered for all non-healing ulcers. The fact the
patient first noticed it after mild trauma may merely be co-incidental. The wound swab showing
skin flora only renders on-going infection unlikely as a cause for persistence. The re-assuring pedal
pulses and atypical location count against ischaemia and the CT angiography.
Silver-based dressings may be useful as an antibacterial agent and VAC dressings may accelerate
wound healing, however, given the lack of response so far, exclusion of a malignant process must
take precedence.
EXAM 2 | ANSWERS 165

Further reading
DermNet NZ. Squamous cell carinoma of the skin: 
Available at: http://dermnetnz.org/lesions/squamous-cell-carcinoma.html
DermNet NZ. Leg ulcers: Available at: http://dermnetnz.org/site-age-specific/leg-ulcers.html

84. A.╇ The history would be consistent with somatic, psychomotor, and psychosocial features
of pain and would score positive with a non-verbal pain assessment tool (e.g. Doloplus-2). The
pressure sore is a likely cause.
It would therefore be appropriate to start some analgesia with ongoing review of the effect.
She is unlikely to be able to manage oral paracetamol, given the non-compliance. Intravenous
or PR paracetamol could be an alternative, but is not listed as an option. A buprenorphine 5 or
10micrograms/hr patch will provide a similar analgesic effect. A 12micrograms fentanyl patch is
likely to prove too strong in an opiate naive patient and would not be advised first line to avoid
toxicity.
The urine dip would not be consistent with bacterial infection and consequently there is no
indication for antibiotics. Analgesia is preferable to antipsychotics in the first line, as it will treat the
cause (and consequently the agitation), rather than merely masking it.

Further reading
Hølen JC, Saltvedt I, Fayers M, et al. Doloplus-2, a valid tool for behavioural pain assessment? BMC
Geriatrics 2007; 7: 29. Available at: http://www.biomedcentral.com/1471-2318/7/29

85. A.╇ There is no specific medical advice for Ménière’s disease and driving. As he had found
this episode quite disabling, he should follow the guidance for those liable to ‘sudden attacks of
unprovoked or unprecipitated disabling giddiness’, to ‘cease driving on diagnosis and resume once
satisfactory control of symptoms achieved’ that, with Meniere’s, can be interpreted as once the
vertigo and imbalance resolve. Physical frailty is not per se a bar to the holding of a driving licence,
neither is vertigo related to middle ear disease. He need not notify the DVLA nor does he need to
take an ‘advanced driving’ course.

Further reading
DVLA. DVLA’s current medical guidelines for professionals—conditions A to C. 
Available at: www.dft.gov.uk/dvla/medical/~/media/pdf/medical/at_a_glance.ashx

86. A.╇ Bullous pemphigoid typically occurs in older age (>70yrs). It may start as small patches of
itchy, erythematous skin, and be mistaken for eczema; blistering usually begins within a week, but
may occur months later. The blisters tend to be tense and dome-shaped; usually filled with clear
fluid, but this can be cloudy or blood-stained. Any area of skin can be affected, but most commonly
the arms, legs, armpits, and groin. It may be several days before the blister bursts, some do not and
the fluid is absorbed with the roof of the blister settling back down on the skin. The blisters usually
heal without forming scars even if they burst. The amount of itch can vary from severe to none
at all.
This would be an unusual age to present with dermatitis herpetiformis, it usually begins in earlier
adult life. This is an extremely itchy blistering rash which is usually symmetrical and may come and
go. The rash is most often on the elbows, knees, back, and buttocks. Dermatitis herpetiformis is
frequently linked to coeliac disease.
166 EXAM 2 | ANSWERS

Impetigo starts as a small patch of blisters that break into red, moist areas that weep fluid. It mostly
affects the face, but may occur on exposed areas of the arms and legs. After a few days the lesions
crust and new blisters appear at the margins.
Pemphigus blisters are fragile and burst easily, leaving raw patches of skin that do not heal without
treatment. If firm pressure is exerted on unaffected skin and a shearing strain applied, a blister will
appear. The blisters are frequently itchy and painful, and feel as if the skin is burning. Pemphigus
vulgaris is the most common form—blisters usually first appear in the mouth. In pemphigus
foliaceus blisters do not occur in the mouth and those on the skin tend to be itchy, rather than
painful.
Stevens–Johnson syndrome may involve the skin and mucous membranes. Cutaneous lesions begin
as macules and eventually develop in to the pathognomic ‘target’ lesions. These may then blister
and rupture. Mucosal lesions are oedematous, sloughing lesions with blisters that ulcerate and
become necrotic.

Further reading
Best Practice. Assessment of vesicular-bullous rash. Available at: http://bestpractice.bmj.com/best-
practice/monograph/775.html
Chan LS. Bullous pemphigoid. Medscape. Available at: http://emedicine.medscape.com/
article/1062391-overview
Foster CS. Stevens-Johnson syndrome. Medscape. Available at: http://emedicine.medscape.com/
article/1197450-overview
Lewis LS. Impetigo. Medscape. Available at: http://emedicine.medscape.com/
article/965254-overview
Miller JL. Dermatitis herpetiformis. Medscape. Available at: http://emedicine.medscape.com/
article/1062640-overview
Schwartz RA. Pemphigus folacious. Medscape. Available at: http://emedicine.medscape.com/
article/1064019-overview

87. B.╇ The ischaemic stroke is likely acting as an epileptogenic focus and he is currently in a post-
ictal phase. The emphasis is palliative and the principle aims are to ensure he is seizure free and
remains comfortable.
NGT insertion is inappropriate in this circumstance and the reactionary use of diazepam will
terminate, but not prevent further seizures.
The use of SC midazolam could achieve both goals. This could also be combined with an anti-
secretory such as glycopyrrhonium in the same syringe driver, thereby addressing the excessive
secretions and ‘death rattle’. There is no pain or respiratory distress indicating a current need for
diamorphine, although this should be available on an ‘as-required’ basis.
Rectal carbamazepine and/or regular rectal diazepam suppositories could also be considered for
seizure control, but their use would be less flexible than a syringe driver in escalation to further
palliative care medications as required.

Further reading
British National Formulary. Available at: http://bnf.org/bnf/index.htm
EXAM 2 | ANSWERS 167

88. C.╇ Whilst it is concerning that she was found wandering, admission under the Mental Health
Act does not apply in this situation:
An application for admission for assessment under the Mental Health Act may be made in
respect of a patient on the grounds that: (a) they are suffering from mental disorder of a
nature or degree which warrants the detention of the patient in a hospital for assessment
(or for assessment followed by medical treatment) for at least a limited period; and (b) they
ought to be so detained in the interests of their own health or safety or with a view to the
protection of other persons.
The daughter is describing chronic cognitive decline and this needs investigating and a formal
diagnosis of a dementia or identification of a treatable condition, such as depression or
hypothyroidism; this can be done as an outpatient in the local memory service and may require
longer-term support of the community mental health team. Urgent medical admission does not
immediately appear to be necessary, although the family may need convincing of this and the
reassurance that ‘something will be done’ and what that ‘something’ is! The first step in unraveling
this is to assess the patient’s capacity to participate in these discussions and her ability to agree a
management strategy.

Further reading
Department of Health. What are the Mental Capacity Act 2005 Deprivation of Liberty Safeguards?
London: DoH. Available at: http://www.dh.gov.uk/prod_consum_dh/groups/dh_digitalassets/@dh/@en/
documents/digitalasset/dh_091867.pdf
HM Government. Mental Capacity Act 2005 (England). Code of Practice. Available at: http://
webarchive.nationalarchives.gov.uk/+/http://www.dca.gov.uk/legal-policy/mental-capacity/mca-cp.pdf
National Archives. Mental Health Act 1983.
Available at: http://www.legislation.gov.uk/ukpga/1983/20/section/2
National Archives. Mental Health Act 2007.
Available at: http://www.legislation.gov.uk/ukpga/2007/12/contents
NICE. Dementia, Clinical guideline CG42. Available at: http://www.nice.org.uk/cg42

89. E.╇ Age-related decline in T-cell function is linked to thymic involution as T-cells are mainly
produced in the thymus. B-cells are constantly replenished throughout life by the bone marrow and
so B-cell function does not decline with aging. The magnitude of swelling of lymphoid tissue, at the
time of infection, is reduced in older age; as the germinal centre is necessary for the production
of high affinity antibodies, its absence in later life is considered to be a cause of the compromised
humoral response seen with aging. Reduction in nasal lymphoid tissue with aging is associated with
reduced production of IgA and increased susceptibility to infection.

Further reading
Pathy MSJ. Principles and Practice of Geriatric Medicine. My Library, 2006. Available at: http://lib.
myilibrary.com/Open.aspx?id=33981&loc=11&srch=undefined&src=2

90. B.╇ The Barthel Index assesses functional not cognitive domains. These include:
• Feeding.
• Bathing.
• Grooming.
• Dressing.
• Bowels.
168 EXAM 2 | ANSWERS

• Bladder.
• Toilet use.
• Transfers (bed to chair and back).
• Mobility on level surfaces and stairs.

Further reading
The Barthel Index.
Available at: http://www.strokecenter.org/wp-content/uploads/2011/08/barthel.pdf

91. A.╇ The ABCDE rule of melanoma helps make an early diagnosis.
• A—Asymmetry.
• B—Border irregularity.
• C—Color variegation.
• D—Diameter greater than 6 mm (tip of pencil head), although melanoma can occur in lesions
less than 6 mm.
• E—Enlargement.
Lentigo maligna lesions are variegated tan-brown. Changes in size or colour of a new pigmented
lesion or an existing mole suggest early melanoma transformation. The commonest sites for
lentigo maligna are the sun-exposed skin of the head and neck, especially the nose and cheek. Less
common sites include the arm, leg, and trunk. Elevation, burning, itching, pain, or bleeding suggest a
more locally-advanced lesion.
Basal cell skin cancer (BCC) typically appear on sun-exposed skin, are slow growing, and rarely
metastasize. They are typically flat, firm, pale, small, raised, pink or red, translucent, shiny, and
waxy, and may bleed following minor injury. They vary in size from a few millimeters to several
centimeters in diameter. Punch biopsy may be necessary to distinguish between pigmented BCC
and melanoma.
Seborrheic keratoses can be annoying due to itch or catching on clothing, but are usually
asymptomatic. They start as one or more sharply defined, light brown, flat macules, which develop
a velvety to finely verrucous surface, and finally develop an uneven warty surface with multiple
plugged follicles, and a dull or lacklustre appearance. They can vary from pale brown with pink
tones to dark brown or black. They can occur on almost anywhere on the body, but not the palms
and soles or mucous membranes.
Pigmented naevi (moles) are usually flesh-coloured, brown or black; can appear anywhere on the
skin, alone or in groups; and may darken after exposure to the sun.

Further reading
Bader RS. Basal cell carcinoma workup. Medscape.
Available at: http://emedicine.medscape.com/article/276624-workup#showall
Balin AK. Seborrhoeic keratosis clinical presentation. Medscape.
Available at: http://emedicine.medscape.com/article/1059477-clinical
Schwartz RA. Lentigo. Medscape.
Available at: http://emedicine.medscape.com/article/1068503-overview
Tan WW. Lentigo maligna melanoma. Medscape.
Available at: http://emedicine.medscape.com/article/279839-overview
EXAM 2 | ANSWERS 169

92. E.╇ Her CHA2DS2-VASc score is 4 and so warfarin is recommended not aspirin. Aspirin
is only recommended if the CHA2DS2-VASc score is 0–1. As she has good rate control and is
asymptomatic there is no reason to change to dronedarone. She has had AF for many years and
so is unlikely to attain rhythm control by drugs or cardioversion, however, catheter ablation is not
appropriate unless she remains symptomatic.

Further reading
Camm AJ, Kirchof P, Lip GY, et al. Guidelines for the management of atrial fibrillation: the task force
for the management of atrial fibrillation of the European Society of Cardiology (ESC). European Heart
Journal 2010; 31: 2369–429. Available at: http://www.escardio.org/guidelines-surveys/esc-guidelines/
GuidelinesDocuments/guidelines-afib-FT.pdf

93. D.╇ Immobility is the major risk factor here for the development of a pressure sore. It is
important to discuss the overall standard of care in the care home with the manager, for it is
possible that this event could not be prevented. The daughter needs to understand risk, risk
assessment, and be reassured that appropriate preventative strategies are in place to minimize risk
of pressure sores developing, e.g. are staff in the care home and the primary care team familiar
with NICE guidance on the prevention and treatment of pressure sores. If the daughter remains
concerned regarding the standard of care then writing to the Care Quality Commission or the local
Social Services, who will most likely be commissioning beds in the home, would be the next most
appropriate step to seek external review of the standards of a care within the home.

Further reading
NICE. Pressure ulcers—prevention and treatment, Clinical guideline CG29.
Available at: http://www.nice.org.uk/nicemedia/pdf/CG029publicinfo.pdf
Waterlow J. Waterlow risk assessment tool.
Available at: http://www.judy-waterlow.co.uk/the-waterlow-score-card.htm
Care Quality Commission. Complain about a care home or service. Available at: http://www.cqc.org.
uk/contactus/howtoraiseaconcernorcomplaint/complainaboutasocialcareservice.cfm

94. A.╇ It is likely that he had a seizure. Unhelpful features differentiating seizure from
syncope: brief twitching and jerking; incontinence; pallor; tongue or lip biting; post-event fatigue.
Helpful features: prolonged post-event confusion; deeply bitten lateral border of tongue; tonic-
clonic movements lasting greater than 1min; deep cyanosis.
Every patient who loses consciousness should have an electrocardiogram. Electroencephalography
should rarely be ordered to answer the question ‘What was this attack?’
MRI head scan is the gold standard and should be performed within 4 weeks.
CT head scan will be performed in the acute situation as it is readily accessible, rapid, and more
sensitive for detecting acute cerebrovascular events.
Serum prolactin is not recommended in routine assessment of a possible seizure.
Holter monitoring should be considered if the resting ECG is abnormal or if there is other reason
to suspect that this was a seizure secondary to cardiac arrhythmia.

Further reading
SIGN. Diagnosis and management of epilepsy in adults, Guideline 70.
Available at: http://www.sign.ac.uk/guidelines/fulltext/70/index.html
170 EXAM 2 | ANSWERS

Morag R. Investigation of syncope. Medscape.


Available at: http://emedicine.medscape.com/article/811669-overview

95. A.╇ Loss of central inhibition to defaecation may occur in Alzheimer’s disease and
cerebrovascular disease, or other frontal lobe disease, with failure of the external sphincter to
contract against spontaneous intrarectal contractions; this may result in the passage of a formed
stool after a meal or cup of coffee.
Prolonged laxative usage, over many decades, may result in degeneration of the myenteric plexus
resulting in laxity of the anal sphincter, and levator ani with resultant failure of the anorectal reflex.
Surgical or obstetric trauma may result in faecal incontinence due to loss of muscle tone in the
levator ani or external sphincter with associated loss of the anorectal angle.
Anal squeeze pressure declines with ageing, this is a measure of the voluntary suppression of
defaecation by contraction of the external sphincter; this may result in a decreased ability to hold
off defaecation, but does not normally result in incontinence.
Damage to the sacral plexus, cauda equina or sacral spinal cord will result in segmental peristalsis
and loss of anal sensation with intermittent faecal incontinence.

Further reading
Nyam DCNK. The current understanding of continence and defecation. Singapore Medical Journal
1998; 39(3): 132–6. Available at: http://www.sma.org.sg/smj/3903/articles/3903ra1.html
Brady CM. Constipation and faecal incontinence. In: Fowler CJ. (ed.) Seminars in Clinical
Neurology, Volume 1 (Chapter 3). Available at: http://www.wfneurology.org//index.
php?cID=85&cType=document

96. D.╇ This woman has had at least three precipitants for delirium: infection, strangulated hernia,
and painful hip. Slow to resolve delirium would be the most likely explanation. Persistent delirium
is more likely the more severe the precipitating illness, the longer the duration of the precipitating
illness, and the more severe any pre-morbid dementia. Whilst dementia is the single commonest
underlying risk factor for delirium you are not given any history to suggest a pre-morbid dementia
and so progression of dementia, over a relatively short time interval is unlikely. Symptoms of
hypoactive delirium mimic depression, but in the absence, of any previous history of depressive
illness it is more likely that she remains delirious than she has developed a depressive illness. Staffing
shortages may slow down the speed of rehabilitative recovery, but would not normally prevent
someone achieving their goals. Personality clash may be likely with one or even several members
of the rehabilitation team, but you are given no reason to suspect this and it would be unusual for
so many personalities to clash that this hindered rehabilitation goals. However, so called ‘difficult’
(aggressive, unco-operative, passive or non-compliant) patients may be perceived by rehabilitation
staff as least worthy of their input and so may be by-passed (disadvantaged) in favour of more
co-operative, ‘willing’ patients.

Further reading
NICE. Delirium: Diagnosis, prevention and management, Clinical guideline CG103.
Available at: http://guidance.nice.org.uk/CG103

97. A.╇ PCR may identify meningococcal (and also streptococcal) disease—facilitating diagnosis. It
should particularly be considered when cerebrospinal fluid (CSF) is difficult to obtain.
EXAM 2 | ANSWERS 171

Benzylpenicillin would be appropriate in a primary care setting, however, 3rd generation


cephalosporins (e.g. ceftriaxone 2g BD or cefotaxime 2g QDS) are usual first line secondary care
agents.
Bacterial meningitis is a notifiable disease, but contact tracing would only usually be undertaken in
cases of meningococcal disease.
The reduced GCS and presence of focal neurological signs raise concern of the possibility of raised
intracranial pressure. CT may be useful in excluding intracranial masses or collections before lumbar
puncture. It should be noted that CT does not, in itself, exclude raised intracranial pressure.
Evidence for the benefit of steroids acutely in meningitis is controversial at best. Studies have also
been based on proven, rather than suspected meningitis, such as in this case.

Further reading
Hasbun R. Meningitis. Medscape.
Available at: http://emedicine.medscape.com/article/784389-overview
Health Protection Agency. Guidance for public health management of meningococcal disease.
Available at: http://www.hpa.org.uk/web/HPAwebFile/HPAweb_C/1194947389261
Newcombe J, Cartwright K, Palmer WH, et al. PCR of peripheral blood for diagnosis of
meningococcal disease. Journal of Clinical Microbiology 1996; 34: 1637–40.
Available at: http://www.ncbi.nlm.nih.gov/pmc/articles/PMC229085/
NICE CG102; Bacterial meningitis and meningococcal septicaemia: Guideline CG102.
Available at: http://guidance.nice.org.uk/CG102/NICEGuidance/pdf/English
Van de Beek D, Farrar JJ, de Gans J, et al. Adjunctive dexamethasone in bacterial meningitis: a meta-
analysis of individual patient data. Lancet Neurology 2010; 9: 254–63.
Available at: http://www.ncbi.nlm.nih.gov/pmc/articles/PMC2835871/

98. C.╇ NICE and SIGN guidelines advise that individuals with transient ischaemic attack are given
300mg aspirin and referred for specialist assessment. If their ABCD2 score is greater than 4 then
assessment within 24hr is advised.

His ABCD2 score is 5:


Age ≥60 years =1
BP ≥ 140/90 =1
Focal weakness =2
Speech disturbance without weakness =1
Lasting ≥60min =2
Lasting 1–59min =1
Diabetes =1
Reprinted from The Lancet, 369, 9558, S Claiborne Johnston et al.,
‘Validation and refinement of scores to predict very early stroke risk after
transient ischaemic attack’, pp. 283–292, Copyright 2007, with permission
from Elsevier.

Further reading
NICE CG68: Stroke: diagnosis and initial management of acute stroke and transient ischaemic attack
(TIA), Clinical guideline CG68, 2008.
Available at: http://www.nice.org.uk/nicemedia/pdf/CG68NICEGuideline.pdf
172 EXAM 2 | ANSWERS

SIGN. Management of patients with stroke or TIA: assessment, investigation, immediate management


and secondary prevention, Guideline 108, 2008. Available at: http://www.sign.ac.uk/pdf/sign108.pdf

99. E.╇ The lesion described appears to be recent and so is most likely either a venous ulcer or
traumatic in origin.
Squamous cell carcinoma is slow growing, may be eroded or nodular, and may have a friable
surface, which bleeds easily. Primary squamous cell carcinoma presenting as ulceration may occur
and squamous cell carcinoma can invade chronic ulcers, causing them to undergo malignant changes
(Marjolin's ulcer).
Pyoderma gangrenosum is usually associated with GI disease, rheumatoid arthritis or myeloma.
It presents acutely, the ulcer has a deep reddish active border, a necrotic, and ulcerated centre,
surrounded by a halo of erythema, and is often extremely painful.
Necrobiosis lipoidica diabeticorum is seen on the anterior surface of the lower legs of diabetic
patients. The lesions may predate the development of diabetes by several years. They are atrophic,
red or yellowish depressed areas with marked telangiectasia; are usually asymptomatic, but can
ulcerate, usually as a result of trauma, and can be very slow to heal.

Further reading
Baldursson B, Sigurgeirsson B, Lindelöf B. Leg ulcers and squamous cell carcinoma: an epidemiological
study and a review of the literature. Acta Dermatologica Venereologie (Stockholm) 1993; 73: 171–4.
Available at: http://hls.is/page2/bs/assets/legulcscclitt.pdf
King B. Diagnosing and treating unusual aetiologies of lower leg ulcers. Nursing Times.Net 2005;
101(4): 44. Available at: http://www.nursingtimes.net/nursing-practice-clinical-research/diagnosing-and-
treating-unusual-aetiologies-of-lower-leg-ulcers/204105.article

100. D.╇ Whilst falls in someone with Parkinson’s may be due to postural instability as part of the
disease itself it is important to check for orthostatic hypotension, either as an adverse drug effect
or due to autonomic dysfunction associated with later stages of the disease.
He clearly needs a comprehensive review of his Parkinson’s, which should include assessments
by physiotherapy and occupational therapy as well as a review of his medication by either the
Parkinson’s nurse or his medical specialist. However, these should follow on from checking his lying
and standing BP and an assessment of other possible causes for his falls.
Falls are common in Parkinson’s patients and so the assessment of these should be undertaken
within the Parkinson’s service and should not require a separate referral to the falls service.

Further reading
Ceravolo R, Rossi C, Kiferle L, Bonuccelli U. Nonmotor Symptoms in Parkinson’s Disease: The Dark
Side of the Moon. Future Neurology 2010; 5(6): 851–71.
Available at: http://www.medscape.com/viewarticle/734227_8
Wood BH, Bilelough JA, Bowron A, Walker RW. Incidence and prediction of falls in Parkinson’s
disease: a prospective multidisciplinary study. Journal of Neurology, Neurosurgery and Psychiatry 2002;
72: 721–5. Available at: http://jnnp.bmj.com/content/72/6/721.full
exam 

3 QUESTIONS

1. Age-related changes to multiple physiological systems are


fundamental to the development of frailty.
Which of the following statements best characterizes the Fried
clinical phenotype of frailty?
A. All five clinical indicators are needed to define frailty
B. Cognitive impairment is a key feature
C. The presence of sarcopenia alone defines frailty
D. The relationship with dementia is uncertain
E. Those with one or two indicators are characterized as normal

2. An 83-year-old female resident of a residential home was admitted to


hospital with hyperosmolar non-ketotic diabetic ketoacidosis. The care
home and her nieces reported that she rarely took her medication, and
in the last month had hardly been eating or drinking.
Once she had been rehydrated and commenced on regular insulin, her
blood sugars stabilized between 6 and 10mmol/L. However, she refused
most food and drink, would not take any pills, and screamed and fought
with the nurses when they tried to give her insulin.
She had been diagnosed with Alzheimer’s dementia 5 years earlier and
on receiving the diagnosis had expressed a wish to go to Dignitas in
Switzerland. Subsequently, she had repeatedly stated that she wished to be
with her husband, who had died a year before her diagnosis of dementia.
She had never attempted to take her own life and was not thought to
be depressed. Her MMSE 2 years prior to this admission had been 15/30.
Currently, she was unable to engage in any discussion about her health care.
Her nieces (jointly and severally) had a Lasting Power of Attorney with a
Health Proxy and asked that the ward staff stopped forcing their aunt to
have medication and injections.
What is the most appropriate next course of action?
A. Insert a NGT to feed her and give oral medication
B. Refer for an Independent Mental Capacity Advocate opinion
C. Respect the nieces’ wishes and withdraw all medication
D. Seek a second opinion before ignoring the nieces’ wishes
E. Seek advice from the Trust lawyers
174 EXAM 3 | QUESTIONS

3. A 69-year-old homeless male was brought to the ED by the police


who had found him in the local park, unable to stand up. He was
cachetic with a BMI of 15.8 (18.5–24.9). There was an extensive
petechial rash over both lower limbs and bleeding gums with poor
dentition. He was infested with lice and fleas. His speech was
slurred, but intelligible and he appeared to be hallucinating. His
breath did not smell of alcohol.
Investigation results
Haemoglobin 9.1g/dL (13-18)
MCV 110fL (80–96)
WCC 2.8 × 109/L (4–11)
Platelets 155 × 109/L (150–400)
PTT 12.0s (11.5–15.5)
Urea 2.3mmol/L (2.5–7.0)
Creatinine 98µmol/L (60–110)
Serum sodium 127mmol/L (137–144)
Serum potassium 2.8mmol/L (3.5–4.9)
Serum corrected calcium 2.2mmol/L (2.2–2.6)
Serum phosphate 0.6mmol/L (0.8–1.4)
Serum albumin 28g/L (37–49)
Serum total bilirubin 8µmol/L (1–22)
Serum alanine aminotransferase 30U/L (5–35)
Serum alkaline phosphatase 158U/L (45–105)
Random plasma glucose 2.1mmol/L (3–6)

Which of the following factors is most closely related to his risk of


the refeeding syndrome?
A. Being dysarthric
B. Having a delirium
C. His BMI
D. His lifestyle
E. Poor dentition
EXAM 3 | QUESTIONS 175

4. An 84-year-old man with type-2 diabetes mellitus had an ulcer


overlying the 1st MTP joint on the right foot. This had been
deteriorating. The patient’s past medical history included
hypertension, angina, type-2 heart block (now paced) and
glaucoma.
On examination, there was a 2 × 2-cm ulcer extending through skin
and muscle, although not obviously through to bone. The ulcer was
sloughy, but there was no frank cellulitis. A swab of the wound was sent
for microbiology. Pedal pulses are present bilaterally. The rest of the
systemic examination was unremarkable.
Blood tests confirmed significantly elevated inflammatory markers.
A bedside urine analysis showed nitrites, but no leucocytes. An
X-ray of the foot was reported as showing ‘no convincing evidence of
osteomyelitis’.
Which of the following is the most appropriate next step in
management?
A. Magnetic resonance imaging of the foot
B. Prolonged IV antibiotics for suspected osteomyelitis
C. Radio-nucleotide white blood cell scan
D. Short course of antibiotics for presumed UTI
E. Wound debridement by chiropodist and discharge to community

5. A 72-year-old man who was on warfarin for atrial fibrillation presented


with significant bleeding, thought to be from his upper GI tract. He was
stabilized in the emergency department and urgent gastroscopy was
arranged. His INR was 8.4 (normal <1.4s).
What is the next most appropriate course of action?
A. Fresh frozen plasma 15mL/kg
B. Omit warfarin and re-check INR in 24hr
C. Prothrombin complex concentrate 50U/kg
D. Prothrombin complex concentrate + 5mg vitamin K
E. Vitamin K 3mg PO stat
176 EXAM 3 | QUESTIONS

6. An 84-year-old woman developed recurrent tonic-clonic seizures


3 months after a left cortical infarct. She was started on sodium
valproate 800mg BD and had no further convulsions. One year
later she was re-referred to the outpatient department for
assessment of postural tremor, generalized muscular rigidity,
bradykinesia, and impaired cognition.
What is the most likely diagnosis?
A. Dementia with Lewy bodies
B. Idiopathic Parkinson’s disease
C. Sodium valproate toxicity
D. Vascular dementia
E. Vascular Parkinsonism

7. The daughter of an 82-year-old woman was concerned that her


mother had been reporting seeing animals and people in her
house. The daughter knew that this was not the case and was
worried that her mother was developing a form of dementia.
When questioned, the mother acknowledged that these
hallucinations were not real, and said that they had not been
upsetting her. Neither reported any other significant cognitive
difficulties.
Past medical history included hypertension, age-related macular
degeneration (registered blind), stable angina, and polymyalgia
rheumatic. Medications included prednisolone, calcium + vitamin D,
alendronic acid, and bendroflumethiazide.
Cardiovascular, respiratory, and abdominal examination were
unremarkable. Her MMSE score was 28/30. Electrolytes, FBC, and
inflammatory markers were all normal.
What is the most appropriate next step in her management?
A. Prescribe a low dose antipsychotic
B. Reassure them that she does not have a dementia
C. Recommend attendance at a day centre
D. Refer her for formal psychiatric evaluation
E. Start her on a cholinesterase inhibitor
EXAM 3 | QUESTIONS 177

8. A 76-year-old man complained of 2 years central chest tightness


and associated fatigue after climbing four flights of stairs, walking
1 mile, or when doing heavy gardening. This was relieved by 5min
rest and then he could continue to the same level of exertion.
He had stopped smoking 5 years earlier and was taking 75mg
aspirin daily. Physical examination was entirely normal. BP was
128/72mmHg. His blood sugar, full blood count, U&Es, thyroid
function and resting ECG were all normal. Chronic stable angina
was diagnosed.
What is the next most appropriate therapy to offer him?
A. A beta blocker
B. A calcium channel blocker
C. A gated potassium channel blocker
D. A selective inhibitor of sinus node pacemaker activity
E. A short-acting nitrate

9. An 83-year-old man with Parkinson’s disease lived with his


daughter. She supervised his medication, and occasionally had to
help him with dressing and cutting up his food. The father was
in receipt of attendance allowance and his daughter asked if she
should also be claiming carer’s allowance.
What is the most appropriate advice to give them about carer’s
allowance?
A. Claiming carer’s allowance will not affect any other benefits she receives
B. Claiming carer’s allowance will not affect her father’s benefits
C. She cannot claim because her father is receiving attendance allowance
D. She must be providing at least 35hr care per week
E. She will have to disclose how much she has in savings
178 EXAM 3 | QUESTIONS

10. A 69-year-old woman was referred to the outpatient clinic. She


had found increasing difficulty with urinary incontinence over the
past few years. She reported having very little warning before
needing to pass urine, and she had noticed that she leaked if she
coughed, sneezed or moved suddenly. She had no problems with
her bowels.
She had a history of two normal vaginal deliveries and was otherwise
well. She was not taking medications. Cardiovascular, respiratory, and
abdominal examinations were unremarkable. There was no evidence of
a vaginal prolapse on inspection.
Which of the following is the most appropriate next step in her
management?
A. Anticholinergic therapy
B. Pads and pants
C. Physical therapies
D. Transvaginal tape
E. Urodynamics

11. A 79-year-old woman with osteoarthritis was troubled with


right knee pain, such that she was struggling to walk to her
local shop. She had lost weight and had a BMI of 24kg/m2. She
exercised as much as the pain would allow. She had a past
history of ischaemic heart disease, chronic renal impairment
(CKD-3a), and hypertension. She was taking paracetamol
1g QDS, aspirin 75mg OD, simvastatin 40mg OD, and
bendroflumethiazide 2.5mg OD.
Which of the following would be the most appropriate next step
in her management?
A. Addition of celecoxib 200mg OD
B. Addition of codeine 30mg QDS
C. Addition of diclofenac 50mg TDS
D. Intra-articular steroid injection
E. Referral for joint replacement surgery
EXAM 3 | QUESTIONS 179

12. A 68-year-old man recovering from a right cerebral hemisphere


infarction had painful spasticity of the left foot limiting his
progress with rehabilitation.
What would be the most appropriate next step in the
management of his spasticity?
A. Baclofen
B. Botulinum toxin
C. Dantrolene
D. Sequential splinting
E. Surgical release

13. A 72-year-old afro-Caribbean male was found to have persistently


raised BP at 158/96mmHg. He had type II diabetes mellitus
and took metformin 1g BD. His BMI was 26.9 (18.5–24.9). His
last glycosylated haemoglobin was 5.3% (< 6%). Urinalysis was
negative for proteinuria and his eGFR 98mL/min (> 90mL/min).
His resting electrocardiogram was normal. He was not on any
antihypertensive treatment.
What is the most appropriate initial treatment for his
hypertension?
A. Angiotensin-converting enzyme inhibitor (ACEI)
B. Angiotensin II receptor blocker
C. Beta-blocker
D. Calcium channel blocker
E. Thiazide diuretic

14. A 76-year-old woman presented with breathlessness. She was an


ex-smoker of 30 years, with a 5 pack year history in total. She had
noticed that she became short of breath on exertion. There had
been a significant day-to-day variability in her symptoms. She had
a chronic cough and found that mornings were worse. She had
woken at night a couple of times and usually used two pillows. She
did not experience palpitations or chest pain.
A CXR and FBC were unremarkable.
Which of the following is the most likely diagnosis?
A. Chronic obstructive pulmonary disease
B. Heart failure
C. Ischaemic heart disease
D. Late-onset asthma
E. Venous thromboembolism
180 EXAM 3 | QUESTIONS

15. An 86-year-old woman with COPD, anxiety, and BMI 34kg/m2


(normal 18–25) lived alone in a sheltered housing complex with
twice daily carers. Her carers called the out-of-hours doctor
because they thought she appeared more short of breath than
usual; they were advised to call for an ambulance to take her
straight to hospital. On arrival at the hospital, the patient was
anxious, but stated that she did not consider her breathing to
be any worse than normal for her. FBC, CRP, and CXR were
all normal. She had home oxygen and received yearly influenza
vaccination. In the last 4 months she had had three similar
admissions. She was discharged home the next day.
Which of the following is most likely to help avoid future similar
admissions?
A. Carer education
B. Community matron
C. GP assessment
D. Telecare
E. Telehealth

16. A 69-year-old man with diet-controlled type-II diabetes


complained of recent onset uncomfortable muscle aches in his
arms. He had been hospitalized 3 months earlier with a NSTEMI.
Since his discharge, he was taking aspirin 75mg daily, simvastatin
40mg daily, bisoprolol 5mg daily, and ramipril 5mg daily. His BP
was 120/72mmHg sitting and standing.
Blood tests obtained
Serum urea 11.0mmol/L (2.5–7.0)
Serum creatinine 125µmol/L (60–110)
TSH 4.0mU/L (0.4–5.0)
Creatine kinase 320U/L (24–195)

What is the most appropriate next step in his management?


A. Assess if the severity of symptoms affect his quality of life
B. Change to ezetimibe OD
C. Change simvastatin to extended release fluvastatin
D. Reduce the dose of simvastatin
E. Stop simvastatin and commence IV fluids
EXAM 3 | QUESTIONS 181

17. A 79-year-old woman with urinary incontinence was seen in


an outpatient clinic. She presented with symptoms of urinary
frequency and urgency. She had to void in excess of 20 times a day
and had also noticed that she ‘leaked’ when lifting heavy objects.
Urinalysis and blood tests were all unremarkable.
Slices through a recent CT abdomen are shown in Fig. 3.1.

Fig. 3.1 Abdomen CT.
What is the next most appropriate step in her management?
A. Bladder training exercises
B. Catheterization
C. Course of co-amoxiclav
D. Initiate anticholinergic therapy
E. Referral to a uro-gynaecologist

18. The first National Audit of Intermediate Care (IC), published in


2012, showed a marked skew towards bed-based IC compared
with home-based IC.
What is the least likely explanation for this imbalance?
A. Failure to engage geriatricians in IC
B. Inability to recruit community staff
C. Patient preference for bed-based IC
D. Population demographics (case-mix)
E. Provision of ‘step-up’ beds in residential homes
182 EXAM 3 | QUESTIONS

19. A 74-year-old man attended for annual review of his Addison’s


disease. It was noted that he had unintentionally lost several
kilograms of weight. He also complained of diarrhoea for the
last few months. Abdominal and rectal examinations were
unremarkable.
Which is the most appropriate next step in his investigation?
A. Arrange a CT scan of his abdomen
B. Arrange a flexible sigmoidoscopy
C. Check coeliac antibodies
D. Check faecal elastase levels
E. Send stool for culture and microscopy

20. An 88-year-old woman with vascular dementia (MMSE 19/30)


was found locked out of her house by her neighbours. She was
not prone to wandering and did not usually venture out of the
house alone. One month earlier she had been diagnosed as
having an adenocarcinoma of the stomach. Her medication was
domperidone 10mg TDS and lansoprazole 30mg BD.
What assistive technology might she most benefit from?
A. Key safe
B. Notices reminding her to take her door key
C. Pill box with alarm function
D. SMS medication reminders
E. Wandering monitor linked to her mobile phone

21. The general practitioner was asked to see a 79-year-old resident


of a dementia-registered residential home. For the last week
he had been aggressive towards the staff and verbally abusive to
other residents. His sleep pattern was disrupted, but he continued
to eat and drink well.
Physical examination, including digital rectal examination, was normal.
Dipstick testing of urine was negative. FBC, U&Es, and serum calcium
were all normal.
What is the most appropriate next step in his management?
A. Ask the home to start a behaviour chart
B. Prescribe a regular antipsychotic
C. Prescribe an acetylcholinesterase inhibitor
D. Refer for assessment by the community geriatrician
E. Transfer to a specialist dementia registered care home
EXAM 3 | QUESTIONS 183

22. The trauma and orthopaedic team asked for assistance in


planning the discharge of a 72-year-old woman who had tripped
on the kerb, and sustained a fracture of her distal right tibia
and a Colles’ fracture of her right arm. The orthopaedic team
recommended that she be toe-touch weight-bearing in plaster
for the next 6 weeks. She lived alone in a first floor flat with no
additional support. She was able to transfer using a rollator frame
and assistance of one other person. She did not feel she would be
able to cope at home at this time.
What is the most appropriate advice to give the
orthopaedic team?
A. Consider transfer to an interim nursing home bed
B. Consider transfer to an interim residential home bed
C. Explain that this is an orthopaedic problem and she should stay there
D. Refer to a community hospital for rehabilitation
E. Transfer to one of your medicine for the elderly beds

23. An 84-year-old woman on warfarin for atrial fibrillation was


found to have a raised international normalized ratio (INR).
She had recently had a short course of clarithromycin for
a chest infection. This had finished 5 days previously. She
was cognitively unimpaired, independent, and lived with her
husband.
She presented to the ED with a minor nose bleed. This settled
spontaneously within 5min, and was estimated to be less than 5mL
in total. Otherwise, she felt well. Examination and observations were
unremarkable.
Her INR was 4.5.
Which of the following is the most appropriate course of action?
A. Fresh frozen plasma 15mL/kg
B. Omit warfarin and re-check INR in 24hr
C. Prothrombin complex concentrate 50U/kg
D. Prothrombin complex concentrate + 5mg vitamin K
E. Vitamin K 3mg PO stat
184 EXAM 3 | QUESTIONS

24. A 79-year-old male was referred to his GP by his niece who was
concerned that her uncle had become increasingly withdrawn
over the last 2 years since the death of his wife. She reported that
he was no longer eating the food she bought for him and that
she had found a large stash of empty vodka bottles in the back
room of his house. He had stopped going upstairs to bed, and
rarely changed his clothes or washed. He took no medication and
had last seen his GP 25 years ago. At the age of 30 he had been
treated for depression in a private psychiatric hospital. Apart
from the weekly visit from his niece, he had no other visitors or
social contact.
When the GP saw him he was dishevelled, cachexic, and smelt of
alcohol. Respiratory, cardiovascular, abdominal, and neurological
examinations were all normal. He was low in mood, anhedonic, and
expressed feelings of low self-worth, stating that he did not deserve
to be alive. Cognitive assessment scored 28/30 on a mini-mental state
examination (MMSE). He believed that his niece was trying to poison
him and that she had installed spy-cameras in the house.
What is the most appropriate next step in his management?
A. Arrange bereavement counselling
B. Refer to old age psychiatry
C. Refer to the community geriatrician
D. Start an selective serotonin reuptake inhibitors (SSRI) antidepressant
E. Treat him for alcohol withdrawal
EXAM 3 | QUESTIONS 185

25. A 74-year-old woman with moderate Alzheimer’s dementia and


diet-controlled type II diabetes, had a 6-month history of leg
pain. The pain was non-specific in nature, but did not stop her
wandering around the care home where she had lived for the last
3 years. She was becoming increasing unsteady on her feet and
needed assistance to rise out of a chair.
Investigations
Haemoglobin 11.3g/dL (11.5–16.6)
Mean cell volume 80fL (80–96)
Erythrocyte sedimentation rate 20mm/1st hour (<30)
Serum corrected calcium 2.10mmol/L (2.2–2.6)
Serum alkaline phosphatase 66U/L (45–105)
Serum albumin 37g/L (37–49)
Serum ferritin 150μg/L (15–300)

What is the most likely cause of her leg pain?


A. Diabetic amyotrophy
B. Iron deficiency anaemia
C. Osteoarthritis
D. Polymyalgia rheumatic
E. Vitamin D deficiency

26. Which of the following statements about the CGA is not true?


A. A geriatrician can complete a CGA independently
B. CGA includes physical and mental health, environment, and social function
C. CGA is associated with lower medication use
D. CGA results in improved independence
E. The goal of CGA is to maximize independence and prevent future disability

27. Which of the following statements is true regarding the management of


atrial fibrillation?
A. ACE-I may improve relapse rates in paroxysmal AF
B. Digoxin provides effective rate control both at rest and while exercising
C. Dronedarone is more effective than amiodarone at rhythm control
D. Flecainide is preferable to beta blockers in patients with acute heart failure
E. Warfarin can be stopped after pulmonary vein ablation
186 EXAM 3 | QUESTIONS

28. At her annual review a 76-year-old female with type II


diabetes mellitus was found to have BP 130/78mmHg, glycated
haemoglobin 6.2% (<6%), eGFR 56mL/min (> 90mL/min), BMI
23.6 (18.5–24.9) and urinary albumin/creatinine ratio 10mg/mmol
(<3.5). She was taking metformin 850mg BD and simvastatin
40mg daily.
What is the most appropriate next step in her management?
A. Add a sulphonylurea
B. Add an ACE-I
C. Advise her to lose weight
D. Increase her metformin
E. Start aspirin

29. A 94-year-old female resident of a specialist dementia care home


was noted to have an ulcerating lesion above the lateral malleolus
of her left ankle. The lesion was largely obscured by a large
haematoma; however, the upper edge was visible, and seemed to
be rolled, friable, and irregular. She had not previously had any
leg ulcers. She was generally aggressive towards staff and other
residents and would not let staff look more closely at the lesion.
Observation shows that she has mild bilateral ankle oedema, but
no chronic skin changes.
What is the most appropriate management plan?
A. Four layer compression bandaging
B. Insist on foot elevation
C. No specific treatment, keep under review
D. Refer for diagnostic biopsy
E. Refer for radiotherapy

30. A 76-year-old female attended the falls clinic after having several
falls in the last few months.
Which of the following has the best evidence base for
reducing falls?
A. Brisk walking therapy
B. Cognitive behavioural therapy
C. Environmental modification
D. Group-based aerobic exercise programme
E. Separate reading and distance spectacles
EXAM 3 | QUESTIONS 187

31. An 87-year-old woman with dementia was considered to be at


risk of harm living alone in her house. She was frequently found
wandering outside of her home and could not tell people where
she lived. She had carers four times a day to supervise her
personal care and medication, and to ensure she ate her meals.
In the last year she had been admitted to hospital six times
after her carers had found her on the floor; on two occasions
she had sustained a fracture. She had also been admitted twice
with an accidental overdose of her antihypertensive medication.
Her family asked for a needs assessment of eligibility for NHS
Continuing Health Care prior to her move to a residential home.
What is the most appropriate response to the family’s request?
A. It is unlikely that she will be eligible for full NHS funding
B. This cannot be done until she moves in to the care home
C. This cannot be done without her consent
D. This must be done before she moves in to the care home
E. There is no point doing the assessment as she will not be eligible

32. A 69-year-old woman was admitted to hospital with a left-sided


hemiparesis. This had started abruptly 8hr previously. She had
a past medical history of type 2 diabetes mellitus, hypertension,
and ischaemic heart disease. She took no regular medication.
She was in sinus rhythm with a BP of 130/80. She had a few right basal
crackles and her oxygen saturations were 96% on air. She had a dense
left hemiparesis and mild left-sided motor and visual neglect. A ward
swallow test showed a degree of aspiration. Her temperature was
37.2°C and BM 15.2mmol/L (range 4.0–7.0mmol/L)
CT head shows a sub-acute right middle cerebral artery territory
infarct.
Which of the following is the least appropriate next step in her
management?
A. 28% oxygen by Venturi mask
B. Initiate clopidogrel 300mg OD
C. Initiate statin therapy at 48hr
D. Maintain glucose between 4 and 11mmol/L
E. Feed via NGT within 24hr of admission
188 EXAM 3 | QUESTIONS

33. The TUG test is often used as part of a comprehensive geriatric


assessment.
In which of the following settings is a TUG test least likely to be
helpful in developing a management strategy?
A. Acute geriatric ward
B. Falls clinic
C. Memory clinic
D. Parkinson’s clinic
E. Stroke rehabilitation

34. A 91-year-old woman was admitted from her nursing home. She
had been found unresponsive by the carers. On examination she
had a Glasgow Coma Score (GCS) of 7/15 and a flaccid left-sided
weakness. An urgent CT head was undertaken. This showed an
old right parietal infarct, but no evidence of an acute bleed.
She was transferred to the stroke unit with the presumptive diagnosis
of ischaemic stroke. On review the next morning she had dramatically
improved to a GCS of 14 with minimal residual neurology. By the
afternoon, she was back at her baseline.
Which of the following is the most likely diagnosis?
A. Dementia with Lewy Bodies
B. Encephalitis
C. Todd’s paresis after a convulsion
D. Transient ischaemic attack
E. Venous sinus thrombosis

35. A 73-year-old woman developed a mark on her sacrum while


being treated for pneumonia. Continence had been difficult to
maintain and she had been using a continence pad.
Examination showed a 4 × 4cm area over the sacrum with loss of the
dermis and blistering. It does not extend to underlying fascia or bone.
Which would be the appropriate classification?
A. Grade 1 ulcer
B. Grade 2 ulcer
C. Grade 3 ulcer
D. Grade 4 ulcer
E. Moisture lesion
EXAM 3 | QUESTIONS 189

36. A 78-year-old man had recurrent falls. He said that he tended to


fall when he tried to turn suddenly. He lived alone and had twice
daily care to help with washing, dressing, and meal preparations.
On examination he was neatly dressed, but smelt strongly of
urine, although he seemed oblivious to this. He wore a right-sided
hearing aid, which was not functioning properly. General physical,
respiratory, heart, and abdominal examinations were normal.
Neurological examination revealed decreased hearing on the
right side, the rest of the cranial nerves were normal. There
was dysdiododyskinesis and past pointing bilaterally. He walked
with a slightly stooped posture, broad base, and shuffling gait.
Hallpike manoeuvre was negative. His MMSE was 20/30 using a
communicator.
What is the most likely diagnosis?
A. Acoustic neuroma
B. Benign positional vertigo
C. Cerebellar tumour
D. Ménière’s disease
E. Normal pressure hydrocephalus

37. Frailty is recognized as a biological marker of poor health


outcomes in older people.
Which of the following characteristics is most commonly
accepted as a marker of frailty?
A. Ageing
B. Gait pattern
C. Male sex
D. Sarcopenia
E. Weight gain
190 EXAM 3 | QUESTIONS

38. An 81-year-old man with chronic renal impairment (CKD-5) had


previously declined dialysis. His U&E and creatinine were stable.
He had been getting increasingly fatigued and short of breath. His
haemoglobin had drifted down to 8.2g/dL (13–18) over the past
few months.
Haematinics results
Ferritin = 80µg/L (100–300 in CKD-5)
B12 = 210ng/L (160–760)
Red cell folate = 240µg/L (160–640)
TSH = 2.7mU/L (0.4–5.0)

He was taking ferrous sulphate 200mg BD until 6 months ago, at which


point it was stopped, owing to GI side effects.
Which of the following is the most appropriate next step in his
management?
A. A course of IM hydroxocobalamin
B. Arrange IV iron infusion
C. Check serum erythropoietin levels
D. Initiate oral ferrous fumarate 210mg TDS
E. Therapy with erythropoiesis-stimulating agents

39. The wife of a 70-year-old male nursing home resident requested


a review by the local Parkinson’s specialist as her husband had
frequent episodes of becoming stiff, with his neck and back
arching backwards so that he could only look at the ceiling; his
eyes also rolled upwards. He remained conscious during these
episodes, but could not swallow if they happened when he was
eating. These episodes happened every day, but at different times
during the day and could last from a few minutes to an hour. He
had developed a Parkinsonian syndrome a year after suffering
the first of three strokes; he was now unable to communicate
and did not recognize his wife. In between these episodes he was
wheelchair bound, needed to be hoisted for transfers, doubly
incontinent, and did not choke when fed by his wife or the staff.
There was no record of exposure to dopamine-depleting drugs.
What is the most appropriate management plan?
A. Advise the staff not to feed him when this happens
B. Commence a benzodiazepine
C. Refer for a feeding gastrostomy tube
D. Refer to speech and language therapist for swallow assessment
E. Stop his levodopa medication
EXAM 3 | QUESTIONS 191

40. A 75-year-old man was noted to be bradykinetic, with a left-sided


pill rolling tremor, rigidity, and impaired postural balance.
A baseline MMSE was 27/30. His only past medical history
was of Ménières disease and hypertension. His medications at
presentation were aspirin 75mg OD, prochlorperazine 10mg TDS,
and bendrofluomethiazide 2.5mg OD. The prochlorperazine was
stopped. Three months later his condition had deteriorated; he
was slower and had increasing difficulty with his personal care. He
was started on co-careldopa titrated to 125 QDS over 4 weeks.
Two months later, there had been minimal improvement in his motor
symptoms. He had become incontinent for urine, and felt very unsteady
and light-headed when standing. His repeat MMSE was 26/30. A head
CT did not show any evidence of focal or generalized ischaemia. A small,
1-cm, frontotemporal meningioma was noticed, with no significant mass
effect or surrounding oedema.
Which of the following is the most likely diagnosis?
A. Idiopathic Parkinson’s disease
B. Dementia with Lewy bodies
C. Multisystem atrophy variant
D. Tumour-related Parkinsonism
E. Vascular Parkinsonism

41. An 82-year-old man with widely disseminated prostate cancer was


admitted for symptom control. He had marked bony pain that the
GP had found hard to control. He had no other significant past
medical history, but had been found to be in acute renal failure
on admission. His blood tests showed an estimated glomerular
filtration rate of 15mL/kg/min.
Which of the following would be the most appropriate opiate
to use?
A. Codeine
B. Diamorphine
C. Fentanyl
D. Oramorph
E. Oxycodone
192 EXAM 3 | QUESTIONS

42. An 81-year-old man with bilateral age-related macular


degeneration and bilateral cataracts complained of 3 months
visual hallucinations, which he knew were not real. His MMSE was
27/30. He was worried he was going mad.
What is the most appropriate treatment option?
A. Cataractectomy
B. Donepezil 5mg daily
C. Haloperidol 0.5mg daily
D. Reassure him he is not mad
E. Risperidone 0.5mg daily

43. A 67-year-old male patient with COPD presented with a sudden


increase in breathlessness. There was some associated pleuritic
chest pain. Examination demonstrated a sinus tachycardia and
normal BP. Oxygen saturations were reduced to 82% on air.
A chest radiograph was normal. Arterial blood gases confirmed
significant type 1 respiratory failure.
Other blood test results
Haemoglobin 13.5g/dL (13–18)
White Cells 6.2 × 109/L (4–11)
Platelets 271 × 109/L (150–400)
Sodium 139mmol/L (137–144)
Potassium 4.2mmol/L (3.5–4.9)
Creatinine 207µmol/L (60–110)
CRP 6mg/L (<10)

Which of the following is the most appropriate next investigation?


A. Arrange a CT pulmonary angiogram
B. Arrange a ventilation-perfusion (VQ) scan
C. Arrange bed-side echocardiography
D. Arrange bilateral Doppler leg ultrasound
E. Perform a d-dimer assay
EXAM 3 | QUESTIONS 193

44. A 77-year-old man with metastatic prostate cancer (bone, lung,


and liver metastases) was referred to the ED having fallen at
home. There was no evidence of any fracture. He was cachexic,
pale, and very weak. He stated that he no longer had any appetite
for food, but was drinking small amounts of liquid (approximately
400mL/day. His haemoglobin was 4.2g/dL (13–18). Blood urea,
electrolytes, and calcium were normal. He declined blood
transfusion and his wife agreed with his expressed wish to be sent
home to die.
Which of the following is least likely to be needed?
A. Cyclizine
B. Diamorphine
C. Hyoscine hydrobromide
D. Midazolam
E. Oxygen

45. A 92-year-old man presented with a fractured neck of femur. He


required 5mg IV morphine to control the pain. He had a past
medical history of mild aortic stenosis and rheumatoid arthritis.
His medications were: 5mg prednisolone OD, aspirin 75mg, and
simvastatin 40mg OD.
The orthopaedic team planned on transferring him to their ward and
scheduled a hemiarthroplasty for the next day. The nursing team were
concerned that he was very confused. He scored 2/10 on an abbreviated
mental test score. His daughter reported that he was slightly forgetful
normally, but nowhere nearly so muddled. Owing to this confusion, a
medical opinion had been requested.
Which of the following is consistent with best practice?
A. Delay surgery until a formal echocardiogram can determine the degree of aortic stenosis
B. Initiate treatment with IV bisphosphonate within 24hr of fracture
C. Referral to on-call anaesthetist to consider performing a regional nerve block
D. Urgent review by the psychogeriatric liaison team to assess capacity to consent for surgery
E. Withhold steroid medication to promote bone and wound healing post-operatively
194 EXAM 3 | QUESTIONS

46. An 87-year-old woman presented following a fall. She was hanging


some washing up and overbalanced. She broke her fall with her
left arm, but now had marked tenderness and swelling over her
distal radius.
Her past medical history included chronic renal impairment (baseline
creatinine 180, eGFR 21mL/min), type 2 diabetes mellitus, and
hypertension. She took gliclazide 40mg OD, aspirin 75mg OD, and
amlodipine 5mg OD.
She did not drink alcohol and was an ex-smoker of 60 years (3 pack
year history). She ate a balanced, non-vegetarian diet. She was a keen
gardener and played bowels twice weekly. She drove a car.
Examination demonstrated no significant cardiovascular, respiratory,
abdominal, or neurological abnormality.
Investigations
Bloods
Sodium 135mmol/L (137–144)
Potassium 4.5mmol/L (3.5–4.9)
Urea 18mmol/L (2.5–7.0)
Creatinine (Cr) 192μmol/L (60–110)
Bilirubin 15μmol/L (1–22)
Alkaline phosphatase (ALP) 150U/L (45–105)
Corrected calcium (Cor. Ca) 2.0mmol/L (2.2–2.6)
Hb 10g/dL (11.5–16.5)
MCV 82fL (80–96)
WCC 5.2 × 109/L (4–11)
Plt 320 × 109/L (150–400)
ECG Sinus rhythm, left axis deviation
X-ray Colles fracture
The fracture was reduced and set in plaster.
Which of the following would be the most appropriate action
regarding her bone health?
A. Alfacalcidol 500ng OD and review in clinic in 2 weeks
B. Arrrange a DEXA scan
C. Initiate treatment with calcium (1.2g) and vitamin D (800U) tablets
D. Secondary osteoporosis prophylaxis with alendronate 70mg weekly
E. Secondary osteoporosis prophylaxis with strontium ranelate 2g every evening
EXAM 3 | QUESTIONS 195

47. A 74-year-old man had been stepped down to the ward from
Intensive Care following a 14-day stay with severe pneumonia. He
had required ventilation and inotropic support for 5 days. He had
been fed through a NGT whilst in intensive care. This dislodged
on transfer to the ward.
Although he is deconditioned and in need of rehabilitation, clinical
examination was otherwise unremarkable on review and blood tests
confirmed resolution of the sepsis. He had lost 3kg of weight during the
admission.
Which is the most appropriate way to proceed?
A. Ask speech and language therapy to assess
B. Initiate high calorie and fat supplements
C. Re-insert bridled NGT
D. Re-insert standard NGT
E. Standard ward nutrition and monitoring

48. An 82-year-old woman has been successfully treated on the ward


for pneumonia. She previously lived alone in her own bungalow
and was supported in meal preparation by carers twice a day. She
had been independently mobile around her home with a walking
stick to assist her.
She was unhappy about returning to this arrangement and wanted to
move to a care home. On the ward, she was able to mobilize to the
toilet independently with a stick and had passed occupational therapy
washing and dressing assessments. The multidisciplinary team thought
that she had returned to her prior level of functioning.
What is the most appropriate course of action?
A. Check-listing for continuing health care
B. Discharge home with a restart of her care package
C. Discharge home with an increase in her care package
D. Refer for consideration of a residential home
E. Refer to a community hospital for ongoing rehabilitation
196 EXAM 3 | QUESTIONS

49. A 73-year-old woman with tremor-dominant Parkinson’s disease


of 6 years duration was taking Madopar® 125 five times a day,
ropinirole 3mg TDS, fludrocortisone 200micrograms BD,
alendronic acid 70mg on Sundays, and Adcal D3 Forte® every day
except Sunday.
Which of the following is least likely to assist her in remaining
independent in the bedroom?
A. Bed cane
B. Inflatable leg lifter
C. Jacob’s ladder
D. Mattress tilter
E. Silk bottom sheet

50. A 72-year-old woman complained of sudden onset distortion of


vision in her left eye. She stated that door and window frames
appeared bent. At her last visit to her optician she had been told
she had a ‘drusen’ in that eye.
What is the most likely diagnosis?
A. Central retinal vein occlusion
B. Lamellar cataract
C. Retinal artery occlusion
D. Vitreal haemorrhage
E. Wet age-related macular degeneration

51. A 79-year-old man with previously stable angina developed


increasing anginal symptoms on exertion. He had no symptoms
at other times. He was taking PRN GTN spray, aspirin 75mg OD,
and simvastatin 40mg OD. His average BP was 110/82mmHg on
24-hr BP monitor. He was an ex-smoker of 5 years, with a 20
pack year history. He had no known drug allergies. He was not
diabetic.
Which of the following would be the most appropriate next step
in his management and investigation?
A. Recommend a cardiospecific beta-blocker
B. Recommend a long-acting nitrate
C. Refer for angiography
D. Suggest he reduces his activity
E. Suggest prophylactic GTN use before exertion
EXAM 3 | QUESTIONS 197

52. A 72-year-old man with Parkinson’s disease was admitted with an


aspiration pneumonia. He had no other past medical history. He
lived with his wife and was supported by twice daily carers. When
he was last seen in clinic he scored 25/30 on an MMSE and neither
he nor his wife reported any significant cognitive difficulties. He
had no overt swallowing problems at that time.
On admission his MMSE was 14/30. His wife reported that his confusion
had deteriorated over the past week. The speech and language
therapists had been assessing him during the admission and had advised
that he be kept nil by mouth. He had subsequently been tolerating NGT
feeding. One week later, the speech therapists reported that there was
limited improvement in his swallow and that it remained unsafe. They
suggested that a percutaneous endoscopic gastrostomy feeding tube
be inserted. The multidisciplinary team did not feel the patient had
capacity to decide whether to proceed with this or not.
Which of the following would be the most appropriate course of
action?
A. Arrange for an IMCA to review
B. Arrange for an old age psychiatry review
C. Continue to pursue NGT feeding
D. Discontinue NGT feeding and pursue the ‘safest oral diet’
E. Refer for PEG

53. A 70-year-old woman complained of reducing mobility due to


chronic low back pain. She took paracetamol 1g QDS, codeine
30mg QDS and one sachet of macrogol on alternate days. On
physical examination there was bilateral weakness of ankle
dorsiflexion and ankle eversion, as well as weakness of hip
abductors and extensors.
Which pattern of pain is most characteristic of lumbar spinal
stenosis?
A. Increases when bathing
B. Increases when bending forward
C. Increases when walking downhill
D. Increases whilst lying in bed
E. Increases with prolonged sitting
198 EXAM 3 | QUESTIONS

54. An 89-year-old female with type II diet-controlled diabetes and


hypertension treated with amlodipine 5mg daily complained
of shortness of breath worsening over the last 4 months and
now limiting her when washing and dressing. She was in atrial
fibrillation at 110beats/min, tachypnoeic at rest, with a normal
jugular venous pressure and normal heart sounds. BP was 160/98.
CXR was normal.
Which of the following would not be in keeping with a diagnosis of
heart failure with normal ejection fraction?
A. History of hypertension
B. Impaired left ventricular diastolic function
C. Left ventricular outflow tract obstruction
D. Normal left ventricular systolic function
E. Shortness of breath on mild exertion

55. An 84 year woman had been diagnosed with an acute confusional


state resulting from a urinary tract infection. She had scored 6/10
on an Abbreviated Mental Test (AMT). Neurological examination
was unremarkable.
What is the most appropriate next step in investigating her
cognitive impairment?
A. Arrange a CT head scan
B. Arrange an MRI of the brain
C. B12, folate, and thyroid function testing
D. More detailed cognitive assessment and collateral history
E. Referral to old age liaison psychiatry team

56. A 75-year-old male resident of a specialist dementia care home


was faecally incontinent. He spent most of the day wandering
around the home and did not recognize his wife or children
when they visited. He ate finger food and would drink with
encouragement. He was on no medication.
What would be the most appropriate long-term management of
his faecal incontinence?
A. Anopexy
B. Kegel exercises
C. Rectopexy
D. Regular enemata
E. Toileting regime
EXAM 3 | QUESTIONS 199

57. A 92-year-old man was having rehabilitation following a stroke,


which had left him with a dyspraxic left upper limb and associated
mild weakness. The occupational therapists were using the
Barthel Index to monitor his progress.
Which of the following activities of daily living would not be
assessed using this scale?
A. Bed to chair transfers
B. Climbing stairs
C. Continence
D. Drink preparation
E. Feeding

58. A 76-year-old woman was admitted from home for treatment of


cellulitis secondary to chronic venous leg ulcers. She was a retired
nurse who had been abused as a child and was wary of strangers.
She had been widowed 5 years and lived alone without any carers.
Her granddaughter shopped for her. She slept downstairs on her
couch as she felt unsafe climbing stairs; although this was a sofa
bed she chose not to open it out as a bed as it was comfortable
enough without doing so. She had a downstairs toilet and stripped
wash at the sink downstairs. Her granddaughter expressed
concerns about her discharge without any care as she thought
her grandmother was neglecting herself. The patient said her
granddaughter was interfering and should let her lead her life as
she wanted to. Her MMSE was 28/30. She was independent on the
ward. Reluctantly, she agreed to allow the district nurses to visit
post-discharge to supervise her leg ulcer dressings.
Which of the following is the most appropriate discharge plan?
A. Arrange a case conference with her and her granddaughter
B. Ask the district nurses to monitor her general well-being
C. Refer to Old Age Psychiatry for a capacity assessment
D. Refer to Old Age Psychiatry to assess for depression
E. Delay her discharge until she agrees to have a care package
200 EXAM 3 | QUESTIONS

59. A 69-year-old man with advanced dementia had four episodes of


aspiration pneumonia in the previous 9 months. The care home
manager was concerned regarding his risk of further aspiration.
He was bed-fast, unable to feed himself but appeared to gain
pleasure from eating. He would become agitated if he was not fed
and resisted any attempts to place a NGT. He had no immediate
family and received no visitors.
Which of the following is the most appropriate management of
his dysphagia?
A. Allow comfort feeding
B. Nasogastric feeding tube
C. Keep nil-by-mouth
D. Percutaneous endoscopic gastrostomy
E. Provide finger foods

60. An 84-year-old man had had three attacks of gout within the past
year. Initially, gout attacks had been rare and easily managed with
ibuprofen. He had marked symptoms of dyspepsia with NSAIDs.
He was currently between episodes.
Which of the following is the next most appropriate course of
action?
A. Colchicine 500micrograms TDS for 3 days during acute flares
B. Prednisolone 20mg to cover acute flares
C. Regular Allopurinol 300mg OD
D. Regular Febuxostat 80mg OD
E. Regular ibuprofen therapy

61. An 86-year-old man has been treated for a LRTI for the past
week in hospital. He reported previously living in a house with his
wife and being independently mobile over short distances with
a frame.
His acute condition seemed to be resolving, but the nursing staff and
physiotherapists reported that they were having to hoist him. Further
examination identified mild fixed flexion deformities of both knees,
bilateral quadriceps wasting and bilateral foot drop. There was no focal
upper limb neurology.
What is the most appropriate course of action?
A. Check creatinine kinase levels
B. Confirm premorbid function
C. Electromyography (EMG)
D. MRI thoracolumbar spine
E. Refer for intensive rehabilitation
EXAM 3 | QUESTIONS 201

62. An 84-year-old woman was seen in the outpatient department


for routine review following admission with pneumonia. Her only
past medical history was of recurrent falls. Having completed her
course of amoxicillin, she took no regular medications.
She told you that the GP was concerned by a BP reading of
150/90mmHg. Consequently, she had been given a home BP monitor,
with which she recorded her BP twice daily for 2 weeks. Her average
reading from this was 138/87.
What is the next most appropriate course of action?
A. Initiate treatment with an ACE-I
B. Initiate treatment with a calcium channel blocker
C. Initiate treatment with a thiazide diuretic
D. Lifestyle advice, reassurance, and periodic BP checks
E. Lying and standing BP recording

63. A 92-year-old man presented following an unwitnessed fall 30min


previously. He was found when his carer heard a crash. He was
unconscious with evidence of a head injury and significant facial
bruising. The carer reported he was unconscious for about 2min.
His GCS had since improved and it was 15/15 in the ED. He had
no relevant past medical history and had not fallen previously. He
took no regular medications.
On examination, there was no focal neurological deficit. Cardiovascular,
respiratory, and abdominal examination was normal. There was no
lying to standing drop in BP at the bedside and his ECG confirmed sinus
rhythm.
Which of the following is the next most appropriate step in his
management?
A. Active stand and Tilt table testing
B. Arrange a CT scan of his head
C. Commence a calcium supplement
D. Request a 24hr cardiac monitor
E. Structured balance training programme
202 EXAM 3 | QUESTIONS

64. You are asked to complete a Do Not Attempt Resuscitation


(DNACPR) order for an 87-year-old woman with multi-organ
failure and severe type 2 respiratory failure. She was confused
with an MMSE of 12/30.
Which of the following statements is true?
A. A patient cannot insist on a DNACPR order if resuscitation is likely to be successful
B. A relative with a lasting power of attorney for health can refuse a DNACPR order
C. It is not compulsory for the medical team to inform the patient of the DNACPR order
D. The DNACPR order means she should not be considered for non-invasive ventilation
E. The DNACPR order should automatically be reversed if the condition improves

65. An 84-year-old woman presented with a 1-year history of


dizziness. She had a past medical history of depression,
hypertension, and osteoarthritis of her cervical spine. Current
medications included citalopram 10mg OD, bendroflumethiazide
2.5mg OD, and paracetamol. She had no known drug allergies.
Since the death of her husband 12 months previously, she had
lived alone in a bungalow, and had no formal social services
support. She did not drink alcohol and was a non-smoker.
She reported that her dizziness could come on at any time and
might occur several times during the day. During an attack, she felt
light-headed and frequently experienced tingling in her fingers. She
did not experience the sensation of the room spinning, although she
sometimes felt that she was ‘distant’ from her body. She had never lost
consciousness during an episode. She did not experience chest pain,
palpitations, or breathlessness. Symptoms occurred when standing,
sitting, or lying.
Examination showed a regular pulse of 70beats/min, with a lying BP
of 132/80mmHg, which rose to 138/82mmHg on standing. Heart
sounds were normal. Respiratory and abdominal examination was
unremarkable. She had normal tone, power and co-ordination in all four
limbs. Plantars were downgoing. Cranial nerves were intact, with no
evidence of nystagmus.
Baseline investigations
• ECG: sinus rhythm.
• Bloods: normal FBC, renal function, corrected calcium and d-dimer.
• Urine dip: unremarkable.
What is the most appropriate next course of action?
A. CT pulmonary angiography
B. Cardiac Holter monitoring
C. Stop bendroflumethiazide
D. Trial of betahistine
E. Trial of propranolol
EXAM 3 | QUESTIONS 203

66. A 72-year-old man presented with a history of left-sided weakness.


He had noticed that his left arm and leg were not moving, and
had gone numb. This resolved over 30min. His only medications
were aspirin 75mg OD and simvastatin 40mg ON. Cardiovascular
and neurological examination were unremarkable. ECG showed
sinus rhythm. Outpatient investigations were arranged.
An MRI head scan was unremarkable. Carotid ultrasound demonstrated
a left-sided internal carotid artery stenosis of 70% with anterograde flow.
The right carotid was unobstructed. Blood tests showed a normal fasting
glucose, HbA1c and a fasting cholesterol of 3.9 mmol/L.
24-hr tape demonstrated sinus rhythm throughout.
Which of the following is the most appropriate next step in TIA
clinic?
A. Add dipyridamole M/R 200mg BD
B. Anticoagulate with warfarin
C. Change the aspirin for clopidogrel
D. Increase simvastatin dose to 80mg ON
E. Referral to vascular surgeons for endarterectomy

67. At a symposium on case management of community-dwelling frail


elders a community matron asks what factor is most likely to prevent
her proposed attempts to engage care home residents in advanced care
planning?
A. Severe dementia
B. Family concerns
C. General practitioner support
D. Staff concerns
E. Time constraints

68. A 73-year-old male with type II diabetes and controlled


hypertension presented with dysphasia, left hemi-neglect, and left
hemiplegia, all being present for 24hr.
Which of the following best defines the type of stroke?
A. Right hemisphere lacunar stroke
B. Right hemisphere partial anterior circulation stroke (PACS)
C. Right hemisphere posterior circulation stroke (POCS)
D. Right hemisphere total anterior circulation stroke (TACS)
E. Right hemisphere transient ischaemic attack (TIA)
204 EXAM 3 | QUESTIONS

69. An 82-year-old man with type II diabetes, hypercholesterolaemia,


and hypertension developed sudden painless loss of vision in his
left eye, whilst out shopping. FBC, ESR, random blood sugar,
U&E, and creatinine were all normal.
What is the most likely explanation?
A. Acute glaucoma
B. Age-related macular degeneration
C. Giant cell arteritis
D. Ischaemic optic neuritis
E. Retinal vein occlusion

70. An 84-year-old man presented with hip pain following a fall. He


had a background of osteoarthritis, hypertension, and diabetes
mellitus. Plain X-ray imaging excluded a fracture, and he was
admitted for analgesia and physiotherapy. He was given regular
paracetamol, tramadol 50mg QDS, and PRN oramorph. 3 days
later, the physiotherapist reported that his rehabilitation was
being limited by ongoing pain on weight-bearing.
What is the most appropriate next step in his management?
A. Arrange for a femoral nerve block
B. Computed tomography (CT) imaging of the hip
C. Increase intensity of physiotherapy to twice daily
D. Magnetic resonance (MR) imaging of the hip
E. Refer for inpatient community rehabilitation

71. An 81-year-old woman was admitted with pneumonia. She was


treated with IV antibiotics. Her albumin on admission was 37g/L
(37–49). Three days later it had fallen to 30g/L. She was not
feeling any better, but was managing to eat all her meals. Her
CRP had remained relatively static at around 200mg/L (<10).
Which of the following is the most appropriate response to her
low albumin?
A. Check a urinary albumin:creatinine ratio
B. Check for urinary legionella antigen
C. Discuss with microbiology regarding antibiotic selection
D. Initiate supplemental NGT feeding
E. Prescribe nutritional and protein supplement drinks
EXAM 3 | QUESTIONS 205

72. A 72-year-old man was admitted with a distended abdomen.


He had recently reported poor sleep, fluctuating confusion, and
a yellow tinge to his skin. Examination demonstrated palmar
erythema, eight spider naevi, conjunctival jaundice, and a
distended abdomen with shifting dullness. His GCS was 10. His
daughter reported that he had drunk in excess of 10U of alcohol a
day for many years and ate a poor diet.
Which of the following is the most important step in his
investigation and management?
A. Alpha fetoprotein (AFP) assay
B. A reducing regime of chlordiazepoxide
C. High dose oral thiamine, Vitamin B Co-Strong, and multivitamins
D. Lactulose +/– phosphate enemas to encourage frequent bowel motions
E. Urgent ascitic tap and microscopy

73. Which of the following is the first to occur in a healthy individual upon
moving from a lying to standing position?
A. Fall in right atrial pressure
B. Increased cardiac output
C. Increased stroke volume
D. Increased sympathetic drive
E. Rise in total peripheral resistance

74. An accepted marker of successful health care and healthy ageing


is ‘compression of morbidity’.
What would best explain a ‘compression of morbidity’?
A. Age-specific morbidity decreasing faster than age-specific mortality
B. Increasing life expectancy after onset of first long-term condition
C. Minimizing the fraction of an illness spent in an acute hospital
D. Reducing disease duration by improved management of long-term conditions
E. Reducing the burden of disease in the whole population

75. A 79-year-old woman presented following a fall. She tripped on a


loose rug. She had had a similar fall 3 months previously when she
slipped on a step. There were no syncopal features. She had a past
medical history of hypertension and took amlodipine 5mg OD.
Examination was unremarkable.
Which of the following is the most appropriate intervention?
A. Arrange calcaneal ultrasound bone density assessment
B. Arrange DEXA imaging
C. Arrange technetium-99m-MDB bone scintography of hip and lumbar spine
D. Calculate a FRAX score with no further initial imaging
E. Prescribe calcium + vitamin D and a monthly bisphosphonate
206 EXAM 3 | QUESTIONS

76. An 88-year-old man presented with mild confusion. He had


had increasing urinary symptoms over the past few days, and a
urine dip was positive for nitrites and leucocytes. He appeared
somewhat unkempt and the paramedics reported that his house
was littered with rubbish.
Blood tests results
Haemoglobin 14.5g/dL (13–18)
White Cells 12.2 × 109/L (4–11)
Platelets 265 × 109/L (150–400)
MCV 102fL (80–96)
Sodium 142mmol/L (137–144)
Potassium 4.7mmol/L (3.5–4.9)
Creatinine 84µmol/L (60–110)
CRP 86mg/L (<10)
Albumin 42fL (80–96)
Bilirubin 17µmol/L (1–22)
ALP 147U/L (45–105)
ALT 72U/L (5–35)
He was started on oral co-amoxiclav. Over the next 48hr, the
inflammatory markers improved. However, he became increasingly
agitated. He was sweaty, tachycardic, and flushed. He became paranoid
that the ward staff were trying to hurt him and was hallucinating. No
additional physical abnormality is evident on re-examination.
Which is the most appropriate course of action?
A. Change to second-line antibiotic
B. Perform a lumbar puncture
C. Prescribe risperidone 500micrograms BD
D. Reducing regime of chlordiazepoxide
E. Request urgent CT head scan
EXAM 3 | QUESTIONS 207

77. An 83-year-old woman was finding it increasingly difficult to


manage on her own at home despite three times a day social
care provision. She had chronic back pain from multiple vertebral
crush fractures and osteoarthritis of the spine, with multiple level
radicular pain. She had recently suffered several falls, thought
to be due to inadvertent self-overdosing with opioid analgesia,
and this had resulted in a decline in her mobility. There was no
suggestion of head injury or declining cognition. After her last fall
she had been assessed in the ED where there was no evidence
of any fracture and FBC, U&Es, mid-stream urine (MSU), and
CXR were all normal. Her daughter also expressed concern as to
how well her mother was eating and thought she might be losing
weight.
Which intermediate care service is most likely to meet her
clinical need?
A. Assessment admission to the acute hospital with later ‘step-down’ intermediate care, once
condition stabilized
B. Hospital at home ‘step-up’ care after obtaining a domiciliary consultation by the community
geriatrician
C. Step-up bed-based care in a community hospital with dedicated daily specialist GP input
and geriatrician visits weekly
D. ‘Step-up’ bed-based care in local residential home with her GP providing medical input
E. ‘Step-up bed-based care in local residential home with regular community
geriatrician rounds
208 EXAM 3 | QUESTIONS

78. An 84-year-old woman was referred to the syncope service


with a history of recurrent falls and loss of consciousness. She
was experiencing up to one episode a week. She experienced
no preceding aura and would suddenly find herself on the floor.
Witnesses had reported that she could be unconscious for up to
5min and there was some rhythmical jerking of her arms. She
denied incontinence or tongue biting. She was confused for up
to half an hour after each episode and felt ‘off colour’ for the
next day or so. On direct questioning, she denied chest pain,
palpitations, and orthostatic light-headedness. She had one
episode while wearing a 24 hour tape arranged through her GP.
This demonstrated sinus rhythm with first-degree heart block
throughout.
A head-up tilt test was performed. This demonstrated an initial normal
BP response. After 20min, she was given a GTN provocation challenge.
Over the following 5min, there was an increase in heart rate from 70
to 110beats/min and a steady fall in systolic BP from 110 to 75mmHg.
The test was terminated at this point as she felt unwell. Haemodynamic
parameters normalized over the following 10min. She was not confused
or unwell.
Which of the following is the most likely diagnosis?
A. Arrhythmia
B. Orthostatic hypotension
C. Postural orthostatic tachycardia syndrome
D. Reflex syncope
E. Seizures
EXAM 3 | QUESTIONS 209

79. A 78-year-old man presented with a lacunar stroke causing


left-sided weakness with power 3/5. He was in atrial fibrillation.
No other significant risk factors were identified.
Over the next 4 weeks on the rehabilitation unit he had been making
good functional recovery, and was able to transfer with assistance of
one and sit out in a chair for 4hr stretches. His MMSE was 29/30. He had
been therapeutically anticoagulated by this stage. The multidisciplinary
team were starting to plan for discharge back home when his functional
abilities deteriorated. He now required two to transfer owing to
unsteadiness and asked to be returned to bed after only a few minutes.
Formal neurological examination was unchanged. Chest auscultation is
unremarkable and he was apyrexial.
Which of the following is the most likely diagnosis?
A. Aspiration pneumonia
B. Dementia
C. Depression
D. Further ischaemic stroke
E. Haemorrhagic transformation

80. Which of the following is the most significant risk factor for developing
dementia?
A. Apolipoprotein E4 mutation
B. Diabetes
C. Head trauma
D. Increasing age
E. Presenilin-1 mutation

81. A 66-year-old, right-handed publican presented to the ED


with sudden onset left hemiparesis. He also had difficulty
expressing himself, struggling to verbalize and find words. He
could, however, write fluently with his right hand. When testing
receptive function there was a lag of up to 30s between issuing
an instruction and him following it, although he then did so
correctly.
A CT perfusion scan excluded haemorrhage and did not show any focal
perfusion or time-to-peak delay.
Which of the following is the most appropriate course of action?
A. Aspirin 300mg stat
B. Clopidogrel 300mg stat
C. Observation
D. Thrombolysis with recombinant tissue plasminogen activator (rTPA)
E. Urgent MRI
210 EXAM 3 | QUESTIONS

82. A 93-year-old woman presented with confusion. She gave a


history of recent onset dysuria, urinary frequency, and urinary
urgency. Her past medical history included type 2 diabetes
mellitus, pernicious anaemia, mixed peripheral neuropathy,
and rheumatoid arthritis. Her current medications consisted
of metformin 500mg BD, folic acid 5mg once weekly and
methotrexate 5mg weekly. She previously developed a rash with
ampicillin.
On examination she was pyrexial at 37.4ºC and had suprapubic
tenderness. Bedside urine analysis was positive for protein, leucocytes,
and nitrites.
Which of the following treatments is most appropriate to start?
A. Ciprofloxacin 500mg BD
B. Cranberry Juice 1L/day
C. Nitrofurantoin 50mg ON
D. Tazocin 4.5g TDS
E. Trimethoprim 200mg BD

83. An 83-year-old man was referred by the tissue viability team


for a non-healing peri-anal ulcer. The ulcer was extremely
painful and had not improved with the use of barrier creams or
a pressure relieving mattress. The patient had a past medical
history of diabetes, hypercholesterolaemia, and ischaemic heart
disease. Medication included metformin 500mg BD, simvastatin
40mg OD, nicorandil 20mg BD, and aspirin 75mg OD. The
patient was fully mobile and independent with all activities of
daily living.
Examination confirmed a discrete perianal ulcer. There was no evidence
of neurovascular compromise in the lower limbs. His serum albumin was
38g/L (37–49) and his HbA1c 6.5% (3.8–6.4) = 48mmol/mol.
Which of the following is the most appropriate intervention?
A. Add a twice daily oral sulphonylurea
B. Admit to hospital for daily dressings and observation
C. Start high protein dietary supplementation
D. Substitute isosorbide mononitrate for nicorandil
E. Supply pressure relieving cushion for chair
EXAM 3 | QUESTIONS 211

84. Which of the following does not increase the susceptibility of the elderly
to hypothermia?
A. Impaired sensation of falling temperature
B. Impaired shivering response
C. Lower resting core body temperature
D. Lower threshold for initiating shivering
E. Reduced skin vasoconstriction

85. A 73-year-old man with restless legs syndrome developed


severe nausea and vomiting, and was unable to keep any food or
fluid down.
Which anti-emetic would be least appropriate to prescribe
for him?
A. Aprepitant
B. Domperidone
C. Granisetron
D. Metoclopramide
E. Ondansetron

86. A 79-year-old man was admitted to hospital after having lost


consciousness. His wife witnessed the event and described him as
staring into space, then falling to the ground jerking from head
to toe. He went blue and was incontinent. When he came round
he was vague and appeared confused for several minutes. Physical
examination was entirely normal. Resting ECG was normal.
What is the most accurate statement regarding anti-epileptic
drugs (AED) for a first ever seizure?
A. AED allow the patient to continue driving
B. AED have no influence on further seizure risk
C. AED should be started if the head CT scan is abnormal
D. Early introduction of AEDs will prevent any further seizures
E. In alcohol withdrawal AED reduce the risk of further seizure
212 EXAM 3 | QUESTIONS

87. An 84-year-old man had recurrent small bowel obstruction


secondary to adhesions. He had declined surgical intervention and
hoped to be managed palliatively. He did not wish to have a NGT
inserted. His principle symptom was persistent nausea. He was
not experiencing any abdominal pain.
Which of the following is the most appropriate first line
antiemetic to use?
A. Cyclizine
B. Domperidone
C. Haloperidol
D. Metoclopramide
E. Ondansetron

88. A 79-year-old woman was admitted from home after being


found wandering on the motorway. No acute illness was
detected, but she remained disorientated in time, place, and
person over the next week. It transpired that she had no family
or friends and had not seen her general practitioner for over
10 years. A member of the ward staff lived in the same village
and said that the patient was well known in the village as being
‘eccentric’. She lived in a dilapidated cottage, and was often
seen wandering in the village and fields in a dishevelled state
in her nightdress. The patient believed that she was capable
of looking after herself and that if she called at any house in
the village for help she would get it. She wished to go home
and stated that she did not need any help. She could offer no
explanation for being on the motorway.
What is the most appropriate next course of action?
A. Arrange discharge home
B. Assess her mental capacity
C. Assess her cognition
D. Refer for an Independent Mental Capacity Advocate
E. Refer to liaison old age psychiatry

89. Sarcopenia is a major age-related physiological change in older


people. Reduced physical activity is not the only causal factor.
Which of the following is least likely to contribute to the
physiological anorexia of ageing?
A. Altered gastric distension
B. Decreased cytokine activity
C. Delayed gastric emptying
D. Diminished sense of smell and taste
E. Increased secretion of cholecystokinin
EXAM 3 | QUESTIONS 213

90. A 71-year-old spinster was seen in the ED because she had


become agitated, restless, and aggressive in her residential home.
She had transferred to the care home the previous day as the
community psychiatric team had found it increasingly difficult to
support her in her own home, because she frequently wandered
and was neglecting herself. Her neighbours had also been
concerned regarding her inappropriate behaviour, she was often
seen naked in her garden and would swear at anyone who visited
her house. She had an 18-month history of dementia, with no
correctable cause identified. She would not take any medication
and so had not been treated with an acetylcholinesterase
inhibitor. There was no known history of recent trauma.
In the ED she was agitated, restless (pacing), verbally abusive, and would
randomly hit out at staff. Intermittently, she was calm, and would accept
staff examining her and allowed them to take urine and blood samples.
She was not obviously hallucinating. It was impossible to sustain any
conversation with her as she was easily distracted and the content of
her speech was unstructured. She did not appear to be in pain. Physical
examination was normal. FBC, U&Es, LFTs, MSU, oxygen saturation,
CXR, and ECG were normal. A head CT-scan showed age-related
changes.
What would be the most appropriate care plan?
A. Admit to the acute hospital for assessment of delirium
B. Return to the care home with intensive support from the community mental health team
C. Transfer to the psychiatric unit for assessment of her behavioural disturbance
D. Urgent placement in a specialist dementia registered residential care home
E. Return to her own home with a 24-hr live-in carer
214 EXAM 3 | QUESTIONS

91. A 75-year-old man was discharged from hospital after 8 days,


having made an uneventful recovery from a community-acquired
atypical pneumonia. He was independent in mobility and self-care
on discharge. He was re-admitted 7 days later with a 2-day history
of double vision and unsteadiness of gait. He had been bedfast for
the last day.
On examination he was alert and orientated, with an AMT score of
9/10. Pulse was 78beats/min and regular, BP 140/77mmHg, respiratory
rate of 14/min with oxygen saturations of 98% on air. Chest was clear
to auscultation and heart sounds were normal. Examination of the
nervous system showed limitation of eye movements and diplopia in
all directions of gaze, incoordination of all limbs, grade 4/5 power in all
muscle groups of all four limbs, and generalized absence of reflexes.
Both plantar responses were flexor. Sensation was slightly impaired to
touch and pin prick in both legs up to the knees. He was unable to walk
in a straight line.
The following investigations were all normal: FBC, urea, creatinine,
electrolytes, LFT, bone profile, thyroid function, B12, folate, CRP,
blood sugar.
What is the most likely diagnosis?
A. Brain stem encephalitis
B. Brain stem stroke
C. Miller Fisher Syndrome
D. Myasthenia gravis
E. Wernicke’s encephalopathy

92. An 83-year-old man with Parkinson’s disease lived with his


daughter. She supervised his medication and, occasionally, had to
help him with dressing and cutting up his food. The Parkinson’s
specialist nurse advised them to claim attendance allowance.
However, they were reluctant to do so, as they did not want to
reveal their financial situation to anyone.
What is the most appropriate advice to give them about
attendance allowance?
A. He is not eligible because he lives in his daughter’s house
B. His care needs do not justify receiving the attendance allowance
C. It does not have to be used to pay for external carers
D. They can claim the attendance allowance when his daughter retires
E. The nurse cannot help them complete the application form
EXAM 3 | QUESTIONS 215

93. A 90-year-old male resident of a specialist dementia care home


was noted to have a raised, centrally crusting lesion with a beaded
edge of <1cm diameter on the tip of his nose.
What is the most appropriate management plan?
A. No further action required
B. No specific treatment, keep under review
C. Regular topical steroid cream
D. Punch biopsy for diagnosis
E. Wide excision and radiotherapy

94. A 77-year-old man was admitted to hospital after falling off his
bicycle whilst cycling with friends. He had no recollection of the
event or of the ambulance journey to hospital. He had suffered a
few grazes and had been incontinent. On arrival at the hospital,
he was fully conscious and there were no abnormal findings
on clinical examination. Nothing like this had ever happened
before. Four years earlier he had suffered a small intracerebral
haemorrhage with no residual functional impairment. He took no
medication.
What is the most appropriate first step in trying to determine
what happened?
A. Contact one of his fellow cyclists
B. Electroencephalography
C. Exercise stress test
D. MRI head scan
E. Twenty-four-hour Holter monitoring

95. When she attended for her yearly influenza vaccine a 70-year-old
female with COPD mentioned that for the last 3 years she had
been prone to leak urine whenever she coughed or strained. She
ensured that she kept her bowels regular by eating plenty of dried
fruit each day. Apart from her steroid and beta-agonist inhalers,
she was on no other medication.
What is the most appropriate advice to give her regarding pelvic
floor exercises?
A. Prior digital vaginal examination can determine response
B. Once continence is regained the exercises can be stopped
C. They are best performed as part of an exercise group
D. They need to be done several times a day until muscles fatigue
E. They will usually be effective within 1–2 months
216 EXAM 3 | QUESTIONS

96. A 73-year-old woman with tremor dominant Parkinson’s disease


of 6 years duration was taking Madopar® 125 five times a day,
ropinirole 3mg TDS, fludrocortisone 200micrograms BD,
alendronic acid 70mg on Sundays and Adcal D3 Forte® every day
except Sunday.
Which of the following is least likely to assist her in remaining
independent in the kitchen?
A. Kettle tipper
B. Perching stool
C. Steady spoon
D. Utensil hand grip
E. Walking trolley

97. An 84-year-old woman developed electrolyte abnormalities,


whilst being treated for a right basal pneumonia. Her oral intake
had been poor and she had been started on a nasogastric feeding
regime. She was currently receiving 2000mL of feed a day, which
was meeting her calorie and protein requirements. She was
drinking up to 500mL water/day.
On admission her serum sodium was normal – 138mmol/L (137–144).
48hr after starting mirtazapine (for a likely depression) it was noticed
to be falling. It continued to fall over the next 3 days and was now
115mmol/L (137–144). The mirtazapine was stopped. Her only other
medication was amoxicillin.
On clinical examination the patient appeared euvolaemic. Serum
osmolality was 246mosmol/kg (278–305). Urinary osmolality was
600mosmol/kg (350–1000). A urine spot sodium was raised at 90mmol/L
Which of the following is the most appropriate next step?
A. Demeclocycline 300mg TDS
B. Intravenous 0.9% NaCl
C. Intravenous 1.8% NaCl
D. Review feeding regime
E. Tolvaptan 15mg OD

98. Which of the following outcomes justifies screening of asymptomatic


individuals for cancer?
A. Decreased cancer specific mortality
B. Decreased morbidity related to screening
C. Improved end of life planning
D. Increased cancer survival after detection
E. More lesions detected at an early stage
EXAM 3 | QUESTIONS 217

╇ 99.╇An 82-year-old widower complained of insomnia, fatigue, weight


loss, and anorexia over the last year. There was no change in
bowel habit. Physical examination was normal. FBC, thyroid
function, LFT, bone profile, electrolytes, serum creatinine, and
CRP were all normal.
What is the most appropriate next step in his management?
A. Arrange a second opinion
B. Check tumour markers
C. Screen for alcohol abuse
D. Screen for depression
E. Screen for myeloma

100.╇The wife of 79-year-old man with Parkinson’s disease contacted


the Parkinson’s Specialist Nurse to ask what she should do, as
in the last week her husband had been asking her why there
were so many children and small animals in the house. He was
usually independent in his personal care, but she also reported
that she had noticed that he smelt of urine and was not eating
or drinking as well as he usually did. His medication was
co-careldopa 125 five times a day and ropinirole MR 8mg BD.
Which of the following is the most appropriate advice to give
his wife?
A. A plan on how to reduce the dose of co-careldopa
B. A plan on how to reduce the dose of his ropinirole
C. He should be seen by his GP and screened for infection
D. He should be started on rivastigmine by the Parkinson’s specialist
E. She must reassure him that these are hallucinations and not real
exam 

3 ANSWERS

1. D.╇ The Fried clinical phenotype of frailty is characterized by:


• Weight loss (>5% per annum or >4.5kg).
• Fatigue/exhaustion (3–4 days per week or most of the time).
• Low energy expenditure.
• Slowness.
• Weakness (reduced grip strength).
Reproduced from Fried et al., ‘Frailty in older adults: evidence for a phenotype’, Journals of
Gerontology Series A: Biological Sciences and Medical Sciences, 2001, 56, 3, 146–156, by permission of
Oxford University Press and The Gerontological Society of America.

People with none of the five indicators are characterized as robust; those with one or two
indicators as pre-frail; those with three or more indicators as frail. People with MMSE less than 18
were excluded from the original cohort from which the Fried model was developed, and so the
relationship between the frailty phenotype and cognitive impairment is uncertain.

Further reading
Fried LP, Tangem CM, Walston J, et al. Cardiovascular Health Study Collaborrative Research Group.
Frailty in older adults: evidence for a phenotype. Journals of Gerontology Series A: Biological Sciences
and Medical Sciences 2001; 56: M146–56.
Available at: http://www.ncbi.nlm.nih.gov/pubmed/11253156

2. C.╇ It would seem that ever since being diagnosed with dementia, this woman has been expressing
an opinion that she would rather be dead than alive and that this has not been thought to be a
manifestation of a depressive illness. Euthanasia is not legal in the UK. It is generally believed to be
illegal to assist someone to access physician-assisted suicide abroad, but the law is not entirely clear
regarding the likelihood of prosecution. Her previous and current refusal to take medication, and
variable food and fluid intake, would seem to be compatible with her frequently expressed wishes
not to have her life prolonged and, therefore, her nieces’ appear to be advocating a course of action
that follows her prior expressed wishes and actions. There is no reason to engage an IMCA as the
Lasting Power of Attorney Health Proxy allows the nieces’ to advocate on their aunt’s behalf.
Inserting a NGT would enable her to be fed, hydrated, and make control of her diabetes easier, if she
did not pull it out! However, this is not obviously in her best interests and seems to be contrary to all her
previous actions and expressed wishes. The dilemma here is that they have never been written down.
As she is refusing medication and resisting her insulin injections, withdrawing these as requested by her
nieces seems to be the right course of action. This may well foreshorten her life, but as the intention is
not to end her life prematurely, but to maintain her dignity, and so far as one can tell, follows her wishes,
this would be the best option here. It would be sensible to reassure yourself (the Bolam and Bolitho
tests) that this is a reasonable and logical course of action by obtaining a second opinion, but not with the
220 EXAM 3 | ANSWERS

intent of going against the nieces’ advocacy on their aunt’s behalf. If you are satisfied that the intended
course of action meets the Bolam and Bolitho tests then there would not appear to be any reason to
involve the Trust lawyers, as there is no conflict here with the nieces’ and medical negligence is unlikely.

Further reading
Department of Health. Independent Mental Capacity Advocate (IMCA) service. Available at:
http://webarchive.nationalarchives.gov.uk/+/www.dh.gov.uk/en/SocialCare/Deliveringsocialcare/
MentalCapacity/IMCA/index.htm
BMA. Medical Ethics Today: The BMA's Handbook of Ethics and Law.
Available at: http://bma.org.uk/practical-support-at-work/ethics/medical-ethics-today
HM Government. Mental Capacity Act 2005 (England). Code of Practice. Available at: http://
webarchive.nationalarchives.gov.uk/+/http://www.dca.gov.uk/legal-policy/mental-capacity/mca-cp.pdf
Independent Mental Capacity Advocate. Making decisions. The Independent Mental Capacity
Advocate (IMCA) Service. Available at: http://webarchive.nationalarchives.gov.uk/+/http://www.dca.
gov.uk/legal-policy/mental-capacity/mibooklets/booklet06.pdf
Patient.co.uk. Clinical negligence. Bolam test and Bolitho test.
Available at: http://www.patient.co.uk/doctor/clinical-negligence
3. C.╇ NICE recommends the following criteria for identifying patients at high risk of refeeding
problems (level D recommendations).
Either the patient has one or more of the following:
• Body mass index (kg/m2) <16.
• Unintentional weight loss >15% in the past 3–6 months.
• Little or no nutritional intake for >10 days.
• Low levels of potassium, phosphate, or magnesium before feeding.
Or the patient has two or more of the following:
• Body mass index <18.5.
• Unintentional weight loss >10% in the past 3–6 months.
• Little or no nutritional intake for >5 days.
• History of alcohol misuse or drugs, including insulin, chemotherapy, antacids, or diuretics.
National Institute for Health and Clinical Excellence (2006). Adapted from ‘CG32 Nutrition support
in adults: oral nutrition support, enteral tube feeding and parenteral nutrition’. London: NICE. Available
from http://guidance.nice.org.uk/CG32. Reproduced with permission.
His dysarthria and delirium are likely reflections of malnutrition and biochemical disturbances.
His poor dentition, along with the bleeding gums and petechial rash are suggestive of vitamin C
deficiency, and may subsequently contribute to poor nutritional intake. Whilst his lifestyle may put
him at risk of malnutrition and is to be likely to have an alcohol problem, in itself, being homeless
does not necessarily mean that he will go hungry or become malnourished.
Further reading
Kraft MD, Btaiche IF, Sacks GS. Review of the refeeding syndrome. Nutrition in Clinical Practice
2005; 20: 625–33. Available at: http://surgery.uc.edu/content/Education/residentresources/SICU-x/
Refeeding%20(Kraft%202005).pdf
Mehanna HM, Moledina J, Travis J. Refeeding syndrome: what it is, and how to prevent and treat it.
British Medical Journal 2008; 336(7659): 1495–8.
Available at: http://www.ncbi.nlm.nih.gov/pmc/articles/PMC2440847/
EXAM 3 | ANSWERS 221

NICE CG32: Nutrition support in adults, Clinical guideline CG32, 2006.


Available at: http://www.nice.org.uk/CG32

4. C.╇ The raised inflammatory markers and lack of other clinical focus raise concern regarding the
possibility of deep-seated infection. X-rays are not sensitive enough to exclude early osteomyelitis.
MRI, while usually the preferred modality, are contraindicated owing to the pacemaker. A white
cell scan should be undertaken to look for bony involvement, before committing the patient to a
prolonged course of treatment.

Further reading
NICE. Diabetic foot problems: inpatient management of diabetic foot problems, Clinical guideline
CG119. Available at: http://publications.nice.org.uk/diabetic-foot-problems-cg119

5. D.╇ Factor prothrombin complex concentrates (e.g. beriplex and octaplex) contain factors II, VII,
IX, and X. They are able to completely reverse warfarin-induced anticoagulation within 10min and
are most appropriate for use in massive bleeding associated with anticoagulation. The factor VII
has a half-life of only 6hr, so the administration of the prothrombin complex concentrate should be
accompanied with 5mg IV vitamin K.
Fresh frozen plasma is more dilute and requires infusion of large volumes with less rapid and inferior
correction of coagulopathy. It is reserved for second line use in massive bleeding when prothrombin
complex concentrate is not available.
Vitamin K will result in significant correction of the INR within 6–8hr of IV administration. This
route produces a more rapid correction than if given orally.
Current UK guidelines for non-bleeding patients with INR > 8 suggest treatment with 1–5mg oral
vitamin K. For patients with INR >5, but <8 should have 1–2 doses of warfarin withheld. The
maintenance dose should be reduced in both cases and the cause of the raised INR investigated.

Further reading
Keeling D, Baglin T, Tait C, et al. Guidelines on anticoagulation with warfarin—fourth edition. British
Journal of Haematology 2011; 154(3): 311–25.
Available at: http://www.bcshguidelines.com/documents/warfarin_4th_ed.pdf
British National Formulary. Available at: http://bnf.org/bnf/index.htm

6. C.╇ Approximately 15% of stroke survivors have convulsions and will need long-term treatment
with anti-epileptic drugs (AED). Sodium valproate is suitable monotherapy for tonic-clonic seizures,
with a usual starting dose of 600mg daily in 1–2 divided doses, increased gradually (in steps of 150–
300mg) every 3 days to a usual maintenance dose of 1–2g daily (20–30mg/kg daily) and a maximum
dose of 2.5g daily. The starting dose of 700mg is very high and, with the absence of further seizures,
should lead you to suspect that the necessary maintenance dose may not need to be this high. As a
general rule, the BNF recommended dose titration interval for most dugs is too fast in older people
as it is based on data derived from a young adult population (remember, start low and go slow).
Common side-effects of valproate include ataxia and tremor. Less common side-effects include
confusion, hallucinations, and very rarely, extrapyramidal symptoms, dementia, and encephalopathy.
The previous history of stroke rules out a diagnosis of idiopathic Parkinson’s disease using UK
Brain Bank Diagnostic Criteria. The absence of hallucinations and profound variability in physical
and mental function makes dementia with Lewy bodies unlikely although a possibility. The patient
does have cerebrovascular disease, cognitive impairment and extrapyramidal signs, so it would be
possible that she has vascular dementia and vascular Parkinsonism, but the combination is not an
option in the answers, and so D and E are incorrect.
222 EXAM 3 | ANSWERS

Further reading
British National Formulary. Available at: http://bnf.org/bnf/bnf/current/3599.htm
Summary of product characteristics. Available at: http://www.sanofi.co.uk/products/Epilim_Crushable.
pdf

7. B.╇ The preserved cognition, well-formed hallucinations, and preserved insight, coupled with
significant visual impairment, are consistent with Charles Bonnet Syndrome, rather than any form of
dementia or psychosis. Reassurance along with improving lighting and safety at home, and reducing
social isolation, are the mainstays of treatment.

Further reading
Jacob A, Prasad S, Boggild M, Chandratre S. Charles Bonnet Syndrome—elderly people and visual
hallucinations. British Medical Journal 2004; 328: 1552–4.
Available at: http://www.bmj.com/content/328/7455/1552.full

8. E.╇ Adjusting lifestyle is the foundation of treatment for stable angina (stop smoking, weight loss,
pacing activities, reducing stress, and anxiety), followed by modification of vascular risk factors, so
he should be offered a statin. As his angina comes on after a reasonable period of activity NICE
guidance advises that he should first be offered a short-acting nitrate, e.g. nitrolingual spray. If he
required regular medication then a beta-blocker or calcium channel blocker would be first choice,
with the two combined as the next step. If he cannot tolerate either of these or additional therapy
is required then drugs such as a long-acting nitrate or ivabradine (D) or nicorandil (C) may be
considered. If symptoms are not controlled on optimal medical therapy then further investigations
should be considered and coronary angiography offered to determine if the individual would
benefit from revascularization.

Further reading
NICE. The management of stable angina, Clinical guideline CG126, 2011.
Available at: http://guidance.nice.org.uk/CG126
NICE. Chest pain of recent onset, Clinical guideline CG95, 2010.
Available at: http://guidance.nice.org.uk/CG95

9. D.╇ The person claiming carer’s allowance must be providing at least 35hr care per week and must
tell the person they are caring for that they are claiming, as it might affect the benefits of the person
they are caring for. If the daughter is receiving any other benefits, e.g. housing benefit, these may be
affected if she claims carer’s allowance. If the person cared for is not receiving attendance allowance
then carer’s allowance cannot be claimed. Carer’s allowance is not affected by any savings.

Further reading
HM Government. Carer’s allowance: Eligibility. Available at: http://www.direct.gov.uk/en/
MoneyTaxAndBenefits/BenefitsTaxCreditsAndOtherSupport/Caringforsomeone/DG_10012525
HM Government. Carer’s allowance: Further information. Available at: http://www.direct.gov.uk/en/
MoneyTaxAndBenefits/BenefitsTaxCreditsAndOtherSupport/Caringforsomeone/DG_10012529

10. C.╇ A history and examination should usually be sufficient to establish whether a patient has
stress, urge, or mixed incontinence. This patient has features of both urgency (little warning) and
stress (leaking) incontinence. NICE recommend supervised pelvic floor muscle and bladder training
as first-line treatment.
EXAM 3 | ANSWERS 223

If conservative measures are unsuccessful in urge (or mixed) incontinence, NICE suggest an
anticholinergic agent as the next step.
In unclear cases, or when planning surgery, urodynamic studies can confirm the nature of the
incontinence.
Transvaginal tape surgery (TVT) and formal colposuspension may restore continence in cases of
stress incontinence by restoring more natural anatomy and raising the urethral pressure. They are
invasive, however, and should only be undertaken in expert hands after failure of conservative and
medical approaches.
‘Pad and Pants’ or other containment procedures are not treatments for incontinence, rather coping
strategies. If possible, the underlying cause should be addressed.

Further reading
NICE. Urinary incontinence: the management of urinary incontinence in women, Clinical guideline
CG40. Available at: http://www.nice.org.uk/guidance/CG40

11. B.╇ The aim is to control her symptoms in the least invasive way. Conservative measures
(weight loss and exercise) and paracetamol are not proving sufficient. A topical non-steroidal cream
or capsaicin cream could be considered next. Were that to also prove insufficient, the next step
would be increased oral analgesia. NSAIDs are relatively contraindicated given the renal impairment
and need for aspirin. Codeine or another weak opiate would, therefore, be preferred.
If increased analgesia is ineffective or not tolerated then specialist referral for possible joint
replacement could be considered, as might steroid injections.

Further reading
NICE. Osteoarthritis: the care and management of osteoarthritis in adults, Clinical guideline CG59.
Available at: http://publications.nice.org.uk/osteoarthritis-cg59/guidance

12. B.╇ A multidisciplinary approach is important to managing spasticity. Physiotherapy, the use
of casts and splints, surgical release of contractures, and medication may all play a part. Careful
positioning, physiotherapy stretching, and splinting may reduce the spasticity and overcome
contractures, but take time, and will not help currently with progressing his mobility. Surgery can
release contractures that have already occurred and facilitate better postures to prevent further
contractures. Antispasmodic drugs, such as baclofen and dantrolene may reduce spasticity, but
there use is limited as they produce generalized weakness, thereby hindering physical function.
Injection of botulinum toxin in to spastic muscles produces local paralysis of selected muscles.
Judiciously applied it can reduce muscle over-activity, while maintaining strength in other muscles.

Further reading
Royal College of Physicians. Guidelines for the use of botulinum toxin (BTX) in the management
of spasticity in adults. Clinical Effectiveness and Evaluation Unit, Royal College of Physicians, 2002.
Available at: http://old.rcplondon.ac.uk/pubs/books/botox/botox.pdf

13. D.╇ BP lowering in type-II diabetes independently reduces cardiovascular risk. NICE
recommends initiating treatment in those aged <80 years with stage 1 hypertension (home BP >
135/85 and 150/95mmHg or clinic BP >140/90 and < 160/100mmHg) and in those with any of
the following:
• Target organ damage.
• Established cardiovascular disease.
224 EXAM 3 | ANSWERS

• Diabetes.
• Renal disease.
• 10-year cardiovascular risk >20%.
In someone with diabetes, NICE recommends a target BP of < 140/80mmHg (130/80mmHg
in the presence of renal, ophthalmic, or cerebrovascular damage). In the presence of
microalbuminuria an ACE-I or angiotensin receptor blocker (ARB) should be commenced,
irrespective of whether there is hypertension. If lifestyle modification does not control BP and
there is no end organ damage, then a calcium channel blocker is the initial drug of choice, in
someone of Afro-Caribbean decent.

Further reading
Royal College of Physicians. Type II diabetes: National clinical guideline for management in primary and
secondary care (update), 2008.
Available at: http://www.nice.org.uk/nicemedia/live/11983/40803/40803.pdf
NICE. Hypertension: clinical management of primary hypertension in adults, Clinical guideline CG127
(update). Available at: http://guidance.nice.org.uk/CG127

14. D.╇ The day-to-day and diurnal variability are suggestive of asthma. Similarly, there are no
features of hyper-expanded lungs or failure on the CXR. On lung-function testing, asthma would
be further suggested by > 400mL response to bronchodilators, >400mL response to 30mg
prednisolone daily for 2 weeks or significant (>20%) diurnal or day-to-day variation in peak flow.

Further reading
NICE. Chronic obstructive pulmonary disease (update): quick reference guide CG101.
Available at: http://guidance.nice.org.uk/CG101/QuickRefGuide/pdf/English

15. C.╇ Although carer education is an important facet of this patient’s management to improve
their confidence in recognizing change in the individual’s well-being it is not reasonable to expect
them to make the decision as to whether medical assessment is required or not; the default must
be that if they are concerned they should seek help and advice. A thorough medical assessment of
this individual should identify whether their condition has changed sufficient that referral for hospital
admission or further assessment (e.g. imaging) is required. Case management by a community
matron may reduce total numbers of hospitalizations and improve overall carer confidence, but
this was an out-of-hours call and so being on the caseload of a community matron would not
have avoided this referral to hospital. Telehealth and telecare may be beneficial in certain disease
specific circumstances to help manage a chronic disease, such as COPD, and may reduce overall
hospitalizations, but would be unlikely to prevent this particular referral.

Further reading
Ward KT, Reuben DB. Comprehensive geriatric assessment.
Available at: http://www.uptodate.com/contents/comprehensive-geriatric-assessment
Martin S, Kelly G, Kernohan WG, et al. Smart home technologies for health and social care support.
Cochrane Database of Systematic Reviews 2005; Issue 4: CD006412.
Available at: http://www.ncbi.nlm.nih.gov/pubmed/18843715
King’s Fund. The impact of telehealth: a review of the evidence. Available at: http://www.kingsfund.org.
uk/topics/technology_and_telecare/telehealth_evidence.html
Poole T. Telecare in older people: Wanless social care review. London: King’s Fund, 2006. Available at:
http://kingsfund.koha-ptfs.eu/cgi-bin/koha/opac-detail.pl?biblionumber=37212
EXAM 3 | ANSWERS 225

16. A.╇ 5–10% of patients given statins in clinical trials suffer from myalgia; the absolute risk in
general populations may be higher. The first step is to decide with the patient the severity of the
symptoms, as this will determine their preferred option of treatment.
Mild myalgia may be acceptable to the patient when balanced against the substantial cardiovascular
benefits of the drug, especially in diabetics (NNT 33). Modestly raised CK <5× upper limit of
normal may be related to exercise and does not demand action. Clinically significant CK rise is
>10× upper limit of normal and is a medical emergency. The minor rise in urea and creatinine,
and small rise in creatine kinase do not suggest a risk of rhabdomyolyisis and renal impairment, IV
fluids are not required, this scenario can be managed in the out-patient setting. As the CK is not
significantly raised, if the symptoms warranted stopping the statin the patient may be willing to try a
rechallenge with a lower dose or to try an alternative statin. If myalgia recurred or the patient was
unwilling to retry a statin then NICE recommends switching to ezetimibe 10mg daily.

Further reading
Buettner C, Davis RB, Leveille SG, et al. Prevalence of musculoskeletal pain and statin use. Journal of
General Internal Medicine 2008; 23(8): 1182–6.
Available at: http://www.ncbi.nlm.nih.gov/pmc/articles/PMC2517983/
Eckel RH. Approach to the patient who is intolerant of statin therapy. Journal of Clinical Endocrinology
and Metabolism 2010; 95: 2015–22. Available at: http://www.ncbi.nlm.nih.gov/pubmed/20444930
NICE. Ezetimibe for the treatment of primary (heterozygous-familial and non-familial)
hypercholesterolaemia, Guideline TA132, 2007. Available at: http://guidance.nice.org.uk/TA132
NICE. Lipid modification: cardiovascular risk assessment and the modification of blood lipids for the
primary and secondary prevention of cardiovascular disease, Clinical guideline CG67, 2008.
Available at: http://guidance.nice.org.uk/CG67

17. E.╇ The CT scan shows a large prolapsed bladder (cystocoele). This will need surgical review.
A cystocoele is a protrusion of the bladder into the vagina due to defects in pelvic support. The
radiographic definition of a cystocoele is descent of the bladder base below the inferior margin of
the symphysis pubis. Cystocoele bladder is a common cause of urinary incontinence in elderly. Risk
factors for pelvic organ prolapse increase with age, parity, and increasing body weight. It is more
common in post-menopausal women because of lack of oestrogen supporting the pelvic floor
muscle.

Further reading
Ghoniem GM. Cystocoele Repair. Medscape.
Available at: http://emedicine.medscape.com/article/1848220-overview
NICE. Urinary incontinence: the management of urinary incontinence in women, Clinical guideline
CG40. Available at: http://www.nice.org.uk/guidance/CG40

18. E.╇ If the local geriatricians are not engaged in IC provision this may skew the distribution/
selection of patients for IC and undermine general practitioner willingness to support ‘step-up’
services. An inability to recruit community rehabilitation staff will limit the provision of home-
based IC. Patient preference for bed-based care or reluctance to have home-based care may skew
service provision as may case-mix; i.e. limited numbers of patients suitable for ‘step-up’ care. The
site of bed-based provision is least likely to explain the skewed distribution unless there are clinical
governance issues regarding the safety of residential bed-based provision.
226 EXAM 3 | ANSWERS

Further reading
Department of Health. Intermediate care—Halfway home. Updated guidance for the NHS and Local
Authorities. Department of Health (England), 2009. Available at: http://www.dh.gov.uk/prod_consum_
dh/groups/dh_digitalassets/@dh/@en/@pg/documents/digitalasset/dh_103154.pdf
National audit of intermediate care report. 2012. Available at: http://www.nhsbenchmarking.nhs.uk/
projects/partnership-projects/National-Audit-of-Intermediate-Care.php

19. C.╇ Addison’s disease is associated with other autoimmune conditions. It is important to
consider these when undertaking a review. It is possible an associated coeliac disease may have
resulted in a malabsorption syndrome and associated weight loss and diarrhoea. This is easily
explored using antibody testing, and results of this are rapidly available and may prove diagnostic
without the need for more invasive or radiology based tests. It should be remembered that there is
a significant false negative rate, particularly if patients are not eating a gluten containing diet.
If the antibody test is negative then further investigation would be warranted.

Further reading
Goebel SU. Celiac Sprue. Medscape, 2012.
Available at: http://emedicine.medscape.com/article/171805-overview
Griffing GT. Addison Disease. Medscape, 2012.
Available at: http://emedicine.medscape.com/article/116467-overview

20. B.╇ We are told she does not wander and that she was found locked out of her house, so the
correct answer is a reminder to take her keys—this could be a simple note on the door for her to
see as she opens the door or a pressure mat near the door, which activates a recorded message
(preferably recorded by someone familiar to her) saying ‘take your keys with you if you are going
out’. She does not need a wandering monitor—mobile phones can be programmed using GPS to
allow dementia sufferers to roam within a specified area and to notify their carers/family if they
roam outside of this area. A key safe has a security code, which she may not remember—these
are usually provided for carers to gain access. We are not told she has problems with taking her
medication, but if she did, then a pill box with alarm function or SMS reminders could be helpful.

Further reading
Kerr B, Hurst K, Clark S. Telehealthcare and mental health: using telehealthcare effectively in the
support of people living with mental disorder. London: King’s Fund, 2011.
Available at: http://kingsfund.koha-ptfs.eu/cgi-bin/koha/opac-detail.pl?biblionumber=103952
King’s Fund. The impact of telehealth: a review of the evidence. London: King’s Fund. Available at:
http://www.kingsfund.org.uk/topics/technology_and_telecare/telehealth_evidence.html
Martin S, Kelly G, Kernohan WG, et al. Smart home technologies for health and social care support.
Cochrane Database of Systematic Reviews 2005; Issue 4: CD006412.
Available at: http://www.ncbi.nlm.nih.gov/pubmed/18843715
Poole T. Telecare in older people: Wanless social care review. London: King’s Fund, 2006.
Available at: http://kingsfund.koha-ptfs.eu/cgi-bin/koha/opac-detail.pl?biblionumber=37212

21. A.╇ It is important to try and identify if there is a trigger or pattern to the behavioural
disturbance; a behaviour chart may do this and point to an appropriate management strategy if
this behaviour persists. Mostly, such behavioural disturbance, in the context of a dementia is short-
lived and will resolve without specific intervention. Prescribing an antipsychotic would only be
EXAM 3 | ANSWERS 227

appropriate if no precipitant for the behaviour were identified and/or the individual were a danger
to themselves or others.
Acetylcholinesterase inhibitors may be useful in the treatment of psychotic symptoms associated
with dementia and, thereby, alter behaviour. The normal examination and initial blood and
urine tests suggest this is not a delirium, although it may turn out to be, and so a referral to the
community geriatrician is not indicated at this point as they will advise keeping a behaviour chart.
It would be inappropriate to move this resident to a specialist unit. These are really for managing
those with dementia and chronic behavioural disturbance.

Further reading
Scottish Intercollegiate Guidelines Network (SIGN). Management of patients with dementia, Guideline
86: Available at: http://www.sign.ac.uk/pdf/sign86.pdf
Banerjee S. The use of antipsychotic medication for people with dementia: time for action. A report
for the Minister of State for Care Services. Available at: http://www.dh.gov.uk/prod_consum_dh/
groups/dh_digitalassets/documents/digitalasset/dh_108302.pdf

22. B.╇ This patient does not need to be in an acute hospital bed. Similarly, rehabilitation in a
community hospital bed is not possible while she cannot weight bear. If available, an interim
placement would be appropriate, given that the patient can transfer with the frame and assistance
of one, her needs could be met in a residential care home setting.
Once the patient can weight bear and engage with rehabilitation, re-admission to a community
hospital for this might be appropriate. Alternatively, intermediate care input in the residential home
or back at her home may be possible.

23. B.╇ The INR is probably raised due to potentiation of the warfarin by the clarithromycin. Given that
the epistaxis is minor, omitting the warfarin and monitoring closely is appropriate. This could be done
on an outpatient basis. If the epistaxis is recurrent, fails to settle, or if the INR continues to rise, then
treatment with vitamin K may be appropriate. Were a significant epistaxis to occur, then reversal with
prothrombin complex concentrate + vitamin K and urgent ENT review for packing would be needed.
Four factor prothrombin complex concentrates (e.g. beriplex and octaplex) contain factors II, VII,
IX, and X. They are able to completely reverse warfarin-induced anticoagulation within 10min and
are most appropriate for use in massive bleeding associated with anticoagulation. The factor VII
has a half-life of only 6hr, so the administration of the prothrombin complex concentrate should be
accompanied with 5mg IV vitamin K.
Fresh frozen plasma is more dilute and requires infusion of large volumes with less rapid and inferior
correction of coagulopathy. It is reserved for second line use in massive bleeding when prothrombin
complex concentrate is not available.
Vitamin K will result in significant correction of the INR within 6–8hr of IV administration. This
route produces a more rapid correction than if given orally.
Current UK guidelines for non-bleeding patients with INR > 8 suggest treatment with 1–5mg oral
vitamin K. For patients with INR >5, but <8 should have 1–2 doses of warfarin withheld. The
maintenance dose should be reduced in both cases and the cause of the raised INR investigated.

Further reading
Keeling D, Baglin T, Tait C, et al. Guidelines on anticoagulation with warfarin, fourth edition. British
Journal of Haematology 2011; 154(3): 311–24.
Available at: http://www.bcshguidelines.com/documents/warfarin_4th_ed.pdf
British National Formulary Available at: http://bnf.org/bnf/index.htm
228 EXAM 3 | ANSWERS

24. B.╇ He has several risk factors for a depressive illness: old age, male, bereavement, alcohol
excess, previous history of depression, and social isolation. The normal MMSE makes a dementia
unlikely. He is psychotic, but there is no history of sudden decline to suggest a delirium and no
obvious precipitant for a delirium other than possible adverse effects of alcohol. As he smells of
alcohol he is hardly likely to be suffering from alcohol withdrawal. As he has no other visitors than
his niece and she was unaware of how much alcohol he has been drinking one must assume he
has been purchasing it. It is more than 2 years since his wife died so this is not a normal grieving
reaction and it would be inappropriate to refer him for bereavement counselling at this stage.
The GP has not identified any physical illness and so, whilst it is possible that he may have some
underlying physical illness a referral to a geriatrician does not seem most appropriate at this stage,
although the GP should perform some blood tests (FBC, U&E, calcium, LFTs and thyroid function),
as well as dip-stick testing a urine sample. In psychotic depression an SSRI may be useful, but it is
more likely that a more complex drug regime will be needed. Referral to the old age psychiatry
team is most appropriate. This should be considered in any older person with depression where
there is a poor response to antidepressants, psychosis, risk of self-harm (including severe neglect as
here), associated cognitive impairment, a need for complex multiprofessional input.

Further reading
NICE. Depression: the treatment and management of depression in adults (update), Clinical guideline
CG90. Available at: http://guidance.nice.org.uk/CG90
NICE. Depression in adults with a chronic physical health problem: treatment and
management, Clinical guideline CG91. Available at: http://publications.nice.org.uk/
depression-in-adults-with-a-chronic-physical-health-problem-cg91
Rodda J, Walker Z, Carter J. Depression in older age. British Medical Journal 2011; 343: d5219.
Available at: http://www.bmj.com/content/343/bmj.d5219.extract
NICE. Depression—clinical knowledge summary from NHS evidence. Clinical Knowledge Summaries.
Available at: http://cks.nice.org.uk/depression

25. E.╇ Diabetic amyotrophy is a form of peripheral neuropathy characterized by painful muscle
wasting and weakness, and would be a close fit to this scenario except that the pain is not
generalized, but usually related to proximal lower limb muscles.
The low grade anaemia with a normal ferritin rules out iron deficiency anaemia.
Osteoarthritis would be expected to give localized pain, although her dementia might hinder her
ability to localize pain, in which case pain-related behaviour would need to be observed.
Polymyalgia rheumatica only rarely occurs with a normal ESR and so would not be the most likely
explanation.
The borderline low calcium with a normal alkaline phosphatase point towards a problem with
calcium metabolism and are in keeping with a diagnosis of vitamin D deficiency (osteomalacia), as is
the history. Having been in a care home for 3 years she is likely to be vitamin D deficient. In a study
from Bradford, calcium was normal in 66%, phosphate in 81%, and alkaline phosphatase in 29%
patients. In only five out of 84 patients were all three indices outside the normal range. The median
parathyroid hormone concentration was significantly greater in patients with abnormal routine
biochemistry than in patients with normal routine biochemistry, but the median 25-hydroxyvitamin
D levels did not differ. Routine biochemistry was normal in 20% of cases.
EXAM 3 | ANSWERS 229

Further reading
Best Practice. Osteomalacia. Available at: http://bestpractice.bmj.com/best-practice/monograph/517.
html
Peacey SR. Routine biochemistry in suspected vitamin D deficiency. Journal of the Royal Society for
Medicine 2004; 97(7): 322–5. Available at: http://www.ncbi.nlm.nih.gov/pmc/articles/PMC1079523/

26. A.╇ CGA is multidisciplinary. The minimum team usually consists of a physician with an interest
in elderly care, a nursing professional, and a social worker. Teams will often be augmented by many
other allied professionals, such as physiotherapists, occupational therapists, dieticians, speech and
language therapists, and psychogeriatricians.

Further reading
Ellis G, Langhorne P. Comprehensive geriatric assessment for older hospital patients’ British Medical
Bulletin 2005; 71(1): 45–59. Available at: http://bmb.oxfordjournals.org/content/71/1/45.full
Ellis G, Whitehead MA, Robinson D, O’Neill D, Langhorne P. Comprehensive geriatric assessment for
older adults admitted to hospital: meta-analysis of randomised controlled trials. British Medical Journal
2011; 343: d6553. Available at: http://www.bmj.com/content/343/bmj.d6553
Stuck AE. Comprehensive geriatric assessment for older. Editorial British Medical Journal 2011; 343:
d6799. Available at: http://www.bmj.com/content/343/bmj.d6799.extract

27. A.╇ Increasing evidence exists to suggest that angiotensin inhibition improves the structural
and electrical properties of the atria, as well as treating the risk factors of heart failure and
hypertension. Evidence is not strong enough to support their use to prevent AF, but they should
be considered if antihypertensive agents are required. Digoxin provides effective resting rate
control, but not with exercise. Dronedarone may have a more preferable safety profile, but is less
efficacious than amidoarone.
Although 60% of patients undergoing pulmonary vein ablation for the first time are free of AF at
1 year, asymptomatic episodes are common, even in those who were previously symptomatic.
Warfarin is still recommended to prevent thromboembolic events in case of relapse.
Both flecainide and beta blockers are contraindicated in acute heart failure.

Further reading
British National Formulary. Available at: http://www.bnf.org
European Society of Cardiology Guidelines. Atrial Fibrillation (Management of) 2010 and Focused
Update (2012). Available at: http://www.escardio.org/guidelines-surveys/esc-guidelines/Pages/atrial-
fibrillation.aspx
Hindricks G, Piorkowski C, Tanner H, et al. Perception of atrial fibrillation before and after
radiofrequency catheter ablation. Circulation 2005; 112: 307–13.
Available at: http://circ.ahajournals.org/content/112/3/307.full
NICE. Dronedarone for the treatment of non-permanent atrial fibrillation, Guideline TA187.
Available at: http://guidance.nice.org.uk/TA197
NICE. The management of atrial fibrillation, Clinical guideline CG36.
Available at: http://guidance.nice.org.uk/CG36

28. B.╇ BP lowering in type-II diabetes independently reduces cardiovascular risk. NICE
recommends a target BP of 140/80mmHg (130/80mmHg in the presence of renal, ophthalmic,
230 EXAM 3 | ANSWERS

or cerebrovascular damage) In the presence of microalbuminuria an ACE-I or ARB should be


commenced irrespective of whether there is hypertension.

Further reading
National Collaborating Centre for Chronic Conditions. Type II diabetes: National clinical guideline for
management in primary and secondary care (update), 2008.
Available at: http://www.nice.org.uk/nicemedia/live/11983/40803/40803.pdf
British National Formulary. Available at: http://bnf.org/bnf/bnf/61/204016.htm

29. C.╇ The lesion described appears to be recent and so is most likely either a venous ulcer or
traumatic in origin. Squamous cell carcinoma is slow growing, may be eroded or nodular, and may
have a friable surface, which bleeds easily. Squamous cell carcinoma can invade chronic ulcers,
causing them to undergo malignant changes (Marjolin’s ulcer)—squamous cell carcinoma can be
recognized by significant changes to the wound bed, followed by a rapid deterioration, which can
indicate it is well established.
She exhibits behavioural and psychological symptoms of dementia and is highly unlikely to comply
with any attempt to further investigate or treat the ulcer. It is appropriate to keep the lesion
under review and if her behaviour changes (perhaps as any pain resolves) reassess the necessity/
appropriateness of further investigation/treatment. At present any intervention would require her
to be sedated, which is difficult to justify as being in her best interests.

Further reading
Baldursson B, Siegurgeirsson B, Lindelöf B. Leg ulcers and squamous cell carcinoma: An
epidemiological study and a review of the literature. Acta Dermatologica Venereologie (Stockholm)
1993; 73: 171–4. Available at: http://hls.is/page2/bs/assets/legulcscclitt.pdf
MPS. Best interests test.
Available at: http://www.medicalprotection.org/uk/england-factsheets/MCA-best-interests-test
Scottish Intercollegiate Guidelines Network (SIGN). Management of chronic venous leg
ulcers: national guideline 120. Available at: http://www.sign.ac.uk/pdf/sign120.pdf

30. C.╇ There is no evidence that brisk walking reduces the risk of falling. However, there may
be other health benefits of brisk walking by older people. There is no evidence that cognitive/
behavioural interventions alone reduce the incidence of falls in community-dwelling older people
of unknown risk status. Such interventions include risk assessment with feedback and counselling,
and individual education discussions. There is no evidence that complex interventions—in which
group activities including education, a behaviour-modification programme aimed at moderating risk,
advice, and exercise interventions—are effective in falls prevention with community-dwelling older
people.
Exercise in groups may be encouraged as a means of health promotion, but there is little evidence
that exercise interventions other than progressive exercise programmes addressing both strength
and balance, such as Otago and T’ai Chi have benefit in falls reduction.
There is no evidence that referral for correction of vision as a single intervention for community-
dwelling older people is effective in reducing the number of people falling. However, vision
assessment and referral has been a component of successful multifactorial falls prevention
programmes. There is a link between falls risk and the wearing of bifocal or varifocal lenses, but this
does not warrant a general recommendation to use separate spectacles.
Home hazard assessment is shown to be effective only in conjunction with follow-up and
intervention, not in isolation.
EXAM 3 | ANSWERS 231

Further reading
NICE CG161: Clinical practice guideline for the assessment and prevention of falls in older people.
Available at: http://www.nice.org.uk/CG161
Otago exercise programme to prevent falls in older adults. Available at: http://www.acc.co.nz/PRD_
EXT_CSMP/groups/external_providers/documents/publications_promotion/prd_ctrb118334.pdf

31. A.╇ Any individual whose health care needs change is eligible to be assessed as to the
complexity of their health needs in order to determine whether they are eligible for continuing
health care funding. This is especially true, if they are changing residence (i.e. moving in to a care
home). This assessment takes in to account the following domains:
• Behaviour.
• Cognition.
• Communication.
• Psychological/emotional needs.
• Mobility.
• Nutrition.
• Continence.
• Skin (including wounds and ulcers).
• Breathing.
• Symptom control through drug therapies and medication (includes a need for supervision of
complex medication regimes).
• Altered states of consciousness.
• Other significant care needs.
Each item is assessed according to its nature, intensity, complexity, and unpredictability. Every effort
should be made to include the individual in the assessment, but if they lack capacity to be involved
then the assessment should still be carried out, acting in their best interests.
Where a person’s primary need is a health need, they are eligible for NHS continuing health care.
Deciding whether this is the case involves looking at the totality of the relevant needs. Where an
individual has a primary health need and is, therefore, eligible for NHS continuing health care, the
NHS is responsible for providing all of that individual’s assessed needs—including accommodation,
if that is part of the overall need.
If the person does not qualify for NHS continuing health care, their local authority will be
responsible for assessing their care needs and providing services if they are eligible. However, if
they don’t qualify for NHS continuing health care, but are assessed as having health care or nursing
needs, they may still receive some care from the NHS. For someone who lives in their own home,
this could be provided as part of a joint package of care, where some services come from the NHS
and some from social services. If the person moves into a nursing home, the NHS may contribute
towards their nursing care costs.
This woman is clearly at risk of harm the longer she remains living alone. The move into residential
care need not be delayed awaiting the assessment of her eligibility for continuing health care
funding. If she were eligible, this could be back-dated to the time of her move into care.

Further reading
Department of Health (England). The National Framework for NHS Continuing Healthcare and
NHS-funded Nursing Care, July 2009 (revised). Available at: http://www.dh.gov.uk/prod_consum_dh/
groups/dh_digitalassets/documents/digitalasset/dh_103161.pdf
232 EXAM 3 | ANSWERS

NHS Choices. Continuing care. Available at: http://www.nhs.uk/CarersDirect/guide/practicalsupport/


Pages/NHSContinuingCare.aspx

32. A.╇ Current NICE guidelines support early antiplatelet therapy in ischaemic stroke, early NGT
feeding in stroke patients with unsafe swallowing and control of blood glucose between 4 and
11mmol/L. Statin therapy should be considered after 48hr.
Oxygen should be used in hypoxic stroke patients, to achieve target saturations 94–98% (88–92% in
COPD patients). It should not be routinely used in all patients. There is some suggestion of harm in
minor or moderate strokes. High oxygen concentrations may lead to an increase in reactive oxygen
species. These may cause tissue damage and be responsible for some of the detrimental effects
observed with high flow oxygen in acute stroke.

Further reading
British Thoracic Society. Emergency oxygen use in adult patients. Available at: http://www.brit-thoracic.
org.uk/guidelines/emergency-oxygen-use-in-adult-patients.aspx
NICE. ‘Do not do’ recommendation details. Available at: http://www.nice.org.uk/usingguidance/
donotdorecommendations/detail.jsp?action=details&dndid=257
NICE. Stroke: diagnosis and initial management of acute stroke and transient ischaemic attack (TIA),
Clinical guideline CG68. Available at: http://guidance.nice.org.uk/CG68

33. C.╇ The TUG test is a measurement of mobility. The original purpose of the TUG was to test
basic mobility skills of frail elderly patients. The test has been used in other populations, including
people with arthritis, stroke, and vertigo. It includes a number of tasks, such as standing from a
seating position, walking, turning, stopping, and sitting down, which are all important tasks needed
for a person to be independently mobile. It provides an opportunity to assess falls risk and may
also be useful in monitoring response to interventions. Cognitively impaired individuals are more
likely to be unable to follow the instructions, but do not presume that everyone with cognitive
impairment will be unable to complete this test. Whilst the test is more commonly used in a falls
clinic or rehabilitation setting, it also has utility in the Parkinson’s clinic and acute care setting. Older
people with dementia may be at risk of falls either due to their dementia or comorbidities, but the
memory clinic is the setting where the TUG test is least likely to be useful as gait abnormalities
associated with vascular dementia or Parkinson’s dementia are likely to be identified without
resorting to a TUG test; and most referrals to a memory clinic are for diagnosis and so are early in
their dementia with lower falls risk.
Abnormalities that may be observed include postural instability, need for external support (person
or walking aid), and gait abnormality.
For the test, the person is asked to stand up from a standard chair and walk a distance of
approximately 10ft (3m), turn around, and walk back to the chair and sit down again. The individual
uses his/her usual footwear and can use any assistive walking device they normally use. The person
is seated with his/her back to the chair, their arms resting on the arm rests, and any walking aid
they may use should be to hand. Timing, using either a wristwatch with a second hand or a stop
watch, begins when the individual starts to rise from the chair and ends when he/she is once again
seated in the chair. The normal time required to finish the test is between 7 and 10sec. Individuals
who cannot complete the task in that time, probably have some mobility problems, especially if
they take more than 20sec.
EXAM 3 | ANSWERS 233

Further reading
American College of Rheumatology. Timed up and go (TUG). Available at: http://www.rheumatology.
org/practice/clinical/clinicianresearchers/outcomes-instrumentation/TUG.asp

34. C.╇ The most likely sequence of events is a seizure precipitated by the damage caused by her
previous stroke. The low GCS was the result of the post-ictal state. The lateralizing neurology is an
unmasking of her previous deficit and/or a Todd’s paresis.
Dementia with Lewy Bodies can result in marked variations in GCS, although lateralizing neurology
would be unusual. TIAs, and, indeed, stroke are not usually associated with an altered state of
consciousness, unless there is brainstem involvement or significant cerebral oedema.

Further reading
Crystal HA. Dementia with Lewy Bodies. Medscape, 2010.
Available at: http://emedicine.medscape.com/article/1135041-overview
NICE. Stroke: diagnosis and initial management of acute stroke and transient ischaemic attack (TIA),
Clinical guideline CG68. Available at: http://www.nice.org.uk/CG68
Rolak LA, Rutecki P, Ashizawa T, Harati Y. Clinical features of Todd’s post-epileptic paralysis. Journal of
Neurology, Neurosurgery and Psychiatry 1992; 55: 63–4.
Available at: http://jnnp.bmj.com/content/55/1/63

35. B.╇ European Pressure Ulcer Advisory Panel (EPUAP) and NICE grading of pressure sores is as
follows:
• Grade 1: non-blanchable erythema of intact skin. Discolouration of the skin, warmth,
oedema, induration, or hardness may also be used as indicators, particularly in individuals with
darker skin.
• Grade 2: partial thickness skin loss involving epidermis, dermis, or both. The ulcer is superficial
and presents clinically as an abrasion or blister.
• Grade 3: full thickness skin loss involving damage to or necrosis of SC tissue that may extend
down to, but not through, underlying fascia.
• Grade 4: extensive destruction, tissue necrosis, or damage to muscle, bone, or supporting
structures with or without full thickness skin loss.
Reproduced with permission from European Pressure Ulcer Advisory Panel and National Pressure
Ulcer Advisory Panel. Prevention and treatment of pressure ulcers: quick reference guide. Washington DC:
National Pressure Ulcer Advisory Panel; 2009.

Further reading
NICE. Pressure ulcers: the management of pressure ulcers in primary and secondary care, Clinical
guideline CG29. Available at: http://www.nice.org.uk/guidance/index.jsp?action=byID&o=10972
European Pressure Ulcer Advisory Panel Guidelines. EPUAP-NPUAP Guidelines.
Available at: http://www.epuap.org/guidelines/

36. E.╇ This man has gait abnormalities, cognitive impairment, and urinary incontinence, which
makes a unifying diagnosis of normal pressure hydrocephalus. Long-standing hearing impairment
as indicated by hearing aid and absence of other cranial nerve deficits make acoustic neuroma less
likely, and it does not explain low MMSE, urinary, and gait symptoms. Negative Hallpike excludes
benign positional vertigo. Falls in Ménière’s disease are associated with vertigo that comes on
spontaneously, and are not associated with posture and turning. Cerebellar signs can manifest
234 EXAM 3 | ANSWERS

in normal pressure hydrocephalus and the absence of other cerebellar signs makes a mid-line
cerebellar tumour unlikely.
Further reading
Benson DF, LeMay M, Patten DH, Rubens AB. Diagnosis of normal-pressure hydrocephalus. New
England Journal of Medicine 1970; 283: 609–15.
Available at: http://www.nejm.org/doi/pdf/10.1056/NEJM197009172831201
Boon AJW, Tans JTJ, Delwel EJA, et al. The Dutch normal-pressure hydrocephalus study. Surgical
Neurology 2000; 53: 201–7. Available at: http://www.surgicalneurology-online.com/article/
S0090-3019(00)00182-8/abstract
Hebb AO, Cusimano MD. Idiopathic normal pressure hydrocephalus: a systematic review of diagnosis
and outcome. Neurosurgery 2001; 49: 1166–86. Available at: http://journals.lww.com/neurosurgery/
Abstract/2001/11000/Idiopathic_Normal_Pressure_Hydrocephalus__A.28.aspx
37. D.╇ Age is not a marker of frailty, although frailty is more common in older age, as is loss of
muscle mass. Slow walking speed, rather than the actual gait pattern is used as a marker of frailty.
Frailty is not gender specific. Sarcopenia is age-related muscle loss, greater than that associated with
normal ageing and is a characteristic marker of frailty being associated with slow walking speed and
poor balance. Weight loss, rather than weight gain is associated with frailty.
Further reading
Fried LP, Tangen CM, Walston J, et al. Frailty in older adults: evidence for a phenotype. Journal of
Gerontology Series A Biological Science, Medical Science 2001; 56(3): M146–57. Available at: http://
biomedgerontology.oxfordjournals.org/content/56/3/M146.short
38. B.╇ The low ferritin is diagnostic of absolute iron deficiency. The ‘normal range’ is higher in
patients with advanced chronic renal disease, owing to the increase in ferritin associated with the
acute phase response.
The first step in treatment should be to correct iron deficiency. While there is no clear indication
of whether oral or IV loading is preferable in absolute deficiency, it is possible that the patient will
not tolerate ferrous fumarate either. In cases of functional iron deficiency (normal ferritin, but low
transferrin saturation or high number of hypochromic cells), IV iron is preferred.
Erythropoietin should not be initiated until the iron deficiency is corrected. NICE suggest that
the evidence does not support routine measurement of erythropoietin levels. The normal B12
precludes the need for hydroxocobalamin.
Further reading
NICE. Anaemia management in people with chronic kidney disease, Clinical guideline CG114.
Available at: http://www.nice.org.uk/Guidance/cg114
39. A.╇ It would seem that he has vascular Parkinsonism with vascular dementia. What is described
is retrocollis with opisthotonic (dystonic) posturing. This is more commonly associated with
antipsychotic usage, tetanus, lithium toxicity, traumatic brain injury, and encephalomyelitis. However,
none of these seem to apply in this case. Such dystonic posturing can be seen in Parkinsonian
syndromes are more likely towards the end of the day. These are often due to reduced
responsiveness to levodopa and may respond to ‘booster’ doses of levodopa.
However, the first step must be to advise not to feed during such an attack as his swallow is clearly
impaired at the time. In this case, it may even be reasonable to reduce or withdraw his levodopa,
especially as he is unlikely to be gaining benefit from it, given the likely aetiology of his Parkinsonism,
and severe physical and mental disability. As this is intermittent, can occur at any time of the day,
is not always associated with feeding, and he can feed safely in between, a swallow assessment
EXAM 3 | ANSWERS 235

and PEG feeding are not necessary. If adjusting the levodopa does not help, treatment with a
benzodiazepine or baclofen may help.

Further reading
Schneider D. Tardive dystonia. Medscape.
Available at: http://emedicine.medscape.com/article/287230-overview
Schneider SA, Udani V, Sankhla CS, Bhatia KP. Recurrent acute dystonic reaction and oculogyric crisis
despite withdrawal of dopamine receptor blocking drugs. Movement Disorders 2009; 24: 1226–9.
Available at: http://onlinelibrary.wiley.com/doi/10.1002/mds.22532/pdf

40. C.╇ The rapid deterioration, apparent lack of response to L-dopa, early urinary incontinence,
and postural hypotension (he has symptoms suggestive of this) are suggestive of multisystem
atrophy.
The rate of decline and lack of response to L-dopa render idiopathic Parkinson’s disease less likely.
The lack of significant cognitive impairment at baseline and the rate of deterioration count against
dementia with Lewy bodies.
Although there are rare case reports of unilateral and bilateral meningioma-induced Parkinsonism,
these are rare and involve large meningiomas, with significant oedema that distort and compress the
basal ganglia.

Further reading
Adhiyaman V, Meara A. Menigioma presenting as bilateral parkinsonism. Age & Ageing 2003; 32:
456–8. Available at: http://ageing.oxfordjournals.org/content/32/4/456.full.pdf
NICE. Parkinson's disease: diagnosis and management in primary and secondary care, Clinical guideline
CG35. Available at: http://guidance.nice.org.uk/CG35
Stefanova N, Bücke P, Duerr S, et al. Multiple system atrophy: an update. Lancet Neurology 2009; 8:
1172–8. Available at: http://www.thelancet.com/journals/laneur/article/PIIS1474-4422(09)70288-1/
fulltext

41. C.╇ Codeine, oxycodone, morphine, and diamorphine all have active metabolites and are
prone to accumulation in cases of significant renal impairment. Fentanyl has inactive metabolites
and may be appropriate in severe renal impairment, although a reduced dose should be used. If
continuous, e.g. SC, analgesia is required, alfentanyl may be more preferable owing to its short
half-life.

Further reading
British National Formulary. Available at: http://www.bnf.org.uk
National Council for Palliative Care. Available at: http://www.ncpc.org.uk/
NHS Scotland Palliative Care Guidelines. Symptom control in patients with chronic kidney
disease/renal disorders. Available at: http://www.palliativecareguidelines.scot.nhs.uk/documents/
RenalPalliativeCarefinal.pdf

42. D.╇ He has Charles Bonnet syndrome—visual hallucinations in the presence of visual
impairment and normal cognition. He should be reassured and informed of the diagnosis and
cause of his hallucinations. These may resolve spontaneously. Drug treatment is ineffective
236 EXAM 3 | ANSWERS

and unnecessary. Correcting the visual impairment is the treatment of choice, but in this case
cataractectomy is unlikely to improve his vision and, therefore, unlikely to affect is hallucinations.

Further reading
Jacod A, Prasad S, Bogglid M, Chandratre S. Charles Bonnet syndrome—elderly people and
visual hallucinations. British Medical Journal 2004; 328: 1552. Available at: http://www.bmj.com/
content/328/7455/1552.full
Manford M, Andermann F. Complex visual hallucinations: clinical and neurobiological insights. Brain
1998; 121: 1819–40. Available at: http://brain.oxfordjournals.org/content/121/10/1819.full.pdf
RNIB. Charles Bonnet syndrome. Available at: http://www.rnib.org.uk/eyehealth/eyeconditions/
conditionsac/Pages/charles_bonnet.aspx

43. D.╇ D-dimer assays are only useful in excluding pulmonary embolism in patients with low
or moderate pre-test probability scores. They should not be undertaken in cases of high clinical
probability.
While CTPA is the recommended initial imaging modality in non-massive PE, there is a risk of
contrast-induced nephropathy given the impaired renal function.
VQ scanning is only reliable in the absence of significant concurrent cardiovascular disease and the
moderate COPD in this case makes a non-diagnostic scan likely.
Echocardiograph is useful in diagnosing massive PE, but less so in diagnosing smaller PEs. It may also
be confounded by other cardiorespiratory diseases.
If a leg ultrasound confirms the presence of deep vein thrombosis, this would be sufficient to
support the diagnosis of venous thromboembolic disease and systemic anticoagulation.

Further reading
British Thoracic Society. Guidelines for the management of suspected pulmonary embolism.
Available at: http://www.brit-thoracic.org.uk/guidelines.aspx

44. E.╇ The emphasis of ongoing treatment is on symptom control. Anticipatory prescriptions of
cyclizine, diamorphine, hyoscine, and midazolam are all appropriate. There is limited and conflicting
evidence for the benefit of oxygen in palliation. Symptom control can often be achieved with the
use of other measures such as opiates and fans.

Further reading
Patient.co.uk. Dyspnoea in palliative care.
Available at: http://www.patient.co.uk/doctor/Dyspnoea-in-Palliative-Care.htm#ref-22

45. C.╇ The acute confusional state is consistent with a delirium. Fractured neck of femur is a
recognized risk factor for this. Any pain associated confusion is likely to have been worsened with
the use of opiate analgesia. If it is possible to spare the use of opiates with a regional nerve block,
this will remove two of the factors driving the delirium. It will also reduce the risk of opiate-induced
constipation, which could further compound the situation.
It is well recognized that delayed surgical repair of fractured neck of femur is associated with
increased peri-operatively morbidity and prolonged recovery. Delaying surgery for a patient with
known pre-existing mild aortic valve disease for repeat echocardiography is inappropriate. In
patients with clinically significant valvular heart disease, echocardiography may aid with planning
an anaesthetic (general versus spinal). However, unless it is readily available, decisions to delay
EXAM 3 | ANSWERS 237

should only be taken after discussion between senior anaesthetic, orthopaedic, and orthogeriatric
clinicians.
The patient will need consideration of secondary osteoporosis investigation and/or treatment
in due course. However, controversy persists regarding the timing, with concern that treatment
too early may impair callus formation and bone healing. It is also essential to confirm adequate
vitamin D levels prior to potent IV bisphosphonates to minimize the risk of severe hypocalcaemia.
Secondary prevention is, therefore, perhaps best considered during the rehabilitation phase or on
follow-up.
Capacity should be assessed by clinicians responsible for an intervention. In cases of uncertainty,
assessment should be escalated to more senior clinicians. Expert psychiatric assessment may be
required in borderline cases.
The patient should, in fact, have their steroid dose increased to cover the stress of surgery. The use
of steroids should be highlighted to the anaesthetist, as IV doses may be required peri-operatively.
Sudden withdrawal of steroids might precipitate an adrenal crisis in a patient on long-term
prednisolone.

Further reading
British Othopoedic Society. The Care of Patients with Fragility Fracture (‘Blue Book’), 2007. Available
at: http://www.bgs.org.uk/index.php?option=com_content&view=article&id=338:bluebook
Del Rosario E, Estyeve N, Semandez MJ, et al. Does femoral nerve analgesia impact the development
of postoperative delirium in the elderly? A retrospective investigation. Acute Pain 2008; 10: 59–64.
Available at: http://www.acutepainjournal.com/article/S1366-0071(08)00017-X/abstract
HM Government. Mental Capacity Act 2005.
Available at: http://www.legislation.gov.uk/ukpga/2005/9/contents
NICE. Delirium: diagnosis, prevention and management, Clinical guideline CG103, 2010.
Available at: http://www.nice.org.uk/nicemedia/live/13060/49909/49909.pdf

46. A.╇ This patient has sustained an osteoporotic fracture. NICE guidelines would currently
suggest that DEXA scanning is not necessary to confirm osteoporosis in female patients over
75 years presenting with such fractures, and a DEXA scan is therefore not indicated. Usual
treatment would be to ensure adequate calcium and vitamin D levels and start an anti-resorbative
treatment (e.g. bisphosphonate or strontium ranelate). If calcium and vitamin D levels are
inadequate, oral supplementation of these may be necessary too.
However, this patient has a significant renal impairment (CKD-4), which means that alendronate
and strontium are both contraindicated. Furthermore, it is like that there is insufficient 1-α
hydroxylation in the kidney, such that activated vitamin D (1-α-hydroxycholecalciferol, e.g.
alfacalcidol) would be required over standard cholecalciferol. The low calcium and raised ALP are
suggestive of this, but confirmation with paired serum PTH, calcium and 1-25-hydroxyvitamin D
levels should be taken. If not already known to a low-clearance renal clinic, she should be referred.
If an anti-resorbative is required, the monoclonal antibody denosumab is not contraindicated
in renal failure, and may be used in such cases. A bone-density measurement is needed to
demonstrate significant osteoporosis to satisfy cost-effectiveness according to NICE guidelines.
Serum calcium must be closely monitored. Hypocalcaemia and vitamin-D deficiency must be
addressed first.
238 EXAM 3 | ANSWERS

Further reading
NICE. Chronic kidney disease: early identification and management of chronic kidney disease in
adults in primary and secondary care, Clinical guideline CG73.
Available at: http://guidance.nice.org.uk/CG73
NICE. Osteoporotic fractures – denosumab, Guideline TA294.
Available at: http://guidance.nice.org.uk/TA204
NICE. Osteoporosis secondary prevention. Alendronate, etidronate, risedronate, raloxifene,
strontium ranelate and teriparatide for the secondary prevention of osteoporotic fragility fractures
in postmenopausal women (amended), Guideline TA161.
Available at: http://guidance.nice.org.uk/TA161

47. A.╇ The patient has a functioning GI tract and there is no reason he should not digest food
normally. He should be able to eat and drink normally, providing his swallow is safe. At that point,
there is no need for further NGT feeding. He is, however, at nutritional risk and his ongoing
progress will need to be monitored by the dieticians who will be able to review whether his oral
intake needs augmentation with supplements or he can step back down to standard ward care.

Further reading
British Association for Parenteral and Enteral Nutrition (BAPEN). Malnutrition Universal Screening
Tool. Available at: http://www.bapen.org.uk/pdfs/must/must_full.pdf
NICE. Nutrition support in adults: oral nutrition support, enteral tube feeding and parenteral nutrition,
Clinical guideline CG32. Available at: http://publications.nice.org.uk/nutrition-support-in-adults-cg32

48. B.╇ There has been no change in the patient’s level of function. Objective evidence shows
that she is back at her baseline. Given that she owns her own property, she is free to self-fund a
residential care home placement should she so wish. However, it is reasonable and safe for her to
be discharged back home with care while she makes arrangements for this.
Ongoing rehabilitation is unlikely to improve her above her baseline. There has been no change
in function, or support need and an increase in care or check-listing are therefore not indicated at
this point.

Further reading
Department of Health. The national framework for NHS continuing health care and NHS funded
nursing care, July 2009 (revised). Available at: http://www.dh.gov.uk/prod_consum_dh/groups/dh_
digitalassets/documents/digitalasset/dh_103161.pdf
HM Government. Social care.
Available at: http://www.dh.gov.uk/health/category/policy-areas/social-care/
HM Government. Health and social care assessments. Available at: http://www.direct.gov.uk/en/
DisabledPeople/HealthAndSupport/ArrangingHealthAndSocialCare/DG_4000436
Department of Health. Legislation. Available at: http://www.dh.gov.uk/en/Publicationsandstatistics/
Legislation/Actsandbills/index.htm

49. D.╇ Parkinson’s patients frequently have problems getting in and out of bed, and also of turning
in bed. These may be compounded by problems associated with arthritis or post-hip fracture.
A bed cane may help with transfers in and out of bed. An inflatable leg lifter may help her getting
into bed. A Jacob’s ladder may help her to sit up in bed from lying down. Silk sheets reduce friction
EXAM 3 | ANSWERS 239

when turning and can make it easier for Parkinson’s patients to turn in bed without needing
assistance, as can silk pyjamas (for men and women).
A mattress tilter is usually placed under the foot end of the mattress to relieve symptoms of
swollen feet, varicose veins, or lower back pain. It is sometimes helpful when used at the head end
of the bed for those with orthopnoea or gastro-oesophageal reflux.

Further reading
The following website provides a useful insight in to the various domestic appliances that might benefit
people with all degrees of disability: Available at: www.essentialaids.com

50. E.╇ The story is typical for wet age-related macular degeneration, and she should be referred
urgently to the eye clinic for assessment and possible treatment with anti-vascular endothelial
growth factor, if she is stand a chance of preserving the vision in that eye.

Further reading
RNIB. Age-related macular degeneration.
Available at: http://www.rnib.org.uk/eyehealth/eyeconditions/conditionsac/Pages/amd.aspx
NICE. Ranibizumab and pegaptanib for the treatment of age-related macular degeneration, Guideline
TA155. Available at: http://www.nice.org.uk/nicemedia/pdf/TA155guidance.pdf

51. E.╇ The predictability of symptoms means that prophylactic short-acting GTN use may be
sufficient to control them, while sparing additional medication burdens. He is also relatively
hypotensive, which additional regular anti-anginals may exacerbate.
Current NICE guidelines recommend the use of fast acting GTN spray to relieve angina attacks
and or prevent them before planned exercise. Thought should be given to secondary prevention
of coronary artery disease with aspirin, statins and anti-hypertensives, which this patient is
appropriately taking.
If a longer-acting preventative agent is needed, NICE advocate the use of beta blockers and or
calcium channel antagonists first line. If these agents are not tolerated, then monotherapy with
a second-line agent such as a long-acting nitrate, ivabradine, nicorandil or ranazoline should
be considered. These should also be added if the symptoms are not controlled with first line
drugs alone.
If symptoms are not controlled with medical treatment, consideration should be given to
angiography, and subsequent angioplasty or coronary artery bypass surgery (CABG). There is
a potential survival benefit for the use of CABG in patients with multi-vessel disease who are
either diabetic, over 65 or have complicated three vessel disease with or without left main stem
involvement.

Further reading
NICE. Stable angina: the management of stable angina, Clinical guideline CG26.
Available at: http://guidance.nice.org.uk/CG126

52. C.╇ The patient is most likely experiencing an infection related delirium and lacks capacity as a
result of this. Given this, it is reasonable to expect he will regain capacity when the infection settles.
He is being successfully fed through an NGT, which means that it is possible to defer a definitive
decision about feeding until he can make it himself. It may also be the case that as the delirium
improves, the swallow safety does too.
240 EXAM 3 | ANSWERS

If the patient does not regain capacity, then a ‘best-interests’ decision will need to be made between
the treating team and the patients representatives—in this case his wife. If the patient were ‘truly
unbefriended’ then an IMCA could be requested to act as such a representative.
A psychiatry opinion is not necessary for every capacity assessment—any individual trained to do
so can assess capacity. However, it may be beneficial in difficult or borderline cases.

Further reading
HM Government. Mental Capacity Act 2005.
Available at: http://www.legislation.gov.uk/ukpga/2005/9/contents
Independent mental capacity advocate (IMCA) service. Making decisions: the Independent Mental
Capacity Advocate (IMCA) Service. Available at: http://webarchive.nationalarchives.gov.uk/+/http://
www.dca.gov.uk/legal-policy/mental-capacity/mibooklets/booklet06.pdf

53. C.╇ Lumbar spinal stenosis is characterized by bony and soft tissue encroachment of the lower
lumbar canal, often compromising the lateral recess of the canal, where the lower lumbar and sacral
roots of the cauda equina are to be found. This can produce lower motor neurone weakness of hip
abductors and ankle dorsi-flexors (L4–5), and hip extensors and ankle evertors (L5, S1).
The lumbar spinal canal increases in size with flexion and decreases with extension. Thus, standing
and walking (on the level or downhill) extends the lumbar spine, increasing the likelihood of pain.
Sitting down, sitting in a bath, bending forward, and lying in bed are all likely to flex the spine and
decrease symptoms.

Further reading
Alvarez JA, Hardy RH. Lumbar spine stenosis: a common cause of back and leg pain. American Family
Physician 1998; 57(8): 1825–34. Available at: http://www.aafp.org/afp/1998/0415/p1825.html
Mayo Clinic staff. Spinal stenosis. Available at: http://mayoclinic.com/health/spinal-stenosis/DS00515

54. C.╇ There are three obligatory conditions for the diagnosis of heart failure with normal ejection
fraction, based on the consensus statement of the Heart Failure and Echocardiography Associations
of the European Society of Cardiology. These are:
• Presence of signs and symptoms of heart failure.
• Presence of normal or mildly abnormal left ventricular function.
• Evidence of diastolic left ventricular dysfunction.
Typically patients are older, female, hypertensive, obese, diabetic, and in AF.

Further reading
Paulus WJ, Tschöpe C, Sandersen JE, et al. How to diagnose diastolic heart failure: a consensus
statement on the diagnosis of heart failure with normal left ventricular ejection fraction by the Heart
Failure and Echocardiography Associations of the European Society of Cardiology. European Heart
Journal 2007; 28(20): 2539–50. Available at: http://eurheartj.oxfordjournals.org/content/28/20/2539.
long

55. D.╇ The most important initial step in the assessment of a confusional state is to determine
if this is acute (delirium) or chronic (most likely a dementia, but other conditions need to be
considered, e.g. depression and hypothyroidism) or acute-on-chronic. A more detailed assessment
of cognition, e.g. with MMSE and clock drawing test, and a detailed collateral history will better
determine the severity and time course of the confusion. Tools such as the confusion assessment
method (CAM), IQCODE, and single question in delirium (SQiD) may also be useful.
EXAM 3 | ANSWERS 241

If the diagnosis remains consistent with delirium and a cause (such as urosepsis in this example)
has been identified, further investigation is unlikely to assist in the acute setting. If the confusion is
longer-standing, if there are focal neurological signs, or there is deviation from the usual disease
course, further investigations are likely to be needed.
All clinicians should be able to recognize and instigate management of delirium. This should not
merely be deferred to old age psychiatry, although their expert advice may prove beneficial.

Further reading
Alagiakrishnan K. Delirium. Medscape.
Available at: http://emedicine.medscape.com/article/288890-overview
British Geriatrics Society. Guidelines for the prevention, diagnosis and management of delirium in
older people in hospital.
Available at: http://www.bgs.org.uk/index.php/clinicalguides/170-clinguidedeliriumtreatment
NICE. Delirium: diagnosis, prevention and management, Clinical guideline CG103.
Available at: http://www.nice.org.uk/guidance/CG103

56. D.╇ Anopexy is a surgical treatment for haemorrhoids.


Kegel exercises may be useful in strengthening pelvic floor muscles to manage urinary incontinence.
Patients with faecal incontinence related to weakened muscles or loss of sensation to have a bowel
movement, may benefit from bowel training involving exercises to strengthen the weakened rectal
muscles.
Rectopexy is a surgical treatment for rectal prolapse. Surgery is the last treatment option for faecal
incontinence, and is generally reserved for the severest cases not responding to other treatments.
Incontinence in demented patients is usually secondary to faecal loading, which should be treated
first and then a regimen of planned defecation should be implemented by the use of an enema
1–3 times per week if there is a continuing tendency to faecal incontinence and/or constipation.
A constipating drug may be needed to prevent leakage between enemas.
Toileting regimes may be useful in managing urinary incontinence.

Further reading
British Geriatrics Society. Faecal incontinence and constipation in the older person. Available at: http://
www.bgs.org.uk/index.php?option=com_content&view=article&id=241:faecalincontinence&catid=8:in
continence&Itemid=146

57. D.╇ The Barthel index assesses patients in 10 domains: bowels, bladder, grooming, toileting,
feeding, bed-chair transfers, walking, dressing, stairs, bathing.

Further reading
Mahoney FI, Barthel D. Functional evaluation: the Barthel Index. Maryland State Medical Journal
1965; 14: 56–61.
Available at: http://www.strokecenter.org/wp-content/uploads/2011/08/barthel_reprint.pdf
Stroke Centre. Available at: http://www.strokecenter.org/wp-content/uploads/2011/08/barthel.pdf

58. B.╇ From the information given she appears to be cognitively intact and is agreeing to
appropriate help from the district nurses, who can monitor her general well-being, and if the grand-
daughter’s concerns are founded they can seek to convince the woman to have appropriate care
or establish that she has capacity to refuse care. She is independent upon the ward and so does not
obviously require a care package. Although the way she is living may not be ideal (sleeping on the
242 EXAM 3 | ANSWERS

couch), this is her choice and must be respected—she is reasoning her decisions, rather than them
necessarily being reasonable, i.e. it might be more reasonable to open out the sofa bed to sleep on.

Further reading
HM Government. Mental Capacity Act 2005 (England).
Available at: http://www.legislation.gov.uk/ukpga/2005/9/contents

59. A.╇ Maintaining dignity is of paramount importance as is respecting the individual’s right to have
access to/be offered nutrition and hydration. As this patient apparently gains pleasure from feeding,
this should be allowed in as safe a manner as possible, without causing undue distress, whilst at the
same time accepting the risk of aspiration. It is not possible to ascertain his views, but it is clear
that he does not tolerate options B and C. In late stage dementia, the evidence would suggest that
options B and D may increase aspiration risk and are associated with shortening of life expectancy.
Option E would be appropriate if he showed evidence of understanding what food is! If there is any
cause for concern as to whether this is the course of action, an IMCA could be appointed to ensure
the team were acting in his best interests.

Further reading
Alzheimer’s Society. End of life care.
Available at: http://alzheimers.org.uk/site/scripts/documents_info.php?documentID=428
British Medical Association. Medical ethics Available at: http://www.bma.org.uk/ethics/
medicalethicstoday.jsp (free access to the book on line for BMA members).
Independent Mental Capacity Advocate Available at: http://webarchive.nationalarchives.gov.uk/+/www.
dh.gov.uk/en/SocialCare/Deliveringsocialcare/MentalCapacity/IMCA/index.htm
Office of the Public Guardian. Making decisions: the Independent Mental Capacity Advocate (IMCA)
service. Available at: http://www.justice.gov.uk/downloads/protecting-the-vulnerable/mca/making-
decisions-opg606-1207.pdf
National Council for Palliative Care. Free downloads.
Available at: http://www.ncpc.org.uk/library?keyword=capacity&date1=&date2=
NICE. Dementia: supporting people with dementia and their carers in health and social care, Clinical
guideline CG42, 2006. Available at: http://www.nice.org.uk/CG42

60. C.╇ Frequent flares of gout warrant prophylactic treatment. Allopurinol is first line for this.
Initiation should be covered with either an NSAID or colchicine to avoid precipitating an attack.
NICE have currently suggested that febuxostat is reserved for patients who are intolerant of
allopurinol, or patients in whom it is contraindicated.

Further reading
NICE. Febuxostat for the management of hyperuricaemia in people with gout, guideline TA164.
Available at: http://guidance.nice.org.uk/TA164
Rothschild BM. Gout and pseudogout.
Available at: http://emedicine.medscape.com/article/329958-overview

61. B.╇ The muscle wasting, bilateral foot drop and fixed flexion deformities are all consistent
with chronicity of greater than a week’s duration. This would suggest that he was not mobile
before admission. Establishing a clear baseline is critical before planning further rehabilitation and
management.
EXAM 3 | ANSWERS 243

62. E╇ NICE guidelines suggest that, with the exception of severe hypertension, ambulatory
or home BP monitoring is essential in reaching a new diagnosis of hypertension, and should be
undertaken if a clinic reading is found above 140/90mmHg. Patients can then be classified according
to ‘degree of hypertension’:
• Stage 1: clinic BP > 140/90mmHg and average home/ambulatory readings 135/85mmHg or
higher.
• Stage 2: clinic BP > 160/100mmHg and average home/ambulatory readings 150/95mmHg or
higher.
• Stage 3: clinic systolic BP > 180 or diastolic > 110.
Antihypertensive therapy should be offered to people under 80 years with stage 1 hypertension,
who have evidence of target organ damage, cardiovascular disease, renal disease, diabetes, or a
10-year cardiovascular risk greater or equal to 20%. Treatment should be offered to anyone with
stage 2 hypertension.
The target BP on treatment should be 140/90mmHg for those under 80 (or 135/85mmHg for
ambulatory/average home readings) and 150/90mmHg for those over 80 (or 145/85mmHg for
ambulatory/average home readings).
The patient’s home BP readings confirm stage 1 hypertension. The NICE guidelines would not
advocate treatment unless the 10-year cardiovascular risk exceeds 20%. In any event, given the
frequent falls, it is important to exclude postural hypotension in the first instance. If the systolic BP
drops by more than 20mmHg, NICE suggest reviewing the medication, and assessing further BPs
standing up.

Further reading
NICE. Hypertension: clinical management of primary hypertension in adults, Clinical guideline CG127.
Available at: http://www.nice.org.uk/CG127

63. B.╇ NICE guidelines suggest that those over 65 years old presenting with a head injury should
have CT imaging if there is evidence of amnesia or loss of consciousness. This should be arranged
before the patient is discharged.
At this stage, there is little available information to further elucidate the cause of the fall and
thereby determine appropriate onward investigation. Given that the baseline assessment is
unremarkable, the patient could be discharged if neuroimaging is satisfactory. They should then be
referred for a multidisciplinary falls assessment, at which point the need for further investigation
could be determined.
Unlike supplementation with calcium and vitamin D, there is no evidence for the benefit of calcium
supplements alone in falls prevention.

Further reading
Bolland MJ, Grey A, Avenell A, et al. Calcium supplements with or without vitamin D and risk of
cardiovascular events: reanalysis of the Women’s Health Initiative limited access dataset and meta-
analysis. British Medical Journal 2011; 342: d2040. Available at: http://dx.doi.org/10.1136/bmj.d2040
NICE. Falls: the assessment and prevention of falls in older people, Clinical guideline CG21.
Available at: http://publications.nice.org.uk/falls-cg21
NICE. Head injury: triage, assessment, investigation and early management of head injury in infants,
children and adults, Clinical guideline CG56. Available at: http://guidance.nice.org.uk/CG56/
244 EXAM 3 | ANSWERS

64. C.╇ Patients who have capacity can refuse treatments, including CPR, even if their clinicians do
not agree with this decision. However, they cannot demand clinically inappropriate treatments. The
decision to offer treatments rests with the most senior clinician involved. This extends to include
resuscitation and patients, or their advocate in the case of recipients of a lasting power of attorney,
cannot demand this intervention.
DNACPR orders only apply to the specific event of cardiopulomonary arrest. It may be, and often
is, appropriate to offer full active treatments short of this. It is often most appropriate to draw up
an escalation plan tailored to the individual patient, addressing which interventions are appropriate
and which are not.
It is best practice to discuss all decisions and information regarding their condition with a patient.
The GMC 2010 guidance suggests that if a patient lacks capacity any legal proxy and others close
to the patient should be informed of a DNACPR decision. For patients with capacity, you should
sensitively explore how willing they might be to know about a DNACPR decision. You should
not withhold information simply because conveying it is difficult or uncomfortable for the health
care team.
DNACRPR orders should be reviewed if the clinical picture changes, but this does not mean they
should be necessarily reversed.

Further reading
GMC. Treatments and Care towards the end of life. Available at: http://www.gmc-uk.org/guidance/
ethical_guidance/end_of_life_care.asp

65. E.╇ The most likely cause for the sensation of dizziness experienced by this patient is anxiety
attacks. The death of her husband may have been the trigger. Associated hyperventilation is likely
to be responsible for the perceived tingling and depersonalization. Beta-blockade may provide
symptomatic relief, while the underlying psychological issues are explored.
The normal d-dimer makes a PE unlikely. The normal ECG, cardiac examination and lack of
palpitations make a primary arrhythmia unlikely. There is no evidence of postural hypotension or
electrolyte abnormality. The patient is not describing vertigo and there are no clinical features of
vestibular dysfunction.

66. A.╇ NICE suggest using aspirin and dipryridamole M/R as antiplatelet treatment for people
with TIAs. Clopidogrel does not have a license for such use. The normal ECG and 24-hr tape make
paroxysmal AF unlikely and, hence, warfarinization inappropriate.
Endarterectomy should be considered in patients who are fit enough with a symptomatic stroke or
TIA. The left-sided carotid artery stenosis might be expected to cause left-sided cerebrovascular
disease, which would cause right limb symptoms. This carotid disease is therefore ‘asymptomatic’.
NICE only suggest increasing the dose of simvastatin if total cholesterol is above the target of
4mmol/L.

Further reading
NICE. Lipid modification: cardiovascular risk assessment and the modification of blood lipids for the
primary and secondary prevention of cardiovascular disease, Clinical guideline CG67.
Available at: http://www.nice.org.uk/CG67
NICE. Stroke: diagnosis and initial management of acute stroke and transient ischaemic attack (TIA),
Clinical guideline CG68. Available at: http://www.nice.org.uk/CG68
NICE. Clopidogrel and modified-release dipyridamole for the prevention of occlusive vascular events,
Guideline TA210 (review of TA90). Available at: http://www.nice.org.uk/TA210
EXAM 3 | ANSWERS 245

67. A.╇ Current guidelines recommend that advanced care planning (ACP) discussions should be
initiated in primary care settings, by an appropriately trained professional. Eliciting the opinion of
care home residents can be difficult and all of the factors listed can potentially hinder discussions
related to ACP. However, potentially all can be overcome with the exception of dementia of such
severity that it prevents such discussions.
Further reading
Royal College of Physicians, National Council for Palliative Care, British Society of Rehabilitation Medicine,
British Geriatrics Society, Alzheimer’s Society, Royal College of Nursing, Royal College of General
Practitioners. Advanced care planning: Concise guidance to good practice series, No. 12. Available at:
http://bookshop.rcplondon.ac.uk/contents/pub267-e5ba7065-2385-49c9-a68e-f64527c15f2a.pdf
Stewart F, Goddard C, Schiff R, Halls S. Advanced care planning in care homes for older
people: A qualitative study of the views of care staff and families. Age & Ageing 2011; 40: 330–5.
Available at: http://www.ncbi.nlm.nih.gov/pubmed/21345840
68. D.╇ Using the Oxford classification of stroke:
• Lacunar: any one of pure motor or pure sensory or sensorimotor deficit.
• PACS: two of contralateral motor or sensory deficit, or homonymous hemianopia or higher
cortical dysfunction.
• POCS: any one of isolated homonymous hemianpia or brain stem signs or cerebellar ataxia.
• TACS: all three of contralateral motor or sensory deficit, homonymous hemianopia, higher
cortical dysfunction.
• TIA: transient focal neurological dysfunction of vascular origin resolving completely in less
than 24hr.
Reprinted from The Lancet, 337, 8756, J. Bamford et al., ‘Classification and natural history of clinically
identifiable subtypes of cerebral infarction’, pp. 1521–1526, Copyright 2007, with permission from Elsevier.
Further reading
Bamford J, Sandercock P, Dennis M, et al. Classification and natural history of clinically identifiable
subtypes of cerebral infarction. Lancet 1991; 337: 1521–6.
Available at: http://www.ncbi.nlm.nih.gov/pubmed/1675378
NICE. Stroke: diagnosis and initial management of acute stroke and transient ischaemic attack (TIA),
Clinical guideline CG68, 2008. Available at: http://www.nice.org.uk/guidance/cg68
Scottish Intercollegiate Guidelines. Management of patients with stroke or TIA: assessment,
investigation, immediate management and secondary prevention, Guideline 108, 2008.
Available at: http://www.sign.ac.uk/pdf/sign108.pdf
69. E.╇ Painless loss or distortion of vision is typical of retinal vein occlusion and this patient has
three major risk factors—hypertension, hypercholesterolaemia, and diabetes. Retinal vein occlusion
is the second commonest cause of retinal vascular disease after diabetic retinopathy, and is caused
by obstruction of the retinal veins either by thrombus or external compression.
Acute glaucoma is intensely painful and presents with a red eye and headache. Vision becomes misty and
visual loss in one or both eyes is rapid. Age-related macular degeneration (ARMD) is the most common
cause of vision loss in those aged over 50 years. It causes a gradual loss of central (but not peripheral)
vision. Giant cell arteritis (GCA), or arteritic ischaemic optic neuritis, can rarely occur with a normal CRP
and ESR, but the normal ESR in this case makes GCA an unlikely cause of visual loss. Ischaemic optic
neuritis presents with painless loss of vision and would be a reasonable confounder in this question,
especially as it shares similar risk factors to retinal vein occlusion. It usually presents upon wakening due
to its association with nocturnal hypotension and so is less likely than retinal vein occlusion in this case.
246 EXAM 3 | ANSWERS

Further reading
Royal College of Opthalmologists. Interim Guidelines for Management of Retinal Vein Occlusion. 2010
Available at: www.rcophth.ac.uk/core/core_picker/download.asp?id=728
Younge BR. Anterior ischemic optic neuropathy clinical presentation.
Available at: http://emedicine.medscape.com/article/1216891-clinical

70. D.╇ NICE recommend the use of MRI in cases where the index of suspicion for fracture
remains high despite negative plain films. If MRI is not available within 24hr, CT imaging should be
considered. After 48hr, plain X-rays may demonstrate a previously missed fracture, owing to bone
resorption or cortical displacement, but the risk of a repeat false negative persists. Radionucleotide
scans may be an option, but are likely to be confounded in the presence of osteoarthritis.

Further reading
NICE. Hip fracture: the management of hip fracture in adults, Clinical guideline CG124.
Available at: http://publications.nice.org.uk/hip-fracture-cg124

71. C.╇ The lack of clinical improvement and persistently elevated CRP despite 3 days of IV
antibiotics suggests the infection is not sensitive. The albumin is most likely to be falling as part of
the acute phase response. The patient is eating well and no further action need be taken concerning
the albumin at this stage, other than to ensure the cause of the acute phase response (i.e. the
infection) is appropriately treated. The albumin will probably recover as the infection resolves. If it
does not, then further investigation may be needed.

Further reading
NICE. Nutrition support in adults: oral nutrition support, enteral tube feeding and parenteral nutrition,
Clinical guideline CG32. Available at: http://publications.nice.org.uk/nutrition-support-in-adults-cg32
Peralta R. Hypoalbuminaemia. Medscape, 2010.
Available at: http://emedicine.medscape.com/article/166724-overview

72. E.╇ The clinical picture is of a sudden decompensation of chronic liver disease. Infection is
a common, potentially life-threatening complication. Urgent ascitic fluid microscopy and culture
should be undertaken to exclude spontaneous bacterial peritonitis and IV antibiotics given if
needed.
AFP may be raised in the presence of hepatocellular carcinoma, but will not affect the acute
management.
Chlordiazepoxide should be available in case of manifestation of alcohol withdrawal. However,
given the low GCS, it would currently be relatively contraindicated.
Given the history of heavy alcohol use and reduced GCS, parenteral thiamine (pabrinex) should be
given in preference to oral.
Regulating bowel motions will be important in reducing the risk of encephalopathy, but excluding
spontaneous bacterial peritonitis must take precedence.

Further reading
Green TE. Spontaneous bacterial peritonitis. Medscape.
Available at: http://emedicine.medscape.com/article/789105-overview

73. A.╇ On standing, 500–1000mL of blood move from the upper body to the lower body. This
reduces venous return, right atrial pressure, atrial stretch, and carotid body stimulation. This results
EXAM 3 | ANSWERS 247

in a rise in sympathetic drive, and thereby heart rate and total peripheral resistance to maintain BP.
(BP = cardiac output × total peripheral resistance, Cardiac output = stroke volume × heart rate).
In a healthy individual, the rate of these homeostatic mechanisms is such that there is no significant
drop in BP, which then rises above supine. In individuals with impaired haemodynamic responses,
the drop can be significant.

Further reading
Gurme M. Idiopathic orthostatic hypotension and other autonomic failure syndromes.
Available at: http://emedicine.medscape.com/article/1154266-overview

74. A.╇ Compression of morbidity refers to increasing the proportion of expected life-expectancy
before ageing manifestations and chronic disease become apparent. This should be the aim of
public health strategies, rather than merely increasing life expectancy. If individuals live longer, but
with more comorbidities, this will increase the overall burden to individuals and the health economy.
The aim should be ‘healthy ageing’ with compression of chronic disease and disability into the
period before death.

Further reading
Fries J. The Compression of Morbidity. Milbank Quarterly 2005; 83(4): 801–23.
Available at: http://onlinelibrary.wiley.com/doi/10.1111/j.1468-0009.2005.00401.x/pdf

75. D.╇ The patient has not previously sustained an osteoporotic fragility fracture and intervention
is therefore on a primary prevention basis. NICE suggest assessing the risk with a validated tool,
such as FRAX or QFracture in the first instance. Depending on the risk score, further assessment
with DEXA testing may be appropriate. Treatment with bisphosphonates would only be warranted
in a high risk subgroup of the population.
A technetium scintogram identifies areas of high bone-turnover, growth or repair, and not bone
quality. It may be useful for identifying metastases, degenerative change, and osteomyelitis.
Calcaneal ultrasound has been correlated with risk of hip fracture, but is not in widespread
clinical use.

Further reading
NICE. Osteoporosis: assessing the risk of fragility fracture, Clinical guideline CG146. Available at:
http://publications.nice.org.uk/osteoporosis-assessing-the-risk-of-fragility-fracture-cg146/guidance
QFracture. Welcome to the QFracture®-2012 risk calculator. Available at: http://www.qfracture.org/
FRAX. WHO Fracture Risk Assessment Tool. Available at: www.shef.ac.uk/FRAX/
Bauer DC, Ewing SK, Cauley JA, et al. Quantitative ultrasound predicts hip and non-spine fracture in
men: the MrOS study. Osteoporosis International 2007; 18: 771–7.
Available at: http://rd.springer.com/article/10.1007/s00198-006-0317-5

76. D.╇ The patient is most likely withdrawing from alcohol. The clinical presentation is consistent
with this, as is the marocytosis and mild liver derangement. A reducing regime of chlordiazepoxide
will improve his symptoms. He should also have supplemental thiamine. Further investigation may
be warranted if there is limited response. The absence of focal neurological deficit makes it unlikely
that a primary intracranial pathology is the cause of his delirium.
248 EXAM 3 | ANSWERS

Further reading
Katona C, Livingston G, Cooper C. Functional psychiatric illness in old age. In: Fillit HM, Rockwood K,
Woodhouse K. (eds) Brocklehurst’s Textbook of Geriatric Medicine and Gerontology, 7th edn (pp. 433–
44). Philadephia: Saunders, 2010.
McKeown NJ. Withdrawal syndromes. Medscape.
Available at: http://emedicine.medscape.com/article/819502-overview

77. C.╇ The recent assessment in the ED seems to have ruled out a need for acute hospital
admission and an ‘admissions avoidance’ service seems appropriate. She is struggling at home with
three times a day care and so home-based care would not seem appropriate. There is a need to
improve her pain control and supervise medication, whilst achieving this goal. It is likely that this will
need daily medical involvement to adjust analgesia with attention to rehabilitation and avoiding side-
effects from her medication. The complexity of her needs would suggest she is best served by an IC
service with dedicated medical cover and geriatrician input.

Further reading
Department of Health (England). Intermediate care—Halfway home. Updated guidance for the
NHS and Local Authorities, 2009. Available at: http://www.dh.gov.uk/prod_consum_dh/groups/
dh_digitalassets/@dh/@en/@pg/documents/digitalasset/dh_103154.pdf
National audit of intermediate care report. 2012. Available at: http://www.nhsbenchmarking.nhs.uk/
projects/partnership-projects/National-Audit-of-Intermediate-Care.php

78. E.╇ The history and witness account strongly supports seizures over syncope or arrhythmia.
Although it is possible seizures could be triggered by an arrhythmia, the normal 24-hr tape during
an episode excludes this. The response during tilt testing is probably a false positive and does not
match her symptomatology.

Further reading
European Society of Cardiology. Syncope (Guidelines on diagnosis and management of).
Available at: http://www.escardio.org/guidelines-surveys/esc-guidelines/Pages/syncope.aspx

79. C.╇ Occult depression is a common barrier to effective rehabilitation and is commonly seen
after stroke. A high index of suspicion should be maintained. Tools such as the geriatric depression
score may be useful in helping identify affected individuals.
The lack of focal neurology make a further cerebrovascular event (infarct or haemorrhage) unlikely.
It may be that his deterioration is due to a delirium; cognitive testing and a confusion-assessment-
method score should be undertaken. The sudden change from a near normal cognitive baseline
would not be consistent with a dementia.
There is no evidence of chest sepsis on examination.

Further reading
Hackett ML, Yapa C, Varsha P. Frequency of depression after stroke. Stroke 2005; 36: 1330–40.
Available at: http://stroke.ahajournals.org/content/36/6/1330
Young J. Caring for older people: rehabilitation and older people. British Medical Journal 1996;
313: 677. Available at: http://www.bmj.com/content/313/7058/677.full
EXAM 3 | ANSWERS 249

80. D.╇ Increasing age is the most potent risk factor for dementia, although the age of onset is
younger in individuals with the mutations and other risk factors listed in the question. 1 in 25
70–79-year-olds in the UK are affected, rising to 1 in 6 aged >80 years.

Further reading
Camicioli R, Rockwood K. Dementia diagnosis. In: Fillit HM, Rockwood K, Woodhouse K
(eds) Brocklehurst’s Textbook of Geriatric Medicine and Gerontology, 7th edn (pp. 385–91).
Philadelphia: Saunders.

81. C.╇ This is likely to be a functional disorder. The normal perfusion scan counts against an acute
cerebrovascular event. In addition, the apparent hemisensory weakness does not match with a
language deficit, which would probably be based in the left hemisphere in a right-handed individual.
Furthermore, a marked expressive dysphasia, but unimpaired written communication would be
unusual.
It would seem prudent to reassure the patient and observe them. Psychiatric and psychological
input may be needed in due course.

Further reading
NICE. Stroke, Clinical guideline CG68. Available at: http://www.nice.org.uk/guidance/cg68

82. A.╇ The low grade fever, suprapubic discomfort and positive urine dip, coupled with the recent-
onset urinary symptoms are suggestive of a UTI, which warrants treatment with antibiotics.
Trimethoprim is contraindicated given the concomitant methotrexate use and risk of severe bone
marrow suppression. Nitofurantoin may precipitate a worsening peripheral neuropathy. Tazocin is a
penicillin and should be avoided given the history of allergy.
Ciprofloxacin gives good Gram negative cover and would be most appropriate given the
contraindications.

Further reading
British National Formulary. Available at: http://bnf.org/bnf/index.htm
Patient.co.uk. Urinary tract infection in adults.
Available at: http://www.patient.co.uk/doctor/Urinary-Tract-Infection-in-Adults.htm

83. D.╇ Pressure ulceration is unlikely in a fully ambulant patient. The diabetic control is adequate.
There is no evidence of benefit of protein supplementation in a patient eating an adequate diet.
Nicorandil is well recognized to cause oral ulceration and is increasingly recognized to cause
perianal ulceration too. A trial of substituting an alternative anti-anginal should be tried.

Further reading
Fayyaz A, Maw A, Bhomwick A. Anal ulceration induced by nicorandil. British Medical Journal 2007;
335: 936–7. Available at: http://www.ncbi.nlm.nih.gov/pmc/articles/PMC2048831/
NICE. Pressure ulcers: the management of pressure ulcers in primary and secondary care, Clinical
guideline CG29. Available at: http://www.nice.org.uk/CG29

84. C.╇ As body temperature falls, the blood flow to the skin is reduced (vasoconstriction) to
mitigate against further heat loss and the heat generated from muscles is increased (shivering).
Both of these counter-regulatory mechanisms and, indeed, detection of a falling environmental
250 EXAM 3 | ANSWERS

temperature are impaired with increasing age. In addition, environmental modification (central
heating, putting on additional layers) may be reduced.
It remains controversial whether the elderly do tend to have a lower core body temperature.
However, it seems increasingly likely that any observable difference is due to confounding factors
(changes in BMI, lean muscle mass, environmental temperature) than an inherent difference in the
‘thermostat’ setting.

Further reading
Frank SM, Srinivasa NR, Bulcao C, et al. Age-related thermoregulatory differences during core cooling
in humans. American Journal of Physiology - Regulatory, Integrative and Comparative Physiology 2000;
279: R349–R354. Available at: http://ajpregu.physiology.org/content/279/1/R349.full.pdf+html
Lui SH, Dai YT. Normal body temperature and the effects of age, sex, ambient temperature and body
mass index on normal oral temperature: a prospective, comparative study. International Journal of
Nursing Studies 2009; 46(5): 661–8. Available at: http://europepmc.org/abstract/MED/19128801

85. D.╇ Metoclopramide, prochlorperazine, antihistamines and antipsychotics should all be avoided
in patients with restless legs syndrome. Other drugs to avoid include, opioids, anticonvulsants,
diphenhydramine, antidepressants (especially norepinephrine or selective serotonin reuptake
inhibitors).

Further reading
European Restless Legs Syndrome Study Group (EURLSSG). Available at: www.eurlssg.org
Garcia-Borreguero D, Stillman P, Benes H, et al. Algorithms for the diagnosis and treatment of restless
legs syndrome in primary care. BMC Neurology 2011; 11: 28–39.
Available at: www.biomedcentral.com/1471-2377/11/28

86. C.╇ Treatment with AED reduces recurrence after a single seizure by up to 50%. Early
introduction of AED does not appear to influence prognosis—this is best predicted by number of
seizures in the first 6 months after diagnosis and response to AED. Whilst those wishing to continue
driving should be offered early treatment with an AED, it is seizure frequency (degree of control)
that governs whether or not they can drive. AEDs do not reduce the risk of further seizure in
alcohol withdrawal.

Further reading
DVLA. Current medical guidelines: DVLA guidance for professionals.
Available at: http://www.dft.gov.uk/dvla/medical/ataglance.aspx
SIGN. Diagnosis and management of epilepsy in adults.
Available at: http://www.sign.ac.uk/guidelines/fulltext/70/index.html

87. D.╇ Choice of anti-emetic should be targeted to the likely cause of the nausea. In this scenario,
GI tract stasis is the most likely cause. If the obstruction is only partial, use of metoclopramide
may increase peristalsis and relieve symptoms. This would be a reasonable first line agent to use.
It should be given parenterally, as GI absorption is compromised. If metoclopramide exacerbates
abdominal pain, its use should be stopped owing to the risk of perforation. In that situation,
octreotide may be used to reduce gastric secretions, dexamethasone to shrink oedema around an
obstructing lesion and haloperidol would be effective as an anti-emetic.
EXAM 3 | ANSWERS 251

Domperidone is not useful in this scenario as GI absorption is ineffective owing to the gastric
stasis. This could, in turn, be worsened by cyclizine. Ondansetron is principally effective on the
chemoreceptor trigger zone in drug-related nausea and would likely not be beneficial.

Further reading
Baines MJ. ABC of Palliative Care: Nausea, Vomiting & Intestinal Obstruction. BMJ 315;1148–50.
Available at: http://www.ncbi.nlm.nih.gov/pmc/articles/PMC2127704/pdf/9374893.pdf
NHS Lothian ‘Nausea/Vomiting in palliative care’.
Available at: http://www.palliativecareguidelines.scot.nhs.uk/documents/NauseaVomiting.pdf

88. B.╇ It sounds as if this woman has significant mental health issues (either dementia, depression,
or a personality disorder); however, she may simply be living on the margins of society and be
‘eccentric’. There is enough in this history to make one cautious about discharging her home, even
though, so far, she has not come to any harm. The Mental Capacity Act 2005 states that a priori all
individuals should be considered to be capable in decision-making unless proven otherwise. Any
individual may assess mental capacity provided they understand the process and feel competent
to do so. If she is shown to lack capacity to determine her own care needs then an IMCA needs
appointing to ensure her best interests are met in any decisions made about her future care.
Assessment of cognition, e.g. MMSE may help support the outcome of mental capacity assessment,
but a low MMSE does not necessarily imply a lack of mental capacity.
A referral to the mental health team may be required to assist with diagnosis and follow-up after
discharge, or if the medical team do not feel competent to assess her capacity, but is not the next
most important step and may not be necessary if she is deemed to be capacious.

Further reading
HM Government. Mental Capacity Act 2005 (England).
Available at: http://www.legislation.gov.uk/ukpga/2005/9/contents

89. B.╇ Ageing is associated with impairment of receptive relaxation of the gastric fundus, causing
rapid antral filling, distension, and earlier satiety. Circulating levels of IL1, IL6, and TNFα have been
shown to be higher in older people and associated with reduced muscle mass. Taste and smell
make food enjoyable and both deteriorate with age. The decline in sense of smell decreases food
intake in older people and can influence the type of food eaten, and a reduced sense of smell is
associated with reduced interest in and intake of food. Older patients with a reduced sense of taste
also tend to have a less varied diet and, consequently, develop micronutrient deficiencies. The loss
of sense of taste is not understood fully but may be caused by a reduced number of taste buds.
Modifications in the olfactory epithelium, receptors, and neural pathways may affect sense of smell.
Drugs such as Parkinson’s medications and antidepressants, affect sense of taste. Improving the
flavour of foods has been shown to improve nutritional intake and body weight in nursing-home
patients.
Neuropeptide Y, secreted by the hypothalamus, mediates hunger and inhibits satiety and Pro-
opiomelacortin, also secreted by the hypothalamus stimulates satiety. Peripheral hormones, such as
cholecystokinin (CCK) and pancreatic polypeptide (PPY), affect the hypothalamus hunger–satiety
control regulation. Both CCK and PPY are enteric peptides involved in GI motility in response to
eating. High levels of fasting and postprandial CCK and PPY may cause prolonged satiety by slowing
antral emptying. CCK is released in the proximal bowel in response to nutrients from the antrum,
particularly lipids and proteins. CCK is increased in older people, and correlates with high levels
of satiety and low hunger. PPY is released by the distal intestine in the presence of nutrients in the
lumen and causes satiety.
252 EXAM 3 | ANSWERS

Further reading
Ahmed T, Haboubi N. Assessment and management of nutrition in older people and its importance
to health. Clinical Interventions in Aging 2010; 5: 207–16.
Available at: http://www.ncbi.nlm.nih.gov/pmc/articles/PMC2920201/
Morley JE. Anorexia of aging: physiologic and pathologic. American Journal of Clinical Nutrition 1997;
66(4): 760–73. Available at: http://www.ajcn.org/content/66/4/760.long

90. B.╇ The duration of dementia is very short and behavioural disturbance an early feature, with
a normal head CT scan, it would seem unlikely that this is either Alzheimer’s or vascular dementia.
It is most likely that this is a frontal dementia. Her current problem seems to be a delirium, with
no apparent cause other than a difficulty adjusting to a new environment. The fact that she is
intermittently calm and responds to staff, and allows examination (physical and tests), suggests
that she would be best managed by returning to her care home with intensive support by the
community health team (for the care home staff ) to see if she will settle in to the home. The most
appropriate management of this case requires complex multidisciplinary team working across
boundaries (interfaces) between the acute hospital, primary care, mental health, and social care.
Assessment in the ED has ruled out other common causes of delirium, so admission to the acute
hospital would not seem to have any benefit for her and is inappropriate. If she does not settle
in the care home, admission to the psychiatric unit, for further assessment of her behavioural
disturbance and dementia, would be appropriate and to consider the possibility of rarer rapidly
progressive dementias, e.g. paraneoplastic syndromes.
Transfer to a specialist dementia registered care home may ultimately be necessary, but would
seem premature at this point in time. She should be given a chance to settle in to the care home,
assuming the original decision regarding placement was appropriate.
If it had been possible to support her in her own home with a live-in carer one would assume this
would have been done, rather than the original transfer to the residential home.

91. C.╇ Miller Fisher syndrome is a rare, acquired nerve disease that is considered to be a variant of
Guillain–Barré syndrome. Ophthalmoplegia, areflexia and ataxia is the classical triad of Miller Fisher
Syndrome. Like Guillain–Barré syndrome, symptoms may be preceded by a viral illness. Additional
symptoms include generalized muscle weakness and respiratory failure. The majority of individuals
with Miller Fisher syndrome have a unique antibody that characterizes the disorder. Treatment for
Miller Fisher syndrome is identical to treatment for Guillain–Barré syndrome: IV immunoglobulin
(IVIg) or plasmapheresis, and supportive care. The prognosis for most individuals with Miller Fisher
syndrome is good. In most cases, recovery begins within 2–4 weeks of the onset of symptoms, and
may be almost complete within 6 months. Some individuals are left with residual deficits. Relapses
may occur rarely (in less than 3% of cases).
Normal responsiveness and normal inflammatory markers exclude brainstem encephalitis and
Wernicke’s encephalopathy in which the patient is confused if not comatose. Brainstem infarct
would give unilateral weakness with ipsi- or contra-lateral cranial nerve signs depending upon the
site of lesion. Myasthenia gravis is characterized by fluctuating muscle weakness depending upon the
level of exertion of the muscles and also the time of the day; in early presentation being negligible
in the morning and after rest.

Further reading
Andary MT. Guillain–Barre Syndrome. Medscape.
Available at: http://emedicine.medscape.com/article/315632-overview
EXAM 3 | ANSWERS 253

Guillain–Barré Syndrome Support Group. Guidelines for Medical Professionals.


Available at: http://www.gbs.org.uk/pdf/0000_Guidelines_for_medical_professionals.pdf

92. C.╇ Attendance allowance is a tax free benefit available to anyone aged over 65 years, who
needs help with personal care, irrespective of their underlying diagnosis. It does matter whether
he lives in his own home or in his daughter’s. It is not means tested and does not require a medical
report. It is meant to assist with the cost of care, but does not have to be used to pay for care.
Sign-posting patients and their carers (families) towards appropriate health and social care, including
assisting them in their applications, is an important part of the support provided by Parkinson’s
specialist nurses.

Further reading
HM Government. Attendance allowance.
Available at: http://www.direct.gov.uk/en/disabledpeople/financialsupport/dg_10012438
Patient.co.uk. Attendance allowance.
Available at: http://www.patient.co.uk/health/Attendance-Allowance.htm

93. B.╇ The lesion described is typical of a basal cell carcinoma. It is small in size, is are slow growing
and rarely metastasize, and so, at this age, can be kept under observation and requires no specific
treatment. Treatment options should be discussed with him if possible (although, as he is resident
in a dementia care home, it is unlikely that he will have capacity to make decisions) or with his next
of kin.

Further reading
Bader RS. Basal cell carcinoma. Medscape.
Available at: http://emedicine.medscape.com/article/276624-overview

94. A.╇ It is possible that he had a convulsion as there has been structural damage to the brain.
Incontinence is unhelpful as this could occur with any cause of loss of consciousness. He could also
have suffered a cardiac arrhythmia or have been concussed after falling off his bicycle. A witness
account would the most appropriate, and often forgotten, next step and would enable appropriate
targeting of onward investigation, avoiding unnecessary tests.
When trying to differentiate seizure from syncope:

Unhelpful features
• Brief twitching and jerking.
• Incontinence.
• Pallor.
• Tongue or lip biting.
• Post-event fatigue.

Helpful features
• Post-event confusion of more than 2min.
• Deeply bitten lateral border of tongue.
• Tonic-clonic movements lasting greater than 1min.
• Deep cyanosis.
254 EXAM 3 | ANSWERS

Further reading
SIGN. Diagnosis and management of epilepsy in adults, 2003.
Available at: http://www.sign.ac.uk/guidelines/fulltext/70/index.html

95. D.╇ Pelvic floor exercises (PFE) are potentially useful for treating stress and mixed urinary
incontinence and so could be of benefit to her. Pelvic floor strength and endurance can adequately
be assessed by either digital rectal or vaginal examination. Clinical guidelines recommend digital
examination of pelvic floor strength in order to direct treatment decisions, although there is no
evidence that this can determine outcome of PFE. PFE regimens are as effective if delivered in
groups or individually, but the exercise programme needs to be tailored to the individual if it is to
be achievable. PFE can be done anywhere the person wishes to and most people can manage to do
them quite discretely. PFE usually require 15–20 weeks to be effective and must be done several
times a day until the pelvic muscles fatigue. Once continence is regained then the individual should
continue with maintenance exercises.

Further reading
NICE. The management of urinary incontinence in women, Clinical guideline CG40.
Available at: http://www.nice.org.uk/CG40
Scottish Intercollegiate Guidelines Network: Management of urinary incontinence in primary care.
Available at: http://www.sign.ac.uk/pdf/sign79.pdf

96. D.╇ As she is tremor dominant a kettle tipper may be useful to minimize spillage and risk of
scolding herself when pouring boiling water.
The fact that she is taking fludrocortisone suggests she has or has had problems with orthostatic
hypotension. The alendronic acid and calcium and vitamin D supplements point towards
osteoporosis and, most probably, she has fallen in the past and sustained a femoral neck fracture.
Thus, a perching stool is likely to be of benefit when she is undertaking jobs in the kitchen that
might otherwise require her to stand for prolonged periods of time. Likewise, a walking trolley may
help with mobility around the kitchen, reducing falls risk and would also reduce spillage of food and
drinks caused by her tremor.
The steady spoon is secured to the hand by an adjustable velcro strap, the spoon bowel swivels to
keep food stable and level, despite tremor and an active counterbalance and weight steadies the
spoon bowel keeping food level.
A utensil hand grip is an eating aid for people who have trouble grasping and holding small utensils.
A spring-action clip fits on to the hand and may be used with different utensils. This may be useful
for those with arthritis of the hands, post-stroke, or with peripheral neuropathy, but would not be
useful for someone with tremor.

Further reading
The following website provides a useful insight in to the various domestic appliances that might benefit
people with all degrees of disability. Available at: www.essentialaids.com

97. D.╇ The low serum osmolality and inappropriately normal urine osmolality are consistent
with a syndrome of inappropriate antidiuretic hormone (SIADH). This is likely being driven by the
pneumonia and mirtazapine.
After addressing the cause, the first line treatment should be to restrict the patient’s fluid intake.
She is receiving a large amount of fluid in her NGT feed. The dieticians should be asked to review
this and a more concentrated feed substituted.
EXAM 3 | ANSWERS 255

She is euvolaemic and not sodium deficient (high urine spot sodium) so IV normal saline is not
appropriate. Hypertonic (1.8%) saline may also result in a rapid rise in serum sodium, which could
precipitate central pontine myelinolysis.
Demeclocycline induces a nephrogenic diabetes insipidus and ‘free-water’ loss. It can be used to
treat SIADH, but should be reserved until after fluid restriction has been tried.
Tolvaptan is a V2 receptor antagonist, which has a similar effect, but addressing the relative excess
fluid intake should be tried first.

Further reading
Simon EE. Hyponatremia. Medscape.
Available at: http://emedicine.medscape.com/article/242166-overview#a0199

98. A.╇ Screening asymptomatic individuals for cancer is most easily justified where mortality
rates for that cancer can be reduced. This is most likely to occur for those cancers where there is
effective treatment and early detection can also reduce metastatic spread (A). The outcome of a
screening trial is expressed as a proportional reduction in mortality from the specific cancer and
then a judgment made as to whether this is worthwhile.
The probability of successful treatment of a cancer may be increased by early diagnosis. However,
detecting more lesions at an early stage does not justify screening if treatments are not available or
do not affect mortality and morbidity from the cancer (D and E).
Early identification of a cancer may enable better end-of-life planning, for those that are incurable,
but does not justify screening asymptomatic individuals (C).
The morbidity (physical and psychological hazard) associated with screening should be as low
as possible, especially in asymptomatic individuals and ideally non-existent (B). However, lower
morbidity in its self does not justify screening.

Further reading
Boyle P, Autier P, Bartelink H, et al. European code against cancer and scientific justification: third
version. Annals of Oncology 2003; 14(7): 973–1005.
Available at: http://annonc.oxfordjournals.org/content/14/7/973.full

99. D.╇ Although you are not told how long he has been a widower, this may be an important clue
to depression as an abnormal post-bereavement reaction. The normal initial blood tests are also
clues to a possible non-organic cause for his symptoms. Whilst it would be important to ask about
alcohol usage, the symptomatology points more to depression and excess alcohol usage may simply
be a manifestation of depression. Myeloma is unlikely in the context of the normal biochemistry,
FBC, and ESR. Tumour markers are not useful as screening tests as they are neither sensitive nor
specific enough when used in isolation.

Further reading
National Cancer Institute. Tumour markers.
Available at: http://www.cancer.gov/cancertopics/factsheet/detection/tumor-markers
NICE. The treatment and management of depression in adults, Clinical guideline CG90.
Available at: http://www.nice.org.uk/CG90
Perkins GL, Slater ED, Sanders GK, Prichard JG. Serum tumor markers. American Family
Physician 2003; 68(6): 1075–82. Available at: http://www.aafp.org/afp/2003/0915/p1075.html
256 EXAM 3 | ANSWERS

Rodda J, Walker Z, Carter J. Depression in older age:


Available at: http://www.bmj.com/content/343/bmj.d5219.extract

100. C.╇ This recent change (poor appetite and fluid intake, hallucinations and decline in
physical function) is consistent with a delirium. He should be assessed for possible causes of
delirium: smelling of urine suggests new onset incontinence, which may be a marker of a urine
infection, but one should consider that it may also be a marker of reduced mobility due to some
other cause of delirium or that, in his delirium, he may not register the need to micturate in a toilet.
If there is no obvious precipitant for a delirium then one should consider the possibility that he is
developing a psychosis within his Parkinson’s disease; this should be given earlier consideration if his
Parkinson’s medication had recently been changed. His wife should attempt reality orientation with
him and reassure him that his hallucinations are not real, but this is not the most important next
step in identifying what is wrong with him.

Further reading
Miyasaki JM, Shannon K, Voon V, et al. Practice parameter: evaluation and treatment of depression,
psychosis, and dementia in Parkinson disease (an evidence-based review). Report of the Quality
Standards Subcommittee of the American Academy of Neurology. Neurology 2006; 66: 996.
Available at: http://www.neurology.org/content/66/7/996.full.pdf
INDEX

Note: Questions appear in bold; answers appear in italics.

Abbey pain scale 95, 142 care for patient when carer is hospitalized 94, 141
Abbreviated Mental Test (AMT) 198 caring for spouse with 90
abdomen, distended 205 loss of central inhibition to defaecation in 170
abscess 84 and urinary tract infection (UTI) 111
accelerated ageing process 121 amantadine 28, 53
acetylcholine 57 amiodarone 7, 131, 164, 229
acetylcholinesterase inhibitors 102, 148, 163, 164, 227 amlodipine 4, 27, 92, 96, 116, 143, 161, 194, 198, 205
n-acetylcysteine (n-AC) 123 amnesia 98, 145, 243
acute coronary syndrome 115 retrograde 144
acute hospital beds 227 transient global amnesia (TGA) 55, 144
Adcal D3 Forte® 196, 216 amoxicillin 201, 216
Addenbrooke’s Cognitive Examination (ACE) 79, 125 ampicillin 210
Addison’s disease 10, 59, 67, 182, 226 anaemia 45, 67, 164, 228
adenocarcinoma 182 pernicious 210
adrenal metastases 119 anal squeeze pressure 170
advanced care planning (ACP) 203, 245 analgesia 56, 111, 165, 235
age-related macular degeneration (ARMD) 41, 79, 99, 111, angina 8, 14, 54, 61, 124, 175, 176, 196, 222, 239
176, 192, 239, 245 angiography 222, 239
ageing 121, 161–2 angiotensin-1 161
acquired immunity reduction and 130 angiotensin-2 161
association with impairment of receptive relaxation of the angiotensin-converting enzyme inhibitors (ACE-I) 56, 81,
gastric fundus 251 115, 224
cardiovascular changes associated with 38 angiotensin inhibition 229
decline in partial pressure of arterial oxygen with 51 angiotensin receptor blockers (ARB) 224
marker of healthy ageing 205 ankle brachial pressure index (ABPI) 43, 102
physiological anorexia of 212 ankle reflexes, attenuated 32
aggression 11, 60, 100, 111, 182, 213 ankles, mild bilateral 134
agitation 25, 29, 40, 53, 104, 150, 206, 213 anopexy 241
akathisia 53 anorexia 8, 217
acute 53 anti-anginals 249
tardive 53 anti-emetic drugs 211, 212, 250
albumin 143, 204, 210, 246 anti-epileptic drugs (AED) 162, 211, 221, 250
alcohol 27, 31, 107, 184, 205, 228, 246, 247, 255 anti-hypertensives 239
aldosterone 81, 143 anti-vascular endothelial growth factor 239
alendronate 3, 67, 100, 107, 237 antibiotics 29, 37, 65, 66, 111, 117, 204, 249
alendronic acid 18, 115, 176, 196, 216, 254 antibody testing 226
alfacalcidol 67, 68, 147, 237 anticholinergics 58, 73, 157, 223
alfentanyl 235 anticoagulation 147
alkaline phosphatase (ALP) 77, 80 anticonvulsants 250
allopurinol 242 antidepressants 144, 250
alpha-2 adrenergic receptor 57 appetite-stimulating antidepressants 154
alpha blockers 157 tricyclic 53
alpha foetoprotein (AFP) 246 antidiuretic hormone 254
Alzheimer’s disease 6, 8, 109, 163, 164, 173, 185, 252 antihistamines 250
258 INDEX

antihypertensives 187, 229, 243 canes 238


antiphospholipid antibody syndrome 137 foam mattresses 148
antiplatelet therapy, in ischaemic stroke 232 mattress tilters 239
antipsychotic (neuroleptic) drugs 53, 56, 58, 71, 83, 144, rails 63, 136
155, 164, 165, 226–7, 250 sheets 136, 238–9
antispasmodic drugs 223 bedsores see pressure sores
anxiety 53, 180, 244 behavioural disturbances 226
aortic stenosis 193 Behcet’s disease 137
aphasia 138 bendroflumethiazide 1, 26, 27, 34, 43, 44, 69, 96, 99, 127,
apixaban 139 143, 176, 178, 191, 202
apomorphine 28 benefits
appetite 19, 45 attendance allowance 83, 177, 214, 222, 253
appetite stimulating anti-depressants 154 carer’s allowance 17, 62, 99, 177, 192, 222
areflexia 252 benign paroxysmal positional vertigo 60
arms, muscle aches in 91, 180 benign prostatic hyperplasia 115, 157
arrhythmia 248 Benson’s syndrome 95
arterial function, age-related change in 61 benzodiazepines 53, 235
arterial oxygen (arterial pO2), decline with ageing 51, 85 benzylpenicillin 50, 171
arteritic ischaemic optic neuritis 245 Berg Balance Scale 153
arthritis 146 beriplex 227
aspiration pneumonia 14, 129, 197, 200 beta-agonist inhalers 215
aspirin 1, 4, 14, 15, 17, 21, 26, 33, 36, 92, 99, 113, 115, beta-agonist/steroid inhalers 21
116, 124, 126, 127, 132, 160, 177, 178, 180, 191, beta-agonists 56, 157
193, 194, 196, 203, 210 beta-blockers 57, 74, 85, 115, 222, 229, 239, 244
as antiplatelet treatment for people with TIAs 244 eye drops 74
and CHA2DS2-VASc score 57, 169 induced syncope 125
and Churg−Strauss syndrome 69 and ischaemic heart disease 65
and dipyridamole 62 lipophilic 53
effect on PT and APTT studies 76 bidets 145
NICE guidelines on 70 bifocal lenses 230
after NSTEMI 51 biochemical disturbances 220
and retinal vein occlusion 137 bisoprolol 14, 21, 33, 57, 91, 180
for secondary prevention of ischaemic heart disease 8, bisphosphonates 60, 76, 77, 152, 237, 247
27, 239 bladder
for transient ischaemic attack 171 incomplete emptying 116
and type 2 diabetes 6, 91, 109 prolapsed bladder (cystoceole) 225
asthma 26, 43, 69, 224 bleeding from gums 19
asymptomatic bacteriuria 53 blindness 41
ataxia 252 blisters 166
atenolol 57 on lower limbs 128
atrial fibrillation (AF) 7, 9, 17, 33, 57, 93, 101, 131, pemphigus foliaceus 166
198, 209 blood clotting 27
angiotensin inhibition and 229 blood glucose 66
management of 185 blood pressure monitoring 243
warfarin and 175, 183 blurred vision 4, 92
atrial-natriutetic polypeptide (ANP) 161 body mass index (BMI) 16, 142, 174, 179, 180
attendance allowance 83, 177, 214, 222, 253 Bolam test 219
avascular necrosis 64 Bolitho test 219
bone density scan 59
B-cell function 167 bone lesions
back pain 207 lytic 55
baclofen 57, 223, 235 sclerotic 80
bacterial meningitis 133, 171 bone pain 21
Barthel Index 130, 167, 199, 241 bone prophylaxis, with bisphosphonates 60
basal cell carcinoma 168, 253 bone scintography 55, 80
bed-based care 225 boredom 83
bed(s) botox injections 19
alternating pressure mattresses 148 botulinum toxin 57, 223
INDEX 259

bowed leg 24, 67 strain on 148


bowel motions 246 cataracts, bilateral 17, 62, 99, 111, 114, 192
bowel resection 47 category fluency test 79
brachytherapy 105 catheterization, temporary 72
bradykinesia 176, 191 cauda equina syndrome 82
Brain Bank Diagnostic Criteria 221 cefotaxime 171
brain natriuretic polypeptide (BNP) 89 ceftriaxone 50, 171
breast cancer 3, 21, 55 cellulitis 199
breast tumours 22 central pontine myelinolysis 255
breathing, deterioration in 5 central retinal vein occlusion 90
breathlessness 3, 179, 192 cephalosporins 171
Breslow thickness 84 cerebral haemorrhage 16
bronchodilators 56, 224 cerebral infarction 7
Bruce protocol 88 cerebral venous sinus thrombosis 62, 139
bruises 20, 117 cerebrovascular disease 170, 221
bilateral periorbital bruising 33 cervical spine injury 60
facial 201 CHA2DS2-VASc score 57, 169
bullous pemphigoid 165 Charles Bonnet syndrome 156, 222, 235
buprenorphine 11, 150, 165 chest
infection 152, 183
C. elegans 161, 162 pain 88, 115
C-reactive protein (CRP) 139 pleuritic 192
calcaneal ultrasound 247 tightness 177
calcium 8, 18, 100, 107, 115, 164, 176, 237, 254 X-ray 89
homeostasis 78 chlordiazepoxide 246, 247
metabolism 228 cholangiocarcinoma 46
serum 8, 55, 237 cholecalciferol 237
supplements 3, 55, 243 cholecystokinin (CCK) 251
calcium channel antagonists 239 cholesterol 15, 33, 137, 244
calcium channel blockers 57, 85, 222, 224 cholinergic deficit 82
Campylobacter-like organism (CLO) test 27 chondroitin 138
cancer chronic
basal cell carcinoma 168, 253 alcoholism 161
bowel 119 back pain 207
breast 21, 104 low 197
breast cancer 3, 21, 55 cardiac insufficiency 97
lung 115, 156 cognitive decline 167
prostate 191, 193 hypomagnesaemia 78
screening of asymptomatic individuals for 216, 255 infection 162
capacity liver disease 246
assessment 158, 240 malnutrition 161
treatment refusal 244 obstructive pulmonary disease see COPD
capsaicin 138, 223 renal impairment 7, 24, 178, 190, 194
carbamazepine 36, 162, 166 stable angina 177
cardiovascular ageing 15 venous leg ulcers 199
cardiovascular disease 15 Churg−Strauss syndrome 69
impaired endothelial relaxation and 77 ciprofloxacin 249
care homes citalopram 202
advanced care planning (ACP) in 203 clarithromycin 183, 227
rehabilitation in 69 clavicles, fractured 20
risk of malnutrition in 95 Clinical Commissioning Group, intermediate care service 123
risk of poor care in 158 clock drawing test 79
care packages 18, 30, 72, 78, 241 clonidine 53
care services 163 clopidogrel 51, 62, 70, 115, 244
carers Clostridium difficile 53, 66
allowance 17, 62, 99, 177, 192, 222 clot retrieval 147
education 224 clotting abnormality 70
low well-being in 90, 138 co-amoxiclav 10, 23, 96, 143, 206
260 INDEX

co-beneldopa 115, 134 core body temperature 250


co-careldopa 29, 95, 191, 217 coronary angiography 222
co-codamol 124 coronary artery bypass surgery (CABG) 239
cocaine 69 coronary artery disease 54, 136, 239
Cochrane review 69, 85 Coroners and Justice Act 2009, 61
codeine 4, 92, 132, 197, 223, 235 corticosteroids 65
codeine phosphate 38 creatine kinase 225
coeliac disease 165 creatinine 6, 7, 56, 225
cognitive crepitus 91
assessment 146, 184, 240, 251 Cushing’s syndrome 67
Montreal Cognitive Assessment 79 cutaneous lesions 166
decline 163 cyclizine 117, 142, 236, 251
function deterioration 125 cystocoele (prolapsed bladder) 225
impairment 9, 58, 73, 79, 102, 176, 198, 221, 232, 233
relationship between frailty phenotype and 219 D-dimer 89, 137, 164, 236, 244
colchicine 242 dabigatran 57, 139
colecalciferol 68, 147 dabigatran etexilate 153
colitis, ulcerative 43 dantrolene 57, 223
collapse 105 deep vein thrombosis (DVT) 109, 153, 236
Colles’ fracture 107, 183 left ileofemoral 130
colonic malignancy 81 defaecation, loss of central inhibition to 170
colostomy 47 deficits accumulation model of frailty 148
colposuspension 223 deforming psoriatic arthropathy 49
coma deforming rheumatoid arthritis 52
myxoedemic 136 dehydration 55, 60, 139, 161
non-myxoedemic 136 delirium 69, 109, 151, 170, 220, 240–1, 247, 252, 256
commodes 84 acute delirium 56, 71, 78
community-dwelling frail elders, case management of 203 anxiety and agitation secondary to 53
community hospitals 227 and cholinergic deficit 82
early discharge to 69 confusion driven by 68
community psychiatric nurse (CPN) 97 due to UTI 2, 25, 132, 155
Comprehensive Geriatric Assessment (CGA) 95, 98, 145, fractured neck of femur as risk factor for 236
185, 229 hypoactive 82
compression bandaging 80 infection related 239
‘compression of morbidity’ 205 peri-surgery 152
compression stockings 153 reducing risk of post-operative 87
computed tomography (CT) rivastigmine and 135
angiogram 54 see also confusion
head scan 169 delusions, erotic 132
imaging 55, 246 demeclocycline 255
pulmonary angiography (CTPA) 236 dementia 5, 129, 130, 151, 155, 176, 213, 219, 252
venography 139 and ability to localize pain 228
confusion 3, 23, 25, 29, 31, 68, 96, 98, 112, 126, 193, 197, advanced 200
206, 210 age as factor for 249
acute confusion 198 Alzheimer’s see Alzheimer’s disease
see also delirium formal diagnosis of 167
confusion assessment method (CAM) 240 frontal 252
congestive cardiac failure 93 lack of interest in food and 154
conjunctival jaundice 205 with Lewy bodies 48, 55, 62, 71, 156, 221, 233, 235
consciousness 29 malnutrition and 147
loss of 74, 89, 105, 131, 169, 207–8, 211, 243 management 119, 182
constipation 21, 66, 80, 82, 110, 150 mild dementia 30
continence pads 157, 188 moderate dementia 48
conveen 59, 84 needs assessment of eligibility for NHS Continuing Health
convulsions 221, 253 Care 93
COPD 21, 65, 116, 180, 192, 215, 236 prescribing drugs for sufferers of 58
management of 157 progression of 53, 55
partially reversible (stage 2 moderate) 56 and rehabilitation 82
INDEX 261

risk factor for development of 209 domperidone 100, 142, 182, 251


risk of harm living alone with 187 donepezil 132, 163
semantic 138 dopamine agonists 102
treatment of psychotic symptoms associated with 227 dopamine dysregulation syndrome 85
vascular 1, 46, 103, 110, 182, 221, 234, 252 dorsalis pedis pulses 32
and visual impairment 79 double vision 16, 214
denosumab 67, 237 doxazosin 157
depression 82, 108, 122, 151, 164, 167, 184, 202, 228, 255 dressing aids 49, 84
occult 248 dressing sleeves 84
psychotic 97, 228 dressing sticks 84
depressive illness see depression leg lifters 84
Deprivation of Liberty Safeguard (DoLS) procedure 60 trouser clips 84
dermatis herpetiformis 165 zip pulls 84
desmopressin 73 dressings 81
DEXA scans 67, 122, 237, 247 activated charcoal dressings 66
dexamethasone 96, 158, 250 silver-based 164
diabetes mellitus 19, 33, 42, 43, 93, 124, 137, 143, 151, tranexamic acid soaked 66
204, 210 driving 165
cardiovascular risk factors in patients with 74 fitness to drive 28, 70–1, 150
diet-controlled 32, 73, 91, 127 seizures and 104
necrobiosis lipoidica diabeticorum on lower legs of patients dronedarone 57, 229
with 172 drooling 36
target BP 224 drowsiness 50, 101, 117
treatment 154 ‘drusen’ 196
type 2 6, 17, 22, 24, 43, 91, 96, 106, 109, 126, 175, dry skin 8
179, 186, 187, 194, 203, 204, 210, 223 Duke’s B adenocarcinoma bowel 47, 87
blood pressure lowering in 229 Duke’s B colonic carcinoma 47
diet-controlled 91, 133, 180, 185, 198 duloxetine 73
diabetic amytrophy 228 duodenal ulcer 26, 27
diabetic ketoacidosis, hyperosmolar non-ketotic 173 DVLA 150
diabetic retinopathy 245 dynamic hip screw 64
diamorphine 68, 81, 115, 117, 156, 158, 166, 235, 236 dysarthria 220
diarrhoea 21, 23, 66, 120, 150, 160, 182, 226 dysdiododyskinesis 189
diazepam 57, 129, 166 dyslexia 30
diclofenac 150 dyspepsia 67, 200
digital rectal examinations (DRE) 66, 132, 154 dysphagia 95, 142, 200
digital vaginal examinations 154 dysphasia 50, 126, 127, 129, 203, 249
dignity 242 dyspnoea 44
digoxin 229 dyspraxic left upper limb 199
diltiazem 139 dysuria 10, 105, 150, 210
Dilute Russell’s Viper Venom Time (dRVVT) test 76–7
diphenhydramine 250 eating aids 254
dipyridamole 244 ECG (electrocardiogram) 89
direct current (DC) cardioversion 164 echocardiography 151, 236
disability 72 eczema 70, 149
discharge from hospitals, care at discharge 63 eGFR 56
discharge placement, for Parkinson’s patient 146 elastic compression stockings 153
discharge plans 199, 241–2 elder abuse 20, 65
discrete perianal ulcers 210 elderly, susceptibility to hypothermia 211
displaced intracapsular fractures 64 electroencephalography 169
disruptive behaviour 48 electrolyte abnormalities 216
diversion therapy 83 encephalopathy 246
diverticulosis 10 endarterectomy 61, 244
dizziness 13, 89, 202, 244 endoscopic decompression 42
see also light-headedness endoscopic retrograde cholangiopancreatography
Do Not Attempt Resuscitation (DNACPR) order 123, (ERCP) 46
202, 244 enoxaparin 96, 161
Doloplus-2 56, 165 eosinophilia 69
262 INDEX

epilepsy 122 pain on weight bearing 38


post-stroke 113 periprosthetic fractures 11
epistaxis 183, 227 sclerotic patch in proximal femur with coarse
Epley manouevre 60 trabeculae 42
erotic delusions 132 fentanyl patch 150, 165
erythrocyte sedimentation rate (ESR) 67 ferritin 234
erythropoietin 234 ferrous fumarate 234
Escherichia coli 2 ferrous sulphate 190
estimated glomerular filtration rate (eGFR) 56 feverishness 50
etidronate 67 financial abuse 34, 74
euthanasia 219 finasteride 157
exenatide 154 fingers, pitting scars on tips 52
exercise 138 fitness to drive 28, 70–1, 150
programmes 230 fixed flexion deformities 242
tolerance 89 flea infestations 174
test 88 flecainide 229
extramedullary implants 64 fludrocortisone 10, 100, 119, 196, 216, 254
eye drops (timolol) 34 fluid intake 46
eyes focal neurology 248
distortion of vision in 196 folic acid 126, 210
meiotic pupils 35 food intake 46
ezetimibe 225 foot
bilateral foot drop 242
facial ulcers 175
bruising 201 forehead, lesions 48
droop 118 forgetfulness 99, 125
injury 74, 163 formal capacity assessment 144
weakness 19 fractures
factor II 221, 227 clavicles 20
factor VII 221, 227 Colles’ fracture of arm 183
factor IX 221, 227 displaced intracapsular 64
factor X 221, 227 distal right tibia 183
factor prothrombin complex 221, 227 femur neck 18, 24, 47, 106, 152, 193, 254
faecal incontinence 94, 132, 170, 198, 241 femur shaft 107
falls 9, 10, 11, 18, 20, 27, 34, 100, 130, 194, 205, 215 fragility 67, 160
brisk walking and risk of 230 minimizing risks of 24
factors contributing to 74 osteoporotic fragility fractures 247
hip pain following 204 periprosthetic femur fractures 11
impaired coordination and 108 pubic ramus 23
LRTI after 41 ribs 20, 117
management of risk of 99 subtrochanteric 152
Parkinson’s disease and 102, 172 use of MRI in cases of suspicion of 246
recurrent falls 20, 37, 111, 114, 124, 189, 201, 207–8 vertebral crush 207
reducing likelihood of falls in hospital settings 17, 63 fragility fractures 67, 160
reducing risk of 68, 76, 114, 146, 186 frailty 19, 35, 75, 165, 173
related to difficulties manoeuvring around furniture 39 Fried clinical phenotype of frailty 173, 219
syncopal falls 126 index of accumulated deficits 103, 148
fans 236 and loss of physical function 19
fatigue 8, 45, 67, 217 markers of 189, 234
febuxostat 242 Framingham risk equation 74
feeding 97 FRAX 247
identifying patients with problems 220 Fried clinical phenotype of frailty 173, 219
feet, painful 32 frontal lobe disease 170
female:male ratio 55 frontalis 19
femidom 145 frontotemporal meningioma 191
femur fungal nail infections 26
fractured neck of 18, 24, 47, 106, 152, 193, 236, 254 fungating breast tumours 22
fractured shaft of 107 furosemide 93
INDEX 263

gait hearing aids 87, 135, 189


abnormalities 233 acoustic feedback 135
rehabilitation 163 hearing impairment 61, 87, 135, 233
speed 75 heart, ageing 77
unsteadiness of 214 heart disease, ischaemic 8, 178
gastric adenocarcinoma 67 heart failure 42, 93, 136, 152
gastrointestinal (GI) disease 172 with normal ejection fraction 198, 240
general hospitals, post-acute care in 26 heart pressures 151
generalized muscular rigidity 176 heels, pressure sores 102, 148
generalized pain and weakness 24 hemiarthroplasty 64, 106, 193
gentamicin 161 hemiparesis 129
Geriatric Depression Scale (GDS) 51, 125, 164, 248 left 117, 118, 187, 209
giant cell arteritis (GCA) 245 hemiplegia
Glasgow Coma Score (GCS) 50, 188 left 203
glaucoma 34, 137, 175 right 133
acute 245 heparin 143
gliclazide 6, 17, 43, 109, 154, 194 low molecular weight (LMWH) 78, 103, 109, 130, 153
glioblastoma 117 hepatocellular insufficiency 161
glucocorticoids 162 hernia
glucosamine 138 strangulated incisional 112
glutamate receptor antagonist 164 strangulated inguinal 103
glyceryl trinitrate (GTN) 8, 14, 196, 239 strangulated para-umbilical 132
glycopyrrhonium 117, 166 high blood pressure 179
glycosylated haemoglobin 179 hips
gold injections 52 bilateral hip replacements 11
Gompertz–Makeham model 149 dynamic hip screw 220
Goodpasture’s syndrome 137 fractures, mortality of 152
gout 200, 242 pain following falls 204
grief reaction 151 replacements 11
growth hormone 161 total hip arthroplasty 37
Guillan–Barré syndrome 252 Holter monitoring 169
gums, bleeding 19, 64 home-based care 225, 248
homonymous hernianopia 96
H. pylori infection 70 hormone replacement therapy (HRT) 28
H2 receptor antagonists 70 hydration 242
haematoma 134, 186 hydrocephalus 233
haemophilia A and B 76 hydrocortisone 10
haemorrhages, splinter 19 hydroxocobalamin 234
haemorrhoids 241 hydroxyproline 80
hair loss 28 1-25-hydroxyvitamin D 237
Hallpike manoeuvre 60, 114 hyoscine 236
hallucinations 17, 40, 115, 134, 156, 174, 176, 206, hyoscine hydrobromide 68
222, 235 hyper-eosinophilic syndrome 69
haloperidol 1, 53, 78, 111, 142, 155, 250 hyperactive episodes 155
hand scratching 103 hypercalcaemia 21, 54–5, 66
handicap 72 hypercholesterolaemia 137, 204, 210
HbA1c 66 hyperglycaemia 66
head injury 13, 60, 201, 207 hyperhomocysteinaemia 137
CT imaging 243 hyperkalaemia 56, 96, 143, 163
head-up tilt test 208 hyperosmolar non-ketotic diabetic ketoacidosis 173
headaches 8, 117 hypertension 3, 4, 6, 17, 19, 24, 26, 27, 32, 33, 34, 43, 92,
and nausea 16, 118, 158 96, 126, 127, 175, 176, 178, 179, 187, 191, 194,
sudden onset 4, 92 202, 203, 204, 205, 230
health care and amlodipine 198
funding 140, 231 bendroflumethiazide and 1, 44
marker of successful healthcare 205 COPD and 116
needs, assessment of 140, 231 diagnosis of 243
patient checklist 158 ischaemic heart disease and 21
264 INDEX

hypertension (Cont.) insomnia 114, 217


management of 111 insulin 154, 173
and retinal vein occlusion 137 glargine 22
stage 1 223 -like growth factor (IGF) 161
stage 2 62 resistance 154
treatment of 16 interdisciplinary teams 153, 163
‘white coat hypertension’ 155 Intermediate Care (IC) services 35, 69, 75, 130, 225, 248
hypertonic saline 67 bed-based 181
hyperventilation 244 home-based 181
hypervitaminosis A 58 National Audit of 181
hypervitaminosis B3 (niacin) 58 International Normalized Ratio (INR) 183, 227
hypervitaminosis B6 58 intra-professional teams 153
hypervitaminosis C 58 intracerebral haemorrhage 215
hypervitaminosis D 58 intracranial pressure 158
hypnotics 56 intracranial tumours 55, 139
hypoactive delirium 82 intramedullary nailing 64
hypoadrenalism 136, 151, 163 IQCODE 71, 240
hypocalcaemia 237 iron deficiency 53, 70, 164, 228, 234
hypoglycaemia 66, 154 irregular heart rhythm 33
hyponatraemia 66–7, 73, 163 ischaemic heart disease 8, 21, 27, 65, 97, 178, 187, 210
hypotension 67, 239 ischaemic stroke 42, 188
orthostatic 100, 172, 254 early antiplatelet therapy in 232
postural 9 isosorbide mononitrate 57
hypothermia 249–50 isotonic saline 67
susceptibility of the elderly to 211 itchy skin 46, 81
hypothyroidism 73, 122, 124, 136, 167 ivabradine 57, 222, 239
borderline 67
hypovolaemia 61, 66 Jacob’s ladder 238
hypoxia 151 jaundice 46
joint replacement 223
ibandronate 60
ibuprofen 38, 96, 143, 200 Kegel exercises 241
idiopathic Parkinson’s disease 235 kettle tippers 254
IL1 251 key safe 226
IL6 251 knee(s)
ileo-femoral deep vein thrombosis (IFDVT) 153 acutely swollen 16
immobility 169 joint, building muscles that support 138
immunoglobulin 252 mild fixed flexion deformities of 200
impaired co-ordination in limbs 35 pain 91, 178
impairment 72 replacement 89
impetigo 166
impulse control disorders 85 lacunar strokes 209
incontinence 2, 9, 14, 58, 72, 78, 116, 188, 215, 253, 256 lamisil 26, 69
in demented patients 241 language deterioration 90
faecal 94, 132, 170, 198, 241 lansoprazole 51, 182
mixed 222–3, 254 Lasting Power of Attorney 173, 219
overflow 157 lateral medullary syndrome (Wallenberg) 75
pelvic floor exercises (PFE) and 110 laxatives 72, 170
stress 53, 58, 141, 222, 223 learning difficulties 26
urge 40, 53, 73, 181, 210, 222 left
urinary 2, 29, 154, 178, 181, 191, 233, 241, 254 cortical infarct 176
Independent Mental Capacity Advocate (IMCA) 60, 79, 242 hemi-neglect 203
indigestion 24 hemiparesis 209
infant mortality, fall in 137 hemiplegia 203
infective endocarditis 84 ileofemoral deep vein thrombosis 109, 130
inflammatory markers 221 internal carotid territory lesion 61
influenza vaccinations 180, 215 middle cerebral artery territory ischaemic strokes 129
inguinal hernia repair 95 -sided internal carotid artery stenosis 203
INDEX 265

-sided weakness 203, 209 malodour reduction 66


ventricular ejection fraction (LVEF) 44 managing at home 120, 206–7
leg(s) Marjolin’s ulcer 230
bowed legs 24, 67 marocytosis 247
claudication 43 maternal caloric restriction 162
lifters 84 medication concordance 59
inflatable 238 melanoma, ABCDE warning signs 83–4, 168
pain 185 memantine 164
ulcers 43, 134, 186 memory clinic 232
chronic venous 199 memory deterioration 129
non-healing 127 men, differential mortality between women
weakness 55 and 103
lesions 172, 230, 253 Ménière’s disease 61, 165, 191, 233
centrally crusting 215 meningism 50
cheek 130 meningitis 50, 55, 139
forehead 48 bacterial meningitis 133, 171
lentigo maligna 168 meningococcal disease 84, 170
lytic bone 55 Mental Capacity Act 162, 251
mucosal 166 mental health
ulcerating 134, 186 issues 251
lethargy 14, 19 team 12
levodopa 78, 102, 234, 235 Mental Health Act 97, 144, 162, 167
levomepromazine 68 mesalazine 69
levothyroxine 122, 124 metastatic
Lewy body dementia 48, 55, 62, 71, 156, 221, 233, 235 bowel cancer 119
lice infestations 174 breast cancer 21, 104
life expectancy 4, 55, 90, 137, 247 prostate disease 80
light-headedness 191 metformin 6, 22, 33, 93, 106, 109, 124, 179,
postural 14 186, 210
see also dizziness methimazole 69
lignocaine 16 methotrexate 126, 164, 210
limb weakness 118 metoclopramide 142, 250
liothyronine 136 microalbuminuria 224, 230
lisinopril 3, 8, 14, 99, 126 microscopic colitis 120, 160
long-term oxygen (LTOT) therapy 65 midazolam 68, 117, 166, 236
loperamide 80, 141 middle cerebral artery territory TIA 15
lorazepam 78, 113, 135 midodrine 100
losartan 56 migraine 159
low molecular weight heparin (LMWH) see heparin mild
lower limbs, blisters on 128 aortic stenosis 193
lower respiratory tract infections (LRTI) 36, 37, 41, 96, 200 cognitive impairment 9, 101
Lubidet 145 concentric hypertrophy of the left
lumbar ventricle 44
puncture 84 dyskinesia 29
spinal canal 240 dysuria 10
spinal stenosis 197, 240 kyphoscoliosis 36
lung cancer 115, 156 Miller Fisher syndrome 252
lupus anticoagulant 76 Mini-Mental State Examination (MMSE) 3, 17, 18, 25, 28,
lying to standing position 205, 246–7 29, 41, 51, 63, 125, 173, 184, 189, 191, 192, 197,
lytic bone lesions 55 202, 209, 228
for the blind (MMblind) 79
macrogol 197 impaired 79
Madopar® 17, 28, 196, 216 and mental capacity 251
magnesium 164 Mini Nutritional Assessment (MNA) 142
malabsorption syndrome 226 mirtazapine 100, 154, 216, 254
malnutrition 60, 147, 220 mixed peripheral neuropathy 210
in care home residents 95 MMSE see Mini-Mental State Examination 
Malnutrition Universal Screening Tool (MUST) 142 (MMSE)
266 INDEX

mobility 242 nitofurantoin 249
decreased 26, 40 nitrates
impaired 9 long-acting 222, 239
measurement of 64–5, 232 short-acting 222
TUG test 20, 64–5, 153, 188, 232 topical 85
modafenil 100 nitrolingual spray 222
moles 168 nocturia 73
monkey poles 136 nocturnal tonic clonic seizure 129
mono-arthritis 16 nocturnal urinary frequency 32
Montreal Cognitive Assessment 79 non-myxoedemic coma 136
morbidity 247 non-specific coagulation inhibitor 76
morphine 193, 235 non-steroidal cream 223
morphine sulphate (MST) 104, 150 non-verbal pain assessment tool 56, 165
mortality, differential mortality of men and women 103 norepinephrine 250
motor restlessness 53 norfloxacin 40
MR venography 139 nose bleeds 183, 227
MRI 169, 221, 246 NSAIDs 21, 52, 138, 150, 200, 223, 242
mucolytics 65 NSTEMI 51, 180
mucosal lesions 166 nursing homes, funding options 118
multi-organ failure 202 nutrition 242
multidisclipinary teams 153 assessment of 95, 142, 147
multiple myeloma 137
multiple sclerosis (MS) 14 occult depression 248
multisystem atrophy 235 occult malignancy 69
murmur, quiet pan-systolic 19 occupational therapy home assessment 160
muscarinic antagonists 21, 56, 157 octaplex 227
muscle aches, arms 180 oedema 158, 186, 250
muscle building 138 oesophageal adenocarcinoma 105
muscle wasting 242 oesophago-gastroduodenoscopy 81
myalgia 139, 225 olanazapine 155
myasthenia gravis 252 omeprazole 27, 107, 122
myeloma 137, 172, 255 ondansetron 142, 251
myeloproliferative disorders 137 ophthalmoplegia 252
myocardial perfusion scans 54 opiates 21, 66, 138, 191, 223, 236
myxoedemic coma 136 opioids 60, 66, 82, 150, 207, 250
oral
nasogastric suction 47 rehydration salts 66
nasogastric tube (NGT) 144, 219 ulcers 249
feeding 216, 219, 232, 238, 239 oramorph 46, 81, 150, 204
insertion 166 oropharyngeal dysphagia 108
National Audit of Intermediate Care (IC) 181 orthopaedic team 183
National Institutes of Health Stroke Scale (NIHSS) 101, 153 orthostatic hypotension 100, 172, 254
nausea 16, 35, 118, 128, 158, 211, 212 osteoarthritis 16, 32, 34, 37, 96, 124, 204
neck rigidity/stiffness 50, 133 hips 49
necrobiosis lipoidica diabeticorum 172 and knee pain 91, 178
necrosis 37 localized pain 228
needs assessment of eligibility for NHS Continuing Health of the spine 202, 207
Care 187 treatment of 138
neglect, left 118, 203 osteomalacia 67, 147, 160, 228
nerve blocks 236 osteomyelitis 95, 221
neuroleptic malignant syndrome 142 osteoporosis 3, 99, 120, 122, 127, 160, 162, 237, 254
neuroleptics see antipsychotic (neuroleptic) drugs Addison’s disease and 59
neuropathic agents 73 Cushing’s syndrome and 67
neuropathy 69, 228 hormone replacement therapy (HRT) and 28
neuropeptide Y 251 treatment 65
NGT see nasogastric tube (NGT) osteoporotic
NHS Continuing Health Care 93, 140, 187, 231 fractures 237
nicorandil 14, 57, 210, 222, 239, 249 crush 76
INDEX 267

fragility 247 penicillin 249
hip 152 perching stools 254
wedge 109 percutaneous endoscopic colostomy 80
overdoses percutaneous endoscopic gastrostomy (PEG) 144, 197, 235
accidental 187 perianal ulcers 249
self-overdoses with opioid analgesia 207 perifollicular haemorrhage with corkscrew hairs 64
oxybutinin 157 peripheral neuropathy 73
oxycodone 235 pernicious anaemia 210
oxygen 180, 232, 236 personality disorder, with schizoid trait 12
consumption 85 petechial rash 174
saturation 89, 192 phenothiazine 68
phenytoin 122, 162
pabrinex 161, 246 photophobia 50, 118
pacemakers 74 physical function, loss of 19
Paget’s disease 38, 42, 67, 77, 80 pigmented naevi (moles) 168
pain pill boxes, with alarm function 226
control 60 pill rolling tremor 191
non-verbal pain assessment tool 56, 165 piracetam 53
radicular 207 pitting scars, on finger tips 52
palliative biliary stenting 46 plasma 227
palliative care 96, 97, 143, 144, 159 plasmapheresis 252
palmar erythema 205 pneumonia 25, 29, 102, 188, 195, 201, 204, 214, 254
pamidronate 21, 55 aspiration 14, 129, 197, 200
pancreatic polypeptide (PPY) 251 left-sided 117
paracetamol 4, 21, 38, 92, 96, 99, 123, 138, 161, 165, 178, right basal 130, 216
197, 202, 204 polyarteritis nodosa 137
paranoia 12, 40, 206 polycythaemia rubra vera (PRV) 4, 55, 92, 139
parasitic infection 69 polymerase chain reaction (PCR) 170
parathyroid hormone (PTH) 38, 54–5, 228, 237 polymorphic ventricular tachycardia 126
Parkinsonism 17 polymyalgia rheumatica 67, 95, 107, 122, 176, 228
vascular 62, 221, 234 pompholyx 149
Parkinson’s disease 2, 29, 40, 47, 78, 115, 134, 142, 156, post-phlebitic syndrome 153
177, 197, 214, 217 postural balance, impaired 191
autonomic dysfunction and 148 postural collapse 151
bilateral 28 postural hypotension 9, 58, 115, 119, 136, 157, 163
cheek brown pigmented lesions 130 postural tremor 176
dementia 71, 100 potassium 1
elective inguinal hernia repair 95 prednisolone 122, 176, 193, 224, 237
and elective resection of a Duke’s B adenocarcinoma pressure sores 2, 6, 54, 56, 117, 131, 148, 165, 169
bowel 87 bilateral grade 2 2, 102
falls and 102, 172 grade 1-2 77
and getting in and out of bed 238 grade 3-4 77
getting medication on time 83, 135 grading of 233
hospital discharge of a patient with dementia 100 risk assessments 2
idiopathic 221, 235 sacral 2, 6, 29, 37, 131
impulse control disorders 85 pressure ulcers see pressure sores
psychosis within 256 primary intracerebral tumour 96
review of patient with 190 primary progressive aphasias 138
and ropinirole 51 primary Raynaud’s phenomenon 85
tremor-dominant 196, 216 prochlorperazine 13, 142, 191, 250
and turning over in bed 88 procyclidine 29, 71
partial ptosis 35 progressive non-fluent aphasia 138
partial thromboplastin time (PTT) 78 prolactin, serum 169
pathology 72 prolapsed bladder (cystoceole) 225
pelvic floor property exit sensors 159
exercises (PFE) 110, 154–5, 215, 254 prophylactic short-acting GTN 239
weakness 141 prophylactic treatment 242
pemphigus foliaceus 166 propiomelacortin 251
268 INDEX

propranolol 53 respiratory function 51


propylthiouracil 69 restless legs syndrome 53, 211, 250
prostaglandins 85 restlessness 1, 213
prostate 9 retinal vascular disease 245
cancer 191, 193 retinal vein occlusion 137, 245
enlargement 150 retrograde amnesia 144
proteinuria 179 rhabdomyolyisis 225
prothrombin 227 rheumatoid arthritis 52, 85, 98, 99, 126, 164, 172,
proton pump inhibitors 70, 162 193, 210
proximal muscle weakness 120 ribs, fractures 20, 117
psoriasis 149 rickets 67
psoriatic arthropathy 49 right
psychosis, acute 53 cerebral hemisphere infarction 179
psychotic depression 97, 228 hemiplegia 125, 133
pubic ramus, fractured 23 rigidity 191
pudendal nerve stimulation 141 risedronate 67
pulmonary embolism, recurrent 136–7 risperidone 155, 164
pulmonary rehabilitation 56 ritanserin 53
pulmonary vein ablation 229 rivaroxaban 139
pumonary embolism 236 rivastigmine 1, 11, 71, 100, 134, 135, 163
pupils, meiotic 35 Rivermead Score 153
pyoderma gangrenosum 172 rope ladders 136
ropinirole 51, 196, 216, 217
QFracture 247 rotigotine patch 83
quadriplegy 131 rousal difficulties 133
Quetelet index 142
quetiapine 155 sacrum, mark on 188
quiet pan-systolic murmur 19 Safeguarding of a Vulnerable Adult (SOVA) 63
safeguarding of individuals 74
RADAR (Royal Association for Disability and Rehabilitation) salbutamol 5, 43, 116
keys 145 saline 255
radionucleotide scans 246 sarcoidosis 137
ramipril 6, 17, 33, 51, 91, 93, 180 sarcopenia 212, 234
ranazoline 239 scabies 149
ranitidine 160 sclerotic bone lesions 80
rash screening, of asymptomatic individuals for
from ampicillin 210 cancer 216
on lower limbs 19 scurvy 64
on palms and soles 103 seborrheic keratoses 168
petechial 174 secondary Raynaud’s phenomenon 85
rectal prolapse 241 seizures 104, 113, 129, 169, 233, 248, 253
rectopexy 241 anti-epileptic drugs (AED) for 211
refeeding syndrome 152, 174, 220 control of 166
reflux oesophagitis 122 recurrent 156
rehabilitation 26, 78, 82 tonic-clonic 16
framework for assessing patients for 72 selective serotonin reuptake inhibitors (SSRIs) 228, 250
services, team working in 108 selegiline 28, 29, 71, 83
rehydration 81 ‘self-management’ 65
relaxation biofeedback 85 self-neglect 12, 60, 106, 151
reminiscence therapy 83 sensorimotor neuropathy 69
renal artery hypoperfusion 161 sepsis 195
renal impairment 68, 137, 225, 237 sertraline 122, 160
renin 161 severe type 2 respiratory failure 202
renin-angiotensin system 121 shortness of breath 14, 89, 179, 180, 198
reperfusion therapies 147 SIADH (syndrome of inappropriate antidiuretic hormone)
replacement arthroplasty 64 254, 255
residential homes 30, 227 sigmoid volvulus 42
funding a placement in 238 sigmoidopexy 80
INDEX 269

simvastatin 6, 8, 14, 15, 17, 21, 27, 36, 62, 91, 99, 109, sub-acute upper GI bleed 61
113, 124, 127, 132, 178, 180, 186, 193, 196, sudden onset left arm paraesthesia 15
203, 210 sudden onset left hemiparesis 101, 209
Sinemet® 78, 100 suicide 61, 173, 219
Sinemet Plus® 40, 47, 87 assisted suicide 14, 219
single question in delirium (SQiD) 240 attempted suicide 61, 123
sinus rhythm 208 sulphonylureas 154
sinus tachycardia 151, 192 swallow 238
sitagliptin 154 assessment 234
skin, itchy 46, 165 at-risk 118, 129
sleep pattern, disrupted 182 syncopal falls 163
small bowel obstruction, recurrent 212 syncope 169, 253
smell, sense of 251 syndrome of inappropriate antidiuretic hormone (SIADH)
smoking 31 254, 255
sodium docusate 18 systemic inflammatory diseases 137
sodium, serum 216, 255
sodium valproate 53, 113, 156, 176, 221 T-cell function, age-related decline in 167
solifenacin 157 tachypnoeic 198
spasticity 7, 57 tamsulosin 115, 157
of the left foot 179 taste, loss of 28
management of 223 taste, sense of 251
speech 36 tazocin 249
muddled 50 technetium scintogram 247
slurred 118 telecare 9, 59, 93, 224
spider naevi 205 telehealth 9, 59, 93, 141, 224
splinter haemorrhages 19 telemedicine 59, 141
squamous cell carcinoma 164, 172, 230 telomere shortening 162
statins 115, 222, 225, 232, 239 temazepam 114
steady spoons 254 temporary respite placement 79
stenosis 61 terazosin 157
steroids 21, 171, 215, 237 theophylline 56
inhalers 21 thiamine 246, 247
injections 223 deficiency 70
Stevens–Johnson syndrome 166 thrombolysis 147
stoma 80 thrombophilia 137
streptococcal disease 170 thromboprophylaxis 78, 96, 109, 130
stress-echo scan 54 thrombotic partial anterior circulation strokes 7, 131
strokes 7, 32, 36, 118, 127, 164, 203 thyroid-stimulating hormone (TSH) 139
acute right middle cerebral artery territory 101 thyrotoxicosis 162
convulsions and 221 thyroxine 136, 162
early NGT feeding in patients 232 timed get up and go test (TUG) 20, 64–5, 153, 188, 232
embolic 139, 149 timolol eye drops 34
ischaemic 166, 188, 232 tingling of fingers and toes 26
lacunar 209 tinnitis 61
left middle cerebral artery territory ischaemic 30, tiredness 26
125, 129 tizanidine 57
NIH Stroke Scale 153 Todd’s paresis 76
Oxford classification of 245 tolcapone 28
post-stroke epilepsy 113 tolterodine 40, 78
posterior circulation 108 tolvaptan 255
and recurrent falls 37 tonic-clonic seizures 16, 129, 176, 221
rehabilitation after 199 torsades 164
right middle cerebral artery ischaemic 49 tramadol 122, 162, 204
risk of 139 trans-disciplinary teams 153
thrombotic partial anterior circulation 7, 131 transient global amnesia (TGA) 55, 144
strontium 60, 237 transient ischaemic attack (TIA) 171
strontium ranelate 67 transvaginal tape surgery (TVT) 223
sub-acute right middle cerebral artery territory infarct 187 trauma team 183
270 INDEX

treatment visual hallucinations 192, 235


decision to decline 162 visual impairment 62, 79, 236
refusal 244 mild refractory 13
triamcinolone 16 vitamin A, deficiency 70
trimethoprim 249 vitamin C
troponin 115 deficiency 70, 220
tuberculosis 26 supplements 64
TUG test 20, 64–5, 153, 188, 232 vitamin D 18, 78, 100, 107, 115, 147, 156, 176, 237,
tumour markers, serum 147 243, 254
tumour necrosis factor alpha (TNFα) 251 deficiency 228, 237
tumours, primary intracerebral 96 supplements 3, 63
type-1 second degree heartblock (Wenckebach) 34 vitamin E 58
type-2 heart block 175 vitamin K 27, 70, 149, 221, 227
vitamin supplements 58
ulcers 230, 249 volvulus 80
bleeding ulcers 66 vomiting 4, 23, 25, 47, 92, 117, 211
duodenal 26, 27
foot 48, 175 walking
heel 148 health benefits of 230
leg 127, 134, 186, 199 speed 234
Marjolin’s 230 walking aids 25, 68, 76, 99, 125
non-healing 80, 127, 164, 210 gutter frames 163
oral 249 quad sticks 76
perianal 210, 249 rollator frames 120, 163
venous 43, 80 walking frames 76, 146
unidisciplinary teams 153 walking sticks 76
upper gastrointestinal bleed 27 walking trolleys 146, 254
upper gastrointestinal tumour 81 Zimmer frames 163
urea 225 Wallenberg Syndrome 75
urinalysis 179 wandering 68, 159, 187, 212
urinary bone markers 77 monitors 226
urinary frequency 10, 104, 181, 210 warfarin 57, 70, 103, 139, 149, 153, 169, 175, 183,
nocturnal 32 227, 229
urinary hydroxyproline 77 weakness 26
urinary retention 18, 47, 82, 150 Weber syndrome 75
urinary tract infections (UTIs) 2, 6, 10, 23, 25, 78, 111, 112, Wegener’s granulomatosis 137
121, 132, 150, 155, 198, 249 weight gain 89, 154
recurrent 98 weight loss 31, 101, 110, 138, 147, 217, 226
urosepsis 71, 161 Wenckebach (type 1 second degree
utensil hand grip 254 heartblock) 34
wet age-related macular
valproate 162, 221 degeneration 239
valproic acid (sodium valproate) 53 wheelchairs 146
varifocal lenses 230 white cell scan 221
venous sinus thrombosis 16, 55, 139, 164 Whiz Freedom 145
venous thromboembolism (VTE) 78, 137, 149, 151, 153 ‘winter vomiting bug’ 23
ventilation-perfusion (VQ) scanning 236 withdrawness 19, 184
verapamil 164 women, differential mortality between men
verbal abuse, from dementia patients 182 and 103
vertigo 60, 61, 128, 165, 233 wrist actigraph 159
vision
correction of 230 X-rays 221, 246
distortion of 196, 245
loss of 90, 204, 245 zinc deficiency 70

Vous aimerez peut-être aussi